Buried Deep

Article Type
Changed
Mon, 11/29/2021 - 11:02
Display Headline
Buried Deep

This icon represents the patient’s case. Each paragraph that follows represents the discussant’s thoughts.

A 56-year-old-woman with a history of HIV and locally invasive ductal carcinoma recently treated with mastectomy and adjuvant doxorubicin and cyclophosphamide, now on paclitaxel, was transferred from another hospital with worsening nausea, epigastric pain, and dyspnea. She had been admitted multiple times to both this hospital and another hospital and had extensive workup over the previous 2 months for gastrointestinal (GI) bleeding and progressive dyspnea with orthopnea and paroxysmal nocturnal dyspnea in the setting of a documented 43-lb weight loss.

The patient presents with two clusters of symptoms in the context of chemotherapy regimens following mastectomy for local breast cancer. The first cluster of nausea, epigastric pain, and weight loss indicates upper-GI dysfunction/pathology, such as gastroparesis from paclitaxel-induced autonomic neuropathy given onset of symptoms following paclitaxel chemotherapy. The pretest probability of this is fairly high; however, in light of the GI bleeding, it is prudent to evaluate for other etiologies. The absence of vomiting does not exclude gastroparesis. Other considerations would be peptic ulcer disease or, less likely, the development of a gastric malignancy. A mechanical cause of gastric outlet obstruction from her known cancer is unlikely in this patient with localized breast cancer and recently completed chemotherapy, as breast cancer is unlikely to metastasize to the stomach. Preliminary workup would consist of computed tomography (CT) scan of the abdomen and an esophagogastroduodenoscopy (EGD) to further evaluate these etiologies. The second cluster of dyspnea, paroxysmal nocturnal dyspnea, and orthopnea suggests heart failure, given recent treatment with doxorubicin, and prompts consideration of anthracycline-induced cardiomyopathy. Work-up would include obtaining a transthoracic echocardiogram (TTE) to look for a decrease in her ejection fraction.

Her past medical history was otherwise significant only for the events of the previous few months. Eight months earlier, she was diagnosed with grade 3 triple-negative (estrogen receptor, progesterone receptor, and human epidermal growth factor receptor 2) invasive ductal carcinoma and underwent mastectomy with negative sentinel lymph node biopsy. She completed four cycles of adjuvant doxorubicin and cyclophosphamide and most recently completed cycle three of paclitaxel chemotherapy.

Her HIV disease was controlled with an antiretroviral regimen of dolutegravir/rilpivirine. She had an undetectable viral load for 20 years (CD4, 239 cells/μL 2 weeks prior to transfer).

Her social history included a 1-pack-year smoking history. She denied alcohol or illicit drug use. Family history included pancreatic cancer in her father and endometrial cancer in her paternal grandmother. She was originally from Mexico but moved to Illinois 27 years earlier.

Work-up for her dyspnea was initiated 7 weeks earlier: noncontrast CT of the chest showed extensive diffuse interstitial thickening and ground-glass opacities bilaterally. Bronchoscopy showed no gross abnormalities, and bronchial washings were negative for bacteria, fungi, Pneumocystis jirovecii , acid-fast bacilli, and cancer. She also had a TTE, which showed an ejection fraction of 65% to 70% and was only significant for a pulmonary artery systolic pressure of 45 mm Hg . She was diagnosed with paclitaxel-induced pneumonitis and was discharged home with prednisone 50 mg daily, dapsone, pantoprazole, and 2 L oxygen via nasal cannula.

Two weeks later, she was admitted for coffee-ground emesis and epigastric pain. Her hemoglobin was 5.9 g/dL, for which she was transfused 3 units of packed red blood cells. EGD showed bleeding from diffuse duodenitis, which was treated with argon plasma coagulation. She was also found to have bilateral pulmonary emboli and lower-extremity deep venous thromboses. An inferior vena cava filter was placed, and she was discharged. One week later, she was readmitted with melena, and repeat EGD showed multiple duodenal ulcers with no active bleeding. Colonoscopy was normal. She was continued on prednisone 40 mg daily, as any attempts at tapering the dose resulted in hypotension.

At the time of transfer, she had presented to the outside hospital with worsening nausea and epigastric pain, increasing postprandial abdominal pain, ongoing weight loss, worsening dyspnea on exertion, paroxysmal nocturnal dyspnea, and orthopnea. She denied symptoms of GI bleeding at that time.

Her imaging is consistent with, albeit not specific for, paclitaxel-induced acute pneumonitis. Her persistent dyspnea may be due to worsening of this pneumonitis. Given her GI bleeding and continued epigastric pain, worsening peptic ulcer disease is the likely culprit. The patient has had multiple episodes of bleeding from multiple duodenal ulcers while on proton pump inhibitors, so we should consider reasons for this progression, such as use of nonsteroidal anti-inflammatory drugs (NSAIDs); Helicobacter pylori, herpes simplex virus (HSV), Epstein-Barr virus (EBV), or cytomegalovirus (CMV) infection; Crohn disease, and antral G-cell hyperfunction, as well as lymphoproliferative disease. I would start with histopathology, urease testing, and viral cultures from biopsy specimens at EGD to help eliminate these possibilities for progression of her peptic ulcer disease. Another consideration, albeit rare for progression of peptic ulcer disease and specifically duodenal ulcers, is a gastrinoma-driven acid hypersecretion state, as in the Zollinger-Ellison syndrome. If the above studies do not reveal a cause for her continued peptic ulcer disease, I would check a fasting serum gastrin level and gastric pH to further evaluate this possibility.

Upon arrival on physical exam, her temperature was 35.4° C, heart rate 112 beats per minute, blood pressure 135/96 mm Hg, respiratory rate 34 breaths per minute, and oxygen saturation 97% on room air. She was ill- appearing and in mild respiratory distress with severe muscle wasting. Cervical and supraclavicular lymphadenopathy were not detected. Heart sounds were normal without murmurs. Her jugular venous pressure was approximately 7 cm H2O. She had no lower-extremity edema. On lung exam, diffuse rhonchi were audible bilaterally with no crackles or wheezing. There was no accessory muscle use. No clubbing was present. Her abdomen was soft and mildly tender in the epigastrium with normal bowel sounds.

Her labs revealed a white blood cell (WBC) count of 5,050/μL (neutrophils, 3,600/μL; lymphocytes, 560/μL; eosinophils, 560/μL; hemoglobin, 8.7 g/dL; mean corpuscular volume, 89.3 fL; and platelets, 402,000/μL). Her CD4 count was 235 cells/μL. Her comprehensive metabolic panel demonstrated a sodium of 127 mmol/L; potassium, 4.0 mmol/L; albumin, 2.0 g/dL; calcium, 8.6 mg/dL; creatinine, 0.41 mg/dL; aspartate aminotransferase (AST), 11 U/L; alanine aminotransferase (ALT), 17 U/L; and serum osmolarity, 258 mOs/kg. Her lipase was 30 U/L, and lactate was 0.8 mmol/L. Urine studies showed creatinine 41 mg/dL, osmolality 503 mOs/kg, and sodium 53 mmol/L.

At this point, the patient has been diagnosed with multiple pulmonary emboli and recurrent GI bleeding from duodenal ulcers with chest imaging suggestive of taxane-induced pulmonary toxicity. She now presents with worsening dyspnea and upper-GI symptoms.

Her dyspnea may represent worsening of her taxane-induced lung disease. However, she may have developed a superimposed infection, heart failure, or further pulmonary emboli. A CT pulmonary angiogram to evaluate for infection, recurrent pulmonary emboli, and worsening of her known taxane-induced pulmonary toxicity should be performed. If the CT demonstrates new areas of consolidation suggestive of a superimposed infection, a bronchoscopy would be warranted to obtain specimens for bacterial, fungal, and viral cultures.

On exam, she is in respiratory distress, almost mildly hypothermic and tachycardic with rhonchi on auscultation. This combination of findings could reflect worsening of her pulmonary disease and/or infection on the background of her cachectic state. Her epigastric tenderness, upper-GI symptoms, and anemia have continued to cause concern for persistent duodenal ulcers. Repeat EGD is warranted to evaluate for ulcers, recent bleed stigmata, and/or complications of ulcers, such as stricture formation, obstruction, or perforation. An abdominal CT scan should be done if the EGD does not show worsening of her peptic ulcer disease or obvious complications. The CT scan is more sensitive for certain complications of peptic ulcer disease, such as perforation or proximal small bowel obstruction due to strictures.

Her anemia could represent ongoing blood loss since her last EGD or an inflammatory state due to infection. Also of concern is her use of dapsone, which can lead to hemolysis with or without glucose-6-phosphate dehydrogenase deficiency (G6PD), and this should be excluded.

She has hypotonic hyponatremia and apparent euvolemia with a high urine sodium and osmolality; this suggests syndrome of inappropriate antidiuretic hormone secretion, which may be due to her ongoing pulmonary disease process.

On day 3 of her hospitalization, her abdominal pain became more diffuse and colicky, with two episodes of associated nonbloody bilious vomiting. During the next 48 hours, her abdominal pain and tenderness worsened diffusely but without rigidity or peritoneal signs. She developed mild abdominal distention. An abdominal X-ray showed moderate to large stool burden and increased bowel dilation concerning for small bowel obstruction. A nasogastric tube was placed, with initial improvement of her abdominal pain and distention. On the morning of day six of hospitalization, she had approximately 100 mL of hematemesis. She immediately became hypotensive to the 50s/20s, and roughly 400 mL of sanguineous fluid was suctioned from her nasogastric tube. She was promptly given intravenous (IV) fluids and 2 units of cross-matched packed red blood cells with normalization of her blood pressure and was transferred to the medical intensive care unit (MICU).

Later that day, she had an EGD that showed copious clots and a severely friable duodenum with duodenal narrowing. Duodenal biopsies were taken.

The duodenal ulcers have led to a complication of stricture formation and obstruction resulting in some degree of small bowel obstruction. EGD with biopsies can shed light on the etiology of these ulcers and can specifically exclude viral, fungal, protozoal, or mycobacterial infection; infiltrative diseases (lymphoma, sarcoidosis, amyloidosis); cancer; and inflammatory noninfectious diseases such as vasculitis/connective tissue disorder. Biopsy specimens should undergo light and electron microscopy (for protozoa-like Cryptosporidium); stains for fungal infections such as histoplasmosis, Candida, and Cryptococcus; and stains for mycobacterium. Immunohistochemistry and polymerase chain reaction (PCR) testing can identify CMV, HIV, HSV, and EBV within the duodenal tissue.

She remained on methylprednisolone 30 mg IV because of her known history of pneumonitis and concern for adrenal insufficiency in the setting of acute illness. Over the next 3 days, she remained normotensive with a stable hemoglobin and had no further episodes of hematemesis. She was transferred to the general medical floor.

One day later, she required an additional unit of cross-matched red blood cells because of a hemoglobin decrease to 6.4 g/dL. The next day, she developed acute-onset respiratory distress and was intubated for hypoxemic respiratory failure and readmitted to the MICU.

Her drop in hemoglobin may reflect ongoing bleeding from the duodenum or may be due to diffuse alveolar hemorrhage (DAH) complicating her pneumonitis. The deterioration in the patient’s respiratory status could represent worsening of her taxane pneumonitis (possibly complicated by DAH or acute respiratory distress syndrome), as fatalities have been reported despite steroid treatment. However, as stated earlier, it is prudent to exclude superimposed pulmonary infection or recurrent pulmonary embolism. Broad-spectrum antibiotics should be provided to cover hospital-acquired pneumonia. Transfusion-related acute lung injury (TRALI) as a cause of her respiratory distress is much less likely given onset after 24 hours from transfusion. Symptoms of TRALI almost always develop within 1 to 2 hours of starting a transfusion, with most starting within minutes. The timing of respiratory distress after 24 hours of transfusion also makes transfusion-associated circulatory overload unlikely, as this presents within 6 to 12 hours of a transfusion being completed and generally in patients receiving large transfusion volumes who have underlying cardiac or renal disease.

Her duodenal pathology revealed Strongyloides stercoralis infection (Figure 1), and she was placed on ivermectin. Steroids were continued due to concern for adrenal insufficiency in the setting of critical illness and later septic shock. Bronchoscopy was also performed, and a specimen grew S stercoralis. She developed septic shock from disseminated S stercoralis infection that required vasopressors. Her sanguineous orogastric output increased, and her abdominal distension worsened, concerning for an intra-abdominal bleed or possible duodenal perforation. As attempts were made to stabilize the patient, ultimately, she experienced cardiac arrest and died.

Ruge10050728e_f1.JPG

The patient succumbed to hyperinfection/dissemination of strongyloidiasis. Her risk factors for superinfection included chemotherapy and high-dose steroids, which led to an unchecked autoinfection.

A high index of suspicion remains the most effective overall diagnostic tool for superinfection, which carries a mortality rate of up to 85% even with treatment. Therefore, prevention is the best treatment. Asymptomatic patients with epidemiological exposure or from endemic areas should be evaluated for empiric treatment of S stercoralis prior to initiation of immunosuppressive treatment.

COMMENTARY

Strongyloides stercoralis is a helminth responsible for one of the most overlooked tropical diseases worldwide.1 It is estimated that 370 million individuals are infected with S stercoralis globally, and prevalence in the endemic tropics and subtropics is 10% to 40%.2,3Strongyloides stercoralis infection is characterized by typically nonspecific cutaneous, pulmonary, and GI symptoms, and chronic infection can often be asymptomatic. Once the infection is established, the entirety of the S stercoralis unique life cycle can occur inside the human host, forming a cycle of endogenous autoinfection that can keep the host chronically infected and infectious for decades (Figure 24). While our patient was likely chronically infected for 27 years, cases of patients being infected for up to 75 years have been reported.5 Though mostly identified in societies where fecal contamination of soil and poor sanitation are common, S stercoralis should be considered among populations who have traveled to endemic areas and are immunocompromised.

JHMVol16No7_Ruge10050728e_f2.JPG

Most chronic S stercoralis infections are asymptomatic, but infection can progress to the life-threatening hyperinfection phase, which has a mortality rate of approximately 85%.6 Hyperinfection and disseminated disease occur when there is a rapid proliferation of larvae within the pulmonary and GI tracts, but in the case of disseminated disease, may travel to the liver, brain, and kidneys.7,8 Typically, this is caused by decreased cellular immunity, often due to preexisting conditions such as human T-cell leukemia virus type 1 (HTLV-1) or medications that allow larvae proliferation to go unchecked.6,7 One common class of medications known to increase risk of progression to hyperinfection is corticosteroids, which are thought to both depress immunity and directly increase larvae population growth.6,9 Our patient had been on a prolonged course of steroids for her pulmonary symptoms, with increased doses during her acute illness because of concern for adrenal insufficiency; this likely further contributed to her progression to hyperinfection syndrome. Furthermore, the patient was also immunocompromised from chemotherapy. In addition, she had HIV, which has a controversial association with S stercoralis infection. While previously an AIDS-defining illness, prevalence data indicate a significant co-infection rate between S stercoralis and HIV, but it is unlikely that HIV increases progression to hyperinfection.3

Diagnosing chronic S stercoralis infection is difficult given the lack of a widely accepted gold standard for diagnosis. Traditionally, diagnosis relied on direct visualization of larvae with stool microscopy studies. However, to obtain adequate sensitivity from this method, up to seven serial stool samples must be examined, which is impractical from patient, cost, and efficiency standpoints.10 While other stool-based techniques, such as enriching the stool sample, stool agar plate culture, or PCR-based stool analysis, improve sensitivity, all stool-based studies are limited by intermittent larvae shedding and low worm burden associated with chronic infection.11 Conversely, serologic studies have higher sensitivity, but concerns exist about lower specificity due to potential cross-reactions with other helminths and the persistence of antibodies even after larvae eradication.11,12 Patients with suspected S stercoralis infection and pulmonary infiltrates on imaging may have larvae visible on sputum cultures. A final diagnostic method is direct visualization via biopsy during endoscopy or bronchoscopy, which is typically recommended in cases where suspicion is high yet stool studies have been negative.13 Our patient’s diagnosis was made by duodenal biopsy after her stool study was negative for S stercoralis.

Deciding who to test is difficult given the nonspecific nature of the symptoms but critically important because of the potential for mortality if the disease progresses to hyperinfection. Diagnosis should be suspected in a patient who has spent time in an endemic area and presents with any combination of pulmonary, dermatologic, or GI symptoms. If suspicion for infection is high in a patient being assessed for solid organ transplant or high-dose steroids, prophylactic treatment with ivermectin should be considered. Given the difficulty in diagnosis, some have suggested using eosinophilia as a key diagnostic element, but this has poor predictive value, particularly if the patient is on corticosteroids.7 This patient did not manifest with significant eosinophilia throughout her hospitalization.

This case highlights the difficulties of S stercoralis diagnosis given the nonspecific and variable symptoms, limitations in testing, and potential for remote travel history to endemic regions. It further underscores the need for provider vigilance when starting patients on immunosuppression, even with steroids, given the potential to accelerate chronic infections that were previously buried deep in the mucosa into a lethal hyperinfectious state.

TEACHING POINTS

  • The cycle of autoinfection by S stercoralis allows it to persist for decades even while asymptomatic. This means patients can present with infection years after travel to endemic regions.
  • Because progression to hyperinfection syndrome carries a high mortality rate and is associated with immunosuppressants, particularly corticosteroids, screening patients from or who have spent time in endemic regions for chronic S stercoralis infection is recommended prior to beginning immunosuppression.
  • Diagnosing chronic S stercoralis infection is difficult given the lack of a highly accurate, gold-standard test. Therefore, if suspicion for infection is high yet low-sensitivity stool studies have been negative, direct visualization with a biopsy is a diagnostic option.

Acknowledgment

The authors thank Dr Nicholas Moore, microbiologist at Rush University Medical Center, for his assistance in obtaining and preparing the histology images.

References

1. Olsen A, van Lieshout L, Marti H, et al. Strongyloidiasis--the most neglected of the neglected tropical diseases? Trans R Soc Trop Med Hyg. 2009;103(10):967-972. https://doi.org/10.1016/j.trstmh.2009.02.013
2. Bisoffi Z, Buonfrate D, Montresor A, et al. Strongyloides stercoralis: a plea for action. PLoS Negl Trop Dis. 2013;7(5):e2214. https://doi.org/10.1371/journal.pntd.0002214
3. Schär F, Trostdorf U, Giardina F, et al. Strongyloides stercoralis: global distribution and risk factors. PLoS Negl Trop Dis. 2013;7(7):e2288. https://doi.org/10.1371/journal.pntd.0002288
4. Silva AJ, Moser M. Life cycle of Strongyloides stercoralis. Accessed June 5, 2020. https://www.cdc.gov/parasites/strongyloides/biology.html
5. Prendki V, Fenaux P, Durand R, Thellier M, Bouchaud O. Strongyloidiasis in man 75 years after initial exposure. Emerg Infect Dis. 2011;17(5):931-932. https://doi.org/10.3201/eid1705.100490
6. Nutman TB. Human infection with Strongyloides stercoralis and other related Strongyloides species. Parasitology. 2017;144(3):263-273. https://doi.org/10.1017/S0031182016000834
7. Naidu P, Yanow SK, Kowalewska-Grochowska KT. Eosinophilia: a poor predictor of Strongyloides infection in refugees. Can J Infect Dis Med Microbiol. 2013;24(2):93-96. https://doi.org/10.1155/2013/290814
8. Kassalik M, Mönkemüller K. Strongyloides stercoralis hyperinfection syndrome and disseminated disease. Gastroenterol Hepatol (N Y). 2011;7(11):766-768.
9. Genta RM. Dysregulation of strongyloidiasis: a new hypothesis. Clin Microbiol Rev. 1992;5(4):345-355. https://doi.org/10.1128/cmr.5.4.345
10. Siddiqui AA, Berk SL. Diagnosis of Strongyloides stercoralis infection. Clin Infect Dis. 2001;33(7):1040-1047. https://doi.org/10.1086/322707
11. Buonfrate D, Requena-Mendez A, Angheben A, et al. Accuracy of molecular biology techniques for the diagnosis of Strongyloides stercoralis infection—a systematic review and meta-analysis. PLoS Negl Trop Dis. 2018;12(2):e0006229. dohttps://doi.org/10.1371/journal.pntd.0006229
12. Arifin N, Hanafiah KM, Ahmad H, Noordin R. Serodiagnosis and early detection of Strongyloides stercoralis infection. J Microbiol Immunol Infect. 2019;52(3):371-378. https://doi.org/10.1016/j.jmii.2018.10.001
13. Lowe RC, Chu JN, Pierce TT, Weil AA, Branda JA. Case 3-2020: a 44-year-old man with weight loss, diarrhea, and abdominal pain. N Engl J Med. 2020;382(4):365-374. https://doi.org/10.1056/NEJMcpc1913473

Article PDF
Author and Disclosure Information

1Department of Medicine, Thomas Jefferson University Hospital, Philadelphia, Pennsylvania; 2Department of Medicine, Rush University Medical Center, Chicago, Illinois; 3Department of Medicine, Northwestern University Feinberg School of Medicine, Chicago, Illinois; 4Department of Infectious Disease, Rush University Medical Center, Chicago, Illinois; 5Department of Pathology, Rush University Medical Center, Chicago, Illinois; 6Department of Medicine, John H Stroger, Jr Hospital of Cook County, Chicago, Illinois.

Disclosures
The authors report having nothing to disclose.

Issue
Journal of Hospital Medicine 16(12)
Publications
Topics
Page Number
757-762. Published Online First July 28, 2021
Sections
Author and Disclosure Information

1Department of Medicine, Thomas Jefferson University Hospital, Philadelphia, Pennsylvania; 2Department of Medicine, Rush University Medical Center, Chicago, Illinois; 3Department of Medicine, Northwestern University Feinberg School of Medicine, Chicago, Illinois; 4Department of Infectious Disease, Rush University Medical Center, Chicago, Illinois; 5Department of Pathology, Rush University Medical Center, Chicago, Illinois; 6Department of Medicine, John H Stroger, Jr Hospital of Cook County, Chicago, Illinois.

Disclosures
The authors report having nothing to disclose.

Author and Disclosure Information

1Department of Medicine, Thomas Jefferson University Hospital, Philadelphia, Pennsylvania; 2Department of Medicine, Rush University Medical Center, Chicago, Illinois; 3Department of Medicine, Northwestern University Feinberg School of Medicine, Chicago, Illinois; 4Department of Infectious Disease, Rush University Medical Center, Chicago, Illinois; 5Department of Pathology, Rush University Medical Center, Chicago, Illinois; 6Department of Medicine, John H Stroger, Jr Hospital of Cook County, Chicago, Illinois.

Disclosures
The authors report having nothing to disclose.

Article PDF
Article PDF
Related Articles

This icon represents the patient’s case. Each paragraph that follows represents the discussant’s thoughts.

A 56-year-old-woman with a history of HIV and locally invasive ductal carcinoma recently treated with mastectomy and adjuvant doxorubicin and cyclophosphamide, now on paclitaxel, was transferred from another hospital with worsening nausea, epigastric pain, and dyspnea. She had been admitted multiple times to both this hospital and another hospital and had extensive workup over the previous 2 months for gastrointestinal (GI) bleeding and progressive dyspnea with orthopnea and paroxysmal nocturnal dyspnea in the setting of a documented 43-lb weight loss.

The patient presents with two clusters of symptoms in the context of chemotherapy regimens following mastectomy for local breast cancer. The first cluster of nausea, epigastric pain, and weight loss indicates upper-GI dysfunction/pathology, such as gastroparesis from paclitaxel-induced autonomic neuropathy given onset of symptoms following paclitaxel chemotherapy. The pretest probability of this is fairly high; however, in light of the GI bleeding, it is prudent to evaluate for other etiologies. The absence of vomiting does not exclude gastroparesis. Other considerations would be peptic ulcer disease or, less likely, the development of a gastric malignancy. A mechanical cause of gastric outlet obstruction from her known cancer is unlikely in this patient with localized breast cancer and recently completed chemotherapy, as breast cancer is unlikely to metastasize to the stomach. Preliminary workup would consist of computed tomography (CT) scan of the abdomen and an esophagogastroduodenoscopy (EGD) to further evaluate these etiologies. The second cluster of dyspnea, paroxysmal nocturnal dyspnea, and orthopnea suggests heart failure, given recent treatment with doxorubicin, and prompts consideration of anthracycline-induced cardiomyopathy. Work-up would include obtaining a transthoracic echocardiogram (TTE) to look for a decrease in her ejection fraction.

Her past medical history was otherwise significant only for the events of the previous few months. Eight months earlier, she was diagnosed with grade 3 triple-negative (estrogen receptor, progesterone receptor, and human epidermal growth factor receptor 2) invasive ductal carcinoma and underwent mastectomy with negative sentinel lymph node biopsy. She completed four cycles of adjuvant doxorubicin and cyclophosphamide and most recently completed cycle three of paclitaxel chemotherapy.

Her HIV disease was controlled with an antiretroviral regimen of dolutegravir/rilpivirine. She had an undetectable viral load for 20 years (CD4, 239 cells/μL 2 weeks prior to transfer).

Her social history included a 1-pack-year smoking history. She denied alcohol or illicit drug use. Family history included pancreatic cancer in her father and endometrial cancer in her paternal grandmother. She was originally from Mexico but moved to Illinois 27 years earlier.

Work-up for her dyspnea was initiated 7 weeks earlier: noncontrast CT of the chest showed extensive diffuse interstitial thickening and ground-glass opacities bilaterally. Bronchoscopy showed no gross abnormalities, and bronchial washings were negative for bacteria, fungi, Pneumocystis jirovecii , acid-fast bacilli, and cancer. She also had a TTE, which showed an ejection fraction of 65% to 70% and was only significant for a pulmonary artery systolic pressure of 45 mm Hg . She was diagnosed with paclitaxel-induced pneumonitis and was discharged home with prednisone 50 mg daily, dapsone, pantoprazole, and 2 L oxygen via nasal cannula.

Two weeks later, she was admitted for coffee-ground emesis and epigastric pain. Her hemoglobin was 5.9 g/dL, for which she was transfused 3 units of packed red blood cells. EGD showed bleeding from diffuse duodenitis, which was treated with argon plasma coagulation. She was also found to have bilateral pulmonary emboli and lower-extremity deep venous thromboses. An inferior vena cava filter was placed, and she was discharged. One week later, she was readmitted with melena, and repeat EGD showed multiple duodenal ulcers with no active bleeding. Colonoscopy was normal. She was continued on prednisone 40 mg daily, as any attempts at tapering the dose resulted in hypotension.

At the time of transfer, she had presented to the outside hospital with worsening nausea and epigastric pain, increasing postprandial abdominal pain, ongoing weight loss, worsening dyspnea on exertion, paroxysmal nocturnal dyspnea, and orthopnea. She denied symptoms of GI bleeding at that time.

Her imaging is consistent with, albeit not specific for, paclitaxel-induced acute pneumonitis. Her persistent dyspnea may be due to worsening of this pneumonitis. Given her GI bleeding and continued epigastric pain, worsening peptic ulcer disease is the likely culprit. The patient has had multiple episodes of bleeding from multiple duodenal ulcers while on proton pump inhibitors, so we should consider reasons for this progression, such as use of nonsteroidal anti-inflammatory drugs (NSAIDs); Helicobacter pylori, herpes simplex virus (HSV), Epstein-Barr virus (EBV), or cytomegalovirus (CMV) infection; Crohn disease, and antral G-cell hyperfunction, as well as lymphoproliferative disease. I would start with histopathology, urease testing, and viral cultures from biopsy specimens at EGD to help eliminate these possibilities for progression of her peptic ulcer disease. Another consideration, albeit rare for progression of peptic ulcer disease and specifically duodenal ulcers, is a gastrinoma-driven acid hypersecretion state, as in the Zollinger-Ellison syndrome. If the above studies do not reveal a cause for her continued peptic ulcer disease, I would check a fasting serum gastrin level and gastric pH to further evaluate this possibility.

Upon arrival on physical exam, her temperature was 35.4° C, heart rate 112 beats per minute, blood pressure 135/96 mm Hg, respiratory rate 34 breaths per minute, and oxygen saturation 97% on room air. She was ill- appearing and in mild respiratory distress with severe muscle wasting. Cervical and supraclavicular lymphadenopathy were not detected. Heart sounds were normal without murmurs. Her jugular venous pressure was approximately 7 cm H2O. She had no lower-extremity edema. On lung exam, diffuse rhonchi were audible bilaterally with no crackles or wheezing. There was no accessory muscle use. No clubbing was present. Her abdomen was soft and mildly tender in the epigastrium with normal bowel sounds.

Her labs revealed a white blood cell (WBC) count of 5,050/μL (neutrophils, 3,600/μL; lymphocytes, 560/μL; eosinophils, 560/μL; hemoglobin, 8.7 g/dL; mean corpuscular volume, 89.3 fL; and platelets, 402,000/μL). Her CD4 count was 235 cells/μL. Her comprehensive metabolic panel demonstrated a sodium of 127 mmol/L; potassium, 4.0 mmol/L; albumin, 2.0 g/dL; calcium, 8.6 mg/dL; creatinine, 0.41 mg/dL; aspartate aminotransferase (AST), 11 U/L; alanine aminotransferase (ALT), 17 U/L; and serum osmolarity, 258 mOs/kg. Her lipase was 30 U/L, and lactate was 0.8 mmol/L. Urine studies showed creatinine 41 mg/dL, osmolality 503 mOs/kg, and sodium 53 mmol/L.

At this point, the patient has been diagnosed with multiple pulmonary emboli and recurrent GI bleeding from duodenal ulcers with chest imaging suggestive of taxane-induced pulmonary toxicity. She now presents with worsening dyspnea and upper-GI symptoms.

Her dyspnea may represent worsening of her taxane-induced lung disease. However, she may have developed a superimposed infection, heart failure, or further pulmonary emboli. A CT pulmonary angiogram to evaluate for infection, recurrent pulmonary emboli, and worsening of her known taxane-induced pulmonary toxicity should be performed. If the CT demonstrates new areas of consolidation suggestive of a superimposed infection, a bronchoscopy would be warranted to obtain specimens for bacterial, fungal, and viral cultures.

On exam, she is in respiratory distress, almost mildly hypothermic and tachycardic with rhonchi on auscultation. This combination of findings could reflect worsening of her pulmonary disease and/or infection on the background of her cachectic state. Her epigastric tenderness, upper-GI symptoms, and anemia have continued to cause concern for persistent duodenal ulcers. Repeat EGD is warranted to evaluate for ulcers, recent bleed stigmata, and/or complications of ulcers, such as stricture formation, obstruction, or perforation. An abdominal CT scan should be done if the EGD does not show worsening of her peptic ulcer disease or obvious complications. The CT scan is more sensitive for certain complications of peptic ulcer disease, such as perforation or proximal small bowel obstruction due to strictures.

Her anemia could represent ongoing blood loss since her last EGD or an inflammatory state due to infection. Also of concern is her use of dapsone, which can lead to hemolysis with or without glucose-6-phosphate dehydrogenase deficiency (G6PD), and this should be excluded.

She has hypotonic hyponatremia and apparent euvolemia with a high urine sodium and osmolality; this suggests syndrome of inappropriate antidiuretic hormone secretion, which may be due to her ongoing pulmonary disease process.

On day 3 of her hospitalization, her abdominal pain became more diffuse and colicky, with two episodes of associated nonbloody bilious vomiting. During the next 48 hours, her abdominal pain and tenderness worsened diffusely but without rigidity or peritoneal signs. She developed mild abdominal distention. An abdominal X-ray showed moderate to large stool burden and increased bowel dilation concerning for small bowel obstruction. A nasogastric tube was placed, with initial improvement of her abdominal pain and distention. On the morning of day six of hospitalization, she had approximately 100 mL of hematemesis. She immediately became hypotensive to the 50s/20s, and roughly 400 mL of sanguineous fluid was suctioned from her nasogastric tube. She was promptly given intravenous (IV) fluids and 2 units of cross-matched packed red blood cells with normalization of her blood pressure and was transferred to the medical intensive care unit (MICU).

Later that day, she had an EGD that showed copious clots and a severely friable duodenum with duodenal narrowing. Duodenal biopsies were taken.

The duodenal ulcers have led to a complication of stricture formation and obstruction resulting in some degree of small bowel obstruction. EGD with biopsies can shed light on the etiology of these ulcers and can specifically exclude viral, fungal, protozoal, or mycobacterial infection; infiltrative diseases (lymphoma, sarcoidosis, amyloidosis); cancer; and inflammatory noninfectious diseases such as vasculitis/connective tissue disorder. Biopsy specimens should undergo light and electron microscopy (for protozoa-like Cryptosporidium); stains for fungal infections such as histoplasmosis, Candida, and Cryptococcus; and stains for mycobacterium. Immunohistochemistry and polymerase chain reaction (PCR) testing can identify CMV, HIV, HSV, and EBV within the duodenal tissue.

She remained on methylprednisolone 30 mg IV because of her known history of pneumonitis and concern for adrenal insufficiency in the setting of acute illness. Over the next 3 days, she remained normotensive with a stable hemoglobin and had no further episodes of hematemesis. She was transferred to the general medical floor.

One day later, she required an additional unit of cross-matched red blood cells because of a hemoglobin decrease to 6.4 g/dL. The next day, she developed acute-onset respiratory distress and was intubated for hypoxemic respiratory failure and readmitted to the MICU.

Her drop in hemoglobin may reflect ongoing bleeding from the duodenum or may be due to diffuse alveolar hemorrhage (DAH) complicating her pneumonitis. The deterioration in the patient’s respiratory status could represent worsening of her taxane pneumonitis (possibly complicated by DAH or acute respiratory distress syndrome), as fatalities have been reported despite steroid treatment. However, as stated earlier, it is prudent to exclude superimposed pulmonary infection or recurrent pulmonary embolism. Broad-spectrum antibiotics should be provided to cover hospital-acquired pneumonia. Transfusion-related acute lung injury (TRALI) as a cause of her respiratory distress is much less likely given onset after 24 hours from transfusion. Symptoms of TRALI almost always develop within 1 to 2 hours of starting a transfusion, with most starting within minutes. The timing of respiratory distress after 24 hours of transfusion also makes transfusion-associated circulatory overload unlikely, as this presents within 6 to 12 hours of a transfusion being completed and generally in patients receiving large transfusion volumes who have underlying cardiac or renal disease.

Her duodenal pathology revealed Strongyloides stercoralis infection (Figure 1), and she was placed on ivermectin. Steroids were continued due to concern for adrenal insufficiency in the setting of critical illness and later septic shock. Bronchoscopy was also performed, and a specimen grew S stercoralis. She developed septic shock from disseminated S stercoralis infection that required vasopressors. Her sanguineous orogastric output increased, and her abdominal distension worsened, concerning for an intra-abdominal bleed or possible duodenal perforation. As attempts were made to stabilize the patient, ultimately, she experienced cardiac arrest and died.

Ruge10050728e_f1.JPG

The patient succumbed to hyperinfection/dissemination of strongyloidiasis. Her risk factors for superinfection included chemotherapy and high-dose steroids, which led to an unchecked autoinfection.

A high index of suspicion remains the most effective overall diagnostic tool for superinfection, which carries a mortality rate of up to 85% even with treatment. Therefore, prevention is the best treatment. Asymptomatic patients with epidemiological exposure or from endemic areas should be evaluated for empiric treatment of S stercoralis prior to initiation of immunosuppressive treatment.

COMMENTARY

Strongyloides stercoralis is a helminth responsible for one of the most overlooked tropical diseases worldwide.1 It is estimated that 370 million individuals are infected with S stercoralis globally, and prevalence in the endemic tropics and subtropics is 10% to 40%.2,3Strongyloides stercoralis infection is characterized by typically nonspecific cutaneous, pulmonary, and GI symptoms, and chronic infection can often be asymptomatic. Once the infection is established, the entirety of the S stercoralis unique life cycle can occur inside the human host, forming a cycle of endogenous autoinfection that can keep the host chronically infected and infectious for decades (Figure 24). While our patient was likely chronically infected for 27 years, cases of patients being infected for up to 75 years have been reported.5 Though mostly identified in societies where fecal contamination of soil and poor sanitation are common, S stercoralis should be considered among populations who have traveled to endemic areas and are immunocompromised.

JHMVol16No7_Ruge10050728e_f2.JPG

Most chronic S stercoralis infections are asymptomatic, but infection can progress to the life-threatening hyperinfection phase, which has a mortality rate of approximately 85%.6 Hyperinfection and disseminated disease occur when there is a rapid proliferation of larvae within the pulmonary and GI tracts, but in the case of disseminated disease, may travel to the liver, brain, and kidneys.7,8 Typically, this is caused by decreased cellular immunity, often due to preexisting conditions such as human T-cell leukemia virus type 1 (HTLV-1) or medications that allow larvae proliferation to go unchecked.6,7 One common class of medications known to increase risk of progression to hyperinfection is corticosteroids, which are thought to both depress immunity and directly increase larvae population growth.6,9 Our patient had been on a prolonged course of steroids for her pulmonary symptoms, with increased doses during her acute illness because of concern for adrenal insufficiency; this likely further contributed to her progression to hyperinfection syndrome. Furthermore, the patient was also immunocompromised from chemotherapy. In addition, she had HIV, which has a controversial association with S stercoralis infection. While previously an AIDS-defining illness, prevalence data indicate a significant co-infection rate between S stercoralis and HIV, but it is unlikely that HIV increases progression to hyperinfection.3

Diagnosing chronic S stercoralis infection is difficult given the lack of a widely accepted gold standard for diagnosis. Traditionally, diagnosis relied on direct visualization of larvae with stool microscopy studies. However, to obtain adequate sensitivity from this method, up to seven serial stool samples must be examined, which is impractical from patient, cost, and efficiency standpoints.10 While other stool-based techniques, such as enriching the stool sample, stool agar plate culture, or PCR-based stool analysis, improve sensitivity, all stool-based studies are limited by intermittent larvae shedding and low worm burden associated with chronic infection.11 Conversely, serologic studies have higher sensitivity, but concerns exist about lower specificity due to potential cross-reactions with other helminths and the persistence of antibodies even after larvae eradication.11,12 Patients with suspected S stercoralis infection and pulmonary infiltrates on imaging may have larvae visible on sputum cultures. A final diagnostic method is direct visualization via biopsy during endoscopy or bronchoscopy, which is typically recommended in cases where suspicion is high yet stool studies have been negative.13 Our patient’s diagnosis was made by duodenal biopsy after her stool study was negative for S stercoralis.

Deciding who to test is difficult given the nonspecific nature of the symptoms but critically important because of the potential for mortality if the disease progresses to hyperinfection. Diagnosis should be suspected in a patient who has spent time in an endemic area and presents with any combination of pulmonary, dermatologic, or GI symptoms. If suspicion for infection is high in a patient being assessed for solid organ transplant or high-dose steroids, prophylactic treatment with ivermectin should be considered. Given the difficulty in diagnosis, some have suggested using eosinophilia as a key diagnostic element, but this has poor predictive value, particularly if the patient is on corticosteroids.7 This patient did not manifest with significant eosinophilia throughout her hospitalization.

This case highlights the difficulties of S stercoralis diagnosis given the nonspecific and variable symptoms, limitations in testing, and potential for remote travel history to endemic regions. It further underscores the need for provider vigilance when starting patients on immunosuppression, even with steroids, given the potential to accelerate chronic infections that were previously buried deep in the mucosa into a lethal hyperinfectious state.

TEACHING POINTS

  • The cycle of autoinfection by S stercoralis allows it to persist for decades even while asymptomatic. This means patients can present with infection years after travel to endemic regions.
  • Because progression to hyperinfection syndrome carries a high mortality rate and is associated with immunosuppressants, particularly corticosteroids, screening patients from or who have spent time in endemic regions for chronic S stercoralis infection is recommended prior to beginning immunosuppression.
  • Diagnosing chronic S stercoralis infection is difficult given the lack of a highly accurate, gold-standard test. Therefore, if suspicion for infection is high yet low-sensitivity stool studies have been negative, direct visualization with a biopsy is a diagnostic option.

Acknowledgment

The authors thank Dr Nicholas Moore, microbiologist at Rush University Medical Center, for his assistance in obtaining and preparing the histology images.

This icon represents the patient’s case. Each paragraph that follows represents the discussant’s thoughts.

A 56-year-old-woman with a history of HIV and locally invasive ductal carcinoma recently treated with mastectomy and adjuvant doxorubicin and cyclophosphamide, now on paclitaxel, was transferred from another hospital with worsening nausea, epigastric pain, and dyspnea. She had been admitted multiple times to both this hospital and another hospital and had extensive workup over the previous 2 months for gastrointestinal (GI) bleeding and progressive dyspnea with orthopnea and paroxysmal nocturnal dyspnea in the setting of a documented 43-lb weight loss.

The patient presents with two clusters of symptoms in the context of chemotherapy regimens following mastectomy for local breast cancer. The first cluster of nausea, epigastric pain, and weight loss indicates upper-GI dysfunction/pathology, such as gastroparesis from paclitaxel-induced autonomic neuropathy given onset of symptoms following paclitaxel chemotherapy. The pretest probability of this is fairly high; however, in light of the GI bleeding, it is prudent to evaluate for other etiologies. The absence of vomiting does not exclude gastroparesis. Other considerations would be peptic ulcer disease or, less likely, the development of a gastric malignancy. A mechanical cause of gastric outlet obstruction from her known cancer is unlikely in this patient with localized breast cancer and recently completed chemotherapy, as breast cancer is unlikely to metastasize to the stomach. Preliminary workup would consist of computed tomography (CT) scan of the abdomen and an esophagogastroduodenoscopy (EGD) to further evaluate these etiologies. The second cluster of dyspnea, paroxysmal nocturnal dyspnea, and orthopnea suggests heart failure, given recent treatment with doxorubicin, and prompts consideration of anthracycline-induced cardiomyopathy. Work-up would include obtaining a transthoracic echocardiogram (TTE) to look for a decrease in her ejection fraction.

Her past medical history was otherwise significant only for the events of the previous few months. Eight months earlier, she was diagnosed with grade 3 triple-negative (estrogen receptor, progesterone receptor, and human epidermal growth factor receptor 2) invasive ductal carcinoma and underwent mastectomy with negative sentinel lymph node biopsy. She completed four cycles of adjuvant doxorubicin and cyclophosphamide and most recently completed cycle three of paclitaxel chemotherapy.

Her HIV disease was controlled with an antiretroviral regimen of dolutegravir/rilpivirine. She had an undetectable viral load for 20 years (CD4, 239 cells/μL 2 weeks prior to transfer).

Her social history included a 1-pack-year smoking history. She denied alcohol or illicit drug use. Family history included pancreatic cancer in her father and endometrial cancer in her paternal grandmother. She was originally from Mexico but moved to Illinois 27 years earlier.

Work-up for her dyspnea was initiated 7 weeks earlier: noncontrast CT of the chest showed extensive diffuse interstitial thickening and ground-glass opacities bilaterally. Bronchoscopy showed no gross abnormalities, and bronchial washings were negative for bacteria, fungi, Pneumocystis jirovecii , acid-fast bacilli, and cancer. She also had a TTE, which showed an ejection fraction of 65% to 70% and was only significant for a pulmonary artery systolic pressure of 45 mm Hg . She was diagnosed with paclitaxel-induced pneumonitis and was discharged home with prednisone 50 mg daily, dapsone, pantoprazole, and 2 L oxygen via nasal cannula.

Two weeks later, she was admitted for coffee-ground emesis and epigastric pain. Her hemoglobin was 5.9 g/dL, for which she was transfused 3 units of packed red blood cells. EGD showed bleeding from diffuse duodenitis, which was treated with argon plasma coagulation. She was also found to have bilateral pulmonary emboli and lower-extremity deep venous thromboses. An inferior vena cava filter was placed, and she was discharged. One week later, she was readmitted with melena, and repeat EGD showed multiple duodenal ulcers with no active bleeding. Colonoscopy was normal. She was continued on prednisone 40 mg daily, as any attempts at tapering the dose resulted in hypotension.

At the time of transfer, she had presented to the outside hospital with worsening nausea and epigastric pain, increasing postprandial abdominal pain, ongoing weight loss, worsening dyspnea on exertion, paroxysmal nocturnal dyspnea, and orthopnea. She denied symptoms of GI bleeding at that time.

Her imaging is consistent with, albeit not specific for, paclitaxel-induced acute pneumonitis. Her persistent dyspnea may be due to worsening of this pneumonitis. Given her GI bleeding and continued epigastric pain, worsening peptic ulcer disease is the likely culprit. The patient has had multiple episodes of bleeding from multiple duodenal ulcers while on proton pump inhibitors, so we should consider reasons for this progression, such as use of nonsteroidal anti-inflammatory drugs (NSAIDs); Helicobacter pylori, herpes simplex virus (HSV), Epstein-Barr virus (EBV), or cytomegalovirus (CMV) infection; Crohn disease, and antral G-cell hyperfunction, as well as lymphoproliferative disease. I would start with histopathology, urease testing, and viral cultures from biopsy specimens at EGD to help eliminate these possibilities for progression of her peptic ulcer disease. Another consideration, albeit rare for progression of peptic ulcer disease and specifically duodenal ulcers, is a gastrinoma-driven acid hypersecretion state, as in the Zollinger-Ellison syndrome. If the above studies do not reveal a cause for her continued peptic ulcer disease, I would check a fasting serum gastrin level and gastric pH to further evaluate this possibility.

Upon arrival on physical exam, her temperature was 35.4° C, heart rate 112 beats per minute, blood pressure 135/96 mm Hg, respiratory rate 34 breaths per minute, and oxygen saturation 97% on room air. She was ill- appearing and in mild respiratory distress with severe muscle wasting. Cervical and supraclavicular lymphadenopathy were not detected. Heart sounds were normal without murmurs. Her jugular venous pressure was approximately 7 cm H2O. She had no lower-extremity edema. On lung exam, diffuse rhonchi were audible bilaterally with no crackles or wheezing. There was no accessory muscle use. No clubbing was present. Her abdomen was soft and mildly tender in the epigastrium with normal bowel sounds.

Her labs revealed a white blood cell (WBC) count of 5,050/μL (neutrophils, 3,600/μL; lymphocytes, 560/μL; eosinophils, 560/μL; hemoglobin, 8.7 g/dL; mean corpuscular volume, 89.3 fL; and platelets, 402,000/μL). Her CD4 count was 235 cells/μL. Her comprehensive metabolic panel demonstrated a sodium of 127 mmol/L; potassium, 4.0 mmol/L; albumin, 2.0 g/dL; calcium, 8.6 mg/dL; creatinine, 0.41 mg/dL; aspartate aminotransferase (AST), 11 U/L; alanine aminotransferase (ALT), 17 U/L; and serum osmolarity, 258 mOs/kg. Her lipase was 30 U/L, and lactate was 0.8 mmol/L. Urine studies showed creatinine 41 mg/dL, osmolality 503 mOs/kg, and sodium 53 mmol/L.

At this point, the patient has been diagnosed with multiple pulmonary emboli and recurrent GI bleeding from duodenal ulcers with chest imaging suggestive of taxane-induced pulmonary toxicity. She now presents with worsening dyspnea and upper-GI symptoms.

Her dyspnea may represent worsening of her taxane-induced lung disease. However, she may have developed a superimposed infection, heart failure, or further pulmonary emboli. A CT pulmonary angiogram to evaluate for infection, recurrent pulmonary emboli, and worsening of her known taxane-induced pulmonary toxicity should be performed. If the CT demonstrates new areas of consolidation suggestive of a superimposed infection, a bronchoscopy would be warranted to obtain specimens for bacterial, fungal, and viral cultures.

On exam, she is in respiratory distress, almost mildly hypothermic and tachycardic with rhonchi on auscultation. This combination of findings could reflect worsening of her pulmonary disease and/or infection on the background of her cachectic state. Her epigastric tenderness, upper-GI symptoms, and anemia have continued to cause concern for persistent duodenal ulcers. Repeat EGD is warranted to evaluate for ulcers, recent bleed stigmata, and/or complications of ulcers, such as stricture formation, obstruction, or perforation. An abdominal CT scan should be done if the EGD does not show worsening of her peptic ulcer disease or obvious complications. The CT scan is more sensitive for certain complications of peptic ulcer disease, such as perforation or proximal small bowel obstruction due to strictures.

Her anemia could represent ongoing blood loss since her last EGD or an inflammatory state due to infection. Also of concern is her use of dapsone, which can lead to hemolysis with or without glucose-6-phosphate dehydrogenase deficiency (G6PD), and this should be excluded.

She has hypotonic hyponatremia and apparent euvolemia with a high urine sodium and osmolality; this suggests syndrome of inappropriate antidiuretic hormone secretion, which may be due to her ongoing pulmonary disease process.

On day 3 of her hospitalization, her abdominal pain became more diffuse and colicky, with two episodes of associated nonbloody bilious vomiting. During the next 48 hours, her abdominal pain and tenderness worsened diffusely but without rigidity or peritoneal signs. She developed mild abdominal distention. An abdominal X-ray showed moderate to large stool burden and increased bowel dilation concerning for small bowel obstruction. A nasogastric tube was placed, with initial improvement of her abdominal pain and distention. On the morning of day six of hospitalization, she had approximately 100 mL of hematemesis. She immediately became hypotensive to the 50s/20s, and roughly 400 mL of sanguineous fluid was suctioned from her nasogastric tube. She was promptly given intravenous (IV) fluids and 2 units of cross-matched packed red blood cells with normalization of her blood pressure and was transferred to the medical intensive care unit (MICU).

Later that day, she had an EGD that showed copious clots and a severely friable duodenum with duodenal narrowing. Duodenal biopsies were taken.

The duodenal ulcers have led to a complication of stricture formation and obstruction resulting in some degree of small bowel obstruction. EGD with biopsies can shed light on the etiology of these ulcers and can specifically exclude viral, fungal, protozoal, or mycobacterial infection; infiltrative diseases (lymphoma, sarcoidosis, amyloidosis); cancer; and inflammatory noninfectious diseases such as vasculitis/connective tissue disorder. Biopsy specimens should undergo light and electron microscopy (for protozoa-like Cryptosporidium); stains for fungal infections such as histoplasmosis, Candida, and Cryptococcus; and stains for mycobacterium. Immunohistochemistry and polymerase chain reaction (PCR) testing can identify CMV, HIV, HSV, and EBV within the duodenal tissue.

She remained on methylprednisolone 30 mg IV because of her known history of pneumonitis and concern for adrenal insufficiency in the setting of acute illness. Over the next 3 days, she remained normotensive with a stable hemoglobin and had no further episodes of hematemesis. She was transferred to the general medical floor.

One day later, she required an additional unit of cross-matched red blood cells because of a hemoglobin decrease to 6.4 g/dL. The next day, she developed acute-onset respiratory distress and was intubated for hypoxemic respiratory failure and readmitted to the MICU.

Her drop in hemoglobin may reflect ongoing bleeding from the duodenum or may be due to diffuse alveolar hemorrhage (DAH) complicating her pneumonitis. The deterioration in the patient’s respiratory status could represent worsening of her taxane pneumonitis (possibly complicated by DAH or acute respiratory distress syndrome), as fatalities have been reported despite steroid treatment. However, as stated earlier, it is prudent to exclude superimposed pulmonary infection or recurrent pulmonary embolism. Broad-spectrum antibiotics should be provided to cover hospital-acquired pneumonia. Transfusion-related acute lung injury (TRALI) as a cause of her respiratory distress is much less likely given onset after 24 hours from transfusion. Symptoms of TRALI almost always develop within 1 to 2 hours of starting a transfusion, with most starting within minutes. The timing of respiratory distress after 24 hours of transfusion also makes transfusion-associated circulatory overload unlikely, as this presents within 6 to 12 hours of a transfusion being completed and generally in patients receiving large transfusion volumes who have underlying cardiac or renal disease.

Her duodenal pathology revealed Strongyloides stercoralis infection (Figure 1), and she was placed on ivermectin. Steroids were continued due to concern for adrenal insufficiency in the setting of critical illness and later septic shock. Bronchoscopy was also performed, and a specimen grew S stercoralis. She developed septic shock from disseminated S stercoralis infection that required vasopressors. Her sanguineous orogastric output increased, and her abdominal distension worsened, concerning for an intra-abdominal bleed or possible duodenal perforation. As attempts were made to stabilize the patient, ultimately, she experienced cardiac arrest and died.

Ruge10050728e_f1.JPG

The patient succumbed to hyperinfection/dissemination of strongyloidiasis. Her risk factors for superinfection included chemotherapy and high-dose steroids, which led to an unchecked autoinfection.

A high index of suspicion remains the most effective overall diagnostic tool for superinfection, which carries a mortality rate of up to 85% even with treatment. Therefore, prevention is the best treatment. Asymptomatic patients with epidemiological exposure or from endemic areas should be evaluated for empiric treatment of S stercoralis prior to initiation of immunosuppressive treatment.

COMMENTARY

Strongyloides stercoralis is a helminth responsible for one of the most overlooked tropical diseases worldwide.1 It is estimated that 370 million individuals are infected with S stercoralis globally, and prevalence in the endemic tropics and subtropics is 10% to 40%.2,3Strongyloides stercoralis infection is characterized by typically nonspecific cutaneous, pulmonary, and GI symptoms, and chronic infection can often be asymptomatic. Once the infection is established, the entirety of the S stercoralis unique life cycle can occur inside the human host, forming a cycle of endogenous autoinfection that can keep the host chronically infected and infectious for decades (Figure 24). While our patient was likely chronically infected for 27 years, cases of patients being infected for up to 75 years have been reported.5 Though mostly identified in societies where fecal contamination of soil and poor sanitation are common, S stercoralis should be considered among populations who have traveled to endemic areas and are immunocompromised.

JHMVol16No7_Ruge10050728e_f2.JPG

Most chronic S stercoralis infections are asymptomatic, but infection can progress to the life-threatening hyperinfection phase, which has a mortality rate of approximately 85%.6 Hyperinfection and disseminated disease occur when there is a rapid proliferation of larvae within the pulmonary and GI tracts, but in the case of disseminated disease, may travel to the liver, brain, and kidneys.7,8 Typically, this is caused by decreased cellular immunity, often due to preexisting conditions such as human T-cell leukemia virus type 1 (HTLV-1) or medications that allow larvae proliferation to go unchecked.6,7 One common class of medications known to increase risk of progression to hyperinfection is corticosteroids, which are thought to both depress immunity and directly increase larvae population growth.6,9 Our patient had been on a prolonged course of steroids for her pulmonary symptoms, with increased doses during her acute illness because of concern for adrenal insufficiency; this likely further contributed to her progression to hyperinfection syndrome. Furthermore, the patient was also immunocompromised from chemotherapy. In addition, she had HIV, which has a controversial association with S stercoralis infection. While previously an AIDS-defining illness, prevalence data indicate a significant co-infection rate between S stercoralis and HIV, but it is unlikely that HIV increases progression to hyperinfection.3

Diagnosing chronic S stercoralis infection is difficult given the lack of a widely accepted gold standard for diagnosis. Traditionally, diagnosis relied on direct visualization of larvae with stool microscopy studies. However, to obtain adequate sensitivity from this method, up to seven serial stool samples must be examined, which is impractical from patient, cost, and efficiency standpoints.10 While other stool-based techniques, such as enriching the stool sample, stool agar plate culture, or PCR-based stool analysis, improve sensitivity, all stool-based studies are limited by intermittent larvae shedding and low worm burden associated with chronic infection.11 Conversely, serologic studies have higher sensitivity, but concerns exist about lower specificity due to potential cross-reactions with other helminths and the persistence of antibodies even after larvae eradication.11,12 Patients with suspected S stercoralis infection and pulmonary infiltrates on imaging may have larvae visible on sputum cultures. A final diagnostic method is direct visualization via biopsy during endoscopy or bronchoscopy, which is typically recommended in cases where suspicion is high yet stool studies have been negative.13 Our patient’s diagnosis was made by duodenal biopsy after her stool study was negative for S stercoralis.

Deciding who to test is difficult given the nonspecific nature of the symptoms but critically important because of the potential for mortality if the disease progresses to hyperinfection. Diagnosis should be suspected in a patient who has spent time in an endemic area and presents with any combination of pulmonary, dermatologic, or GI symptoms. If suspicion for infection is high in a patient being assessed for solid organ transplant or high-dose steroids, prophylactic treatment with ivermectin should be considered. Given the difficulty in diagnosis, some have suggested using eosinophilia as a key diagnostic element, but this has poor predictive value, particularly if the patient is on corticosteroids.7 This patient did not manifest with significant eosinophilia throughout her hospitalization.

This case highlights the difficulties of S stercoralis diagnosis given the nonspecific and variable symptoms, limitations in testing, and potential for remote travel history to endemic regions. It further underscores the need for provider vigilance when starting patients on immunosuppression, even with steroids, given the potential to accelerate chronic infections that were previously buried deep in the mucosa into a lethal hyperinfectious state.

TEACHING POINTS

  • The cycle of autoinfection by S stercoralis allows it to persist for decades even while asymptomatic. This means patients can present with infection years after travel to endemic regions.
  • Because progression to hyperinfection syndrome carries a high mortality rate and is associated with immunosuppressants, particularly corticosteroids, screening patients from or who have spent time in endemic regions for chronic S stercoralis infection is recommended prior to beginning immunosuppression.
  • Diagnosing chronic S stercoralis infection is difficult given the lack of a highly accurate, gold-standard test. Therefore, if suspicion for infection is high yet low-sensitivity stool studies have been negative, direct visualization with a biopsy is a diagnostic option.

Acknowledgment

The authors thank Dr Nicholas Moore, microbiologist at Rush University Medical Center, for his assistance in obtaining and preparing the histology images.

References

1. Olsen A, van Lieshout L, Marti H, et al. Strongyloidiasis--the most neglected of the neglected tropical diseases? Trans R Soc Trop Med Hyg. 2009;103(10):967-972. https://doi.org/10.1016/j.trstmh.2009.02.013
2. Bisoffi Z, Buonfrate D, Montresor A, et al. Strongyloides stercoralis: a plea for action. PLoS Negl Trop Dis. 2013;7(5):e2214. https://doi.org/10.1371/journal.pntd.0002214
3. Schär F, Trostdorf U, Giardina F, et al. Strongyloides stercoralis: global distribution and risk factors. PLoS Negl Trop Dis. 2013;7(7):e2288. https://doi.org/10.1371/journal.pntd.0002288
4. Silva AJ, Moser M. Life cycle of Strongyloides stercoralis. Accessed June 5, 2020. https://www.cdc.gov/parasites/strongyloides/biology.html
5. Prendki V, Fenaux P, Durand R, Thellier M, Bouchaud O. Strongyloidiasis in man 75 years after initial exposure. Emerg Infect Dis. 2011;17(5):931-932. https://doi.org/10.3201/eid1705.100490
6. Nutman TB. Human infection with Strongyloides stercoralis and other related Strongyloides species. Parasitology. 2017;144(3):263-273. https://doi.org/10.1017/S0031182016000834
7. Naidu P, Yanow SK, Kowalewska-Grochowska KT. Eosinophilia: a poor predictor of Strongyloides infection in refugees. Can J Infect Dis Med Microbiol. 2013;24(2):93-96. https://doi.org/10.1155/2013/290814
8. Kassalik M, Mönkemüller K. Strongyloides stercoralis hyperinfection syndrome and disseminated disease. Gastroenterol Hepatol (N Y). 2011;7(11):766-768.
9. Genta RM. Dysregulation of strongyloidiasis: a new hypothesis. Clin Microbiol Rev. 1992;5(4):345-355. https://doi.org/10.1128/cmr.5.4.345
10. Siddiqui AA, Berk SL. Diagnosis of Strongyloides stercoralis infection. Clin Infect Dis. 2001;33(7):1040-1047. https://doi.org/10.1086/322707
11. Buonfrate D, Requena-Mendez A, Angheben A, et al. Accuracy of molecular biology techniques for the diagnosis of Strongyloides stercoralis infection—a systematic review and meta-analysis. PLoS Negl Trop Dis. 2018;12(2):e0006229. dohttps://doi.org/10.1371/journal.pntd.0006229
12. Arifin N, Hanafiah KM, Ahmad H, Noordin R. Serodiagnosis and early detection of Strongyloides stercoralis infection. J Microbiol Immunol Infect. 2019;52(3):371-378. https://doi.org/10.1016/j.jmii.2018.10.001
13. Lowe RC, Chu JN, Pierce TT, Weil AA, Branda JA. Case 3-2020: a 44-year-old man with weight loss, diarrhea, and abdominal pain. N Engl J Med. 2020;382(4):365-374. https://doi.org/10.1056/NEJMcpc1913473

References

1. Olsen A, van Lieshout L, Marti H, et al. Strongyloidiasis--the most neglected of the neglected tropical diseases? Trans R Soc Trop Med Hyg. 2009;103(10):967-972. https://doi.org/10.1016/j.trstmh.2009.02.013
2. Bisoffi Z, Buonfrate D, Montresor A, et al. Strongyloides stercoralis: a plea for action. PLoS Negl Trop Dis. 2013;7(5):e2214. https://doi.org/10.1371/journal.pntd.0002214
3. Schär F, Trostdorf U, Giardina F, et al. Strongyloides stercoralis: global distribution and risk factors. PLoS Negl Trop Dis. 2013;7(7):e2288. https://doi.org/10.1371/journal.pntd.0002288
4. Silva AJ, Moser M. Life cycle of Strongyloides stercoralis. Accessed June 5, 2020. https://www.cdc.gov/parasites/strongyloides/biology.html
5. Prendki V, Fenaux P, Durand R, Thellier M, Bouchaud O. Strongyloidiasis in man 75 years after initial exposure. Emerg Infect Dis. 2011;17(5):931-932. https://doi.org/10.3201/eid1705.100490
6. Nutman TB. Human infection with Strongyloides stercoralis and other related Strongyloides species. Parasitology. 2017;144(3):263-273. https://doi.org/10.1017/S0031182016000834
7. Naidu P, Yanow SK, Kowalewska-Grochowska KT. Eosinophilia: a poor predictor of Strongyloides infection in refugees. Can J Infect Dis Med Microbiol. 2013;24(2):93-96. https://doi.org/10.1155/2013/290814
8. Kassalik M, Mönkemüller K. Strongyloides stercoralis hyperinfection syndrome and disseminated disease. Gastroenterol Hepatol (N Y). 2011;7(11):766-768.
9. Genta RM. Dysregulation of strongyloidiasis: a new hypothesis. Clin Microbiol Rev. 1992;5(4):345-355. https://doi.org/10.1128/cmr.5.4.345
10. Siddiqui AA, Berk SL. Diagnosis of Strongyloides stercoralis infection. Clin Infect Dis. 2001;33(7):1040-1047. https://doi.org/10.1086/322707
11. Buonfrate D, Requena-Mendez A, Angheben A, et al. Accuracy of molecular biology techniques for the diagnosis of Strongyloides stercoralis infection—a systematic review and meta-analysis. PLoS Negl Trop Dis. 2018;12(2):e0006229. dohttps://doi.org/10.1371/journal.pntd.0006229
12. Arifin N, Hanafiah KM, Ahmad H, Noordin R. Serodiagnosis and early detection of Strongyloides stercoralis infection. J Microbiol Immunol Infect. 2019;52(3):371-378. https://doi.org/10.1016/j.jmii.2018.10.001
13. Lowe RC, Chu JN, Pierce TT, Weil AA, Branda JA. Case 3-2020: a 44-year-old man with weight loss, diarrhea, and abdominal pain. N Engl J Med. 2020;382(4):365-374. https://doi.org/10.1056/NEJMcpc1913473

Issue
Journal of Hospital Medicine 16(12)
Issue
Journal of Hospital Medicine 16(12)
Page Number
757-762. Published Online First July 28, 2021
Page Number
757-762. Published Online First July 28, 2021
Publications
Publications
Topics
Article Type
Display Headline
Buried Deep
Display Headline
Buried Deep
Sections
Article Source

© 2021 Society of Hospital Medicine

Disallow All Ads
Correspondence Location
Max Ruge, MD; Email: rugem15@gmail.com; Telephone: 703-346-9870.
Content Gating
Gated (full article locked unless allowed per User)
Alternative CME
Disqus Comments
Default
Use ProPublica
Hide sidebar & use full width
render the right sidebar.
Conference Recap Checkbox
Not Conference Recap
Clinical Edge
Display the Slideshow in this Article
Gating Strategy
First Page Free
Medscape Article
Display survey writer
Reuters content
Disable Inline Native ads
WebMD Article
Article PDF Media
Image
Teambase ID
18001F2E.SIG
Disable zoom
Off

Out of Sight, Not Out of Mind

Article Type
Changed
Thu, 07/29/2021 - 11:43
Display Headline
Out of Sight, Not Out of Mind

A 73-year-old man presented to clinic with 6 weeks of headache. He occasionally experienced generalized headaches throughout his life that resolved with naproxen. His new headache was characterized by a progressively worsening sensation of left-eye pressure with radiation to the left temple. Over the previous week, he had intermittent diplopia, left ptosis, and left lacrimation. He denied head trauma, fever, vision loss, photophobia, dysphagia, dysarthria, nausea, vomiting, or jaw claudication.

Primary headaches include tension type, migraine, and trigeminal autonomic cephalalgias (eg, cluster headache). A new headache in an older patient, particularly if protracted and progressive, prioritizes consideration of a secondary headache, which may reflect pathology within the brain parenchyma (eg, intracranial mass), blood vessels (eg, giant cell arteritis), meninges (eg, meningitis), or ventricles (eg, intraventricular cyst). Eye pain may arise from ocular and extraocular disease. Corneal abrasions, infectious keratitis, scleritis, uveitis, or acute angle-closure glaucoma are painful, although the latter is less likely given the prolonged duration of symptoms. Thyroid eye disease or other infiltrative disorders of the orbit can also cause eye discomfort.

Ptosis commonly results from degeneration of the levator aponeurosis. Other causes include third cranial nerve palsy and myasthenia gravis. Interruption of sympathetic innervation of the eyelid by lesions in the brain stem, spinal cord, lung (eg, Pancoast tumor), or cavernous sinus also can result in ptosis.

Whether the patient has monocular or binocular diplopia is uncertain. Monocular diplopia persists with only one eye open and can arise from uncorrected refractive error, corneal irregularities, lenticular opacities, or unilateral macular disease. Binocular diplopia develops from ocular misalignment due to neuromuscular weakness, extraocular muscle entrapment, or an orbital mass displacing the globe. An orbital mass would also explain the unilateral headache and unilateral ptosis.

His medical history included coronary artery disease, seronegative rheumatoid arthritis, osteoporosis, benign prostatic hypertrophy, and ureteral strictures from chronic nephrolithiasis. Following a cholecystectomy for gallstone pancreatitis 13 years earlier, he was hospitalized five more times for pancreatitis. The last episode was 6 years prior to this presentation. At that time, magnetic resonance cholangiopancreatography (MRCP) did not reveal pancreatic divisum, annular pancreas, biliary strictures, or a pancreatic mass. Esophagogastroduodenoscopy peformed during the same hospitalization showed mild gastritis. His recurrent pancreatitis was deemed idiopathic.

His medications were folic acid, cholecalciferol, lisinopril, metoprolol, omeprazole, simvastatin, aspirin, and weekly methotrexate. His sister had breast and ovarian cancer, and his brother had gastric cancer. He had two subcentimeter tubular adenomas removed during a screening colonoscopy 3 years prior. He had a 30 pack-year smoking history and quit 28 years earlier. He did not use alcohol or drugs. He was a retired chemical plant worker.

Choledocholithiasis (as discrete stones or biliary sludge) can trigger pancreatitis despite a cholecystectomy, but the recurrent episodes and negative MRCP should prompt consideration of other causes, such as alcohol. Hypercalcemia, hypertriglyceridemia, and medications are infrequent causes of pancreatic inflammation. IgG4-related disease (IgG4-RD) causes autoimmune pancreatitis and can infiltrate the eyelids, lacrimal glands, extraocular muscles, or orbital connective tissue. Malignancy of the pancreas or ampulla can trigger pancreatitis by causing pancreatic duct obstruction but would not go undetected for 13 years.

The patient was evaluated by an ophthalmologist and a neurologist. His heart rate was 52 beats per minute and blood pressure, 174/70 mm Hg; other vital signs were normal. He had conjunctival chemosis, ptosis, and nonpulsatile proptosis of the left eye with tenderness and increased resistance to retropulsion compared to the right eye (Figure 1). Visual acuity was 20/25 for the right eye and hand motions only in the left eye. The pupils were reactive and symmetric without afferent pupillary defect. There was no optic nerve swelling or pallor. Abduction, adduction, and elevation of the left eye were restricted and associated with diplopia. Movement of the right eye was unrestricted. There was no other facial asymmetry. Facial sensation was normal. Corneal reflexes were intact. Shoulder shrug strength was equal and symmetric. Tongue protrusion was midline. Olfaction and hearing were not assessed. Strength, sensation, and deep tendon reflexes were normal in all extremities. The plantar response was flexor bilaterally.

jhm01608502_f1.JPG

Unilateral ptosis, chemosis, proptosis, ophthalmoplegia, eye tenderness, and visual loss collectively point to a space-occupying orbital disease. Orbital masses are caused by cancers, infections such as mucormycosis (usually in an immunocompromised host), and inflammatory disorders such as thyroid orbitopathy, sarcoidosis, IgG4-related orbitopathy, granulomatosis with polyangiitis, and orbital pseudotumor (idiopathic inflammation of the orbit). Chemosis reflects edema of the conjunctiva, which can arise from direct conjunctival injury (eg, allergy, infection, or trauma), interruption of the venous drainage of the conjunctiva by vascular disorders (eg, cavernous sinus thrombosis or carotid-cavernous fistula), or space-occupying diseases of the orbit. Monocular visual loss arises from a prechiasmal lesion, and acute monocular visual loss is more commonly caused by posterior ocular pathology (eg, retina or optic nerve) than anterior disease (eg, keratitis). Visual loss in the presence of an orbital process suggests a compressive or infiltrative disease of the optic nerve.

Complete blood count, comprehensive metabolic panel, erythrocyte sedimentation rate, C-reactive protein, and thyroid function tests were normal. Interferon-gamma release assay, HIV antibody, rapid plasma reagin, Lyme antibody, antinuclear antibody, and antineutrophil cytoplasmic antibody (ANCA) tests were negative. A noncontrast computed tomography (CT) scan of the head revealed thickening of the left inferior rectus muscle. Orbital magnetic resonance imaging (MRI) with gadolinium and fluid-attenuated inversion recovery imaging demonstrated a T2 hyperintense, heterogeneous 1.4-cm mass in the left inferior rectus muscle (Figure 2). There was no carotid-cavernous fistula, brain mass, or meningeal enhancement.

jhm01608502_f2.JPG

An isolated mass in one ocular muscle raises the probability of a cancer. The most common malignant orbital tumor is B-cell lymphoma. Metastatic cancer to the eye is rare; breast, prostate, and lung cancer account for the majority of cases. The family history of breast and ovarian cancer raises the possibility of a BRCA mutation, which is also associated with gastric, pancreatic, and prostate malignancies. Granulomatosis with polyangiitis may be ANCA negative in localized sino-orbital disease. Biopsy of the orbital mass is the next step.

The patient underwent transconjunctival orbitotomy with excision of the left inferior rectus mass. Two days later, he presented to the emergency department with acute onset epigastric pain, nausea, and vomiting. A comprehensive review of systems, which had not been performed until this visit, revealed an unintentional 20-lb weight loss over the previous 3 months. He had a progressive ache in the left anterior groin that was dull, tender, nonradiating, and worse with weight bearing. He denied melena or hematochezia.

His temperature was 37 °C; heart rate, 98 beats per minute; and blood pressure, 128/63 mm Hg. He had midepigastric tenderness and point tenderness over the anterior iliac spine. White blood cell count was 12,600/μL; hemo globin, 14.5 g/dL; and platelet count, 158,000/μL. Serum lipase was 7,108 U/L. Serum creatinine, calcium, and triglyceride levels were normal. Alkaline phosphatase was 117 U/L (normal, 34-104 U/L); total bilirubin, 1.1 mg/dL; alanine aminotransferase (ALT), 119 U/L (normal, 7-52 U/L); and aspartate aminotransferase (AST), 236 U/L (normal, 13-39 U/L). Troponin I was undetectable, and an electrocardiogram demonstrated sinus tachycardia. Urinalysis was normal.

Concomitant pancreatitis and hepatitis with an elevated AST-to-ALT ratio should prompt evaluation of recurrent choledocholithiasis and a repeat inquiry about alcohol use. His medications should be reviewed for an association with pancreatitis. Anterior groin discomfort usually reflects osteoarthritis of the hip joint, inguinal hernia, or inguinal lymphadenopathy. Groin pain may be referred from spinal nerve root compression, aortoiliac occlusion, or nephrolithiasis. Weight loss in the presence of an inferior rectus mass suggests one of the aforementioned systemic diseases with orbital manifestations. Pancreatitis and groin discomfort may be important clues, but the chronicity of the recurrent pancreatitis and the high prevalence of hip osteoarthritis make it equally likely that they are unrelated to the eye disease.

CT scan of the abdomen and pelvis with contrast showed peripancreatic edema with fat stranding but no pancreatic or hepatobiliary mass. The common bile duct was normal. A 2.2×1.3-cm mass in the right posterior subphrenic space, a lytic lesion in the left anterior inferior iliac spine, and right nonobstructive nephrolithiasis were identified. CT scan of the chest with contrast showed multiple subpleural nodules and innumerable parenchymal nodules. Subcentimeter hilar, mediastinal, and prevascular lymphadenopathy were present, as well as multiple sclerotic lesions in the right fourth and sixth ribs. Prostate-specific antigen was 0.7 ng/mL (normal, ≤ 4.0 ng/mL). Cancer antigen 19-9 level was 5.5 U/mL (normal, < 37.0 U/mL), and carcinoembryonic antigen (CEA) was 100.1 ng/mL (normal, 0-3 U/mL).

Widespread pulmonary nodules, diffuse lymphadenopathy, and bony lesions raise concern for a metastatic malignancy. There is no evidence of a primary carcinoma. The lack of hepatic involvement reduces the likelihood of a gastrointestinal tumor, although a rectal cancer, which may drain directly into the inferior vena cava and bypass the portal circulation, could present as lung metastases on CT imaging. Lymphoma is plausible given the diffuse lymphadenopathy and orbital mass. Sarcoidosis and histiocytic disorders (eg, Langerhans cell histiocytosis) also cause orbital disease, pulmonary nodules, lymphadenopathy, and bone lesions, although a subphrenic mass would be atypical for both disorders; furthermore, the majority of patients with adult Langerhans cell histiocytosis smoke cigarettes. The elevated CEA makes a metastatic solid tumor more likely than lymphoma but does not specify the location of the primary tumor.

Pathology of the inferior rectus muscle mass showed well-differentiated adenocarcinoma (Figure 3A and 3B). A CT-guided biopsy of the left anterior inferior iliac spine revealed well-differentiated adenocarcinoma (Figure 3C). Adenocarcinoma of unknown primary wasdiagnosed.

Subsequent immunohistochemical (IHC) staining was positive for cytokeratin 7 (CK7) and mucicarmine (Figure 3D and 3E) and negative for cytokeratin 20 (CK20) and thyroid transcription factor 1 (TTF1). This IHC profile suggested pancreatic or upper gastrointestinal tract lineage. Positron emission tomography–CT (PET-CT) scan was aborted because of dyspnea and chest pressure following contrast administration. He declined further imaging or endoscopy. He received palliative radiation and three cycles of paclitaxel and gemcitabine for cancer of unknown primary (CUP). Two months later, he developed bilateral upper-arm weakness due to C7 and T2 cord compression from vertebral and epidural metastases; his symptoms progressed despite salvage chemotherapy. He was transitioned to comfort care and died at home 9 months after diagnosis.

jhm01608502_f3.JPG

DISCUSSION

This patient’s new headache and ocular abnormalities led to the discovery of an inferior rectus muscle mass. Initially unrecognized unintentional weight loss and hip pain recast a localized orbital syndrome as a systemic disease with pancreatic, ocular, pulmonary, lymph node, and skeletal pathology. Biopsies of the orbital rectus muscle and iliac bone demonstrated metastatic adenocarcinoma. Imaging studies did not identify a primary cancer, but IHC analysis suggested carcinoma of upper gastrointestinal or pancreatic origin.

Acute and chronic pancreatitis are both associated with pancreatic cancer.1 Chronic pancreatitis is associated with an increasing cumulative risk of pancreatic cancer; a potential mechanism is chronic inflammation with malignant transformation.2,3 There is also a 20-fold increased risk of pancreatic cancer in the first 2 years following an episode of acute pancreatitis,4 which may develop from malignant pancreatic duct obstruction. Although the post–acute pancreatitis risk of pancreatic cancer attenuates over time, a two-fold increased risk of pancreatic cancer remains after 10 years,4 which suggests that acute pancreatitis (particularly when idiopathic) either contributes to or shares pathogenesis with pancreatic adenocarcinoma. In elderly patients without gallstones or alcohol use, an abdominal CT scan or MRI shortly after resolution of the acute pancreatitis may be considered to assess for an underlying pancreatic tumor.5

CUP is a histologically defined malignancy without a known primary anatomic site despite an extensive evaluation. CUP accounts for up to 10% of all cancer diagnoses.6 CUP is ascribed to a primary cancer that remains too small to be detected or spontaneous regression of the primary cancer.7 Approximately 70% of autopsies of patients with CUP identify the primary tumor, which most commonly originates in the lung, gastrointestinal tract, breast, or pancreas.8

When a metastatic focus of cancer is found but the initial diagnostic evaluation (including CT scan of the chest, abdomen, and pelvis) fails to locate a primary cancer, the next step in searching for the tissue of origin is an IHC analysis of the tumor specimen. IHC analysis is a multistep staining process that can identify major categories of cancer, including carcinoma (adenocarcinoma, squamous cell carcinoma, and neuroendocrine carcinoma) and poorly or undifferentiated neoplasms (including carcinoma, lymphoma, sarcoma, or melanoma). Eighty-five percent of CUP cases are adenocarcinoma, 10% are squamous cell carcinoma, and the remaining 5% are undifferentiated neoplasms.9

There are no consensus guidelines for imaging in patients with CUP who have already undergone a CT scan of the chest, abdomen, and pelvis. Mammography is indicated in women with metastatic adenocarcinoma or axillary lymphadenopathy.7 MRI of the breast is obtained when mammography is nondiagnostic and the suspicion for breast cancer is high. Small clinical studies and meta-analyses support the use of PET-CT scans,7 although one study found that a PET-CT scan was not superior to CT imaging in identifying the primary tumor site in CUP.10 Endoscopy of the upper airway or gastrointestinal tract is rarely diagnostic in the absence of referable symptoms or a suggestive IHC profile (eg, CK7−, CK20+ suggestive of colon cancer).6

Molecular cancer classification has emerged as a useful diagnostic technique in CUP. Cancer cells retain gene expression patterns based on cellular origin, and a tumor’s profile can be compared with a reference database of known cancers, aiding in the identification of the primary tumor type. Molecular cancer classifier assays that use gene expression profiling can accurately determine a primary site11 and have been shown to be concordant with IHC testing.12 Molecular cancer classification is distinct from genetic assays that identify mutations for which there are approved therapies. Serum tumor markers are generally not useful in establishing the primary tumor and should be considered based on the clinical presentation (eg, prostate-specific antigen testing in a man with adenocarcinoma of unknown primary and osteoblastic metastases).

CUP is classified as favorable or unfavorable based on the IHC, pattern of spread, and serum markers in certain cases.6 Approximately 20% of CUP patients can be categorized into favorable subsets, such as adenocarcinoma in a single axillary lymph node in a female patient suggestive of a breast primary cancer, or squamous cell carcinoma in a cervical lymph node suggestive of a head or neck primary cancer.7 The remaining 80% of cases are categorized as unfavorable CUP and often have multiple metastases. Our patient’s pattern of spread and limited response to chemotherapy is characteristic of the unfavorable subset of CUP. The median survival of this group is 9 months, and only 25% of patients survive longer than 1 year.13

Biomarker-driven treatment of specific molecular targets independent of the tissue of origin (tissue-agnostic therapy) has shown promising results in the treatment of skin, lung, thyroid, colorectal, and gastric cancers.14 Pembrolizumab was the first drug approved by the US Food and Drug Administration based on a tumor’s biomarker without regard to its primary location. Data to support this approach for treating CUP are evolving and offer hope for patients with specific molecular targets.

Following the focused neuro-ophthalmologic evaluations, with focused examination and imaging, the hospitalist’s review of systems at the time of the final admission for pancreatitis set in motion an evaluation that led to a diagnosis of metastatic cancer. The risk factor of recurrent pancreatitis and IHC results suggested that pancreatic adenocarcinoma was the most likely primary tumor. As the focus of cancer treatment shifts away from the tissue of origin and toward molecular and genetic profiles, the search for the primary site may decrease in importance. In the future, even when we do not know the cancer’s origin, we may still know precisely what to do. But for now, as in this patient, our treatments continue to be based on a tumor that is out of sight, but not out of mind.

KEY TEACHING POINTS

  • Acute and chronic pancreatitis are associated with an increased risk of pancreatic adenocarcinoma.
  • CUP is a cancer in which diagnostic testing does not identify a primary tumor site. Immunohistochemistry and molecular analysis, imaging, and endoscopy are utilized selectively to identify a primary tumor type.
  • Treatment of CUP currently depends on the suspected tissue of origin and pattern of spread.
  • Tissue-agnostic therapy could allow for treatment for CUP patients independent of the tissue of origin.

Acknowledgments

We thank Andrew Mick, OD, for his review of an earlier version of this manuscript and Peter Phillips, MD, for his interpretation of the pathologic images.

References

1. Sadr-Azodi O, Oskarsson V, Discacciati A, Videhult P, Askling J, Ekbom A. Pancreatic cancer following acute pancreatitis: a population-based matched cohort study. Am J Gastroenterol. 2018;113(111):1711-1719. https://doi.org/10.1038/s41395-018-0255-9
2. Duell EJ, Lucenteforte E, Olson SH, et al. Pancreatitis and pancreatic cancer risk: a pooled analysis in the International Pancreatic Cancer Case-Control Consortium (PanC4). Ann Oncol. 2012;23(11):2964-2970. https://doi.org/10.1093/annonc/mds140
3. Ekbom A, McLaughlin JK, Nyren O. Pancreatitis and the risk of pancreatic cancer. N Engl J Med. 1993;329(20):1502-1503. https://doi.org/10.1056/NEJM199311113292016
4. Kirkegard J, Cronin-Fenton D, Heide-Jorgensen U, Mortensen FV. Acute pancreatitis and pancreatic cancer risk: a nationwide matched-cohort study in Denmark. Gastroenterology. 2018;154(156):1729-1736. https://doi.org/10.1053/j.gastro.2018.02.011
5. Frampas E, Morla O, Regenet N, Eugene T, Dupas B, Meurette G. A solid pancreatic mass: tumour or inflammation? Diagn Interv Imaging. 2013;94(7-8):741-755. https://doi.org/10.1016/j.diii.2013.03.013
6. Varadhachary GR, Raber MN. Cancer of unknown primary site. N Engl J Med. 2014;371(8):757-765. https://doi.org/10.1056/NEJMra1303917
7. Bochtler T, Löffler H, Krämer A. Diagnosis and management of metastatic neoplasms with unknown primary. Semin Diagn Pathol. 2017. 2018;35(3):199-206. https://doi.org//10.1053/j.semdp.2017.11.013
8. Pentheroudakis G, Golfinopoulos V, Pavlidis N. Switching benchmarks in cancer of unknown primary: from autopsy to microarray. Eur J Cancer. 2007;43(14):2026-2036. https://doi.org/10.1016/j.ejca.2007.06.023
9. Pavlidis N, Fizazi K. Carcinoma of unknown primary (CUP). Crit Rev Oncol Hematol. 2009;69(3):271-278. https://doi.org/10.1016/j.critrevonc.2008.09.005
10. Moller AK, Loft A, Berthelsen AK, et al. A prospective comparison of 18F-FDG PET/CT and CT as diagnostic tools to identify the primary tumor site in patients with extracervical carcinoma of unknown primary site. Oncologist. 2012;17(9):1146-1154. https://doi.org/10.1634/theoncologist.2011-0449
11. Economopoulou P, Mountzios G, Pavlidis N, Pentheroudakis G. Cancer of unknown primary origin in the genomic era: elucidating the dark box of cancer. Cancer Treat Rev. 2015;41(7):598-604. https://doi.org/10.1016/j.ctrv.2015.05.010
12. Greco FA. Molecular diagnosis of the tissue of origin in cancer of unknown primary site: useful in patient management. Curr Treat Options Oncol. 2013;14(4):634-642. https://doi.org/10.1007/s11864-013-0257-1
13. Massard C, Loriot Y, Fizazi K. Carcinomas of an unknown primary origin—diagnosis and treatment. Nat Rev Clin Oncol. 2011;8(12):701-710. https://doi.org/10.1038/nrclinonc.2011.158
14. Luoh SW, Flaherty KT. When tissue is no longer the issue: tissue-agnostic cancer therapy comes of age. Ann Intern Med. 2018;169(4):233-239. https://doi.org/10.7326/M17-2832

Article PDF
Author and Disclosure Information

1Department of Medicine, Warren Alpert Medical School of Brown University and The Miriam Hospital, Providence, Rhode Island; 2Department of Medicine, Northwestern University School of Medicine, Chicago, Illinois; 3Department of Medicine, University of California, San Francisco, San Francisco, California; 4Medical Service, San Francisco VA Medical Center, San Francisco, California; 5Division of Hematology and Oncology, University of California, San Francisco, San Francisco, California.

Disclosures
Drs Santos, Manesh, Hsu, and Geha have no disclosures. Dr. Dhaliwal reports receiving honoraria from ISMIE Mutual Insurance Company and GE Healthcare.

Funding
Dr Dhaliwal is a US federal government employee and prepared the paper as part of his official duties.

Issue
Journal of Hospital Medicine 16(8)
Publications
Topics
Page Number
502-506. Published Online First July 21, 2021
Sections
Author and Disclosure Information

1Department of Medicine, Warren Alpert Medical School of Brown University and The Miriam Hospital, Providence, Rhode Island; 2Department of Medicine, Northwestern University School of Medicine, Chicago, Illinois; 3Department of Medicine, University of California, San Francisco, San Francisco, California; 4Medical Service, San Francisco VA Medical Center, San Francisco, California; 5Division of Hematology and Oncology, University of California, San Francisco, San Francisco, California.

Disclosures
Drs Santos, Manesh, Hsu, and Geha have no disclosures. Dr. Dhaliwal reports receiving honoraria from ISMIE Mutual Insurance Company and GE Healthcare.

Funding
Dr Dhaliwal is a US federal government employee and prepared the paper as part of his official duties.

Author and Disclosure Information

1Department of Medicine, Warren Alpert Medical School of Brown University and The Miriam Hospital, Providence, Rhode Island; 2Department of Medicine, Northwestern University School of Medicine, Chicago, Illinois; 3Department of Medicine, University of California, San Francisco, San Francisco, California; 4Medical Service, San Francisco VA Medical Center, San Francisco, California; 5Division of Hematology and Oncology, University of California, San Francisco, San Francisco, California.

Disclosures
Drs Santos, Manesh, Hsu, and Geha have no disclosures. Dr. Dhaliwal reports receiving honoraria from ISMIE Mutual Insurance Company and GE Healthcare.

Funding
Dr Dhaliwal is a US federal government employee and prepared the paper as part of his official duties.

Article PDF
Article PDF
Related Articles

A 73-year-old man presented to clinic with 6 weeks of headache. He occasionally experienced generalized headaches throughout his life that resolved with naproxen. His new headache was characterized by a progressively worsening sensation of left-eye pressure with radiation to the left temple. Over the previous week, he had intermittent diplopia, left ptosis, and left lacrimation. He denied head trauma, fever, vision loss, photophobia, dysphagia, dysarthria, nausea, vomiting, or jaw claudication.

Primary headaches include tension type, migraine, and trigeminal autonomic cephalalgias (eg, cluster headache). A new headache in an older patient, particularly if protracted and progressive, prioritizes consideration of a secondary headache, which may reflect pathology within the brain parenchyma (eg, intracranial mass), blood vessels (eg, giant cell arteritis), meninges (eg, meningitis), or ventricles (eg, intraventricular cyst). Eye pain may arise from ocular and extraocular disease. Corneal abrasions, infectious keratitis, scleritis, uveitis, or acute angle-closure glaucoma are painful, although the latter is less likely given the prolonged duration of symptoms. Thyroid eye disease or other infiltrative disorders of the orbit can also cause eye discomfort.

Ptosis commonly results from degeneration of the levator aponeurosis. Other causes include third cranial nerve palsy and myasthenia gravis. Interruption of sympathetic innervation of the eyelid by lesions in the brain stem, spinal cord, lung (eg, Pancoast tumor), or cavernous sinus also can result in ptosis.

Whether the patient has monocular or binocular diplopia is uncertain. Monocular diplopia persists with only one eye open and can arise from uncorrected refractive error, corneal irregularities, lenticular opacities, or unilateral macular disease. Binocular diplopia develops from ocular misalignment due to neuromuscular weakness, extraocular muscle entrapment, or an orbital mass displacing the globe. An orbital mass would also explain the unilateral headache and unilateral ptosis.

His medical history included coronary artery disease, seronegative rheumatoid arthritis, osteoporosis, benign prostatic hypertrophy, and ureteral strictures from chronic nephrolithiasis. Following a cholecystectomy for gallstone pancreatitis 13 years earlier, he was hospitalized five more times for pancreatitis. The last episode was 6 years prior to this presentation. At that time, magnetic resonance cholangiopancreatography (MRCP) did not reveal pancreatic divisum, annular pancreas, biliary strictures, or a pancreatic mass. Esophagogastroduodenoscopy peformed during the same hospitalization showed mild gastritis. His recurrent pancreatitis was deemed idiopathic.

His medications were folic acid, cholecalciferol, lisinopril, metoprolol, omeprazole, simvastatin, aspirin, and weekly methotrexate. His sister had breast and ovarian cancer, and his brother had gastric cancer. He had two subcentimeter tubular adenomas removed during a screening colonoscopy 3 years prior. He had a 30 pack-year smoking history and quit 28 years earlier. He did not use alcohol or drugs. He was a retired chemical plant worker.

Choledocholithiasis (as discrete stones or biliary sludge) can trigger pancreatitis despite a cholecystectomy, but the recurrent episodes and negative MRCP should prompt consideration of other causes, such as alcohol. Hypercalcemia, hypertriglyceridemia, and medications are infrequent causes of pancreatic inflammation. IgG4-related disease (IgG4-RD) causes autoimmune pancreatitis and can infiltrate the eyelids, lacrimal glands, extraocular muscles, or orbital connective tissue. Malignancy of the pancreas or ampulla can trigger pancreatitis by causing pancreatic duct obstruction but would not go undetected for 13 years.

The patient was evaluated by an ophthalmologist and a neurologist. His heart rate was 52 beats per minute and blood pressure, 174/70 mm Hg; other vital signs were normal. He had conjunctival chemosis, ptosis, and nonpulsatile proptosis of the left eye with tenderness and increased resistance to retropulsion compared to the right eye (Figure 1). Visual acuity was 20/25 for the right eye and hand motions only in the left eye. The pupils were reactive and symmetric without afferent pupillary defect. There was no optic nerve swelling or pallor. Abduction, adduction, and elevation of the left eye were restricted and associated with diplopia. Movement of the right eye was unrestricted. There was no other facial asymmetry. Facial sensation was normal. Corneal reflexes were intact. Shoulder shrug strength was equal and symmetric. Tongue protrusion was midline. Olfaction and hearing were not assessed. Strength, sensation, and deep tendon reflexes were normal in all extremities. The plantar response was flexor bilaterally.

jhm01608502_f1.JPG

Unilateral ptosis, chemosis, proptosis, ophthalmoplegia, eye tenderness, and visual loss collectively point to a space-occupying orbital disease. Orbital masses are caused by cancers, infections such as mucormycosis (usually in an immunocompromised host), and inflammatory disorders such as thyroid orbitopathy, sarcoidosis, IgG4-related orbitopathy, granulomatosis with polyangiitis, and orbital pseudotumor (idiopathic inflammation of the orbit). Chemosis reflects edema of the conjunctiva, which can arise from direct conjunctival injury (eg, allergy, infection, or trauma), interruption of the venous drainage of the conjunctiva by vascular disorders (eg, cavernous sinus thrombosis or carotid-cavernous fistula), or space-occupying diseases of the orbit. Monocular visual loss arises from a prechiasmal lesion, and acute monocular visual loss is more commonly caused by posterior ocular pathology (eg, retina or optic nerve) than anterior disease (eg, keratitis). Visual loss in the presence of an orbital process suggests a compressive or infiltrative disease of the optic nerve.

Complete blood count, comprehensive metabolic panel, erythrocyte sedimentation rate, C-reactive protein, and thyroid function tests were normal. Interferon-gamma release assay, HIV antibody, rapid plasma reagin, Lyme antibody, antinuclear antibody, and antineutrophil cytoplasmic antibody (ANCA) tests were negative. A noncontrast computed tomography (CT) scan of the head revealed thickening of the left inferior rectus muscle. Orbital magnetic resonance imaging (MRI) with gadolinium and fluid-attenuated inversion recovery imaging demonstrated a T2 hyperintense, heterogeneous 1.4-cm mass in the left inferior rectus muscle (Figure 2). There was no carotid-cavernous fistula, brain mass, or meningeal enhancement.

jhm01608502_f2.JPG

An isolated mass in one ocular muscle raises the probability of a cancer. The most common malignant orbital tumor is B-cell lymphoma. Metastatic cancer to the eye is rare; breast, prostate, and lung cancer account for the majority of cases. The family history of breast and ovarian cancer raises the possibility of a BRCA mutation, which is also associated with gastric, pancreatic, and prostate malignancies. Granulomatosis with polyangiitis may be ANCA negative in localized sino-orbital disease. Biopsy of the orbital mass is the next step.

The patient underwent transconjunctival orbitotomy with excision of the left inferior rectus mass. Two days later, he presented to the emergency department with acute onset epigastric pain, nausea, and vomiting. A comprehensive review of systems, which had not been performed until this visit, revealed an unintentional 20-lb weight loss over the previous 3 months. He had a progressive ache in the left anterior groin that was dull, tender, nonradiating, and worse with weight bearing. He denied melena or hematochezia.

His temperature was 37 °C; heart rate, 98 beats per minute; and blood pressure, 128/63 mm Hg. He had midepigastric tenderness and point tenderness over the anterior iliac spine. White blood cell count was 12,600/μL; hemo globin, 14.5 g/dL; and platelet count, 158,000/μL. Serum lipase was 7,108 U/L. Serum creatinine, calcium, and triglyceride levels were normal. Alkaline phosphatase was 117 U/L (normal, 34-104 U/L); total bilirubin, 1.1 mg/dL; alanine aminotransferase (ALT), 119 U/L (normal, 7-52 U/L); and aspartate aminotransferase (AST), 236 U/L (normal, 13-39 U/L). Troponin I was undetectable, and an electrocardiogram demonstrated sinus tachycardia. Urinalysis was normal.

Concomitant pancreatitis and hepatitis with an elevated AST-to-ALT ratio should prompt evaluation of recurrent choledocholithiasis and a repeat inquiry about alcohol use. His medications should be reviewed for an association with pancreatitis. Anterior groin discomfort usually reflects osteoarthritis of the hip joint, inguinal hernia, or inguinal lymphadenopathy. Groin pain may be referred from spinal nerve root compression, aortoiliac occlusion, or nephrolithiasis. Weight loss in the presence of an inferior rectus mass suggests one of the aforementioned systemic diseases with orbital manifestations. Pancreatitis and groin discomfort may be important clues, but the chronicity of the recurrent pancreatitis and the high prevalence of hip osteoarthritis make it equally likely that they are unrelated to the eye disease.

CT scan of the abdomen and pelvis with contrast showed peripancreatic edema with fat stranding but no pancreatic or hepatobiliary mass. The common bile duct was normal. A 2.2×1.3-cm mass in the right posterior subphrenic space, a lytic lesion in the left anterior inferior iliac spine, and right nonobstructive nephrolithiasis were identified. CT scan of the chest with contrast showed multiple subpleural nodules and innumerable parenchymal nodules. Subcentimeter hilar, mediastinal, and prevascular lymphadenopathy were present, as well as multiple sclerotic lesions in the right fourth and sixth ribs. Prostate-specific antigen was 0.7 ng/mL (normal, ≤ 4.0 ng/mL). Cancer antigen 19-9 level was 5.5 U/mL (normal, < 37.0 U/mL), and carcinoembryonic antigen (CEA) was 100.1 ng/mL (normal, 0-3 U/mL).

Widespread pulmonary nodules, diffuse lymphadenopathy, and bony lesions raise concern for a metastatic malignancy. There is no evidence of a primary carcinoma. The lack of hepatic involvement reduces the likelihood of a gastrointestinal tumor, although a rectal cancer, which may drain directly into the inferior vena cava and bypass the portal circulation, could present as lung metastases on CT imaging. Lymphoma is plausible given the diffuse lymphadenopathy and orbital mass. Sarcoidosis and histiocytic disorders (eg, Langerhans cell histiocytosis) also cause orbital disease, pulmonary nodules, lymphadenopathy, and bone lesions, although a subphrenic mass would be atypical for both disorders; furthermore, the majority of patients with adult Langerhans cell histiocytosis smoke cigarettes. The elevated CEA makes a metastatic solid tumor more likely than lymphoma but does not specify the location of the primary tumor.

Pathology of the inferior rectus muscle mass showed well-differentiated adenocarcinoma (Figure 3A and 3B). A CT-guided biopsy of the left anterior inferior iliac spine revealed well-differentiated adenocarcinoma (Figure 3C). Adenocarcinoma of unknown primary wasdiagnosed.

Subsequent immunohistochemical (IHC) staining was positive for cytokeratin 7 (CK7) and mucicarmine (Figure 3D and 3E) and negative for cytokeratin 20 (CK20) and thyroid transcription factor 1 (TTF1). This IHC profile suggested pancreatic or upper gastrointestinal tract lineage. Positron emission tomography–CT (PET-CT) scan was aborted because of dyspnea and chest pressure following contrast administration. He declined further imaging or endoscopy. He received palliative radiation and three cycles of paclitaxel and gemcitabine for cancer of unknown primary (CUP). Two months later, he developed bilateral upper-arm weakness due to C7 and T2 cord compression from vertebral and epidural metastases; his symptoms progressed despite salvage chemotherapy. He was transitioned to comfort care and died at home 9 months after diagnosis.

jhm01608502_f3.JPG

DISCUSSION

This patient’s new headache and ocular abnormalities led to the discovery of an inferior rectus muscle mass. Initially unrecognized unintentional weight loss and hip pain recast a localized orbital syndrome as a systemic disease with pancreatic, ocular, pulmonary, lymph node, and skeletal pathology. Biopsies of the orbital rectus muscle and iliac bone demonstrated metastatic adenocarcinoma. Imaging studies did not identify a primary cancer, but IHC analysis suggested carcinoma of upper gastrointestinal or pancreatic origin.

Acute and chronic pancreatitis are both associated with pancreatic cancer.1 Chronic pancreatitis is associated with an increasing cumulative risk of pancreatic cancer; a potential mechanism is chronic inflammation with malignant transformation.2,3 There is also a 20-fold increased risk of pancreatic cancer in the first 2 years following an episode of acute pancreatitis,4 which may develop from malignant pancreatic duct obstruction. Although the post–acute pancreatitis risk of pancreatic cancer attenuates over time, a two-fold increased risk of pancreatic cancer remains after 10 years,4 which suggests that acute pancreatitis (particularly when idiopathic) either contributes to or shares pathogenesis with pancreatic adenocarcinoma. In elderly patients without gallstones or alcohol use, an abdominal CT scan or MRI shortly after resolution of the acute pancreatitis may be considered to assess for an underlying pancreatic tumor.5

CUP is a histologically defined malignancy without a known primary anatomic site despite an extensive evaluation. CUP accounts for up to 10% of all cancer diagnoses.6 CUP is ascribed to a primary cancer that remains too small to be detected or spontaneous regression of the primary cancer.7 Approximately 70% of autopsies of patients with CUP identify the primary tumor, which most commonly originates in the lung, gastrointestinal tract, breast, or pancreas.8

When a metastatic focus of cancer is found but the initial diagnostic evaluation (including CT scan of the chest, abdomen, and pelvis) fails to locate a primary cancer, the next step in searching for the tissue of origin is an IHC analysis of the tumor specimen. IHC analysis is a multistep staining process that can identify major categories of cancer, including carcinoma (adenocarcinoma, squamous cell carcinoma, and neuroendocrine carcinoma) and poorly or undifferentiated neoplasms (including carcinoma, lymphoma, sarcoma, or melanoma). Eighty-five percent of CUP cases are adenocarcinoma, 10% are squamous cell carcinoma, and the remaining 5% are undifferentiated neoplasms.9

There are no consensus guidelines for imaging in patients with CUP who have already undergone a CT scan of the chest, abdomen, and pelvis. Mammography is indicated in women with metastatic adenocarcinoma or axillary lymphadenopathy.7 MRI of the breast is obtained when mammography is nondiagnostic and the suspicion for breast cancer is high. Small clinical studies and meta-analyses support the use of PET-CT scans,7 although one study found that a PET-CT scan was not superior to CT imaging in identifying the primary tumor site in CUP.10 Endoscopy of the upper airway or gastrointestinal tract is rarely diagnostic in the absence of referable symptoms or a suggestive IHC profile (eg, CK7−, CK20+ suggestive of colon cancer).6

Molecular cancer classification has emerged as a useful diagnostic technique in CUP. Cancer cells retain gene expression patterns based on cellular origin, and a tumor’s profile can be compared with a reference database of known cancers, aiding in the identification of the primary tumor type. Molecular cancer classifier assays that use gene expression profiling can accurately determine a primary site11 and have been shown to be concordant with IHC testing.12 Molecular cancer classification is distinct from genetic assays that identify mutations for which there are approved therapies. Serum tumor markers are generally not useful in establishing the primary tumor and should be considered based on the clinical presentation (eg, prostate-specific antigen testing in a man with adenocarcinoma of unknown primary and osteoblastic metastases).

CUP is classified as favorable or unfavorable based on the IHC, pattern of spread, and serum markers in certain cases.6 Approximately 20% of CUP patients can be categorized into favorable subsets, such as adenocarcinoma in a single axillary lymph node in a female patient suggestive of a breast primary cancer, or squamous cell carcinoma in a cervical lymph node suggestive of a head or neck primary cancer.7 The remaining 80% of cases are categorized as unfavorable CUP and often have multiple metastases. Our patient’s pattern of spread and limited response to chemotherapy is characteristic of the unfavorable subset of CUP. The median survival of this group is 9 months, and only 25% of patients survive longer than 1 year.13

Biomarker-driven treatment of specific molecular targets independent of the tissue of origin (tissue-agnostic therapy) has shown promising results in the treatment of skin, lung, thyroid, colorectal, and gastric cancers.14 Pembrolizumab was the first drug approved by the US Food and Drug Administration based on a tumor’s biomarker without regard to its primary location. Data to support this approach for treating CUP are evolving and offer hope for patients with specific molecular targets.

Following the focused neuro-ophthalmologic evaluations, with focused examination and imaging, the hospitalist’s review of systems at the time of the final admission for pancreatitis set in motion an evaluation that led to a diagnosis of metastatic cancer. The risk factor of recurrent pancreatitis and IHC results suggested that pancreatic adenocarcinoma was the most likely primary tumor. As the focus of cancer treatment shifts away from the tissue of origin and toward molecular and genetic profiles, the search for the primary site may decrease in importance. In the future, even when we do not know the cancer’s origin, we may still know precisely what to do. But for now, as in this patient, our treatments continue to be based on a tumor that is out of sight, but not out of mind.

KEY TEACHING POINTS

  • Acute and chronic pancreatitis are associated with an increased risk of pancreatic adenocarcinoma.
  • CUP is a cancer in which diagnostic testing does not identify a primary tumor site. Immunohistochemistry and molecular analysis, imaging, and endoscopy are utilized selectively to identify a primary tumor type.
  • Treatment of CUP currently depends on the suspected tissue of origin and pattern of spread.
  • Tissue-agnostic therapy could allow for treatment for CUP patients independent of the tissue of origin.

Acknowledgments

We thank Andrew Mick, OD, for his review of an earlier version of this manuscript and Peter Phillips, MD, for his interpretation of the pathologic images.

A 73-year-old man presented to clinic with 6 weeks of headache. He occasionally experienced generalized headaches throughout his life that resolved with naproxen. His new headache was characterized by a progressively worsening sensation of left-eye pressure with radiation to the left temple. Over the previous week, he had intermittent diplopia, left ptosis, and left lacrimation. He denied head trauma, fever, vision loss, photophobia, dysphagia, dysarthria, nausea, vomiting, or jaw claudication.

Primary headaches include tension type, migraine, and trigeminal autonomic cephalalgias (eg, cluster headache). A new headache in an older patient, particularly if protracted and progressive, prioritizes consideration of a secondary headache, which may reflect pathology within the brain parenchyma (eg, intracranial mass), blood vessels (eg, giant cell arteritis), meninges (eg, meningitis), or ventricles (eg, intraventricular cyst). Eye pain may arise from ocular and extraocular disease. Corneal abrasions, infectious keratitis, scleritis, uveitis, or acute angle-closure glaucoma are painful, although the latter is less likely given the prolonged duration of symptoms. Thyroid eye disease or other infiltrative disorders of the orbit can also cause eye discomfort.

Ptosis commonly results from degeneration of the levator aponeurosis. Other causes include third cranial nerve palsy and myasthenia gravis. Interruption of sympathetic innervation of the eyelid by lesions in the brain stem, spinal cord, lung (eg, Pancoast tumor), or cavernous sinus also can result in ptosis.

Whether the patient has monocular or binocular diplopia is uncertain. Monocular diplopia persists with only one eye open and can arise from uncorrected refractive error, corneal irregularities, lenticular opacities, or unilateral macular disease. Binocular diplopia develops from ocular misalignment due to neuromuscular weakness, extraocular muscle entrapment, or an orbital mass displacing the globe. An orbital mass would also explain the unilateral headache and unilateral ptosis.

His medical history included coronary artery disease, seronegative rheumatoid arthritis, osteoporosis, benign prostatic hypertrophy, and ureteral strictures from chronic nephrolithiasis. Following a cholecystectomy for gallstone pancreatitis 13 years earlier, he was hospitalized five more times for pancreatitis. The last episode was 6 years prior to this presentation. At that time, magnetic resonance cholangiopancreatography (MRCP) did not reveal pancreatic divisum, annular pancreas, biliary strictures, or a pancreatic mass. Esophagogastroduodenoscopy peformed during the same hospitalization showed mild gastritis. His recurrent pancreatitis was deemed idiopathic.

His medications were folic acid, cholecalciferol, lisinopril, metoprolol, omeprazole, simvastatin, aspirin, and weekly methotrexate. His sister had breast and ovarian cancer, and his brother had gastric cancer. He had two subcentimeter tubular adenomas removed during a screening colonoscopy 3 years prior. He had a 30 pack-year smoking history and quit 28 years earlier. He did not use alcohol or drugs. He was a retired chemical plant worker.

Choledocholithiasis (as discrete stones or biliary sludge) can trigger pancreatitis despite a cholecystectomy, but the recurrent episodes and negative MRCP should prompt consideration of other causes, such as alcohol. Hypercalcemia, hypertriglyceridemia, and medications are infrequent causes of pancreatic inflammation. IgG4-related disease (IgG4-RD) causes autoimmune pancreatitis and can infiltrate the eyelids, lacrimal glands, extraocular muscles, or orbital connective tissue. Malignancy of the pancreas or ampulla can trigger pancreatitis by causing pancreatic duct obstruction but would not go undetected for 13 years.

The patient was evaluated by an ophthalmologist and a neurologist. His heart rate was 52 beats per minute and blood pressure, 174/70 mm Hg; other vital signs were normal. He had conjunctival chemosis, ptosis, and nonpulsatile proptosis of the left eye with tenderness and increased resistance to retropulsion compared to the right eye (Figure 1). Visual acuity was 20/25 for the right eye and hand motions only in the left eye. The pupils were reactive and symmetric without afferent pupillary defect. There was no optic nerve swelling or pallor. Abduction, adduction, and elevation of the left eye were restricted and associated with diplopia. Movement of the right eye was unrestricted. There was no other facial asymmetry. Facial sensation was normal. Corneal reflexes were intact. Shoulder shrug strength was equal and symmetric. Tongue protrusion was midline. Olfaction and hearing were not assessed. Strength, sensation, and deep tendon reflexes were normal in all extremities. The plantar response was flexor bilaterally.

jhm01608502_f1.JPG

Unilateral ptosis, chemosis, proptosis, ophthalmoplegia, eye tenderness, and visual loss collectively point to a space-occupying orbital disease. Orbital masses are caused by cancers, infections such as mucormycosis (usually in an immunocompromised host), and inflammatory disorders such as thyroid orbitopathy, sarcoidosis, IgG4-related orbitopathy, granulomatosis with polyangiitis, and orbital pseudotumor (idiopathic inflammation of the orbit). Chemosis reflects edema of the conjunctiva, which can arise from direct conjunctival injury (eg, allergy, infection, or trauma), interruption of the venous drainage of the conjunctiva by vascular disorders (eg, cavernous sinus thrombosis or carotid-cavernous fistula), or space-occupying diseases of the orbit. Monocular visual loss arises from a prechiasmal lesion, and acute monocular visual loss is more commonly caused by posterior ocular pathology (eg, retina or optic nerve) than anterior disease (eg, keratitis). Visual loss in the presence of an orbital process suggests a compressive or infiltrative disease of the optic nerve.

Complete blood count, comprehensive metabolic panel, erythrocyte sedimentation rate, C-reactive protein, and thyroid function tests were normal. Interferon-gamma release assay, HIV antibody, rapid plasma reagin, Lyme antibody, antinuclear antibody, and antineutrophil cytoplasmic antibody (ANCA) tests were negative. A noncontrast computed tomography (CT) scan of the head revealed thickening of the left inferior rectus muscle. Orbital magnetic resonance imaging (MRI) with gadolinium and fluid-attenuated inversion recovery imaging demonstrated a T2 hyperintense, heterogeneous 1.4-cm mass in the left inferior rectus muscle (Figure 2). There was no carotid-cavernous fistula, brain mass, or meningeal enhancement.

jhm01608502_f2.JPG

An isolated mass in one ocular muscle raises the probability of a cancer. The most common malignant orbital tumor is B-cell lymphoma. Metastatic cancer to the eye is rare; breast, prostate, and lung cancer account for the majority of cases. The family history of breast and ovarian cancer raises the possibility of a BRCA mutation, which is also associated with gastric, pancreatic, and prostate malignancies. Granulomatosis with polyangiitis may be ANCA negative in localized sino-orbital disease. Biopsy of the orbital mass is the next step.

The patient underwent transconjunctival orbitotomy with excision of the left inferior rectus mass. Two days later, he presented to the emergency department with acute onset epigastric pain, nausea, and vomiting. A comprehensive review of systems, which had not been performed until this visit, revealed an unintentional 20-lb weight loss over the previous 3 months. He had a progressive ache in the left anterior groin that was dull, tender, nonradiating, and worse with weight bearing. He denied melena or hematochezia.

His temperature was 37 °C; heart rate, 98 beats per minute; and blood pressure, 128/63 mm Hg. He had midepigastric tenderness and point tenderness over the anterior iliac spine. White blood cell count was 12,600/μL; hemo globin, 14.5 g/dL; and platelet count, 158,000/μL. Serum lipase was 7,108 U/L. Serum creatinine, calcium, and triglyceride levels were normal. Alkaline phosphatase was 117 U/L (normal, 34-104 U/L); total bilirubin, 1.1 mg/dL; alanine aminotransferase (ALT), 119 U/L (normal, 7-52 U/L); and aspartate aminotransferase (AST), 236 U/L (normal, 13-39 U/L). Troponin I was undetectable, and an electrocardiogram demonstrated sinus tachycardia. Urinalysis was normal.

Concomitant pancreatitis and hepatitis with an elevated AST-to-ALT ratio should prompt evaluation of recurrent choledocholithiasis and a repeat inquiry about alcohol use. His medications should be reviewed for an association with pancreatitis. Anterior groin discomfort usually reflects osteoarthritis of the hip joint, inguinal hernia, or inguinal lymphadenopathy. Groin pain may be referred from spinal nerve root compression, aortoiliac occlusion, or nephrolithiasis. Weight loss in the presence of an inferior rectus mass suggests one of the aforementioned systemic diseases with orbital manifestations. Pancreatitis and groin discomfort may be important clues, but the chronicity of the recurrent pancreatitis and the high prevalence of hip osteoarthritis make it equally likely that they are unrelated to the eye disease.

CT scan of the abdomen and pelvis with contrast showed peripancreatic edema with fat stranding but no pancreatic or hepatobiliary mass. The common bile duct was normal. A 2.2×1.3-cm mass in the right posterior subphrenic space, a lytic lesion in the left anterior inferior iliac spine, and right nonobstructive nephrolithiasis were identified. CT scan of the chest with contrast showed multiple subpleural nodules and innumerable parenchymal nodules. Subcentimeter hilar, mediastinal, and prevascular lymphadenopathy were present, as well as multiple sclerotic lesions in the right fourth and sixth ribs. Prostate-specific antigen was 0.7 ng/mL (normal, ≤ 4.0 ng/mL). Cancer antigen 19-9 level was 5.5 U/mL (normal, < 37.0 U/mL), and carcinoembryonic antigen (CEA) was 100.1 ng/mL (normal, 0-3 U/mL).

Widespread pulmonary nodules, diffuse lymphadenopathy, and bony lesions raise concern for a metastatic malignancy. There is no evidence of a primary carcinoma. The lack of hepatic involvement reduces the likelihood of a gastrointestinal tumor, although a rectal cancer, which may drain directly into the inferior vena cava and bypass the portal circulation, could present as lung metastases on CT imaging. Lymphoma is plausible given the diffuse lymphadenopathy and orbital mass. Sarcoidosis and histiocytic disorders (eg, Langerhans cell histiocytosis) also cause orbital disease, pulmonary nodules, lymphadenopathy, and bone lesions, although a subphrenic mass would be atypical for both disorders; furthermore, the majority of patients with adult Langerhans cell histiocytosis smoke cigarettes. The elevated CEA makes a metastatic solid tumor more likely than lymphoma but does not specify the location of the primary tumor.

Pathology of the inferior rectus muscle mass showed well-differentiated adenocarcinoma (Figure 3A and 3B). A CT-guided biopsy of the left anterior inferior iliac spine revealed well-differentiated adenocarcinoma (Figure 3C). Adenocarcinoma of unknown primary wasdiagnosed.

Subsequent immunohistochemical (IHC) staining was positive for cytokeratin 7 (CK7) and mucicarmine (Figure 3D and 3E) and negative for cytokeratin 20 (CK20) and thyroid transcription factor 1 (TTF1). This IHC profile suggested pancreatic or upper gastrointestinal tract lineage. Positron emission tomography–CT (PET-CT) scan was aborted because of dyspnea and chest pressure following contrast administration. He declined further imaging or endoscopy. He received palliative radiation and three cycles of paclitaxel and gemcitabine for cancer of unknown primary (CUP). Two months later, he developed bilateral upper-arm weakness due to C7 and T2 cord compression from vertebral and epidural metastases; his symptoms progressed despite salvage chemotherapy. He was transitioned to comfort care and died at home 9 months after diagnosis.

jhm01608502_f3.JPG

DISCUSSION

This patient’s new headache and ocular abnormalities led to the discovery of an inferior rectus muscle mass. Initially unrecognized unintentional weight loss and hip pain recast a localized orbital syndrome as a systemic disease with pancreatic, ocular, pulmonary, lymph node, and skeletal pathology. Biopsies of the orbital rectus muscle and iliac bone demonstrated metastatic adenocarcinoma. Imaging studies did not identify a primary cancer, but IHC analysis suggested carcinoma of upper gastrointestinal or pancreatic origin.

Acute and chronic pancreatitis are both associated with pancreatic cancer.1 Chronic pancreatitis is associated with an increasing cumulative risk of pancreatic cancer; a potential mechanism is chronic inflammation with malignant transformation.2,3 There is also a 20-fold increased risk of pancreatic cancer in the first 2 years following an episode of acute pancreatitis,4 which may develop from malignant pancreatic duct obstruction. Although the post–acute pancreatitis risk of pancreatic cancer attenuates over time, a two-fold increased risk of pancreatic cancer remains after 10 years,4 which suggests that acute pancreatitis (particularly when idiopathic) either contributes to or shares pathogenesis with pancreatic adenocarcinoma. In elderly patients without gallstones or alcohol use, an abdominal CT scan or MRI shortly after resolution of the acute pancreatitis may be considered to assess for an underlying pancreatic tumor.5

CUP is a histologically defined malignancy without a known primary anatomic site despite an extensive evaluation. CUP accounts for up to 10% of all cancer diagnoses.6 CUP is ascribed to a primary cancer that remains too small to be detected or spontaneous regression of the primary cancer.7 Approximately 70% of autopsies of patients with CUP identify the primary tumor, which most commonly originates in the lung, gastrointestinal tract, breast, or pancreas.8

When a metastatic focus of cancer is found but the initial diagnostic evaluation (including CT scan of the chest, abdomen, and pelvis) fails to locate a primary cancer, the next step in searching for the tissue of origin is an IHC analysis of the tumor specimen. IHC analysis is a multistep staining process that can identify major categories of cancer, including carcinoma (adenocarcinoma, squamous cell carcinoma, and neuroendocrine carcinoma) and poorly or undifferentiated neoplasms (including carcinoma, lymphoma, sarcoma, or melanoma). Eighty-five percent of CUP cases are adenocarcinoma, 10% are squamous cell carcinoma, and the remaining 5% are undifferentiated neoplasms.9

There are no consensus guidelines for imaging in patients with CUP who have already undergone a CT scan of the chest, abdomen, and pelvis. Mammography is indicated in women with metastatic adenocarcinoma or axillary lymphadenopathy.7 MRI of the breast is obtained when mammography is nondiagnostic and the suspicion for breast cancer is high. Small clinical studies and meta-analyses support the use of PET-CT scans,7 although one study found that a PET-CT scan was not superior to CT imaging in identifying the primary tumor site in CUP.10 Endoscopy of the upper airway or gastrointestinal tract is rarely diagnostic in the absence of referable symptoms or a suggestive IHC profile (eg, CK7−, CK20+ suggestive of colon cancer).6

Molecular cancer classification has emerged as a useful diagnostic technique in CUP. Cancer cells retain gene expression patterns based on cellular origin, and a tumor’s profile can be compared with a reference database of known cancers, aiding in the identification of the primary tumor type. Molecular cancer classifier assays that use gene expression profiling can accurately determine a primary site11 and have been shown to be concordant with IHC testing.12 Molecular cancer classification is distinct from genetic assays that identify mutations for which there are approved therapies. Serum tumor markers are generally not useful in establishing the primary tumor and should be considered based on the clinical presentation (eg, prostate-specific antigen testing in a man with adenocarcinoma of unknown primary and osteoblastic metastases).

CUP is classified as favorable or unfavorable based on the IHC, pattern of spread, and serum markers in certain cases.6 Approximately 20% of CUP patients can be categorized into favorable subsets, such as adenocarcinoma in a single axillary lymph node in a female patient suggestive of a breast primary cancer, or squamous cell carcinoma in a cervical lymph node suggestive of a head or neck primary cancer.7 The remaining 80% of cases are categorized as unfavorable CUP and often have multiple metastases. Our patient’s pattern of spread and limited response to chemotherapy is characteristic of the unfavorable subset of CUP. The median survival of this group is 9 months, and only 25% of patients survive longer than 1 year.13

Biomarker-driven treatment of specific molecular targets independent of the tissue of origin (tissue-agnostic therapy) has shown promising results in the treatment of skin, lung, thyroid, colorectal, and gastric cancers.14 Pembrolizumab was the first drug approved by the US Food and Drug Administration based on a tumor’s biomarker without regard to its primary location. Data to support this approach for treating CUP are evolving and offer hope for patients with specific molecular targets.

Following the focused neuro-ophthalmologic evaluations, with focused examination and imaging, the hospitalist’s review of systems at the time of the final admission for pancreatitis set in motion an evaluation that led to a diagnosis of metastatic cancer. The risk factor of recurrent pancreatitis and IHC results suggested that pancreatic adenocarcinoma was the most likely primary tumor. As the focus of cancer treatment shifts away from the tissue of origin and toward molecular and genetic profiles, the search for the primary site may decrease in importance. In the future, even when we do not know the cancer’s origin, we may still know precisely what to do. But for now, as in this patient, our treatments continue to be based on a tumor that is out of sight, but not out of mind.

KEY TEACHING POINTS

  • Acute and chronic pancreatitis are associated with an increased risk of pancreatic adenocarcinoma.
  • CUP is a cancer in which diagnostic testing does not identify a primary tumor site. Immunohistochemistry and molecular analysis, imaging, and endoscopy are utilized selectively to identify a primary tumor type.
  • Treatment of CUP currently depends on the suspected tissue of origin and pattern of spread.
  • Tissue-agnostic therapy could allow for treatment for CUP patients independent of the tissue of origin.

Acknowledgments

We thank Andrew Mick, OD, for his review of an earlier version of this manuscript and Peter Phillips, MD, for his interpretation of the pathologic images.

References

1. Sadr-Azodi O, Oskarsson V, Discacciati A, Videhult P, Askling J, Ekbom A. Pancreatic cancer following acute pancreatitis: a population-based matched cohort study. Am J Gastroenterol. 2018;113(111):1711-1719. https://doi.org/10.1038/s41395-018-0255-9
2. Duell EJ, Lucenteforte E, Olson SH, et al. Pancreatitis and pancreatic cancer risk: a pooled analysis in the International Pancreatic Cancer Case-Control Consortium (PanC4). Ann Oncol. 2012;23(11):2964-2970. https://doi.org/10.1093/annonc/mds140
3. Ekbom A, McLaughlin JK, Nyren O. Pancreatitis and the risk of pancreatic cancer. N Engl J Med. 1993;329(20):1502-1503. https://doi.org/10.1056/NEJM199311113292016
4. Kirkegard J, Cronin-Fenton D, Heide-Jorgensen U, Mortensen FV. Acute pancreatitis and pancreatic cancer risk: a nationwide matched-cohort study in Denmark. Gastroenterology. 2018;154(156):1729-1736. https://doi.org/10.1053/j.gastro.2018.02.011
5. Frampas E, Morla O, Regenet N, Eugene T, Dupas B, Meurette G. A solid pancreatic mass: tumour or inflammation? Diagn Interv Imaging. 2013;94(7-8):741-755. https://doi.org/10.1016/j.diii.2013.03.013
6. Varadhachary GR, Raber MN. Cancer of unknown primary site. N Engl J Med. 2014;371(8):757-765. https://doi.org/10.1056/NEJMra1303917
7. Bochtler T, Löffler H, Krämer A. Diagnosis and management of metastatic neoplasms with unknown primary. Semin Diagn Pathol. 2017. 2018;35(3):199-206. https://doi.org//10.1053/j.semdp.2017.11.013
8. Pentheroudakis G, Golfinopoulos V, Pavlidis N. Switching benchmarks in cancer of unknown primary: from autopsy to microarray. Eur J Cancer. 2007;43(14):2026-2036. https://doi.org/10.1016/j.ejca.2007.06.023
9. Pavlidis N, Fizazi K. Carcinoma of unknown primary (CUP). Crit Rev Oncol Hematol. 2009;69(3):271-278. https://doi.org/10.1016/j.critrevonc.2008.09.005
10. Moller AK, Loft A, Berthelsen AK, et al. A prospective comparison of 18F-FDG PET/CT and CT as diagnostic tools to identify the primary tumor site in patients with extracervical carcinoma of unknown primary site. Oncologist. 2012;17(9):1146-1154. https://doi.org/10.1634/theoncologist.2011-0449
11. Economopoulou P, Mountzios G, Pavlidis N, Pentheroudakis G. Cancer of unknown primary origin in the genomic era: elucidating the dark box of cancer. Cancer Treat Rev. 2015;41(7):598-604. https://doi.org/10.1016/j.ctrv.2015.05.010
12. Greco FA. Molecular diagnosis of the tissue of origin in cancer of unknown primary site: useful in patient management. Curr Treat Options Oncol. 2013;14(4):634-642. https://doi.org/10.1007/s11864-013-0257-1
13. Massard C, Loriot Y, Fizazi K. Carcinomas of an unknown primary origin—diagnosis and treatment. Nat Rev Clin Oncol. 2011;8(12):701-710. https://doi.org/10.1038/nrclinonc.2011.158
14. Luoh SW, Flaherty KT. When tissue is no longer the issue: tissue-agnostic cancer therapy comes of age. Ann Intern Med. 2018;169(4):233-239. https://doi.org/10.7326/M17-2832

References

1. Sadr-Azodi O, Oskarsson V, Discacciati A, Videhult P, Askling J, Ekbom A. Pancreatic cancer following acute pancreatitis: a population-based matched cohort study. Am J Gastroenterol. 2018;113(111):1711-1719. https://doi.org/10.1038/s41395-018-0255-9
2. Duell EJ, Lucenteforte E, Olson SH, et al. Pancreatitis and pancreatic cancer risk: a pooled analysis in the International Pancreatic Cancer Case-Control Consortium (PanC4). Ann Oncol. 2012;23(11):2964-2970. https://doi.org/10.1093/annonc/mds140
3. Ekbom A, McLaughlin JK, Nyren O. Pancreatitis and the risk of pancreatic cancer. N Engl J Med. 1993;329(20):1502-1503. https://doi.org/10.1056/NEJM199311113292016
4. Kirkegard J, Cronin-Fenton D, Heide-Jorgensen U, Mortensen FV. Acute pancreatitis and pancreatic cancer risk: a nationwide matched-cohort study in Denmark. Gastroenterology. 2018;154(156):1729-1736. https://doi.org/10.1053/j.gastro.2018.02.011
5. Frampas E, Morla O, Regenet N, Eugene T, Dupas B, Meurette G. A solid pancreatic mass: tumour or inflammation? Diagn Interv Imaging. 2013;94(7-8):741-755. https://doi.org/10.1016/j.diii.2013.03.013
6. Varadhachary GR, Raber MN. Cancer of unknown primary site. N Engl J Med. 2014;371(8):757-765. https://doi.org/10.1056/NEJMra1303917
7. Bochtler T, Löffler H, Krämer A. Diagnosis and management of metastatic neoplasms with unknown primary. Semin Diagn Pathol. 2017. 2018;35(3):199-206. https://doi.org//10.1053/j.semdp.2017.11.013
8. Pentheroudakis G, Golfinopoulos V, Pavlidis N. Switching benchmarks in cancer of unknown primary: from autopsy to microarray. Eur J Cancer. 2007;43(14):2026-2036. https://doi.org/10.1016/j.ejca.2007.06.023
9. Pavlidis N, Fizazi K. Carcinoma of unknown primary (CUP). Crit Rev Oncol Hematol. 2009;69(3):271-278. https://doi.org/10.1016/j.critrevonc.2008.09.005
10. Moller AK, Loft A, Berthelsen AK, et al. A prospective comparison of 18F-FDG PET/CT and CT as diagnostic tools to identify the primary tumor site in patients with extracervical carcinoma of unknown primary site. Oncologist. 2012;17(9):1146-1154. https://doi.org/10.1634/theoncologist.2011-0449
11. Economopoulou P, Mountzios G, Pavlidis N, Pentheroudakis G. Cancer of unknown primary origin in the genomic era: elucidating the dark box of cancer. Cancer Treat Rev. 2015;41(7):598-604. https://doi.org/10.1016/j.ctrv.2015.05.010
12. Greco FA. Molecular diagnosis of the tissue of origin in cancer of unknown primary site: useful in patient management. Curr Treat Options Oncol. 2013;14(4):634-642. https://doi.org/10.1007/s11864-013-0257-1
13. Massard C, Loriot Y, Fizazi K. Carcinomas of an unknown primary origin—diagnosis and treatment. Nat Rev Clin Oncol. 2011;8(12):701-710. https://doi.org/10.1038/nrclinonc.2011.158
14. Luoh SW, Flaherty KT. When tissue is no longer the issue: tissue-agnostic cancer therapy comes of age. Ann Intern Med. 2018;169(4):233-239. https://doi.org/10.7326/M17-2832

Issue
Journal of Hospital Medicine 16(8)
Issue
Journal of Hospital Medicine 16(8)
Page Number
502-506. Published Online First July 21, 2021
Page Number
502-506. Published Online First July 21, 2021
Publications
Publications
Topics
Article Type
Display Headline
Out of Sight, Not Out of Mind
Display Headline
Out of Sight, Not Out of Mind
Sections
Article Source

© 2021 Society of Hospital Medicine

Disallow All Ads
Correspondence Location
Michael A Santos, MD; Email: michael_santos2@brown.edu; Twitter: @masMD2171
Content Gating
Gated (full article locked unless allowed per User)
Alternative CME
Disqus Comments
Default
Use ProPublica
Hide sidebar & use full width
render the right sidebar.
Conference Recap Checkbox
Not Conference Recap
Clinical Edge
Display the Slideshow in this Article
Gating Strategy
First Page Free
Medscape Article
Display survey writer
Reuters content
Disable Inline Native ads
WebMD Article
Article PDF Media
Image
Teambase ID
18001D79.SIG
Disable zoom
Off

Supine-Related Pseudoanemia in Hospitalized Patients

Article Type
Changed
Tue, 03/30/2021 - 14:01
Display Headline
Supine-Related Pseudoanemia in Hospitalized Patients

The World Health Organization (WHO) defines anemia as a hemoglobin value less than 12 g/dL in women and less than 13 g/dL in men.1 Hospital-acquired anemia is loosely defined as normal hemoglobin levels on admission that, at their nadir during hospitalization or on discharge, are less than WHO sex-defined cutoffs. Hospital-acquired anemia or significant decreases in hemoglobin are often identified during hospitalization.2-6 Potential causes include blood loss from phlebotomy, occult gastrointestinal bleeding, hemolysis, anemia of inflammation, and hemodilution due to fluid resuscitation. Of these causes, some are dangerous to patients, some are iatrogenic, and some are due to laboratory error.7 Physicians often evaluate decreases in hemoglobin, which could otherwise be explained by laboratory error, hemodilution, or expected decrease in hemoglobin due to hospitalization, to identify causes that may lead to potential harm.

Jacob et al8 demonstrated the effect of posture on hemoglobin concentrations in healthy volunteers, showing an average 11% relative increase in hemoglobin when going from lying to standing. This increase was attributed to shifts in plasma volume to the vascular space with recumbence. They hypothesized that the initial hemoglobin on admission is measured when patients are upright or recently upright, whereas after admission, patients are more likely to be supine, resulting in lower hemoglobin concentrations. Others have also demonstrated similar effects of patient posture on hemoglobin concentration.9-13 However, these prior results are not readily generalizable to hospitalized patients. These prior studies enrolled healthy volunteers, and most examined postural changes from the supine and standing positions; blood is rarely obtained from hospitalized patients when they are standing.

The aim of this study was to investigate whether postural changes in hemoglobin can be demonstrated in positions that patients routinely encountered during in-hospital phlebotomy: upright in a chair or recumbent in a bed. Patient position, which is not standardized during blood draws, may contribute to lower measured hemoglobin concentrations in some patients, especially sicker individuals who are recumbent more frequently. We hypothesized that going from supine to upright in a chair would result in a relative increase in hemoglobin concentration of 5% to 6%, approximately half the value of going from supine to standing.8 To investigate this, we conducted a quasi-experimental study exploring the effect of position (supine or sitting in chair) on hemoglobin concentrations in medical inpatients.

METHODS

Participants

Patients were enrolled in this single-center study between October 2017 and August 2018. Patients aged 18 years or older who were hospitalized on the general internal medicine wards were screened to determine if they met the following inclusion criteria: hospitalized for <5 days, had blood work scheduled as part of routine care (in order to decrease phlebotomy required by this study), had baseline hemoglobin >8 g/dL, and were able to remain supine without interruption overnight and able to sit in a chair for at least 1 hour the following morning. Patients were excluded from the study if they had a hematologic malignancy, were at risk of >100 mL of blood loss (eg, admitted for gastrointestinal bleeding, planned surgery), had a transfusion requirement, or received intravascular modifiers such as fluid (>100 cc/h) or intravenous diuretics. The Johns Hopkins Institutional Review Board approved this study, and all patients provided written informed consent.

Study Design

Patients enrolled in this quasi-experimental study were asked to remain supine for at least 6 hours overnight. Adherence to the recumbent position was tracked by patient self-report and by corroboration with the patient’s nurse overnight. Any interruptions to supine positioning resulted in exclusion from the study. The following morning, a member of the study team performed phlebotomy while the patient remained supine. Patients were then asked to sit comfortably in a chair for at least 1 hour with their feet on the ground; the blood draw was then repeated. All blood samples were acquired by venipuncture. Prior to each blood draw, a tourniquet was placed over the upper arm below the axilla. An antecubital vein on either arm was visualized under ultrasound guidance, and a 23-G × 3/4” butterfly needle was used for venipuncture. The vials of blood were immediately inverted after blood collection. Hemoglobin assays were processed and analyzed using Sysmex XN-10 analyzer (Sysmex Corporation). The reference range for hemoglobin in our facility was 12.0 to 15.0 g/dL for women and 13.9 to 16.3 g/dL for men. Laboratory technicians were blinded to and uninvolved in the study.

We determined, a priori, that 33 enrolled patients would provide 80% power (alpha 0.05) to detect an average hemoglobin change of 4.1%, assuming that the standard deviation of the hemoglobin change was twice the mean (ie, SD = 8.2%). The Wilcoxon signed-rank test was used to test the significance of postural hemoglobin changes. Analyses were conducted using JMP Pro 13.0 (SAS) and GraphPad Prism 8 (GraphPad Software). Significance was defined at P < .05 for all analyses.

RESULTS

Thirty-nine patients were consented and enrolled in the study; four patients were excluded prior to blood draw (two patients because of interruption of supine time, two patients because of refusal in the morning). Of the 35 patients who completed the study, 13 were women (37%); median age was 49 years (range, 25-83 years). Median supine hemoglobin concentration in our sample was 11.7 g/dL (range, 9.3-18.1 g/dL), and median baseline creatinine level was 0.70 mg/dL (range, 0.5-2.5 mg/dL). Median supine hemoglobin levels were 11.7 g/dL (range, 9.6-13.2 g/dL) in women and 11.8 g/dL (range, 9.3-18.1 g/dL) in men. In aggregate, patients had a median increase in hemoglobin concentration of 0.60 g/dL (range, –0.6 to 1.4 g/dL) with sitting, a 5.2% (range, –4.5% to 15.1%) relative change (P < .001) (Figure 1).

derakhshan10470317_f1.jpg
Women had a median increase in hemoglobin concentration of 0.60 g/dL (range, –0.6 to 1.4 g/dL) with sitting, a relative change of 5.3% (range, –4.5% to 12.0%) (P = .02). Men had a median increase in hemoglobin concentration of 0.55 g/dL (range, –0.1 to 1.4 g/dL) with sitting, a 5.0% (range, –0.6% to 15.1%) relative change (P < .001). Ten of 35 participants (29%) exhibited an increase in hemoglobin level of 1.0 g/dL or more (Figure 2).
derakhshan10470317_f2.jpg

DISCUSSION

International blood collection guidelines acknowledge postural changes in laboratory values and recommend standardization of patient position to either sitting in a chair or lying flat in a bed, without changes in position for 15 minutes prior to blood draw.14 When these positional accommodations cannot be met, documenting positional disruptions is recommended so that laboratory values can be interpreted accordingly. To the best of our knowledge, no hospital in the United States has standardized patient position as part of phlebotomy procedure such that patient position is documented and can be made available to interpreting providers.

Relative increases in hemoglobin or hematocrit range from 7% to 12% when patients go from supine to standing.8,9,11 The reverse relationship has also been shown, where upright-to-supine position results in decreases in hemoglobin concentrations.10,13 We found that going from supine to a seated position resulted in significant increases in hemoglobin of 0.6 g/dL and in a more than 1 g/dL increase in 29% of the patients. Although four of the 35 patients experienced either no change or a slight decrease in their hemoglobin concentration when going from supine to upright and not all patients saw a uniform effect, providers should be aware that the patient’s position can contribute to changes in hemoglobin concentration in the hospitalized setting. Providers may be able to use this information to avoid an extensive diagnostic workup when anemia is identified in hospitalized patients, although more research is needed to identify patient subsets who are at higher risk for this effect.

Until hospitals implement protocols that require phlebotomists to report patient position during phlebotomy in a standardized fashion, providers should be alert to the fact that supine positioning may result in a hemoglobin level that is significantly lower than that when drawn in a sitting position, and in almost one-third of patients, this difference may be 1.0 g/dL or greater.

Given our study criteria requiring supine positions of at least 6 hours and a baseline hemoglobin concentration >8 g/dL, our sample of patients may have been younger and healthier than the average hospitalized patient on general internal medicine wards. Since greater relative changes in plasma volume shifts and hemoglobin might be seen in patients with lower baseline hemoglobin and lower baseline plasma protein, this selection bias may underestimate the effects of position on hemoglobin changes for the average inpatient population. Additionally, we intentionally sought to obtain sitting hemoglobin levels after the supine samples to avoid the possibility of incorrectly attributing dropping hemoglobin levels to progressive hospital-acquired anemia from phlebotomy or illness. Any concomitant trend of falling hemoglobin levels in our patients would be expected to lead to a systematic underestimation of the positional change in hemoglobin we observed. We did not objectively observe adherence to supine and upright position and instead relied on patient self-reporting, which is one possible contributor to the variable effects of position on hemoglobin concentration, with some patients having no change or decreases in hemoglobin concentrations.

CONCLUSION

Posture can significantly influence hemoglobin levels in hospitalized patients on general medicine wards. Further research can determine whether it would be cost and time effective to standardize patient positions prior to phlebotomy, or at least to report patient positioning with the laboratory testing results.

References

1. DeMaeyer E, Adiels-Tegman M. The prevalence of anaemia in the world. World Health Stat Q. 1985;38(3):302-316.
2. Martin ND, Scantling D. Hospital-acquired anemia. J Infus Nurs. 2015;38(5):330-338. https://doi.org/10.1097/NAN.0000000000000121
3. Thavendiranathan P, Bagai A, Ebidia A, Detsky AS, Choudhry NK. Do blood tests cause anemia in hospitalized patients? The effect of diagnostic phlebotomy on hemoglobin and hematocrit levels. J Gen Intern Med. 2005;20(6):520-524. https://doi.org/10.1111/j.1525-1497.2005.0094.x
4. Salisbury AC, Reid KJ, Alexander KP, et al. Diagnostic blood loss from phlebotomy and hospital-acquired anemia during acute myocardial infarction. Arch Intern Med. 2011;171(18):1646-1653. https://doi.org/10.1001/archinternmed.2011.361
5. Languasco A, Cazap N, Marciano S, et al. Hemoglobin concentration variations over time in general medical inpatients. J Hosp Med. 2010;5(5):283-288. https://doi.org/10.1002/jhm.650
6. van der Bom JG, Cannegieter SC. Hospital-acquired anemia: the contribution of diagnostic blood loss. J Thromb Haemost. 2015;13(6):1157-1159. https://doi.org/10.1111/jth.12886
7. Berkow L. Factors affecting hemoglobin measurement. J Clin Monit Comput. 2013;27(5):499-508. https://doi.org/10.1007/s10877-013-9456-3
8. Jacob G, Raj SR, Ketch T, et al. Postural pseudoanemia: posture-dependent change in hematocrit. Mayo Clin Proc. 2005;80(5):611-614. https://doi.org/10.4065/80.5.611
9. Fawcett JK, Wynn V. Effects of posture on plasma volume and some blood constituents. J Clin Pathol. 1960;13(4):304-310. https://doi.org/10.1136/jcp.13.4.304
10. Tombridge TL. Effect of posture on hematology results. Am J ClinPathol. 1968;49(4):491-493. https://doi.org/10.1093/ajcp/49.4.491
11. Hagan RD, Diaz FJ, Horvath SM. Plasma volume changes with movement to supine and standing positions. J Appl Physiol. 1978;45(3):414-417. https://doi.org/10.1152/jappl.1978.45.3.414
12. Maw GJ, Mackenzie IL, Taylor NA. Redistribution of body fluids during postural manipulations. Acta Physiol Scand. 1995;155(2):157-163. https://doi.org/10.1111/j.1748-1716.1995.tb09960.x
13. Lima-Oliveira G, Guidi GC, Salvagno GL, Danese E, Montagnana M, Lippi G. Patient posture for blood collection by venipuncture: recall for standardization after 28 years. Rev Bras Hematol Hemoter. 2017;39(2):127-132. https://doi.org/10.1016/j.bjhh.2017.01.004
14. Simundic AM, Bölenius K, Cadamuro J, et al. Working Group for Preanalytical Phase (WG-PRE), of the European Federation of Clinical Chemistry and Laboratory Medicine (EFLM) and Latin American Working Group for Preanalytical Phase (WG-PRE-LATAM) of the Latin America Confederation of Clinical Biochemistry (COLABIOCLI). Joint EFLM-COLABIOCLI recommendation for venous blood sampling. Clin Chem Lab Med. 2018;56(12):2015-2038. https://doi.org/10.1515/cclm-2018-0602

Article PDF
Author and Disclosure Information

1Department of Internal Medicine, Case Western Reserve University School of Medicine, University Hospital Cleveland Medical Center, Cleveland, Ohio; 2Department of Internal Medicine, Johns Hopkins University School of Medicine, Baltimore, Maryland; 3Department of Internal Medicine, Saint Joseph’s Medical Center, Towson, Maryland; 4Division of Cardiology, Department of Medicine, University of South Florida, Morsani College of Medicine, Tampa, Florida; 5Department of Pathology, Johns Hopkins University School of Medicine, Baltimore, Maryland; 6Department of Medicine, Johns Hopkins University School of Medicine, Baltimore, Maryland.

Disclosures

The authors have no financial relationships or conflicts of interest relevant to this article to disclose.

Issue
Journal of Hospital Medicine 16(4)
Publications
Topics
Page Number
219-222. Published Online First March 17, 2021
Sections
Author and Disclosure Information

1Department of Internal Medicine, Case Western Reserve University School of Medicine, University Hospital Cleveland Medical Center, Cleveland, Ohio; 2Department of Internal Medicine, Johns Hopkins University School of Medicine, Baltimore, Maryland; 3Department of Internal Medicine, Saint Joseph’s Medical Center, Towson, Maryland; 4Division of Cardiology, Department of Medicine, University of South Florida, Morsani College of Medicine, Tampa, Florida; 5Department of Pathology, Johns Hopkins University School of Medicine, Baltimore, Maryland; 6Department of Medicine, Johns Hopkins University School of Medicine, Baltimore, Maryland.

Disclosures

The authors have no financial relationships or conflicts of interest relevant to this article to disclose.

Author and Disclosure Information

1Department of Internal Medicine, Case Western Reserve University School of Medicine, University Hospital Cleveland Medical Center, Cleveland, Ohio; 2Department of Internal Medicine, Johns Hopkins University School of Medicine, Baltimore, Maryland; 3Department of Internal Medicine, Saint Joseph’s Medical Center, Towson, Maryland; 4Division of Cardiology, Department of Medicine, University of South Florida, Morsani College of Medicine, Tampa, Florida; 5Department of Pathology, Johns Hopkins University School of Medicine, Baltimore, Maryland; 6Department of Medicine, Johns Hopkins University School of Medicine, Baltimore, Maryland.

Disclosures

The authors have no financial relationships or conflicts of interest relevant to this article to disclose.

Article PDF
Article PDF
Related Articles

The World Health Organization (WHO) defines anemia as a hemoglobin value less than 12 g/dL in women and less than 13 g/dL in men.1 Hospital-acquired anemia is loosely defined as normal hemoglobin levels on admission that, at their nadir during hospitalization or on discharge, are less than WHO sex-defined cutoffs. Hospital-acquired anemia or significant decreases in hemoglobin are often identified during hospitalization.2-6 Potential causes include blood loss from phlebotomy, occult gastrointestinal bleeding, hemolysis, anemia of inflammation, and hemodilution due to fluid resuscitation. Of these causes, some are dangerous to patients, some are iatrogenic, and some are due to laboratory error.7 Physicians often evaluate decreases in hemoglobin, which could otherwise be explained by laboratory error, hemodilution, or expected decrease in hemoglobin due to hospitalization, to identify causes that may lead to potential harm.

Jacob et al8 demonstrated the effect of posture on hemoglobin concentrations in healthy volunteers, showing an average 11% relative increase in hemoglobin when going from lying to standing. This increase was attributed to shifts in plasma volume to the vascular space with recumbence. They hypothesized that the initial hemoglobin on admission is measured when patients are upright or recently upright, whereas after admission, patients are more likely to be supine, resulting in lower hemoglobin concentrations. Others have also demonstrated similar effects of patient posture on hemoglobin concentration.9-13 However, these prior results are not readily generalizable to hospitalized patients. These prior studies enrolled healthy volunteers, and most examined postural changes from the supine and standing positions; blood is rarely obtained from hospitalized patients when they are standing.

The aim of this study was to investigate whether postural changes in hemoglobin can be demonstrated in positions that patients routinely encountered during in-hospital phlebotomy: upright in a chair or recumbent in a bed. Patient position, which is not standardized during blood draws, may contribute to lower measured hemoglobin concentrations in some patients, especially sicker individuals who are recumbent more frequently. We hypothesized that going from supine to upright in a chair would result in a relative increase in hemoglobin concentration of 5% to 6%, approximately half the value of going from supine to standing.8 To investigate this, we conducted a quasi-experimental study exploring the effect of position (supine or sitting in chair) on hemoglobin concentrations in medical inpatients.

METHODS

Participants

Patients were enrolled in this single-center study between October 2017 and August 2018. Patients aged 18 years or older who were hospitalized on the general internal medicine wards were screened to determine if they met the following inclusion criteria: hospitalized for <5 days, had blood work scheduled as part of routine care (in order to decrease phlebotomy required by this study), had baseline hemoglobin >8 g/dL, and were able to remain supine without interruption overnight and able to sit in a chair for at least 1 hour the following morning. Patients were excluded from the study if they had a hematologic malignancy, were at risk of >100 mL of blood loss (eg, admitted for gastrointestinal bleeding, planned surgery), had a transfusion requirement, or received intravascular modifiers such as fluid (>100 cc/h) or intravenous diuretics. The Johns Hopkins Institutional Review Board approved this study, and all patients provided written informed consent.

Study Design

Patients enrolled in this quasi-experimental study were asked to remain supine for at least 6 hours overnight. Adherence to the recumbent position was tracked by patient self-report and by corroboration with the patient’s nurse overnight. Any interruptions to supine positioning resulted in exclusion from the study. The following morning, a member of the study team performed phlebotomy while the patient remained supine. Patients were then asked to sit comfortably in a chair for at least 1 hour with their feet on the ground; the blood draw was then repeated. All blood samples were acquired by venipuncture. Prior to each blood draw, a tourniquet was placed over the upper arm below the axilla. An antecubital vein on either arm was visualized under ultrasound guidance, and a 23-G × 3/4” butterfly needle was used for venipuncture. The vials of blood were immediately inverted after blood collection. Hemoglobin assays were processed and analyzed using Sysmex XN-10 analyzer (Sysmex Corporation). The reference range for hemoglobin in our facility was 12.0 to 15.0 g/dL for women and 13.9 to 16.3 g/dL for men. Laboratory technicians were blinded to and uninvolved in the study.

We determined, a priori, that 33 enrolled patients would provide 80% power (alpha 0.05) to detect an average hemoglobin change of 4.1%, assuming that the standard deviation of the hemoglobin change was twice the mean (ie, SD = 8.2%). The Wilcoxon signed-rank test was used to test the significance of postural hemoglobin changes. Analyses were conducted using JMP Pro 13.0 (SAS) and GraphPad Prism 8 (GraphPad Software). Significance was defined at P < .05 for all analyses.

RESULTS

Thirty-nine patients were consented and enrolled in the study; four patients were excluded prior to blood draw (two patients because of interruption of supine time, two patients because of refusal in the morning). Of the 35 patients who completed the study, 13 were women (37%); median age was 49 years (range, 25-83 years). Median supine hemoglobin concentration in our sample was 11.7 g/dL (range, 9.3-18.1 g/dL), and median baseline creatinine level was 0.70 mg/dL (range, 0.5-2.5 mg/dL). Median supine hemoglobin levels were 11.7 g/dL (range, 9.6-13.2 g/dL) in women and 11.8 g/dL (range, 9.3-18.1 g/dL) in men. In aggregate, patients had a median increase in hemoglobin concentration of 0.60 g/dL (range, –0.6 to 1.4 g/dL) with sitting, a 5.2% (range, –4.5% to 15.1%) relative change (P < .001) (Figure 1).

derakhshan10470317_f1.jpg
Women had a median increase in hemoglobin concentration of 0.60 g/dL (range, –0.6 to 1.4 g/dL) with sitting, a relative change of 5.3% (range, –4.5% to 12.0%) (P = .02). Men had a median increase in hemoglobin concentration of 0.55 g/dL (range, –0.1 to 1.4 g/dL) with sitting, a 5.0% (range, –0.6% to 15.1%) relative change (P < .001). Ten of 35 participants (29%) exhibited an increase in hemoglobin level of 1.0 g/dL or more (Figure 2).
derakhshan10470317_f2.jpg

DISCUSSION

International blood collection guidelines acknowledge postural changes in laboratory values and recommend standardization of patient position to either sitting in a chair or lying flat in a bed, without changes in position for 15 minutes prior to blood draw.14 When these positional accommodations cannot be met, documenting positional disruptions is recommended so that laboratory values can be interpreted accordingly. To the best of our knowledge, no hospital in the United States has standardized patient position as part of phlebotomy procedure such that patient position is documented and can be made available to interpreting providers.

Relative increases in hemoglobin or hematocrit range from 7% to 12% when patients go from supine to standing.8,9,11 The reverse relationship has also been shown, where upright-to-supine position results in decreases in hemoglobin concentrations.10,13 We found that going from supine to a seated position resulted in significant increases in hemoglobin of 0.6 g/dL and in a more than 1 g/dL increase in 29% of the patients. Although four of the 35 patients experienced either no change or a slight decrease in their hemoglobin concentration when going from supine to upright and not all patients saw a uniform effect, providers should be aware that the patient’s position can contribute to changes in hemoglobin concentration in the hospitalized setting. Providers may be able to use this information to avoid an extensive diagnostic workup when anemia is identified in hospitalized patients, although more research is needed to identify patient subsets who are at higher risk for this effect.

Until hospitals implement protocols that require phlebotomists to report patient position during phlebotomy in a standardized fashion, providers should be alert to the fact that supine positioning may result in a hemoglobin level that is significantly lower than that when drawn in a sitting position, and in almost one-third of patients, this difference may be 1.0 g/dL or greater.

Given our study criteria requiring supine positions of at least 6 hours and a baseline hemoglobin concentration >8 g/dL, our sample of patients may have been younger and healthier than the average hospitalized patient on general internal medicine wards. Since greater relative changes in plasma volume shifts and hemoglobin might be seen in patients with lower baseline hemoglobin and lower baseline plasma protein, this selection bias may underestimate the effects of position on hemoglobin changes for the average inpatient population. Additionally, we intentionally sought to obtain sitting hemoglobin levels after the supine samples to avoid the possibility of incorrectly attributing dropping hemoglobin levels to progressive hospital-acquired anemia from phlebotomy or illness. Any concomitant trend of falling hemoglobin levels in our patients would be expected to lead to a systematic underestimation of the positional change in hemoglobin we observed. We did not objectively observe adherence to supine and upright position and instead relied on patient self-reporting, which is one possible contributor to the variable effects of position on hemoglobin concentration, with some patients having no change or decreases in hemoglobin concentrations.

CONCLUSION

Posture can significantly influence hemoglobin levels in hospitalized patients on general medicine wards. Further research can determine whether it would be cost and time effective to standardize patient positions prior to phlebotomy, or at least to report patient positioning with the laboratory testing results.

The World Health Organization (WHO) defines anemia as a hemoglobin value less than 12 g/dL in women and less than 13 g/dL in men.1 Hospital-acquired anemia is loosely defined as normal hemoglobin levels on admission that, at their nadir during hospitalization or on discharge, are less than WHO sex-defined cutoffs. Hospital-acquired anemia or significant decreases in hemoglobin are often identified during hospitalization.2-6 Potential causes include blood loss from phlebotomy, occult gastrointestinal bleeding, hemolysis, anemia of inflammation, and hemodilution due to fluid resuscitation. Of these causes, some are dangerous to patients, some are iatrogenic, and some are due to laboratory error.7 Physicians often evaluate decreases in hemoglobin, which could otherwise be explained by laboratory error, hemodilution, or expected decrease in hemoglobin due to hospitalization, to identify causes that may lead to potential harm.

Jacob et al8 demonstrated the effect of posture on hemoglobin concentrations in healthy volunteers, showing an average 11% relative increase in hemoglobin when going from lying to standing. This increase was attributed to shifts in plasma volume to the vascular space with recumbence. They hypothesized that the initial hemoglobin on admission is measured when patients are upright or recently upright, whereas after admission, patients are more likely to be supine, resulting in lower hemoglobin concentrations. Others have also demonstrated similar effects of patient posture on hemoglobin concentration.9-13 However, these prior results are not readily generalizable to hospitalized patients. These prior studies enrolled healthy volunteers, and most examined postural changes from the supine and standing positions; blood is rarely obtained from hospitalized patients when they are standing.

The aim of this study was to investigate whether postural changes in hemoglobin can be demonstrated in positions that patients routinely encountered during in-hospital phlebotomy: upright in a chair or recumbent in a bed. Patient position, which is not standardized during blood draws, may contribute to lower measured hemoglobin concentrations in some patients, especially sicker individuals who are recumbent more frequently. We hypothesized that going from supine to upright in a chair would result in a relative increase in hemoglobin concentration of 5% to 6%, approximately half the value of going from supine to standing.8 To investigate this, we conducted a quasi-experimental study exploring the effect of position (supine or sitting in chair) on hemoglobin concentrations in medical inpatients.

METHODS

Participants

Patients were enrolled in this single-center study between October 2017 and August 2018. Patients aged 18 years or older who were hospitalized on the general internal medicine wards were screened to determine if they met the following inclusion criteria: hospitalized for <5 days, had blood work scheduled as part of routine care (in order to decrease phlebotomy required by this study), had baseline hemoglobin >8 g/dL, and were able to remain supine without interruption overnight and able to sit in a chair for at least 1 hour the following morning. Patients were excluded from the study if they had a hematologic malignancy, were at risk of >100 mL of blood loss (eg, admitted for gastrointestinal bleeding, planned surgery), had a transfusion requirement, or received intravascular modifiers such as fluid (>100 cc/h) or intravenous diuretics. The Johns Hopkins Institutional Review Board approved this study, and all patients provided written informed consent.

Study Design

Patients enrolled in this quasi-experimental study were asked to remain supine for at least 6 hours overnight. Adherence to the recumbent position was tracked by patient self-report and by corroboration with the patient’s nurse overnight. Any interruptions to supine positioning resulted in exclusion from the study. The following morning, a member of the study team performed phlebotomy while the patient remained supine. Patients were then asked to sit comfortably in a chair for at least 1 hour with their feet on the ground; the blood draw was then repeated. All blood samples were acquired by venipuncture. Prior to each blood draw, a tourniquet was placed over the upper arm below the axilla. An antecubital vein on either arm was visualized under ultrasound guidance, and a 23-G × 3/4” butterfly needle was used for venipuncture. The vials of blood were immediately inverted after blood collection. Hemoglobin assays were processed and analyzed using Sysmex XN-10 analyzer (Sysmex Corporation). The reference range for hemoglobin in our facility was 12.0 to 15.0 g/dL for women and 13.9 to 16.3 g/dL for men. Laboratory technicians were blinded to and uninvolved in the study.

We determined, a priori, that 33 enrolled patients would provide 80% power (alpha 0.05) to detect an average hemoglobin change of 4.1%, assuming that the standard deviation of the hemoglobin change was twice the mean (ie, SD = 8.2%). The Wilcoxon signed-rank test was used to test the significance of postural hemoglobin changes. Analyses were conducted using JMP Pro 13.0 (SAS) and GraphPad Prism 8 (GraphPad Software). Significance was defined at P < .05 for all analyses.

RESULTS

Thirty-nine patients were consented and enrolled in the study; four patients were excluded prior to blood draw (two patients because of interruption of supine time, two patients because of refusal in the morning). Of the 35 patients who completed the study, 13 were women (37%); median age was 49 years (range, 25-83 years). Median supine hemoglobin concentration in our sample was 11.7 g/dL (range, 9.3-18.1 g/dL), and median baseline creatinine level was 0.70 mg/dL (range, 0.5-2.5 mg/dL). Median supine hemoglobin levels were 11.7 g/dL (range, 9.6-13.2 g/dL) in women and 11.8 g/dL (range, 9.3-18.1 g/dL) in men. In aggregate, patients had a median increase in hemoglobin concentration of 0.60 g/dL (range, –0.6 to 1.4 g/dL) with sitting, a 5.2% (range, –4.5% to 15.1%) relative change (P < .001) (Figure 1).

derakhshan10470317_f1.jpg
Women had a median increase in hemoglobin concentration of 0.60 g/dL (range, –0.6 to 1.4 g/dL) with sitting, a relative change of 5.3% (range, –4.5% to 12.0%) (P = .02). Men had a median increase in hemoglobin concentration of 0.55 g/dL (range, –0.1 to 1.4 g/dL) with sitting, a 5.0% (range, –0.6% to 15.1%) relative change (P < .001). Ten of 35 participants (29%) exhibited an increase in hemoglobin level of 1.0 g/dL or more (Figure 2).
derakhshan10470317_f2.jpg

DISCUSSION

International blood collection guidelines acknowledge postural changes in laboratory values and recommend standardization of patient position to either sitting in a chair or lying flat in a bed, without changes in position for 15 minutes prior to blood draw.14 When these positional accommodations cannot be met, documenting positional disruptions is recommended so that laboratory values can be interpreted accordingly. To the best of our knowledge, no hospital in the United States has standardized patient position as part of phlebotomy procedure such that patient position is documented and can be made available to interpreting providers.

Relative increases in hemoglobin or hematocrit range from 7% to 12% when patients go from supine to standing.8,9,11 The reverse relationship has also been shown, where upright-to-supine position results in decreases in hemoglobin concentrations.10,13 We found that going from supine to a seated position resulted in significant increases in hemoglobin of 0.6 g/dL and in a more than 1 g/dL increase in 29% of the patients. Although four of the 35 patients experienced either no change or a slight decrease in their hemoglobin concentration when going from supine to upright and not all patients saw a uniform effect, providers should be aware that the patient’s position can contribute to changes in hemoglobin concentration in the hospitalized setting. Providers may be able to use this information to avoid an extensive diagnostic workup when anemia is identified in hospitalized patients, although more research is needed to identify patient subsets who are at higher risk for this effect.

Until hospitals implement protocols that require phlebotomists to report patient position during phlebotomy in a standardized fashion, providers should be alert to the fact that supine positioning may result in a hemoglobin level that is significantly lower than that when drawn in a sitting position, and in almost one-third of patients, this difference may be 1.0 g/dL or greater.

Given our study criteria requiring supine positions of at least 6 hours and a baseline hemoglobin concentration >8 g/dL, our sample of patients may have been younger and healthier than the average hospitalized patient on general internal medicine wards. Since greater relative changes in plasma volume shifts and hemoglobin might be seen in patients with lower baseline hemoglobin and lower baseline plasma protein, this selection bias may underestimate the effects of position on hemoglobin changes for the average inpatient population. Additionally, we intentionally sought to obtain sitting hemoglobin levels after the supine samples to avoid the possibility of incorrectly attributing dropping hemoglobin levels to progressive hospital-acquired anemia from phlebotomy or illness. Any concomitant trend of falling hemoglobin levels in our patients would be expected to lead to a systematic underestimation of the positional change in hemoglobin we observed. We did not objectively observe adherence to supine and upright position and instead relied on patient self-reporting, which is one possible contributor to the variable effects of position on hemoglobin concentration, with some patients having no change or decreases in hemoglobin concentrations.

CONCLUSION

Posture can significantly influence hemoglobin levels in hospitalized patients on general medicine wards. Further research can determine whether it would be cost and time effective to standardize patient positions prior to phlebotomy, or at least to report patient positioning with the laboratory testing results.

References

1. DeMaeyer E, Adiels-Tegman M. The prevalence of anaemia in the world. World Health Stat Q. 1985;38(3):302-316.
2. Martin ND, Scantling D. Hospital-acquired anemia. J Infus Nurs. 2015;38(5):330-338. https://doi.org/10.1097/NAN.0000000000000121
3. Thavendiranathan P, Bagai A, Ebidia A, Detsky AS, Choudhry NK. Do blood tests cause anemia in hospitalized patients? The effect of diagnostic phlebotomy on hemoglobin and hematocrit levels. J Gen Intern Med. 2005;20(6):520-524. https://doi.org/10.1111/j.1525-1497.2005.0094.x
4. Salisbury AC, Reid KJ, Alexander KP, et al. Diagnostic blood loss from phlebotomy and hospital-acquired anemia during acute myocardial infarction. Arch Intern Med. 2011;171(18):1646-1653. https://doi.org/10.1001/archinternmed.2011.361
5. Languasco A, Cazap N, Marciano S, et al. Hemoglobin concentration variations over time in general medical inpatients. J Hosp Med. 2010;5(5):283-288. https://doi.org/10.1002/jhm.650
6. van der Bom JG, Cannegieter SC. Hospital-acquired anemia: the contribution of diagnostic blood loss. J Thromb Haemost. 2015;13(6):1157-1159. https://doi.org/10.1111/jth.12886
7. Berkow L. Factors affecting hemoglobin measurement. J Clin Monit Comput. 2013;27(5):499-508. https://doi.org/10.1007/s10877-013-9456-3
8. Jacob G, Raj SR, Ketch T, et al. Postural pseudoanemia: posture-dependent change in hematocrit. Mayo Clin Proc. 2005;80(5):611-614. https://doi.org/10.4065/80.5.611
9. Fawcett JK, Wynn V. Effects of posture on plasma volume and some blood constituents. J Clin Pathol. 1960;13(4):304-310. https://doi.org/10.1136/jcp.13.4.304
10. Tombridge TL. Effect of posture on hematology results. Am J ClinPathol. 1968;49(4):491-493. https://doi.org/10.1093/ajcp/49.4.491
11. Hagan RD, Diaz FJ, Horvath SM. Plasma volume changes with movement to supine and standing positions. J Appl Physiol. 1978;45(3):414-417. https://doi.org/10.1152/jappl.1978.45.3.414
12. Maw GJ, Mackenzie IL, Taylor NA. Redistribution of body fluids during postural manipulations. Acta Physiol Scand. 1995;155(2):157-163. https://doi.org/10.1111/j.1748-1716.1995.tb09960.x
13. Lima-Oliveira G, Guidi GC, Salvagno GL, Danese E, Montagnana M, Lippi G. Patient posture for blood collection by venipuncture: recall for standardization after 28 years. Rev Bras Hematol Hemoter. 2017;39(2):127-132. https://doi.org/10.1016/j.bjhh.2017.01.004
14. Simundic AM, Bölenius K, Cadamuro J, et al. Working Group for Preanalytical Phase (WG-PRE), of the European Federation of Clinical Chemistry and Laboratory Medicine (EFLM) and Latin American Working Group for Preanalytical Phase (WG-PRE-LATAM) of the Latin America Confederation of Clinical Biochemistry (COLABIOCLI). Joint EFLM-COLABIOCLI recommendation for venous blood sampling. Clin Chem Lab Med. 2018;56(12):2015-2038. https://doi.org/10.1515/cclm-2018-0602

References

1. DeMaeyer E, Adiels-Tegman M. The prevalence of anaemia in the world. World Health Stat Q. 1985;38(3):302-316.
2. Martin ND, Scantling D. Hospital-acquired anemia. J Infus Nurs. 2015;38(5):330-338. https://doi.org/10.1097/NAN.0000000000000121
3. Thavendiranathan P, Bagai A, Ebidia A, Detsky AS, Choudhry NK. Do blood tests cause anemia in hospitalized patients? The effect of diagnostic phlebotomy on hemoglobin and hematocrit levels. J Gen Intern Med. 2005;20(6):520-524. https://doi.org/10.1111/j.1525-1497.2005.0094.x
4. Salisbury AC, Reid KJ, Alexander KP, et al. Diagnostic blood loss from phlebotomy and hospital-acquired anemia during acute myocardial infarction. Arch Intern Med. 2011;171(18):1646-1653. https://doi.org/10.1001/archinternmed.2011.361
5. Languasco A, Cazap N, Marciano S, et al. Hemoglobin concentration variations over time in general medical inpatients. J Hosp Med. 2010;5(5):283-288. https://doi.org/10.1002/jhm.650
6. van der Bom JG, Cannegieter SC. Hospital-acquired anemia: the contribution of diagnostic blood loss. J Thromb Haemost. 2015;13(6):1157-1159. https://doi.org/10.1111/jth.12886
7. Berkow L. Factors affecting hemoglobin measurement. J Clin Monit Comput. 2013;27(5):499-508. https://doi.org/10.1007/s10877-013-9456-3
8. Jacob G, Raj SR, Ketch T, et al. Postural pseudoanemia: posture-dependent change in hematocrit. Mayo Clin Proc. 2005;80(5):611-614. https://doi.org/10.4065/80.5.611
9. Fawcett JK, Wynn V. Effects of posture on plasma volume and some blood constituents. J Clin Pathol. 1960;13(4):304-310. https://doi.org/10.1136/jcp.13.4.304
10. Tombridge TL. Effect of posture on hematology results. Am J ClinPathol. 1968;49(4):491-493. https://doi.org/10.1093/ajcp/49.4.491
11. Hagan RD, Diaz FJ, Horvath SM. Plasma volume changes with movement to supine and standing positions. J Appl Physiol. 1978;45(3):414-417. https://doi.org/10.1152/jappl.1978.45.3.414
12. Maw GJ, Mackenzie IL, Taylor NA. Redistribution of body fluids during postural manipulations. Acta Physiol Scand. 1995;155(2):157-163. https://doi.org/10.1111/j.1748-1716.1995.tb09960.x
13. Lima-Oliveira G, Guidi GC, Salvagno GL, Danese E, Montagnana M, Lippi G. Patient posture for blood collection by venipuncture: recall for standardization after 28 years. Rev Bras Hematol Hemoter. 2017;39(2):127-132. https://doi.org/10.1016/j.bjhh.2017.01.004
14. Simundic AM, Bölenius K, Cadamuro J, et al. Working Group for Preanalytical Phase (WG-PRE), of the European Federation of Clinical Chemistry and Laboratory Medicine (EFLM) and Latin American Working Group for Preanalytical Phase (WG-PRE-LATAM) of the Latin America Confederation of Clinical Biochemistry (COLABIOCLI). Joint EFLM-COLABIOCLI recommendation for venous blood sampling. Clin Chem Lab Med. 2018;56(12):2015-2038. https://doi.org/10.1515/cclm-2018-0602

Issue
Journal of Hospital Medicine 16(4)
Issue
Journal of Hospital Medicine 16(4)
Page Number
219-222. Published Online First March 17, 2021
Page Number
219-222. Published Online First March 17, 2021
Publications
Publications
Topics
Article Type
Display Headline
Supine-Related Pseudoanemia in Hospitalized Patients
Display Headline
Supine-Related Pseudoanemia in Hospitalized Patients
Sections
Article Source

© 2021 Society of Hospital Medicine

Disallow All Ads
Correspondence Location
Arsalan Derakhshan, MD; Email: Arsalan.Derakhshan@UHhospitals.org; Twitter: @ArsalanMedEd.
Content Gating
Gated (full article locked unless allowed per User)
Alternative CME
Disqus Comments
Default
Use ProPublica
Hide sidebar & use full width
render the right sidebar.
Conference Recap Checkbox
Not Conference Recap
Clinical Edge
Display the Slideshow in this Article
Gating Strategy
First Page Free
Medscape Article
Display survey writer
Reuters content
Article PDF Media

More Is Less

Article Type
Changed
Tue, 03/30/2021 - 14:13

A 64-year-old man presented with a 2-month history of a nonproductive cough, weight loss, and subjective fevers. He had no chest pain, hemoptysis, or shortness of breath. He also described worsening anorexia and a 15-pound weight loss over the previous 3 months. He had no arthralgias, myalgias, abdominal pain, nausea, emesis, or diarrhea.

Two weeks prior to his presentation, he was diagnosed with pneumonia and given a 5-day course of azithromycin. His symptoms did not improve, so he presented to the emergency room. 

He had not been seen regularly by a physician in decades and had no known medical conditions. He did not take any medications. He immigrated from China 3 years prior and lived with his wife in California. He had a 30 pack-year smoking history. He drank a shot glass of liquor daily and denied any drug use.

Weight loss might result from inflammatory disorders like cancer or noninflammatory causes such as decreased oral intake (eg, diminished appetite) or malabsorption (eg, celiac disease). However, his fevers suggest inflammation, which usually reflects an underlying infection, cancer, or autoimmune process. While chronic cough typically results from upper airway cough syndrome (allergic or nonallergic rhinitis), gastroesophageal reflux disease, or asthma, it can also point to pathology of the lung, which may be intrinsic (bronchiectasis) or extrinsic (mediastinal mass). The duration of 2 months makes a typical infectious process like pneumococcal pneumonia unlikely. Atypical infections such as tuberculosis, melioidosis, and talaromycosis are possible given his immigration from East Asia, and coccidioidomycosis given his residence in California. He might have undiagnosed medical conditions, such as diabetes, that could be relevant to his current presentation and classify him as immunocompromised. His smoking history prompts consideration of lung cancer.

His temperature was 36.5 oC, heart rate 70 beats per minute, blood pressure 118/66 mm Hg, respiratory rate 16 breaths per minute, oxygen saturation 98% on room air, and body mass index 23 kg/m2. He was in no acute distress. The findings from the cardiac, lung, abdominal, and neurological exams were normal.

Skin examination found a fixed, symmetric, 5-cm, firm nodule at top of sternum (Figure 1A). In addition, he had two 1-cm, mobile, firm, subcutaneous nodules, one on his anterior left chest and another underneath his right axilla. He also had two 2-cm, erythematous, tender nodules on his left anterior forearm and a 1-cm nodule with a central black plug on the dorsal surface of his right hand (Figure 1B). He did not have any edema.

chan02320923e_f1.jpg

The white blood cell count was 10,500/mm3 (42% neutrophils, 37% lymphocytes, 16.4% monocytes, and 2.9% eosinophils), hemoglobin was 12.2 g/dL with a mean corpuscular volume of 91 fL, and the platelet count was 441,000/mm3. Basic metabolic panel, aminotransferase, bilirubin, and alkaline phosphatase were within reference ranges. Serum albumin was 3.1 g/dL. Serum total protein was elevated at 8.8 g/dL. Serum calcium was 9.0 mg/dL. Urinalysis results were normal.

The slightly low albumin, mildly elevated platelet count, monocytosis, and normocytic anemia suggest inflammation, although monocytosis might represent a hematologic malignancy like chronic myelomonocytic leukemia (CMML). His subjective fevers and weight loss further corroborate underlying inflammation. What is driving the inflammation? There are two localizing findings: cough and nodular skin lesions.

His lack of dyspnea and normal oxygen saturation, respiratory rate, and lung exam make an extrapulmonary cause of cough such as lymphadenopathy or mediastinal infection possible. The number of nodular skin lesions, wide-spread distribution, and appearance (eg, erythematous, tender) point to either a primary cutaneous disease with systemic manifestations (eg, cutaneous lymphoma) or a systemic disease with cutaneous features (eg, sarcoidosis).

Three categories—inflammatory, infectious, and neoplastic—account for most nodular skin lesions. Usually microscopic evaluation is necessary for definitive diagnosis, though epidemiology, associated symptoms, and characteristics of the nodules help prioritize the differential diagnosis. Tender nodules might reflect a panniculitis; erythema nodosum is the most common type, and while this classically develops on the anterior shins, it may also occur on the forearm. His immigration from China prompts consideration of tuberculosis and cutaneous leishmaniasis. Coccidioidomycosis can lead to inflammation and nodular skin lesions. Other infections such as nontuberculous mycobacteria, nocardiosis, and cryptococcosis may cause disseminated infection with pulmonary and skin manifestations. His smoking puts him at risk of lung cancer, which rarely results in metastatic subcutaneous infiltrates.

A chest radiograph demonstrated a prominent density in the right paratracheal region of the mediastinum with adjacent streaky opacities. A computed tomography scan of the chest with intravenous contrast demonstrated centrilobular emphysematous changes and revealed a 2.6 × 4.7-cm necrotic mass in the anterior chest wall with erosion into the manubrium, a 3.8 × 2.1-cm centrally necrotic soft-tissue mass in the right hilum, a 5-mm left upper-lobe noncalcified solid pulmonary nodule, and prominent subcarinal, paratracheal, hilar, and bilateral supraclavicular lymphadenopathy (Figure 2). 

chan02320923e_f2.jpg

Flow cytometry of the peripheral blood did not demonstrate a lymphoproliferative disorder. Blood smear demonstrated normal red blood cell, white blood cell, and platelet morphology. HIV antibody was negative. Hemoglobin A1c was 6.1%. Smear microscopy for acid-fast bacilli (AFB) was negative and sputum AFB samples were sent for culture. Bacterial, fungal, and AFB blood cultures were collected and pending. 

Causes of necrotizing pneumonia include liquid (eg, lymphoma) and solid (eg, squamous cell carcinoma) cancers, infections, and noninfectious inflammatory processes such as granulomatosis with polyangiitis (GPA). Given his subacute presentation and extrapulmonary cutaneous manifestations, consideration of mycobacteria, fungi (eg, Coccidioides, Aspergillus, and Cryptococcus), and filamentous bacteria (eg, Nocardia and Actinomyces) is prioritized among the myriad of infections that can cause a lung cavity. His smoking history and centrilobular emphysematous changes are highly suggestive of chronic obstructive pulmonary disease, which puts him at increased risk of bacterial colonization and recurrent pulmonary infections. Tuberculosis is still possible despite three negative AFB-sputa smears given the sensitivity of smear microscopy (with three specimens) is roughly 70% in an immunocompetent host.

The lymphadenopathy likely reflects spread from the necrotic lung mass. The frequency of non-Hodgkin lymphoma increases with age. The results of the peripheral flow cytometry do not exclude the possibility of an aggressive lymphoma with pulmonary and cutaneous manifestations.

The erosive property of the chest wall mass makes an autoimmune process like GPA unlikely. An aggressive and disseminated infection or cancer is most likely. A pathologic process that originated in the lung and then spread to the lymph nodes and skin is more likely than a disorder which started in the skin. It would be unlikely for a primary cutaneous disorder to cause such a well-defined necrotic lung mass. Lung cancer rarely metastasizes to the skin and, instead, preferentially involves the chest. Ultimately, ascertaining what the patient experienced first (ie, respiratory or cutaneous symptoms) will determine where the pathology originated.

Computed tomography scan of the abdomen and pelvis with intravenous contrast demonstrated multiple ill-­defined lytic lesions in the pelvis, including a 12-mm lesion of the left sacral ala and multiple subcentimeter lesions in the medial left iliac bone and superior right acetabulum. In addition, there were two 1-cm, rim-enhancing, hypodense nodules in the subcutaneous fat of the right flank at the level of L5 and the left lower quadrant, respectively. There was also a 2.2 × 1.9-cm faintly rim-enhancing hypodensity within the left iliopsoas muscle belly.

These imaging findings further corroborate a widely metastatic process probably originating in the lung and spreading to the lymph nodes, skin, muscles, and bones. The characterization of lesions as lytic as opposed to blastic is less helpful because many diseases can cause both. It does prompt consideration of multiple myeloma; however, multiple myeloma less commonly manifests with extramedullary plasmacytomas and is less likely given his normal renal function and calcium level. Bone lesions lessen the likelihood of GPA, and his necrotic lung mass makes sarcoidosis unlikely. Atypical infections and cancers are the prime suspect of his multisystemic disease.

There are no data yet to suggest a weakened immune system, which would increase his risk for atypical infections. His chronic lung disease, identified on imaging, is a risk factor for nocardiosis. This gram-positive, weakly acid-fast bacterium can involve any organ, although lung, brain, and skin are most commonly involved. Disseminated nocardiosis can result from a pulmonary or cutaneous site of origin. Mycobacteria; Actinomyces; dimorphic fungi like Histoplasma, Coccidioides, and Blastomyces; and molds such as Aspergillus can also cause disseminated disease with pulmonary, cutaneous, and musculoskeletal manifestations.

While metastases to muscle itself are rare, they can occur with primary lung cancers. Primary lung cancer with extrapulmonary features is feasible. Squamous cell lung cancer is the most likely to cavitate, although it rarely spreads to the skin. An aggressive lymphoma like diffuse large B-cell lymphoma or cutaneous T-cell lymphoma (higher occurrence in Asians) might also explain his constellation of findings. If culture data remain negative, then biopsy of the chest wall mass might be the safest and highest-yield target.

On hospital day 2, the patient developed new-onset severe neck pain. Magnetic resonance imaging of the cervical, thoracic, and lumbar spine revealed multilevel, bony, lytic lesions with notable cortical breakthrough of the C2 and C3 vertebrae into the prevertebral space, as well as epidural extension and paraspinal soft-tissue extension of the thoracic and lumbar vertebral lesions (Figure 3). 

chan02320923e_f3.jpg

On hospital day 3, the patient reported increased tenderness in his skin nodules with one on his left forearm spontaneously draining purulent fluid. Repeat complete blood count demonstrated a white blood cell count of 12,600/mm3 (45% neutrophils, 43% lymphocytes, 8.4% monocytes, and 4.3% eosinophils), hemoglobin of 16 g/dL, and platelet count of 355,000/mm3.

The erosion into the manubrium and cortical destruction of the cervical spine attests to the aggressiveness of the underlying disease process. Noncutaneous lymphoma and lung cancer are unlikely to have such prominent skin findings; the visceral pathology, necrotizing lung mass, and bone lesions make cutaneous lymphoma less likely. At this point, a disseminated infectious process is most likely. Leading considerations based on his emigration from China and residence in California are tuberculosis and coccidioidomycosis, respectively. Tuberculous spondylitis most commonly involves the lower thoracic and upper lumbar region, and less commonly the cervical spine. His three negative AFB sputa samples further reduce its posttest probability. Ultimately microbiologic data are needed to distinguish between a disseminated fungal process, like coccidioidomycosis, or tuberculosis.

Given the concern for malignancy, a fine needle aspiration of the left supraclavicular lymph node was pursued. This revealed fungal microorganisms morphologically compatible with Coccidioides spp. with a background of necrotizing granulomas and acute inflammation. Fungal blood cultures grew Coccidioides immitis. AFB blood cultures were discontinued due to overgrowth of mold. The Coccidioides immitis antibody immunodiffusion titer was positive at 1:256. 

During the remainder of the hospitalization, the patient was treated with oral fluconazole 800 mg daily. The patient underwent surgical debridement of the manubrium. In addition, given the concern for cervical spine instability, neurosurgery recommended follow-up with interval imaging. Since his discharge from the hospital, the patient continues to take oral fluconazole with resolution of his cutaneous lesions and respiratory symptoms. His titers have incrementally decreased from 1:256 to 1:16 after 8 months of treatment. 

COMMENTARY

This elderly gentleman from China presented with subacute symptoms and was found to have numerous cutaneous nodules, lymphadenopathy, and diffuse osseous lesions. This multisystem illness posed a diagnostic challenge, forcing our discussant to search for a disease process that could lead to such varied findings. Ultimately, epidemiologic and clinical clues suggested a diagnosis of disseminated coccidioidomycosis, which was later confirmed on lymph node biopsy.

Coccidioides species are important fungal pathogens in the Western Hemisphere. This organism exhibits dimorphism, existing as mycelia (with arthroconidia) in soil and spherules in tissues. Coccidioides spp are endemic to the Southwestern United States, particularly California’s central valley and parts of Arizona; it additionally remains an important pathogen in Mexico, Central America, and South America.1 Newer epidemiologic studies have raised concerns that the incidence of coccidioidomycosis is increasing and that its geographic range may be more extensive than previously appreciated, with it now being found as far north as Washington state.2 

Coccidioidal infection can take several forms. One-half to two-thirds of infections may be asymptomatic.3 Clinically significant infections can include an acute self-limiting respiratory illness, pulmonary nodules and cavities, chronic fibrocavitary pneumonia, and infections with extrapulmonary dissemination. Early respiratory infection is often indistinguishable from typical community-acquired pneumonia (10%-15% of pneumonia in endemic areas) but can be associated with certain suggestive features, such as erythema nodosum, erythema multiforme, prominent arthralgias (ie, “desert rheumatism”), and a peripheral eosinophilia.4,5 

Extrapulmonary dissemination is rare and most commonly associated with immunocompromising states.6 However, individuals of African or Filipino ancestry also appear to be at increased risk for disseminated disease, which led to a California court decision that excluded African American inmates from state prisons located in Coccidioides endemic areas.7 The most common sites of extrapulmonary dissemination include the skin and soft tissues, bones and joints, and the central nervous system (CNS).6 CNS disease has a predilection to manifest as a chronic basilar meningitis, most often complicated by hydrocephalus, vasculitic infarction, and spinal arachnoiditis.8

Cutaneous manifestations of coccidioidomycosis can occur as immunologic phenomenon associated with pulmonary disease or represent skin and soft tissue foci of disseminated infection.9 In primary pulmonary infection, skin findings can range from a nonspecific exanthem to erythema nodosum and erythema multiforme, which are thought to represent hypersensitivity responses. In contrast, Coccidioides spp can infect the skin either through direct inoculation (as in primary cutaneous coccidioidomycosis) or via hematogenous dissemination.9,10 A variety of lesions have been described, with painless nodules being the most frequently encountered morphotype in one study.11,12 On histopathologic examination, these lesions often have features of granulomatous dermatitis, eosinophilic infiltration, gummatous necrosis, microabscesses, or perivascular inflammation.13

Another common and highly morbid site of extrapulmonary dissemination is the musculoskeletal system. Bone and joint coccidioidomycosis most frequently affect the axial skeleton, although peripheral skeletal structures and joints can also be involved.6,12 Vertebral coccidioidomycosis is associated with significant morbidity. A study describing the magnetic resonance imaging findings of patients with vertebral coccidioidomycosis found that Coccidioides spp appeared to have a predilection for the thoracic vertebrae (in up to 80% of the study’s cohort).14 Skip lesions with noncontiguously involved vertebrae occurred in roughly half of patients, highlighting the usefulness of imaging the total spine in suspected cases. 

The diagnosis of coccidioidomycosis is often established through serologic testing or by isolation of Coccidioides spp. on histopathology or culture. Obtaining sputum or tissue may be difficult, so clinicians often rely on noninvasive diagnostic tests such as coccidioidal antigen and serologies by enzyme immunoassays, immunodiffusion, and complement fixation. Enzyme immunoassays IgM and IgG results are positive early in the disease process and need to be confirmed with immunodiffusion or complement fixation testing. Complement fixation IgG is additionally useful to monitor disease activity over time and can help inform risk of disseminated disease.15 The gold standard of diagnosis of disseminated coccidioidomycosis infection remains histopathologic confirmation either by direct visualization of a spherule or growth in fungal cultures.16 Polymerase chain reaction testing of sputum samples is an emerging diagnostic technique that has been found to have similar sensitivity rates to fungal culture.17

Treatment decisions in coccidioidomycosis are complex and vary by site of infection, immune status of the host, and extent of disease.16 While uncomplicated primary pulmonary infections can often be managed with observation alone, prolonged medical therapy with azole antifungals is often recommended for complicated pulmonary infections, symptomatic cavitary disease, and virtually all forms of extrapulmonary disease. Intravenous liposomal amphotericin is often used as initial therapy in immunosuppressed individuals, pregnant women, and those with extensive disease. CNS disease represents a particularly challenging treatment scenario and requires lifelong azole therapy.8,16 

The patient in this case initially presented with vague inflammatory symptoms, with each aliquot revealing further evidence of a metastatic disease process. Such multisystem presentations are diagnostically challenging and force clinicians to reach for some feature around which to build their differential diagnosis. It is with this in mind that we are often taught to “localize the lesion” in order to focus our search for a unifying diagnosis. Yet, in this case, the sheer number of disease foci ultimately helped the discussant to narrow the range of diagnostic possibilities because only a limited number of conditions could present with such widespread, multisystem manifestations. Therefore, this case serves as a reminder that, sometimes in clinical reasoning, “more is less.”

KEY TEACHING POINTS

  • Coccidioidomycosis is a fungal infection that can present with pulmonary or extrapulmonary disease. Risk of extrapulmonary dissemination is greatest among immunocompromised individuals and those of African or Filipino ancestry.3,7
  • The most common sites of extrapulmonary dissemination include the skin and soft tissues, bones and joints, and the CNS.6
  • While serologic testing can be diagnostically useful, the gold standard for diagnosis of disseminated coccidioidomycosis infection remains histopathologic confirmation with direct visualization of a spherule or growth in fungal cultures.16
 
References

1. Benedict K, McCotter OZ, Brady S, et al. Surveillance for Coccidioidomycosis - United States, 2011-2017. MMWR Surveill Summ. 2019;68(No. SS-7):1-15. http://dx.doi.org/10.15585/mmwr.ss6807a1
2. McCotter OZ, Benedict K, Engelthaler DM, et al. Update on the epidemiology of coccidioidomycosis in the United States. Med Mycol. 2019;57(Suppl 1):S30-s40. https://doi.org/10.1093/mmy/myy095
3. Galgiani JN, Ampel NM, Blair JE, et al. Coccidioidomycosis. Clin Infect Dis. 2005;41(9):1217-1223. https://doi.org/10.1086/496991
4. Chang DC, Anderson S, Wannemuehler K, et al. Testing for coccidioidomycosis among patients with community-acquired pneumonia. Emerg Infect Dis. 2008;14(7):1053-1059. https://doi.org/10.3201/eid1407.070832
5. Saubolle MA, McKellar PP, Sussland D. Epidemiologic, clinical, and diagnostic aspects of coccidioidomycosis. J Clin Microbiol. 2007;45(1):26-30. https://doi.org/10.1128/jcm.02230-06
6. Adam RD, Elliott SP, Taljanovic MS. The spectrum and presentation of disseminated coccidioidomycosis. Am J Med. 2009;122(8):770-777. https://doi.org/10.1016/j.amjmed.2008.12.024
7. Wheeler C, Lucas KD, Mohle-Boetani JC. Rates and risk factors for Coccidioidomycosis among prison inmates, California, USA, 2011. Emerg Infect Dis. 2015;21(1):70-75. https://doi.org/10.3201/eid2101.140836
8. Johnson RH, Einstein HE. Coccidioidal meningitis. Clin Infect Dis. 2006;42(1):103-107. https://doi.org/10.1086/497596
9. Blair JE. State-of-the-art treatment of coccidioidomycosis: skin and soft-­tissue infections. Ann N Y Acad Sci. 2007;1111:411-421. https://doi.org/10.1196/annals.1406.010
10. Chang A, Tung RC, McGillis TS, Bergfeld WF, Taylor JS. Primary cutaneous coccidioidomycosis. J Am Acad Dermatol. 2003;49(5):944-949. https://doi.org/10.1016/s0190-9622(03)00462-6
11. Quimby SR, Connolly SM, Winkelmann RK, Smilack JD. Clinicopathologic spectrum of specific cutaneous lesions of disseminated coccidioidomycosis. J Am Acad Dermatol. 1992;26(1):79-85. https://doi.org/10.1016/0190-9622(92)70011-4
12. Crum NF, Lederman ER, Stafford CM, Parrish JS, Wallace MR. Coccidioidomycosis: a descriptive survey of a reemerging disease. clinical characteristics and current controversies. Medicine (Baltimore). 2004;83(3):149-175. https://doi.org/10.1097/01.md.0000126762.91040.fd
13. Carpenter JB, Feldman JS, Leyva WH, DiCaudo DJ. Clinical and pathologic characteristics of disseminated cutaneous coccidioidomycosis. J Am Acad Dermatol. 2010;62(5):831-837. https://doi.org/10.1016/j.jaad.2008.07.031
14. Crete RN, Gallmann W, Karis JP, Ross J. Spinal coccidioidomycosis: MR imaging findings in 41 patients. AJNR Am J Neuroradiol. 2018;39(11):2148-2153. https://doi.org/10.3174/ajnr.a5818
15. McHardy IH, Dinh BN, Waldman S, et al. Coccidioidomycosis complement fixation titer trends in the age of antifungals. J Clin Microbiol. 2018;56(12):e01318-18. https://doi.org/10.1128/jcm.01318-18
16. Galgiani JN, Ampel NM, Blair JE, et al. 2016 Infectious Diseases Society of America (IDSA) clinical practice guideline for the treatment of coccidioidomycosis. Clin Infect Dis. 2016;63(6):e112-e146. https://doi.org/10.1093/cid/ciw360
17. Vucicevic D, Blair JE, Binnicker MJ, et al. The utility of Coccidioides polymerase chain reaction testing in the clinical setting. Mycopathologia. 2010;170(5):345-351. https://doi.org/10.1007/s11046-010-9327-0

Article PDF
Author and Disclosure Information

1Department of Medicine, University of California, San Francisco, California; 2Division of Infectious Diseases, Johns Hopkins University School of Medicine, Baltimore, Maryland; 3Division of General Internal Medicine, Johns Hopkins University School of Medicine, Baltimore, Maryland; 4Medical Service, San Francisco VA Medical Center, San Francisco, California.

Disclosures

There are no conflicts of interests reported by all authors.

Issue
Journal of Hospital Medicine 16(4)
Publications
Topics
Page Number
239-242. Published Online First September 23, 2020
Sections
Author and Disclosure Information

1Department of Medicine, University of California, San Francisco, California; 2Division of Infectious Diseases, Johns Hopkins University School of Medicine, Baltimore, Maryland; 3Division of General Internal Medicine, Johns Hopkins University School of Medicine, Baltimore, Maryland; 4Medical Service, San Francisco VA Medical Center, San Francisco, California.

Disclosures

There are no conflicts of interests reported by all authors.

Author and Disclosure Information

1Department of Medicine, University of California, San Francisco, California; 2Division of Infectious Diseases, Johns Hopkins University School of Medicine, Baltimore, Maryland; 3Division of General Internal Medicine, Johns Hopkins University School of Medicine, Baltimore, Maryland; 4Medical Service, San Francisco VA Medical Center, San Francisco, California.

Disclosures

There are no conflicts of interests reported by all authors.

Article PDF
Article PDF
Related Articles

A 64-year-old man presented with a 2-month history of a nonproductive cough, weight loss, and subjective fevers. He had no chest pain, hemoptysis, or shortness of breath. He also described worsening anorexia and a 15-pound weight loss over the previous 3 months. He had no arthralgias, myalgias, abdominal pain, nausea, emesis, or diarrhea.

Two weeks prior to his presentation, he was diagnosed with pneumonia and given a 5-day course of azithromycin. His symptoms did not improve, so he presented to the emergency room. 

He had not been seen regularly by a physician in decades and had no known medical conditions. He did not take any medications. He immigrated from China 3 years prior and lived with his wife in California. He had a 30 pack-year smoking history. He drank a shot glass of liquor daily and denied any drug use.

Weight loss might result from inflammatory disorders like cancer or noninflammatory causes such as decreased oral intake (eg, diminished appetite) or malabsorption (eg, celiac disease). However, his fevers suggest inflammation, which usually reflects an underlying infection, cancer, or autoimmune process. While chronic cough typically results from upper airway cough syndrome (allergic or nonallergic rhinitis), gastroesophageal reflux disease, or asthma, it can also point to pathology of the lung, which may be intrinsic (bronchiectasis) or extrinsic (mediastinal mass). The duration of 2 months makes a typical infectious process like pneumococcal pneumonia unlikely. Atypical infections such as tuberculosis, melioidosis, and talaromycosis are possible given his immigration from East Asia, and coccidioidomycosis given his residence in California. He might have undiagnosed medical conditions, such as diabetes, that could be relevant to his current presentation and classify him as immunocompromised. His smoking history prompts consideration of lung cancer.

His temperature was 36.5 oC, heart rate 70 beats per minute, blood pressure 118/66 mm Hg, respiratory rate 16 breaths per minute, oxygen saturation 98% on room air, and body mass index 23 kg/m2. He was in no acute distress. The findings from the cardiac, lung, abdominal, and neurological exams were normal.

Skin examination found a fixed, symmetric, 5-cm, firm nodule at top of sternum (Figure 1A). In addition, he had two 1-cm, mobile, firm, subcutaneous nodules, one on his anterior left chest and another underneath his right axilla. He also had two 2-cm, erythematous, tender nodules on his left anterior forearm and a 1-cm nodule with a central black plug on the dorsal surface of his right hand (Figure 1B). He did not have any edema.

chan02320923e_f1.jpg

The white blood cell count was 10,500/mm3 (42% neutrophils, 37% lymphocytes, 16.4% monocytes, and 2.9% eosinophils), hemoglobin was 12.2 g/dL with a mean corpuscular volume of 91 fL, and the platelet count was 441,000/mm3. Basic metabolic panel, aminotransferase, bilirubin, and alkaline phosphatase were within reference ranges. Serum albumin was 3.1 g/dL. Serum total protein was elevated at 8.8 g/dL. Serum calcium was 9.0 mg/dL. Urinalysis results were normal.

The slightly low albumin, mildly elevated platelet count, monocytosis, and normocytic anemia suggest inflammation, although monocytosis might represent a hematologic malignancy like chronic myelomonocytic leukemia (CMML). His subjective fevers and weight loss further corroborate underlying inflammation. What is driving the inflammation? There are two localizing findings: cough and nodular skin lesions.

His lack of dyspnea and normal oxygen saturation, respiratory rate, and lung exam make an extrapulmonary cause of cough such as lymphadenopathy or mediastinal infection possible. The number of nodular skin lesions, wide-spread distribution, and appearance (eg, erythematous, tender) point to either a primary cutaneous disease with systemic manifestations (eg, cutaneous lymphoma) or a systemic disease with cutaneous features (eg, sarcoidosis).

Three categories—inflammatory, infectious, and neoplastic—account for most nodular skin lesions. Usually microscopic evaluation is necessary for definitive diagnosis, though epidemiology, associated symptoms, and characteristics of the nodules help prioritize the differential diagnosis. Tender nodules might reflect a panniculitis; erythema nodosum is the most common type, and while this classically develops on the anterior shins, it may also occur on the forearm. His immigration from China prompts consideration of tuberculosis and cutaneous leishmaniasis. Coccidioidomycosis can lead to inflammation and nodular skin lesions. Other infections such as nontuberculous mycobacteria, nocardiosis, and cryptococcosis may cause disseminated infection with pulmonary and skin manifestations. His smoking puts him at risk of lung cancer, which rarely results in metastatic subcutaneous infiltrates.

A chest radiograph demonstrated a prominent density in the right paratracheal region of the mediastinum with adjacent streaky opacities. A computed tomography scan of the chest with intravenous contrast demonstrated centrilobular emphysematous changes and revealed a 2.6 × 4.7-cm necrotic mass in the anterior chest wall with erosion into the manubrium, a 3.8 × 2.1-cm centrally necrotic soft-tissue mass in the right hilum, a 5-mm left upper-lobe noncalcified solid pulmonary nodule, and prominent subcarinal, paratracheal, hilar, and bilateral supraclavicular lymphadenopathy (Figure 2). 

chan02320923e_f2.jpg

Flow cytometry of the peripheral blood did not demonstrate a lymphoproliferative disorder. Blood smear demonstrated normal red blood cell, white blood cell, and platelet morphology. HIV antibody was negative. Hemoglobin A1c was 6.1%. Smear microscopy for acid-fast bacilli (AFB) was negative and sputum AFB samples were sent for culture. Bacterial, fungal, and AFB blood cultures were collected and pending. 

Causes of necrotizing pneumonia include liquid (eg, lymphoma) and solid (eg, squamous cell carcinoma) cancers, infections, and noninfectious inflammatory processes such as granulomatosis with polyangiitis (GPA). Given his subacute presentation and extrapulmonary cutaneous manifestations, consideration of mycobacteria, fungi (eg, Coccidioides, Aspergillus, and Cryptococcus), and filamentous bacteria (eg, Nocardia and Actinomyces) is prioritized among the myriad of infections that can cause a lung cavity. His smoking history and centrilobular emphysematous changes are highly suggestive of chronic obstructive pulmonary disease, which puts him at increased risk of bacterial colonization and recurrent pulmonary infections. Tuberculosis is still possible despite three negative AFB-sputa smears given the sensitivity of smear microscopy (with three specimens) is roughly 70% in an immunocompetent host.

The lymphadenopathy likely reflects spread from the necrotic lung mass. The frequency of non-Hodgkin lymphoma increases with age. The results of the peripheral flow cytometry do not exclude the possibility of an aggressive lymphoma with pulmonary and cutaneous manifestations.

The erosive property of the chest wall mass makes an autoimmune process like GPA unlikely. An aggressive and disseminated infection or cancer is most likely. A pathologic process that originated in the lung and then spread to the lymph nodes and skin is more likely than a disorder which started in the skin. It would be unlikely for a primary cutaneous disorder to cause such a well-defined necrotic lung mass. Lung cancer rarely metastasizes to the skin and, instead, preferentially involves the chest. Ultimately, ascertaining what the patient experienced first (ie, respiratory or cutaneous symptoms) will determine where the pathology originated.

Computed tomography scan of the abdomen and pelvis with intravenous contrast demonstrated multiple ill-­defined lytic lesions in the pelvis, including a 12-mm lesion of the left sacral ala and multiple subcentimeter lesions in the medial left iliac bone and superior right acetabulum. In addition, there were two 1-cm, rim-enhancing, hypodense nodules in the subcutaneous fat of the right flank at the level of L5 and the left lower quadrant, respectively. There was also a 2.2 × 1.9-cm faintly rim-enhancing hypodensity within the left iliopsoas muscle belly.

These imaging findings further corroborate a widely metastatic process probably originating in the lung and spreading to the lymph nodes, skin, muscles, and bones. The characterization of lesions as lytic as opposed to blastic is less helpful because many diseases can cause both. It does prompt consideration of multiple myeloma; however, multiple myeloma less commonly manifests with extramedullary plasmacytomas and is less likely given his normal renal function and calcium level. Bone lesions lessen the likelihood of GPA, and his necrotic lung mass makes sarcoidosis unlikely. Atypical infections and cancers are the prime suspect of his multisystemic disease.

There are no data yet to suggest a weakened immune system, which would increase his risk for atypical infections. His chronic lung disease, identified on imaging, is a risk factor for nocardiosis. This gram-positive, weakly acid-fast bacterium can involve any organ, although lung, brain, and skin are most commonly involved. Disseminated nocardiosis can result from a pulmonary or cutaneous site of origin. Mycobacteria; Actinomyces; dimorphic fungi like Histoplasma, Coccidioides, and Blastomyces; and molds such as Aspergillus can also cause disseminated disease with pulmonary, cutaneous, and musculoskeletal manifestations.

While metastases to muscle itself are rare, they can occur with primary lung cancers. Primary lung cancer with extrapulmonary features is feasible. Squamous cell lung cancer is the most likely to cavitate, although it rarely spreads to the skin. An aggressive lymphoma like diffuse large B-cell lymphoma or cutaneous T-cell lymphoma (higher occurrence in Asians) might also explain his constellation of findings. If culture data remain negative, then biopsy of the chest wall mass might be the safest and highest-yield target.

On hospital day 2, the patient developed new-onset severe neck pain. Magnetic resonance imaging of the cervical, thoracic, and lumbar spine revealed multilevel, bony, lytic lesions with notable cortical breakthrough of the C2 and C3 vertebrae into the prevertebral space, as well as epidural extension and paraspinal soft-tissue extension of the thoracic and lumbar vertebral lesions (Figure 3). 

chan02320923e_f3.jpg

On hospital day 3, the patient reported increased tenderness in his skin nodules with one on his left forearm spontaneously draining purulent fluid. Repeat complete blood count demonstrated a white blood cell count of 12,600/mm3 (45% neutrophils, 43% lymphocytes, 8.4% monocytes, and 4.3% eosinophils), hemoglobin of 16 g/dL, and platelet count of 355,000/mm3.

The erosion into the manubrium and cortical destruction of the cervical spine attests to the aggressiveness of the underlying disease process. Noncutaneous lymphoma and lung cancer are unlikely to have such prominent skin findings; the visceral pathology, necrotizing lung mass, and bone lesions make cutaneous lymphoma less likely. At this point, a disseminated infectious process is most likely. Leading considerations based on his emigration from China and residence in California are tuberculosis and coccidioidomycosis, respectively. Tuberculous spondylitis most commonly involves the lower thoracic and upper lumbar region, and less commonly the cervical spine. His three negative AFB sputa samples further reduce its posttest probability. Ultimately microbiologic data are needed to distinguish between a disseminated fungal process, like coccidioidomycosis, or tuberculosis.

Given the concern for malignancy, a fine needle aspiration of the left supraclavicular lymph node was pursued. This revealed fungal microorganisms morphologically compatible with Coccidioides spp. with a background of necrotizing granulomas and acute inflammation. Fungal blood cultures grew Coccidioides immitis. AFB blood cultures were discontinued due to overgrowth of mold. The Coccidioides immitis antibody immunodiffusion titer was positive at 1:256. 

During the remainder of the hospitalization, the patient was treated with oral fluconazole 800 mg daily. The patient underwent surgical debridement of the manubrium. In addition, given the concern for cervical spine instability, neurosurgery recommended follow-up with interval imaging. Since his discharge from the hospital, the patient continues to take oral fluconazole with resolution of his cutaneous lesions and respiratory symptoms. His titers have incrementally decreased from 1:256 to 1:16 after 8 months of treatment. 

COMMENTARY

This elderly gentleman from China presented with subacute symptoms and was found to have numerous cutaneous nodules, lymphadenopathy, and diffuse osseous lesions. This multisystem illness posed a diagnostic challenge, forcing our discussant to search for a disease process that could lead to such varied findings. Ultimately, epidemiologic and clinical clues suggested a diagnosis of disseminated coccidioidomycosis, which was later confirmed on lymph node biopsy.

Coccidioides species are important fungal pathogens in the Western Hemisphere. This organism exhibits dimorphism, existing as mycelia (with arthroconidia) in soil and spherules in tissues. Coccidioides spp are endemic to the Southwestern United States, particularly California’s central valley and parts of Arizona; it additionally remains an important pathogen in Mexico, Central America, and South America.1 Newer epidemiologic studies have raised concerns that the incidence of coccidioidomycosis is increasing and that its geographic range may be more extensive than previously appreciated, with it now being found as far north as Washington state.2 

Coccidioidal infection can take several forms. One-half to two-thirds of infections may be asymptomatic.3 Clinically significant infections can include an acute self-limiting respiratory illness, pulmonary nodules and cavities, chronic fibrocavitary pneumonia, and infections with extrapulmonary dissemination. Early respiratory infection is often indistinguishable from typical community-acquired pneumonia (10%-15% of pneumonia in endemic areas) but can be associated with certain suggestive features, such as erythema nodosum, erythema multiforme, prominent arthralgias (ie, “desert rheumatism”), and a peripheral eosinophilia.4,5 

Extrapulmonary dissemination is rare and most commonly associated with immunocompromising states.6 However, individuals of African or Filipino ancestry also appear to be at increased risk for disseminated disease, which led to a California court decision that excluded African American inmates from state prisons located in Coccidioides endemic areas.7 The most common sites of extrapulmonary dissemination include the skin and soft tissues, bones and joints, and the central nervous system (CNS).6 CNS disease has a predilection to manifest as a chronic basilar meningitis, most often complicated by hydrocephalus, vasculitic infarction, and spinal arachnoiditis.8

Cutaneous manifestations of coccidioidomycosis can occur as immunologic phenomenon associated with pulmonary disease or represent skin and soft tissue foci of disseminated infection.9 In primary pulmonary infection, skin findings can range from a nonspecific exanthem to erythema nodosum and erythema multiforme, which are thought to represent hypersensitivity responses. In contrast, Coccidioides spp can infect the skin either through direct inoculation (as in primary cutaneous coccidioidomycosis) or via hematogenous dissemination.9,10 A variety of lesions have been described, with painless nodules being the most frequently encountered morphotype in one study.11,12 On histopathologic examination, these lesions often have features of granulomatous dermatitis, eosinophilic infiltration, gummatous necrosis, microabscesses, or perivascular inflammation.13

Another common and highly morbid site of extrapulmonary dissemination is the musculoskeletal system. Bone and joint coccidioidomycosis most frequently affect the axial skeleton, although peripheral skeletal structures and joints can also be involved.6,12 Vertebral coccidioidomycosis is associated with significant morbidity. A study describing the magnetic resonance imaging findings of patients with vertebral coccidioidomycosis found that Coccidioides spp appeared to have a predilection for the thoracic vertebrae (in up to 80% of the study’s cohort).14 Skip lesions with noncontiguously involved vertebrae occurred in roughly half of patients, highlighting the usefulness of imaging the total spine in suspected cases. 

The diagnosis of coccidioidomycosis is often established through serologic testing or by isolation of Coccidioides spp. on histopathology or culture. Obtaining sputum or tissue may be difficult, so clinicians often rely on noninvasive diagnostic tests such as coccidioidal antigen and serologies by enzyme immunoassays, immunodiffusion, and complement fixation. Enzyme immunoassays IgM and IgG results are positive early in the disease process and need to be confirmed with immunodiffusion or complement fixation testing. Complement fixation IgG is additionally useful to monitor disease activity over time and can help inform risk of disseminated disease.15 The gold standard of diagnosis of disseminated coccidioidomycosis infection remains histopathologic confirmation either by direct visualization of a spherule or growth in fungal cultures.16 Polymerase chain reaction testing of sputum samples is an emerging diagnostic technique that has been found to have similar sensitivity rates to fungal culture.17

Treatment decisions in coccidioidomycosis are complex and vary by site of infection, immune status of the host, and extent of disease.16 While uncomplicated primary pulmonary infections can often be managed with observation alone, prolonged medical therapy with azole antifungals is often recommended for complicated pulmonary infections, symptomatic cavitary disease, and virtually all forms of extrapulmonary disease. Intravenous liposomal amphotericin is often used as initial therapy in immunosuppressed individuals, pregnant women, and those with extensive disease. CNS disease represents a particularly challenging treatment scenario and requires lifelong azole therapy.8,16 

The patient in this case initially presented with vague inflammatory symptoms, with each aliquot revealing further evidence of a metastatic disease process. Such multisystem presentations are diagnostically challenging and force clinicians to reach for some feature around which to build their differential diagnosis. It is with this in mind that we are often taught to “localize the lesion” in order to focus our search for a unifying diagnosis. Yet, in this case, the sheer number of disease foci ultimately helped the discussant to narrow the range of diagnostic possibilities because only a limited number of conditions could present with such widespread, multisystem manifestations. Therefore, this case serves as a reminder that, sometimes in clinical reasoning, “more is less.”

KEY TEACHING POINTS

  • Coccidioidomycosis is a fungal infection that can present with pulmonary or extrapulmonary disease. Risk of extrapulmonary dissemination is greatest among immunocompromised individuals and those of African or Filipino ancestry.3,7
  • The most common sites of extrapulmonary dissemination include the skin and soft tissues, bones and joints, and the CNS.6
  • While serologic testing can be diagnostically useful, the gold standard for diagnosis of disseminated coccidioidomycosis infection remains histopathologic confirmation with direct visualization of a spherule or growth in fungal cultures.16
 

A 64-year-old man presented with a 2-month history of a nonproductive cough, weight loss, and subjective fevers. He had no chest pain, hemoptysis, or shortness of breath. He also described worsening anorexia and a 15-pound weight loss over the previous 3 months. He had no arthralgias, myalgias, abdominal pain, nausea, emesis, or diarrhea.

Two weeks prior to his presentation, he was diagnosed with pneumonia and given a 5-day course of azithromycin. His symptoms did not improve, so he presented to the emergency room. 

He had not been seen regularly by a physician in decades and had no known medical conditions. He did not take any medications. He immigrated from China 3 years prior and lived with his wife in California. He had a 30 pack-year smoking history. He drank a shot glass of liquor daily and denied any drug use.

Weight loss might result from inflammatory disorders like cancer or noninflammatory causes such as decreased oral intake (eg, diminished appetite) or malabsorption (eg, celiac disease). However, his fevers suggest inflammation, which usually reflects an underlying infection, cancer, or autoimmune process. While chronic cough typically results from upper airway cough syndrome (allergic or nonallergic rhinitis), gastroesophageal reflux disease, or asthma, it can also point to pathology of the lung, which may be intrinsic (bronchiectasis) or extrinsic (mediastinal mass). The duration of 2 months makes a typical infectious process like pneumococcal pneumonia unlikely. Atypical infections such as tuberculosis, melioidosis, and talaromycosis are possible given his immigration from East Asia, and coccidioidomycosis given his residence in California. He might have undiagnosed medical conditions, such as diabetes, that could be relevant to his current presentation and classify him as immunocompromised. His smoking history prompts consideration of lung cancer.

His temperature was 36.5 oC, heart rate 70 beats per minute, blood pressure 118/66 mm Hg, respiratory rate 16 breaths per minute, oxygen saturation 98% on room air, and body mass index 23 kg/m2. He was in no acute distress. The findings from the cardiac, lung, abdominal, and neurological exams were normal.

Skin examination found a fixed, symmetric, 5-cm, firm nodule at top of sternum (Figure 1A). In addition, he had two 1-cm, mobile, firm, subcutaneous nodules, one on his anterior left chest and another underneath his right axilla. He also had two 2-cm, erythematous, tender nodules on his left anterior forearm and a 1-cm nodule with a central black plug on the dorsal surface of his right hand (Figure 1B). He did not have any edema.

chan02320923e_f1.jpg

The white blood cell count was 10,500/mm3 (42% neutrophils, 37% lymphocytes, 16.4% monocytes, and 2.9% eosinophils), hemoglobin was 12.2 g/dL with a mean corpuscular volume of 91 fL, and the platelet count was 441,000/mm3. Basic metabolic panel, aminotransferase, bilirubin, and alkaline phosphatase were within reference ranges. Serum albumin was 3.1 g/dL. Serum total protein was elevated at 8.8 g/dL. Serum calcium was 9.0 mg/dL. Urinalysis results were normal.

The slightly low albumin, mildly elevated platelet count, monocytosis, and normocytic anemia suggest inflammation, although monocytosis might represent a hematologic malignancy like chronic myelomonocytic leukemia (CMML). His subjective fevers and weight loss further corroborate underlying inflammation. What is driving the inflammation? There are two localizing findings: cough and nodular skin lesions.

His lack of dyspnea and normal oxygen saturation, respiratory rate, and lung exam make an extrapulmonary cause of cough such as lymphadenopathy or mediastinal infection possible. The number of nodular skin lesions, wide-spread distribution, and appearance (eg, erythematous, tender) point to either a primary cutaneous disease with systemic manifestations (eg, cutaneous lymphoma) or a systemic disease with cutaneous features (eg, sarcoidosis).

Three categories—inflammatory, infectious, and neoplastic—account for most nodular skin lesions. Usually microscopic evaluation is necessary for definitive diagnosis, though epidemiology, associated symptoms, and characteristics of the nodules help prioritize the differential diagnosis. Tender nodules might reflect a panniculitis; erythema nodosum is the most common type, and while this classically develops on the anterior shins, it may also occur on the forearm. His immigration from China prompts consideration of tuberculosis and cutaneous leishmaniasis. Coccidioidomycosis can lead to inflammation and nodular skin lesions. Other infections such as nontuberculous mycobacteria, nocardiosis, and cryptococcosis may cause disseminated infection with pulmonary and skin manifestations. His smoking puts him at risk of lung cancer, which rarely results in metastatic subcutaneous infiltrates.

A chest radiograph demonstrated a prominent density in the right paratracheal region of the mediastinum with adjacent streaky opacities. A computed tomography scan of the chest with intravenous contrast demonstrated centrilobular emphysematous changes and revealed a 2.6 × 4.7-cm necrotic mass in the anterior chest wall with erosion into the manubrium, a 3.8 × 2.1-cm centrally necrotic soft-tissue mass in the right hilum, a 5-mm left upper-lobe noncalcified solid pulmonary nodule, and prominent subcarinal, paratracheal, hilar, and bilateral supraclavicular lymphadenopathy (Figure 2). 

chan02320923e_f2.jpg

Flow cytometry of the peripheral blood did not demonstrate a lymphoproliferative disorder. Blood smear demonstrated normal red blood cell, white blood cell, and platelet morphology. HIV antibody was negative. Hemoglobin A1c was 6.1%. Smear microscopy for acid-fast bacilli (AFB) was negative and sputum AFB samples were sent for culture. Bacterial, fungal, and AFB blood cultures were collected and pending. 

Causes of necrotizing pneumonia include liquid (eg, lymphoma) and solid (eg, squamous cell carcinoma) cancers, infections, and noninfectious inflammatory processes such as granulomatosis with polyangiitis (GPA). Given his subacute presentation and extrapulmonary cutaneous manifestations, consideration of mycobacteria, fungi (eg, Coccidioides, Aspergillus, and Cryptococcus), and filamentous bacteria (eg, Nocardia and Actinomyces) is prioritized among the myriad of infections that can cause a lung cavity. His smoking history and centrilobular emphysematous changes are highly suggestive of chronic obstructive pulmonary disease, which puts him at increased risk of bacterial colonization and recurrent pulmonary infections. Tuberculosis is still possible despite three negative AFB-sputa smears given the sensitivity of smear microscopy (with three specimens) is roughly 70% in an immunocompetent host.

The lymphadenopathy likely reflects spread from the necrotic lung mass. The frequency of non-Hodgkin lymphoma increases with age. The results of the peripheral flow cytometry do not exclude the possibility of an aggressive lymphoma with pulmonary and cutaneous manifestations.

The erosive property of the chest wall mass makes an autoimmune process like GPA unlikely. An aggressive and disseminated infection or cancer is most likely. A pathologic process that originated in the lung and then spread to the lymph nodes and skin is more likely than a disorder which started in the skin. It would be unlikely for a primary cutaneous disorder to cause such a well-defined necrotic lung mass. Lung cancer rarely metastasizes to the skin and, instead, preferentially involves the chest. Ultimately, ascertaining what the patient experienced first (ie, respiratory or cutaneous symptoms) will determine where the pathology originated.

Computed tomography scan of the abdomen and pelvis with intravenous contrast demonstrated multiple ill-­defined lytic lesions in the pelvis, including a 12-mm lesion of the left sacral ala and multiple subcentimeter lesions in the medial left iliac bone and superior right acetabulum. In addition, there were two 1-cm, rim-enhancing, hypodense nodules in the subcutaneous fat of the right flank at the level of L5 and the left lower quadrant, respectively. There was also a 2.2 × 1.9-cm faintly rim-enhancing hypodensity within the left iliopsoas muscle belly.

These imaging findings further corroborate a widely metastatic process probably originating in the lung and spreading to the lymph nodes, skin, muscles, and bones. The characterization of lesions as lytic as opposed to blastic is less helpful because many diseases can cause both. It does prompt consideration of multiple myeloma; however, multiple myeloma less commonly manifests with extramedullary plasmacytomas and is less likely given his normal renal function and calcium level. Bone lesions lessen the likelihood of GPA, and his necrotic lung mass makes sarcoidosis unlikely. Atypical infections and cancers are the prime suspect of his multisystemic disease.

There are no data yet to suggest a weakened immune system, which would increase his risk for atypical infections. His chronic lung disease, identified on imaging, is a risk factor for nocardiosis. This gram-positive, weakly acid-fast bacterium can involve any organ, although lung, brain, and skin are most commonly involved. Disseminated nocardiosis can result from a pulmonary or cutaneous site of origin. Mycobacteria; Actinomyces; dimorphic fungi like Histoplasma, Coccidioides, and Blastomyces; and molds such as Aspergillus can also cause disseminated disease with pulmonary, cutaneous, and musculoskeletal manifestations.

While metastases to muscle itself are rare, they can occur with primary lung cancers. Primary lung cancer with extrapulmonary features is feasible. Squamous cell lung cancer is the most likely to cavitate, although it rarely spreads to the skin. An aggressive lymphoma like diffuse large B-cell lymphoma or cutaneous T-cell lymphoma (higher occurrence in Asians) might also explain his constellation of findings. If culture data remain negative, then biopsy of the chest wall mass might be the safest and highest-yield target.

On hospital day 2, the patient developed new-onset severe neck pain. Magnetic resonance imaging of the cervical, thoracic, and lumbar spine revealed multilevel, bony, lytic lesions with notable cortical breakthrough of the C2 and C3 vertebrae into the prevertebral space, as well as epidural extension and paraspinal soft-tissue extension of the thoracic and lumbar vertebral lesions (Figure 3). 

chan02320923e_f3.jpg

On hospital day 3, the patient reported increased tenderness in his skin nodules with one on his left forearm spontaneously draining purulent fluid. Repeat complete blood count demonstrated a white blood cell count of 12,600/mm3 (45% neutrophils, 43% lymphocytes, 8.4% monocytes, and 4.3% eosinophils), hemoglobin of 16 g/dL, and platelet count of 355,000/mm3.

The erosion into the manubrium and cortical destruction of the cervical spine attests to the aggressiveness of the underlying disease process. Noncutaneous lymphoma and lung cancer are unlikely to have such prominent skin findings; the visceral pathology, necrotizing lung mass, and bone lesions make cutaneous lymphoma less likely. At this point, a disseminated infectious process is most likely. Leading considerations based on his emigration from China and residence in California are tuberculosis and coccidioidomycosis, respectively. Tuberculous spondylitis most commonly involves the lower thoracic and upper lumbar region, and less commonly the cervical spine. His three negative AFB sputa samples further reduce its posttest probability. Ultimately microbiologic data are needed to distinguish between a disseminated fungal process, like coccidioidomycosis, or tuberculosis.

Given the concern for malignancy, a fine needle aspiration of the left supraclavicular lymph node was pursued. This revealed fungal microorganisms morphologically compatible with Coccidioides spp. with a background of necrotizing granulomas and acute inflammation. Fungal blood cultures grew Coccidioides immitis. AFB blood cultures were discontinued due to overgrowth of mold. The Coccidioides immitis antibody immunodiffusion titer was positive at 1:256. 

During the remainder of the hospitalization, the patient was treated with oral fluconazole 800 mg daily. The patient underwent surgical debridement of the manubrium. In addition, given the concern for cervical spine instability, neurosurgery recommended follow-up with interval imaging. Since his discharge from the hospital, the patient continues to take oral fluconazole with resolution of his cutaneous lesions and respiratory symptoms. His titers have incrementally decreased from 1:256 to 1:16 after 8 months of treatment. 

COMMENTARY

This elderly gentleman from China presented with subacute symptoms and was found to have numerous cutaneous nodules, lymphadenopathy, and diffuse osseous lesions. This multisystem illness posed a diagnostic challenge, forcing our discussant to search for a disease process that could lead to such varied findings. Ultimately, epidemiologic and clinical clues suggested a diagnosis of disseminated coccidioidomycosis, which was later confirmed on lymph node biopsy.

Coccidioides species are important fungal pathogens in the Western Hemisphere. This organism exhibits dimorphism, existing as mycelia (with arthroconidia) in soil and spherules in tissues. Coccidioides spp are endemic to the Southwestern United States, particularly California’s central valley and parts of Arizona; it additionally remains an important pathogen in Mexico, Central America, and South America.1 Newer epidemiologic studies have raised concerns that the incidence of coccidioidomycosis is increasing and that its geographic range may be more extensive than previously appreciated, with it now being found as far north as Washington state.2 

Coccidioidal infection can take several forms. One-half to two-thirds of infections may be asymptomatic.3 Clinically significant infections can include an acute self-limiting respiratory illness, pulmonary nodules and cavities, chronic fibrocavitary pneumonia, and infections with extrapulmonary dissemination. Early respiratory infection is often indistinguishable from typical community-acquired pneumonia (10%-15% of pneumonia in endemic areas) but can be associated with certain suggestive features, such as erythema nodosum, erythema multiforme, prominent arthralgias (ie, “desert rheumatism”), and a peripheral eosinophilia.4,5 

Extrapulmonary dissemination is rare and most commonly associated with immunocompromising states.6 However, individuals of African or Filipino ancestry also appear to be at increased risk for disseminated disease, which led to a California court decision that excluded African American inmates from state prisons located in Coccidioides endemic areas.7 The most common sites of extrapulmonary dissemination include the skin and soft tissues, bones and joints, and the central nervous system (CNS).6 CNS disease has a predilection to manifest as a chronic basilar meningitis, most often complicated by hydrocephalus, vasculitic infarction, and spinal arachnoiditis.8

Cutaneous manifestations of coccidioidomycosis can occur as immunologic phenomenon associated with pulmonary disease or represent skin and soft tissue foci of disseminated infection.9 In primary pulmonary infection, skin findings can range from a nonspecific exanthem to erythema nodosum and erythema multiforme, which are thought to represent hypersensitivity responses. In contrast, Coccidioides spp can infect the skin either through direct inoculation (as in primary cutaneous coccidioidomycosis) or via hematogenous dissemination.9,10 A variety of lesions have been described, with painless nodules being the most frequently encountered morphotype in one study.11,12 On histopathologic examination, these lesions often have features of granulomatous dermatitis, eosinophilic infiltration, gummatous necrosis, microabscesses, or perivascular inflammation.13

Another common and highly morbid site of extrapulmonary dissemination is the musculoskeletal system. Bone and joint coccidioidomycosis most frequently affect the axial skeleton, although peripheral skeletal structures and joints can also be involved.6,12 Vertebral coccidioidomycosis is associated with significant morbidity. A study describing the magnetic resonance imaging findings of patients with vertebral coccidioidomycosis found that Coccidioides spp appeared to have a predilection for the thoracic vertebrae (in up to 80% of the study’s cohort).14 Skip lesions with noncontiguously involved vertebrae occurred in roughly half of patients, highlighting the usefulness of imaging the total spine in suspected cases. 

The diagnosis of coccidioidomycosis is often established through serologic testing or by isolation of Coccidioides spp. on histopathology or culture. Obtaining sputum or tissue may be difficult, so clinicians often rely on noninvasive diagnostic tests such as coccidioidal antigen and serologies by enzyme immunoassays, immunodiffusion, and complement fixation. Enzyme immunoassays IgM and IgG results are positive early in the disease process and need to be confirmed with immunodiffusion or complement fixation testing. Complement fixation IgG is additionally useful to monitor disease activity over time and can help inform risk of disseminated disease.15 The gold standard of diagnosis of disseminated coccidioidomycosis infection remains histopathologic confirmation either by direct visualization of a spherule or growth in fungal cultures.16 Polymerase chain reaction testing of sputum samples is an emerging diagnostic technique that has been found to have similar sensitivity rates to fungal culture.17

Treatment decisions in coccidioidomycosis are complex and vary by site of infection, immune status of the host, and extent of disease.16 While uncomplicated primary pulmonary infections can often be managed with observation alone, prolonged medical therapy with azole antifungals is often recommended for complicated pulmonary infections, symptomatic cavitary disease, and virtually all forms of extrapulmonary disease. Intravenous liposomal amphotericin is often used as initial therapy in immunosuppressed individuals, pregnant women, and those with extensive disease. CNS disease represents a particularly challenging treatment scenario and requires lifelong azole therapy.8,16 

The patient in this case initially presented with vague inflammatory symptoms, with each aliquot revealing further evidence of a metastatic disease process. Such multisystem presentations are diagnostically challenging and force clinicians to reach for some feature around which to build their differential diagnosis. It is with this in mind that we are often taught to “localize the lesion” in order to focus our search for a unifying diagnosis. Yet, in this case, the sheer number of disease foci ultimately helped the discussant to narrow the range of diagnostic possibilities because only a limited number of conditions could present with such widespread, multisystem manifestations. Therefore, this case serves as a reminder that, sometimes in clinical reasoning, “more is less.”

KEY TEACHING POINTS

  • Coccidioidomycosis is a fungal infection that can present with pulmonary or extrapulmonary disease. Risk of extrapulmonary dissemination is greatest among immunocompromised individuals and those of African or Filipino ancestry.3,7
  • The most common sites of extrapulmonary dissemination include the skin and soft tissues, bones and joints, and the CNS.6
  • While serologic testing can be diagnostically useful, the gold standard for diagnosis of disseminated coccidioidomycosis infection remains histopathologic confirmation with direct visualization of a spherule or growth in fungal cultures.16
 
References

1. Benedict K, McCotter OZ, Brady S, et al. Surveillance for Coccidioidomycosis - United States, 2011-2017. MMWR Surveill Summ. 2019;68(No. SS-7):1-15. http://dx.doi.org/10.15585/mmwr.ss6807a1
2. McCotter OZ, Benedict K, Engelthaler DM, et al. Update on the epidemiology of coccidioidomycosis in the United States. Med Mycol. 2019;57(Suppl 1):S30-s40. https://doi.org/10.1093/mmy/myy095
3. Galgiani JN, Ampel NM, Blair JE, et al. Coccidioidomycosis. Clin Infect Dis. 2005;41(9):1217-1223. https://doi.org/10.1086/496991
4. Chang DC, Anderson S, Wannemuehler K, et al. Testing for coccidioidomycosis among patients with community-acquired pneumonia. Emerg Infect Dis. 2008;14(7):1053-1059. https://doi.org/10.3201/eid1407.070832
5. Saubolle MA, McKellar PP, Sussland D. Epidemiologic, clinical, and diagnostic aspects of coccidioidomycosis. J Clin Microbiol. 2007;45(1):26-30. https://doi.org/10.1128/jcm.02230-06
6. Adam RD, Elliott SP, Taljanovic MS. The spectrum and presentation of disseminated coccidioidomycosis. Am J Med. 2009;122(8):770-777. https://doi.org/10.1016/j.amjmed.2008.12.024
7. Wheeler C, Lucas KD, Mohle-Boetani JC. Rates and risk factors for Coccidioidomycosis among prison inmates, California, USA, 2011. Emerg Infect Dis. 2015;21(1):70-75. https://doi.org/10.3201/eid2101.140836
8. Johnson RH, Einstein HE. Coccidioidal meningitis. Clin Infect Dis. 2006;42(1):103-107. https://doi.org/10.1086/497596
9. Blair JE. State-of-the-art treatment of coccidioidomycosis: skin and soft-­tissue infections. Ann N Y Acad Sci. 2007;1111:411-421. https://doi.org/10.1196/annals.1406.010
10. Chang A, Tung RC, McGillis TS, Bergfeld WF, Taylor JS. Primary cutaneous coccidioidomycosis. J Am Acad Dermatol. 2003;49(5):944-949. https://doi.org/10.1016/s0190-9622(03)00462-6
11. Quimby SR, Connolly SM, Winkelmann RK, Smilack JD. Clinicopathologic spectrum of specific cutaneous lesions of disseminated coccidioidomycosis. J Am Acad Dermatol. 1992;26(1):79-85. https://doi.org/10.1016/0190-9622(92)70011-4
12. Crum NF, Lederman ER, Stafford CM, Parrish JS, Wallace MR. Coccidioidomycosis: a descriptive survey of a reemerging disease. clinical characteristics and current controversies. Medicine (Baltimore). 2004;83(3):149-175. https://doi.org/10.1097/01.md.0000126762.91040.fd
13. Carpenter JB, Feldman JS, Leyva WH, DiCaudo DJ. Clinical and pathologic characteristics of disseminated cutaneous coccidioidomycosis. J Am Acad Dermatol. 2010;62(5):831-837. https://doi.org/10.1016/j.jaad.2008.07.031
14. Crete RN, Gallmann W, Karis JP, Ross J. Spinal coccidioidomycosis: MR imaging findings in 41 patients. AJNR Am J Neuroradiol. 2018;39(11):2148-2153. https://doi.org/10.3174/ajnr.a5818
15. McHardy IH, Dinh BN, Waldman S, et al. Coccidioidomycosis complement fixation titer trends in the age of antifungals. J Clin Microbiol. 2018;56(12):e01318-18. https://doi.org/10.1128/jcm.01318-18
16. Galgiani JN, Ampel NM, Blair JE, et al. 2016 Infectious Diseases Society of America (IDSA) clinical practice guideline for the treatment of coccidioidomycosis. Clin Infect Dis. 2016;63(6):e112-e146. https://doi.org/10.1093/cid/ciw360
17. Vucicevic D, Blair JE, Binnicker MJ, et al. The utility of Coccidioides polymerase chain reaction testing in the clinical setting. Mycopathologia. 2010;170(5):345-351. https://doi.org/10.1007/s11046-010-9327-0

References

1. Benedict K, McCotter OZ, Brady S, et al. Surveillance for Coccidioidomycosis - United States, 2011-2017. MMWR Surveill Summ. 2019;68(No. SS-7):1-15. http://dx.doi.org/10.15585/mmwr.ss6807a1
2. McCotter OZ, Benedict K, Engelthaler DM, et al. Update on the epidemiology of coccidioidomycosis in the United States. Med Mycol. 2019;57(Suppl 1):S30-s40. https://doi.org/10.1093/mmy/myy095
3. Galgiani JN, Ampel NM, Blair JE, et al. Coccidioidomycosis. Clin Infect Dis. 2005;41(9):1217-1223. https://doi.org/10.1086/496991
4. Chang DC, Anderson S, Wannemuehler K, et al. Testing for coccidioidomycosis among patients with community-acquired pneumonia. Emerg Infect Dis. 2008;14(7):1053-1059. https://doi.org/10.3201/eid1407.070832
5. Saubolle MA, McKellar PP, Sussland D. Epidemiologic, clinical, and diagnostic aspects of coccidioidomycosis. J Clin Microbiol. 2007;45(1):26-30. https://doi.org/10.1128/jcm.02230-06
6. Adam RD, Elliott SP, Taljanovic MS. The spectrum and presentation of disseminated coccidioidomycosis. Am J Med. 2009;122(8):770-777. https://doi.org/10.1016/j.amjmed.2008.12.024
7. Wheeler C, Lucas KD, Mohle-Boetani JC. Rates and risk factors for Coccidioidomycosis among prison inmates, California, USA, 2011. Emerg Infect Dis. 2015;21(1):70-75. https://doi.org/10.3201/eid2101.140836
8. Johnson RH, Einstein HE. Coccidioidal meningitis. Clin Infect Dis. 2006;42(1):103-107. https://doi.org/10.1086/497596
9. Blair JE. State-of-the-art treatment of coccidioidomycosis: skin and soft-­tissue infections. Ann N Y Acad Sci. 2007;1111:411-421. https://doi.org/10.1196/annals.1406.010
10. Chang A, Tung RC, McGillis TS, Bergfeld WF, Taylor JS. Primary cutaneous coccidioidomycosis. J Am Acad Dermatol. 2003;49(5):944-949. https://doi.org/10.1016/s0190-9622(03)00462-6
11. Quimby SR, Connolly SM, Winkelmann RK, Smilack JD. Clinicopathologic spectrum of specific cutaneous lesions of disseminated coccidioidomycosis. J Am Acad Dermatol. 1992;26(1):79-85. https://doi.org/10.1016/0190-9622(92)70011-4
12. Crum NF, Lederman ER, Stafford CM, Parrish JS, Wallace MR. Coccidioidomycosis: a descriptive survey of a reemerging disease. clinical characteristics and current controversies. Medicine (Baltimore). 2004;83(3):149-175. https://doi.org/10.1097/01.md.0000126762.91040.fd
13. Carpenter JB, Feldman JS, Leyva WH, DiCaudo DJ. Clinical and pathologic characteristics of disseminated cutaneous coccidioidomycosis. J Am Acad Dermatol. 2010;62(5):831-837. https://doi.org/10.1016/j.jaad.2008.07.031
14. Crete RN, Gallmann W, Karis JP, Ross J. Spinal coccidioidomycosis: MR imaging findings in 41 patients. AJNR Am J Neuroradiol. 2018;39(11):2148-2153. https://doi.org/10.3174/ajnr.a5818
15. McHardy IH, Dinh BN, Waldman S, et al. Coccidioidomycosis complement fixation titer trends in the age of antifungals. J Clin Microbiol. 2018;56(12):e01318-18. https://doi.org/10.1128/jcm.01318-18
16. Galgiani JN, Ampel NM, Blair JE, et al. 2016 Infectious Diseases Society of America (IDSA) clinical practice guideline for the treatment of coccidioidomycosis. Clin Infect Dis. 2016;63(6):e112-e146. https://doi.org/10.1093/cid/ciw360
17. Vucicevic D, Blair JE, Binnicker MJ, et al. The utility of Coccidioides polymerase chain reaction testing in the clinical setting. Mycopathologia. 2010;170(5):345-351. https://doi.org/10.1007/s11046-010-9327-0

Issue
Journal of Hospital Medicine 16(4)
Issue
Journal of Hospital Medicine 16(4)
Page Number
239-242. Published Online First September 23, 2020
Page Number
239-242. Published Online First September 23, 2020
Publications
Publications
Topics
Article Type
Sections
Article Source

© 2020 Society of Hospital Medicine

Disallow All Ads
Correspondence Location
Hilary Chan, MD; Email: Hilary.chan@ucsf.edu; Phone: 415-476-1528; Twitter: @hillyc13.
Content Gating
Gated (full article locked unless allowed per User)
Alternative CME
Disqus Comments
Default
Use ProPublica
Hide sidebar & use full width
render the right sidebar.
Conference Recap Checkbox
Not Conference Recap
Clinical Edge
Display the Slideshow in this Article
Gating Strategy
First Page Free
Medscape Article
Display survey writer
Reuters content
Article PDF Media

Left Out in the Cold

Article Type
Changed
Thu, 03/18/2021 - 13:03

A previously healthy 4-year-old boy presented to his pediatrician for nasal congestion, left ear pain, and intermittent fevers, which he’d been experiencing for 2 days. His exam was consistent with acute otitis media. Cefdinir was prescribed given a rash allergy to amoxicillin. His fever, congestion, and otalgia improved the next day.

Three days later he developed abdominal pain, fever, and labored breathing; his mother brought him to the emergency department (ED). His temperature was 38.0 °C, heart rate 141 beats per minute, blood pressure 117/71 mm Hg, respiratory rate 22 breaths per minute; he had oxygen saturation of 96% on ambient air. Despite mild accessory muscle use, he appeared comfortable and interactive. His left tympanic membrane was bulging without erythema. His neck was supple and mucous membranes moist. He had neither cervical lymphadenopathy nor conjunctival pallor. The cardiopulmonary exam was normal except for tachycardia. His abdomen was soft and not distended without organomegaly or tenderness.

Upper respiratory tract symptoms are commonly encountered in pediatrics and most often result from self-limited viral processes. Evaluation of a child with upper respiratory tract symptoms aims to identify serious causes like meningitis, as well as assessing the need for antimicrobial therapy. Supportive management is often appropriate in otitis media. His new, more concerning symptoms portend either a progression of the original process causing his upper respiratory tract symptoms or a separate etiology. It is key to determine which signs and symptoms are associated with the primary process and which are compensatory or secondary. If he were to be more ill appearing, for example, it is possible that his respiratory distress may be related to an underlying systemic illness rather than a primary lung process. Respiratory distress, abdominal pain, and fever could be a result of sepsis from an intrabdominal process such as ruptured appendicitis, intussusception, or malrotation with volvulus. Other causes of sepsis, such as meningitis or severe mastoiditis, both rare complications of otitis media, should be considered, although he does not appear severely ill. Acute myelogenous leukemia or other malignancies and illnesses associated with immunodeficiency can present with sepsis and chloromas in the middle ear that can be misconstrued as otitis media.

A chest radiograph demonstrated left lower lobe patchy opacities concerning for pneumonia. Rapid respiratory syncytial virus and influenza antigen test results were negative. Laboratory testing for general bloodwork was not obtained. He was administered a single dose of intramuscular ceftriaxone, prescribed a 5-day course of azithromycin, and discharged home. The child’s breathing gradually improved, but he continued to have subjective fevers. Two days later, he developed dark red urine. His mother brought him back to the outpatient clinic.

At the time of the ED visit, a diagnosis of community-acquired pneumonia was plausible given fever, mildly increased work of breathing, and an opacification on chest radiography. Most community-acquired pneumonia is caused by viruses; common bacterial causes for his age include Streptococcus pneumoniae and Moraxella catarrhalis. The first-line treatment for uncomplicated community-acquired pneumonia in children is amoxicillin, but this was appropriately avoided given his allergy.

The persistent fevers are surprising. The improvement in breathing corresponds to the treatment (and resolution) of community-acquired pneumonia. However, the development of dark urine does not. Red urine—in the absence of ingested pigments (such as those found in beets)—usually results from hematuria, hemoglobinuria, or myoglobinuria. Gross hematuria can originate from the kidneys to the urethral meatus. Abdominal masses, kidney trauma, or underlying kidney disease may all present with gross hematuria (or microscopic hematuria, seen only on urinalysis). The urine should be examined for the presence of heme, protein, and for evidence of infection; microscopy should be performed to examine for cellular casts and dysmorphic red cells. Tests of renal function, a comprehensive metabolic panel, evaluation of hematologic indexes, and assessments of inflammatory markers should be performed.

The child lived with his parents and had no siblings. He experienced no physical trauma, and there was no family history of kidney disease or hematuria. His father had a persistent cough and fever for 1 month, but recovered around the time the patient began to experience his initial symptoms. This was the patient’s third diagnosis of pneumonia. He had not traveled and was up to date with immunizations. He attended day care.

The fact that this is not the first episode of “pneumonia” raises important possibilities. The most likely one is that the child has had multiple viral infections; however, he could have an underlying primary immunodeficiency (PI) that predisposes him to recurrent infections. More severe PIs often present with recurrent sepsis, bacteremia, and failure to thrive, none of which were present in this case. Less severe PIs (such as selective IgA deficiency) could be possible. Another possibility is that these recurrent episodes of pneumonia are a relapsing and remitting noninfectious process, such as an antineutrophil cytoplasmic antibodies–associated vasculitis or anti–glomerular basement membrane disease. The patient’s father’s recent prolonged respiratory symptoms may be suggestive of pertussis or a “walking pneumonia” potentially caused by Mycoplasma or another atypical bacterium.

His temperature was 36.9 °C, heart rate 107 beats per minute, blood pressure was 106/67 mm Hg, and respiratory rate was 24 breaths per minute with oxygen saturation of 100% on ambient air. He was well appearing. His mucous membranes were moist, and oropharynx was clear. He had scleral icterus. The cardiopulmonary exam was normal. He had no significant lymphadenopathy, hepatosplenomegaly, or rashes.

The finding of jaundice is an important diagnostic pivot point, especially when combined with hematuria. The next step is determining if the jaundice is resulting from unconjugated or conjugated hyperbilirubinemia; the former most often stems from hemolysis or impairment in conjugation, while the latter results from intrahepatic or extrahepatic biliary defects. Tests for hepatobiliary injury including evaluations of alanine and aspartate aminotransferases and alkaline phosphatase, as well as for hepatic function such as tests of coagulation, should be performed.

The patient was referred to the ED and admitted for further evaluation. A complete blood count revealed a white blood cell (WBC) count of 10,700/µL (61% polymorphonuclear neutrophils, 30% lymphocytes, 5% monocytes, 3% eosinophils, 1% basophils), hemoglobin count was 10.3 g/dL (reticulocyte 2% with absolute reticulocyte count 58,400/μL), and platelet count was 265,000/µL. Components of the basic metabolic panel were within reference ranges except for a mildly elevated blood urea nitrogen level of 14 mg/dL with normal creatinine level of 0.3 mg/dL. Total protein was 6.7 g/dL (reference range, 6.4-8.3) and albumin 3.9 g/dL (reference range, 3.4-4.8). Alkaline phosphatase level was 188 U/L (reference range, 44-147), aspartate aminotransferase level 76 U/L (reference range, 0-40), and alanine aminotransferase level 12 U/L (reference range, 7-40). Total bilirubin level was 2.4 mg/dL (reference range, less than 1.5) with direct bilirubin level of 0.4 mg/dL. His C-reactive protein level was 1.5 mg/mL (reference range, 0-0.75). Creatinine kinase (CK) level was 2,550 U/L (reference range, 2-198). International Normalized Ratio (INR) was 1.0. Urinalysis was notable for 2+ proteinuria, large hemoglobin pigment, and 6 red blood cells per high power field (reference range, 0-4).

His blood urea nitrogen is elevated, reflecting either prerenal azotemia or increased absorption of nitrogenous products. Unconjugated hyperbilirubinemia may result from impaired hepatic bilirubin uptake (such as in heart failure or portosystemic shunts), impaired bilirubin conjugation (resulting from genetic conditions or drugs), or excess bilirubin production (such as in hemolysis); his anemia and lack of other evidence of hepatic dysfunction point to hemolysis as the etiology. The reticulocyte production index is approximately 1%, which suggests that an increase in erythrocyte generation is present but inadequate. This, however, does not mean that an erythrocyte production abnormality is present since reticulocytosis can be delayed in many cases of acute hemolytic anemia. It is also possible that the same hemolytic process is affecting mature and immature erythrocytes. A peripheral blood smear should be reviewed for evidence of intravascular hemolysis and testing for autoimmune hemolysis should be performed. Notably, his white blood cell and platelet counts are preserved, which makes a bone marrow–involved malignancy or infiltrative process less likely. The alkaline phosphatase elevation may result from either intrahepatic or extrahepatic biliopathy; bone damage is also possible. The elevation of aspartate aminotransferase, CK, and potassium, along with marked urinary heme pigment, may indicate muscle damage; the most common myositis in children is benign acute childhood myositis resulting from viral infection. However, the moderate level of CK elevation seen in this case is nonspecific and can result from many different etiologies. A metabolic myopathy, such as carnitine palmitoyltransferase II deficiency, can be made worse by metabolic stress and result in rhabdomyolysis; the presentations of inborn errors of metabolism are varied and a planned-out, stepwise approach in evaluation is fundamental.

Lactic acid dehydrogenase (LDH) level was 1,457 U/L (reference range, 140-280), and haptoglobin level was less than 6 mg/dL (reference range, 30-200). Peripheral blood smear demonstrated occasional atypical, reactive-appearing lymphocytes with red cell clumping and agglutination, as well as rare target, burr, and fragmented red cells. Test results for urine myoglobin were negative. Results of urine culture were negative. No blood culture was collected.

The elevated LDH, decreased haptoglobin, and findings on the peripheral blood smear confirm hemolysis. The clumping of erythrocytes can be artifactual in the preparation of peripheral smears, but when considered in the context of hemolysis, may be clinically important. Clumping of erythrocytes on the peripheral smear indicates the binding of a protein to antigens on the erythrocyte membrane; when this occurs below body temperature, this is consistent with the presence of a “cold agglutinin,” usually an IgM antibody directed at erythrocyte surface antigens that causes agglutination and destruction, especially in cooler areas of the body. This is a well-known complication of Mycoplasma pneumoniae infections as well as Epstein-Barr virus (EBV) infections; it may also occur with lymphoid malignancies or autoimmune disease.

Direct Coombs IgG test findings were negative, direct Coombs C3 test was positive, and direct Coombs polyspecific test was positive. M pneumoniae IgG antibody level was 1.4 mg/dL (reference ranges: <0.9, negative; 0.91-1.09, equivocal; >1.1, positive); M pneumoniae IgM level was 529 U/mL (reference range: <770, negative). EBV capsid IgM and IgG levels were undetectable. EBV nuclear antigen IgG level was also undetectable. EBV viral load was fewer than 10 copies/mL. Antinuclear antibodies (ANA) level was negative. General IgE and IgM levels were normal, at 11 and 81 mg/dL, respectively. Repeat complete blood count showed WBC of 7,800/µL, hemoglobin of 8.7 g/dL, and platelet count of 341,000/µL. The patient’s hemoglobin remained stable during hospitalization.

This directed testing is helpful in further classifying the patient’s hemolytic anemia. Autoimmune hemolytic anemias are classified into warm antibody–mediated, cold antibody–mediated, and mixed-type forms; drug-induced and alloimmune hemolytic anemias also occur. In addition, both systemic lupus erythematosus and antiphospholipid antibody syndrome can have hemolytic anemia with variable Coombs testing results; neither fit well in this case. The absence of red blood cell–directed IgG antibodies substantially decreases the likelihood of warm antibody–mediated hemolytic anemia. In cold antibody–mediated hemolytic anemia, antibodies bind to the erythrocyte membrane and then adhere to complement C3, which leads to both intravascular and extravascular hemolysis. Important types of cold antibody–mediated hemolytic anemia in children are primary and secondary cold agglutinin disease, along with paroxysmal cold hemoglobinuria. The Donath-Landsteiner test can be helpful in differentiating these conditions. Antibodies to Mycoplasma may be delayed in response to acute infection, and a child who is reinfected may only produce IgG antibodies. Given the patient’s clinical stability and previous health, the most likely diagnosis is Mycoplasma-induced cold antibody–mediated hemolytic anemia. It may be helpful to check convalescent titers to Mycoplasma in 2 to 4 weeks.

Donath-Landsteiner (D-L) antibody test results were positive. Medication-derived hemolytic anemia testing was conducted, but the presence of positive D-L antibody makes the test results inconclusive. This ultimately led to a diagnosis of paroxysmal cold hemoglobinuria (PCH), presumably triggered by a viral syndrome. Convalescent titers to Mycoplasma were not checked given clinical improvement. Because the patient’s hemoglobin was stable during hospitalization, he was not treated with steroids. His parents were counseled on avoiding cold temperatures for several days. Within 1 month, his hemoglobin had recovered without further evidence of hemolysis.

DISCUSSION

Hemolytic anemia refers to the accelerated destruction of red blood cells and can be further classified as acquired or hereditary.1 Hereditary conditions causing hemolytic anemia include enzymopathies (eg, glucose-6-phosphate dehydrogenase deficiency), hemoglobinopathies (eg, sickle cell disease), and membrane abnormalities (eg, hereditary spherocytosis). Acquired pathologies include microangiopathic hemolytic anemia (MAHA), anemias directly caused by certain infections such as malaria, and immune-mediated (Coombs-positive) hemolytic anemias.

MAHA can sometimes be life-threatening and is therefore important to identify quickly. In the right clinical context, such processes may be rapidly recognized by the presence of schistocytes on blood smear in addition to an elevated serum LDH level. Schistocytes suggest mechanical destruction of erythrocytes in the vasculature, the hallmark of MAHA. Important MAHAs include thrombocytopenic purpura, hemolytic-uremic syndrome, and disseminated intravascular coagulation. Though this patient did have a mildly elevated LDH, MAHA was less likely because there were no schistocytes on the blood smear.

Autoimmune hemolytic anemias (AIHAs) are another important subset of acquired hemolytic anemias. AIHAs occur when there is antibody-mediated destruction of erythrocytes. The direct Coombs test evaluates for antibody- or complement-­coated erythrocytes. After administration of anti-IgG and anti-­C3 serum, the test evaluates for agglutination of the red cells caused by attached antibodies or complement. Coombs-­positive AIHA can also be categorized by the temperature of agglutination. “Warm” hemolysis often involves IgG autoantibodies (ie, warm agglutinins), while “cold” antibodies, usually IgM autoantibodies, bind at colder temperatures (0-4 °C) and activate complements, including C3. In this patient, the Coombs C3 was positive while the Coombs IgG was negative, which is more suggestive of a cold complement–mediated pathway.

Cold AIHA can be further categorized into primary cold agglutinin disease, secondary cold agglutinin disease, and PCH. Primary cold agglutinin disease is an autoimmune disorder that mostly occurs in adults. Secondary cold AIHA can often be triggered by bacterial infection (commonly M pneumoniae) or viruses including EBV, measles, and mumps.2 Medications, including penicillin and cephalosporins, can also be implicated. Secondary cold AIHA is also linked with autoimmune diseases, such as systemic lupus erythematosus and lymphoproliferative disorders. PCH can be identified with the unique presence of a specific autoantibody (ie, D-L autoantibody) that agglutinates at cold temperatures but dissociates on subsequent rewarming.3 Complement remains affixed and activates hemolysis.

The D-L antibody responsible for PCH is an IgG antibody to the P-antigen present on the erythrocyte surface. Since the Coombs test is conducted at normal temperature, it will be positive for the affixed complement but not for IgG. The underlying mechanism for PCH was proposed by Julius Donath, MD, and Karl Landsteiner, MD, in 1904 and is considered to be the first description of autoimmune disease being precipitated by antibodies.4 The D-L antibody test itself is uncommonly performed and somewhat difficult to interpret, particularly in adults, and may lead to false-negative results.5

PCH is an acquired, cold AIHA more common to children6,7 and may account for up to 33% of pediatric AIHA cases.8 Typical presentation is after an upper respiratory tract illness; however, the trigger is often not identified. Implicated triggers include a number of viruses.9 Clinical presentation includes findings of intravascular hemolysis similar to those in our patient. The pathogenic IgG autoantibody is polyclonal and is likely formed because of immune stimulation, which is consistent with the predominance of nonmalignant triggers of this disease process.10 Hemolysis and associated symptoms are often exacerbated with cold exposure; both typically resolve within 2 weeks. In recurrent cases, which are a minority, immunosuppression may be considered.10

PCH remains an often-understated cause of hemolytic anemia particularly in children. Lacking obvious pathognomonic clinical symptoms, it may be overlooked for other forms of AIHA or MAHA. However, with a structured approach to evaluation, as with this patient who had hematuria and jaundice, early diagnosis can prevent an unnecessarily extensive workup and can provide reassurance to patient and parents. By understanding the basic categories of hemolytic anemia, the relevant blood testing available, and interpretation of Coombs test results, clinicians can ensure that PCH is a diagnosis that is not left out in the cold.

KEY TEACHING POINTS

  • Examination for schistocytes on a blood smear can help identify life-threatening causes of hemolytic anemia.
  • Characterization of cold AIHA includes defining the underlying etiology as primary cold agglutinin disease, secondary cold agglutinin disease, or PCH.
  • PCH is a cold AIHA that is an underrecognized cause of hemolytic anemia in children. The diagnosis of PCH is made by testing for the presence of the D-L antibody.
 
References

1. Dhaliwal G, Cornett PA, Tierney LM Jr. Hemolytic anemia. Am Fam Physician. 2004;69(11):2599-2606.
2. Djaldetti M. Paroxysmal cold hemoglobinuria. CRC Crit Rev Clin Lab Sci. 1978;9(1):49-83. https://doi.org/10.3109/10408367809150915
3. Levine P, Celano MJ, Falkowski F. The specificity of the antibody in paroxysmal cold hemoglobinuria (P.C.H.). Transfusion. 1963;3(4):278-280. https://doi.org/10.1111/j.1537-2995.1963.tb04643.x
4. Donath J, Landsteiner K. Uber Paroxysmale Hamoglobinurie. Munch Med Wochenschr. 1904;51:1590-1593
5. Zeller MP, Arnold DM, Al Habsi K, et al. Paroxysmal cold hemoglobinuria: a difficult diagnosis in adult patients. Transfusion. 2017;57(1):137-143. https://doi.org/10.1111/trf.13888
6. Göttsche B, Salama A, Mueller-Eckhardt C. Donath-Landsteiner autoimmune hemolytic anemia in children. a study of 22 cases. Vox Sang. 1990;58(4):281-286. https://doi.org/10.1111/j.1423-0410.1990.tb05000.x
7. Sokol RJ, Booker DJ, Stamps R. Erythropoiesis: paroxysmal cold haemoglobinuria: a clinico-pathological study of patients with a positive Donath-­Landsteiner test. Hematology. 1999;4(2):137-164. https://doi.org/10.1080/10245332.1999.11746439
8. Petz LD. Cold antibody autoimmune hemolytic anemias. Blood Rev. 2008;22(1):1-15. https://doi.org/10.1016/j.blre.2007.08.002
9. Leibrandt R, Angelino K, Vizel-Schwartz M, Shapira I. Paroxysmal cold hemoglobinuria in an adult with respiratory syncytial virus. Case Rep Hematol. 2018;2018:1-3. https://doi.org/10.1155/2018/7586719
10. Gertz MA. Management of cold haemolytic syndrome. Br J Haematol. 2007;138(4):422-429. https://doi.org/10.1111/j.1365-2141.2007.06664.x

Article PDF
Author and Disclosure Information

1Departments of Medicine and Pediatrics, Warren Alpert School of Medicine at Brown University, Providence, Rhode Island; 2Departments of Medicine and Pediatrics, University of Minnesota Medical School, Minneapolis, Minnesota; 3Department of Medicine, University of California San Francisco, San Francisco, California; 4Medical Service, San Francisco VA Medical Center, San Francisco, California; 5Section of Hematology-Oncology, Department of Medicine, University of Chicago, Chicago, Illinois; 6Department of Internal Medicine, Johns Hopkins Hospital, Baltimore, Maryland.

Disclosures

Dr Patel reported receiving an honorarium from the Human Diagnosis Project. The other authors reported having nothing to disclose.

Issue
Journal of Hospital Medicine 16(2)
Publications
Topics
Page Number
105-108. Published Online First August 19, 2020
Sections
Author and Disclosure Information

1Departments of Medicine and Pediatrics, Warren Alpert School of Medicine at Brown University, Providence, Rhode Island; 2Departments of Medicine and Pediatrics, University of Minnesota Medical School, Minneapolis, Minnesota; 3Department of Medicine, University of California San Francisco, San Francisco, California; 4Medical Service, San Francisco VA Medical Center, San Francisco, California; 5Section of Hematology-Oncology, Department of Medicine, University of Chicago, Chicago, Illinois; 6Department of Internal Medicine, Johns Hopkins Hospital, Baltimore, Maryland.

Disclosures

Dr Patel reported receiving an honorarium from the Human Diagnosis Project. The other authors reported having nothing to disclose.

Author and Disclosure Information

1Departments of Medicine and Pediatrics, Warren Alpert School of Medicine at Brown University, Providence, Rhode Island; 2Departments of Medicine and Pediatrics, University of Minnesota Medical School, Minneapolis, Minnesota; 3Department of Medicine, University of California San Francisco, San Francisco, California; 4Medical Service, San Francisco VA Medical Center, San Francisco, California; 5Section of Hematology-Oncology, Department of Medicine, University of Chicago, Chicago, Illinois; 6Department of Internal Medicine, Johns Hopkins Hospital, Baltimore, Maryland.

Disclosures

Dr Patel reported receiving an honorarium from the Human Diagnosis Project. The other authors reported having nothing to disclose.

Article PDF
Article PDF
Related Articles

A previously healthy 4-year-old boy presented to his pediatrician for nasal congestion, left ear pain, and intermittent fevers, which he’d been experiencing for 2 days. His exam was consistent with acute otitis media. Cefdinir was prescribed given a rash allergy to amoxicillin. His fever, congestion, and otalgia improved the next day.

Three days later he developed abdominal pain, fever, and labored breathing; his mother brought him to the emergency department (ED). His temperature was 38.0 °C, heart rate 141 beats per minute, blood pressure 117/71 mm Hg, respiratory rate 22 breaths per minute; he had oxygen saturation of 96% on ambient air. Despite mild accessory muscle use, he appeared comfortable and interactive. His left tympanic membrane was bulging without erythema. His neck was supple and mucous membranes moist. He had neither cervical lymphadenopathy nor conjunctival pallor. The cardiopulmonary exam was normal except for tachycardia. His abdomen was soft and not distended without organomegaly or tenderness.

Upper respiratory tract symptoms are commonly encountered in pediatrics and most often result from self-limited viral processes. Evaluation of a child with upper respiratory tract symptoms aims to identify serious causes like meningitis, as well as assessing the need for antimicrobial therapy. Supportive management is often appropriate in otitis media. His new, more concerning symptoms portend either a progression of the original process causing his upper respiratory tract symptoms or a separate etiology. It is key to determine which signs and symptoms are associated with the primary process and which are compensatory or secondary. If he were to be more ill appearing, for example, it is possible that his respiratory distress may be related to an underlying systemic illness rather than a primary lung process. Respiratory distress, abdominal pain, and fever could be a result of sepsis from an intrabdominal process such as ruptured appendicitis, intussusception, or malrotation with volvulus. Other causes of sepsis, such as meningitis or severe mastoiditis, both rare complications of otitis media, should be considered, although he does not appear severely ill. Acute myelogenous leukemia or other malignancies and illnesses associated with immunodeficiency can present with sepsis and chloromas in the middle ear that can be misconstrued as otitis media.

A chest radiograph demonstrated left lower lobe patchy opacities concerning for pneumonia. Rapid respiratory syncytial virus and influenza antigen test results were negative. Laboratory testing for general bloodwork was not obtained. He was administered a single dose of intramuscular ceftriaxone, prescribed a 5-day course of azithromycin, and discharged home. The child’s breathing gradually improved, but he continued to have subjective fevers. Two days later, he developed dark red urine. His mother brought him back to the outpatient clinic.

At the time of the ED visit, a diagnosis of community-acquired pneumonia was plausible given fever, mildly increased work of breathing, and an opacification on chest radiography. Most community-acquired pneumonia is caused by viruses; common bacterial causes for his age include Streptococcus pneumoniae and Moraxella catarrhalis. The first-line treatment for uncomplicated community-acquired pneumonia in children is amoxicillin, but this was appropriately avoided given his allergy.

The persistent fevers are surprising. The improvement in breathing corresponds to the treatment (and resolution) of community-acquired pneumonia. However, the development of dark urine does not. Red urine—in the absence of ingested pigments (such as those found in beets)—usually results from hematuria, hemoglobinuria, or myoglobinuria. Gross hematuria can originate from the kidneys to the urethral meatus. Abdominal masses, kidney trauma, or underlying kidney disease may all present with gross hematuria (or microscopic hematuria, seen only on urinalysis). The urine should be examined for the presence of heme, protein, and for evidence of infection; microscopy should be performed to examine for cellular casts and dysmorphic red cells. Tests of renal function, a comprehensive metabolic panel, evaluation of hematologic indexes, and assessments of inflammatory markers should be performed.

The child lived with his parents and had no siblings. He experienced no physical trauma, and there was no family history of kidney disease or hematuria. His father had a persistent cough and fever for 1 month, but recovered around the time the patient began to experience his initial symptoms. This was the patient’s third diagnosis of pneumonia. He had not traveled and was up to date with immunizations. He attended day care.

The fact that this is not the first episode of “pneumonia” raises important possibilities. The most likely one is that the child has had multiple viral infections; however, he could have an underlying primary immunodeficiency (PI) that predisposes him to recurrent infections. More severe PIs often present with recurrent sepsis, bacteremia, and failure to thrive, none of which were present in this case. Less severe PIs (such as selective IgA deficiency) could be possible. Another possibility is that these recurrent episodes of pneumonia are a relapsing and remitting noninfectious process, such as an antineutrophil cytoplasmic antibodies–associated vasculitis or anti–glomerular basement membrane disease. The patient’s father’s recent prolonged respiratory symptoms may be suggestive of pertussis or a “walking pneumonia” potentially caused by Mycoplasma or another atypical bacterium.

His temperature was 36.9 °C, heart rate 107 beats per minute, blood pressure was 106/67 mm Hg, and respiratory rate was 24 breaths per minute with oxygen saturation of 100% on ambient air. He was well appearing. His mucous membranes were moist, and oropharynx was clear. He had scleral icterus. The cardiopulmonary exam was normal. He had no significant lymphadenopathy, hepatosplenomegaly, or rashes.

The finding of jaundice is an important diagnostic pivot point, especially when combined with hematuria. The next step is determining if the jaundice is resulting from unconjugated or conjugated hyperbilirubinemia; the former most often stems from hemolysis or impairment in conjugation, while the latter results from intrahepatic or extrahepatic biliary defects. Tests for hepatobiliary injury including evaluations of alanine and aspartate aminotransferases and alkaline phosphatase, as well as for hepatic function such as tests of coagulation, should be performed.

The patient was referred to the ED and admitted for further evaluation. A complete blood count revealed a white blood cell (WBC) count of 10,700/µL (61% polymorphonuclear neutrophils, 30% lymphocytes, 5% monocytes, 3% eosinophils, 1% basophils), hemoglobin count was 10.3 g/dL (reticulocyte 2% with absolute reticulocyte count 58,400/μL), and platelet count was 265,000/µL. Components of the basic metabolic panel were within reference ranges except for a mildly elevated blood urea nitrogen level of 14 mg/dL with normal creatinine level of 0.3 mg/dL. Total protein was 6.7 g/dL (reference range, 6.4-8.3) and albumin 3.9 g/dL (reference range, 3.4-4.8). Alkaline phosphatase level was 188 U/L (reference range, 44-147), aspartate aminotransferase level 76 U/L (reference range, 0-40), and alanine aminotransferase level 12 U/L (reference range, 7-40). Total bilirubin level was 2.4 mg/dL (reference range, less than 1.5) with direct bilirubin level of 0.4 mg/dL. His C-reactive protein level was 1.5 mg/mL (reference range, 0-0.75). Creatinine kinase (CK) level was 2,550 U/L (reference range, 2-198). International Normalized Ratio (INR) was 1.0. Urinalysis was notable for 2+ proteinuria, large hemoglobin pigment, and 6 red blood cells per high power field (reference range, 0-4).

His blood urea nitrogen is elevated, reflecting either prerenal azotemia or increased absorption of nitrogenous products. Unconjugated hyperbilirubinemia may result from impaired hepatic bilirubin uptake (such as in heart failure or portosystemic shunts), impaired bilirubin conjugation (resulting from genetic conditions or drugs), or excess bilirubin production (such as in hemolysis); his anemia and lack of other evidence of hepatic dysfunction point to hemolysis as the etiology. The reticulocyte production index is approximately 1%, which suggests that an increase in erythrocyte generation is present but inadequate. This, however, does not mean that an erythrocyte production abnormality is present since reticulocytosis can be delayed in many cases of acute hemolytic anemia. It is also possible that the same hemolytic process is affecting mature and immature erythrocytes. A peripheral blood smear should be reviewed for evidence of intravascular hemolysis and testing for autoimmune hemolysis should be performed. Notably, his white blood cell and platelet counts are preserved, which makes a bone marrow–involved malignancy or infiltrative process less likely. The alkaline phosphatase elevation may result from either intrahepatic or extrahepatic biliopathy; bone damage is also possible. The elevation of aspartate aminotransferase, CK, and potassium, along with marked urinary heme pigment, may indicate muscle damage; the most common myositis in children is benign acute childhood myositis resulting from viral infection. However, the moderate level of CK elevation seen in this case is nonspecific and can result from many different etiologies. A metabolic myopathy, such as carnitine palmitoyltransferase II deficiency, can be made worse by metabolic stress and result in rhabdomyolysis; the presentations of inborn errors of metabolism are varied and a planned-out, stepwise approach in evaluation is fundamental.

Lactic acid dehydrogenase (LDH) level was 1,457 U/L (reference range, 140-280), and haptoglobin level was less than 6 mg/dL (reference range, 30-200). Peripheral blood smear demonstrated occasional atypical, reactive-appearing lymphocytes with red cell clumping and agglutination, as well as rare target, burr, and fragmented red cells. Test results for urine myoglobin were negative. Results of urine culture were negative. No blood culture was collected.

The elevated LDH, decreased haptoglobin, and findings on the peripheral blood smear confirm hemolysis. The clumping of erythrocytes can be artifactual in the preparation of peripheral smears, but when considered in the context of hemolysis, may be clinically important. Clumping of erythrocytes on the peripheral smear indicates the binding of a protein to antigens on the erythrocyte membrane; when this occurs below body temperature, this is consistent with the presence of a “cold agglutinin,” usually an IgM antibody directed at erythrocyte surface antigens that causes agglutination and destruction, especially in cooler areas of the body. This is a well-known complication of Mycoplasma pneumoniae infections as well as Epstein-Barr virus (EBV) infections; it may also occur with lymphoid malignancies or autoimmune disease.

Direct Coombs IgG test findings were negative, direct Coombs C3 test was positive, and direct Coombs polyspecific test was positive. M pneumoniae IgG antibody level was 1.4 mg/dL (reference ranges: <0.9, negative; 0.91-1.09, equivocal; >1.1, positive); M pneumoniae IgM level was 529 U/mL (reference range: <770, negative). EBV capsid IgM and IgG levels were undetectable. EBV nuclear antigen IgG level was also undetectable. EBV viral load was fewer than 10 copies/mL. Antinuclear antibodies (ANA) level was negative. General IgE and IgM levels were normal, at 11 and 81 mg/dL, respectively. Repeat complete blood count showed WBC of 7,800/µL, hemoglobin of 8.7 g/dL, and platelet count of 341,000/µL. The patient’s hemoglobin remained stable during hospitalization.

This directed testing is helpful in further classifying the patient’s hemolytic anemia. Autoimmune hemolytic anemias are classified into warm antibody–mediated, cold antibody–mediated, and mixed-type forms; drug-induced and alloimmune hemolytic anemias also occur. In addition, both systemic lupus erythematosus and antiphospholipid antibody syndrome can have hemolytic anemia with variable Coombs testing results; neither fit well in this case. The absence of red blood cell–directed IgG antibodies substantially decreases the likelihood of warm antibody–mediated hemolytic anemia. In cold antibody–mediated hemolytic anemia, antibodies bind to the erythrocyte membrane and then adhere to complement C3, which leads to both intravascular and extravascular hemolysis. Important types of cold antibody–mediated hemolytic anemia in children are primary and secondary cold agglutinin disease, along with paroxysmal cold hemoglobinuria. The Donath-Landsteiner test can be helpful in differentiating these conditions. Antibodies to Mycoplasma may be delayed in response to acute infection, and a child who is reinfected may only produce IgG antibodies. Given the patient’s clinical stability and previous health, the most likely diagnosis is Mycoplasma-induced cold antibody–mediated hemolytic anemia. It may be helpful to check convalescent titers to Mycoplasma in 2 to 4 weeks.

Donath-Landsteiner (D-L) antibody test results were positive. Medication-derived hemolytic anemia testing was conducted, but the presence of positive D-L antibody makes the test results inconclusive. This ultimately led to a diagnosis of paroxysmal cold hemoglobinuria (PCH), presumably triggered by a viral syndrome. Convalescent titers to Mycoplasma were not checked given clinical improvement. Because the patient’s hemoglobin was stable during hospitalization, he was not treated with steroids. His parents were counseled on avoiding cold temperatures for several days. Within 1 month, his hemoglobin had recovered without further evidence of hemolysis.

DISCUSSION

Hemolytic anemia refers to the accelerated destruction of red blood cells and can be further classified as acquired or hereditary.1 Hereditary conditions causing hemolytic anemia include enzymopathies (eg, glucose-6-phosphate dehydrogenase deficiency), hemoglobinopathies (eg, sickle cell disease), and membrane abnormalities (eg, hereditary spherocytosis). Acquired pathologies include microangiopathic hemolytic anemia (MAHA), anemias directly caused by certain infections such as malaria, and immune-mediated (Coombs-positive) hemolytic anemias.

MAHA can sometimes be life-threatening and is therefore important to identify quickly. In the right clinical context, such processes may be rapidly recognized by the presence of schistocytes on blood smear in addition to an elevated serum LDH level. Schistocytes suggest mechanical destruction of erythrocytes in the vasculature, the hallmark of MAHA. Important MAHAs include thrombocytopenic purpura, hemolytic-uremic syndrome, and disseminated intravascular coagulation. Though this patient did have a mildly elevated LDH, MAHA was less likely because there were no schistocytes on the blood smear.

Autoimmune hemolytic anemias (AIHAs) are another important subset of acquired hemolytic anemias. AIHAs occur when there is antibody-mediated destruction of erythrocytes. The direct Coombs test evaluates for antibody- or complement-­coated erythrocytes. After administration of anti-IgG and anti-­C3 serum, the test evaluates for agglutination of the red cells caused by attached antibodies or complement. Coombs-­positive AIHA can also be categorized by the temperature of agglutination. “Warm” hemolysis often involves IgG autoantibodies (ie, warm agglutinins), while “cold” antibodies, usually IgM autoantibodies, bind at colder temperatures (0-4 °C) and activate complements, including C3. In this patient, the Coombs C3 was positive while the Coombs IgG was negative, which is more suggestive of a cold complement–mediated pathway.

Cold AIHA can be further categorized into primary cold agglutinin disease, secondary cold agglutinin disease, and PCH. Primary cold agglutinin disease is an autoimmune disorder that mostly occurs in adults. Secondary cold AIHA can often be triggered by bacterial infection (commonly M pneumoniae) or viruses including EBV, measles, and mumps.2 Medications, including penicillin and cephalosporins, can also be implicated. Secondary cold AIHA is also linked with autoimmune diseases, such as systemic lupus erythematosus and lymphoproliferative disorders. PCH can be identified with the unique presence of a specific autoantibody (ie, D-L autoantibody) that agglutinates at cold temperatures but dissociates on subsequent rewarming.3 Complement remains affixed and activates hemolysis.

The D-L antibody responsible for PCH is an IgG antibody to the P-antigen present on the erythrocyte surface. Since the Coombs test is conducted at normal temperature, it will be positive for the affixed complement but not for IgG. The underlying mechanism for PCH was proposed by Julius Donath, MD, and Karl Landsteiner, MD, in 1904 and is considered to be the first description of autoimmune disease being precipitated by antibodies.4 The D-L antibody test itself is uncommonly performed and somewhat difficult to interpret, particularly in adults, and may lead to false-negative results.5

PCH is an acquired, cold AIHA more common to children6,7 and may account for up to 33% of pediatric AIHA cases.8 Typical presentation is after an upper respiratory tract illness; however, the trigger is often not identified. Implicated triggers include a number of viruses.9 Clinical presentation includes findings of intravascular hemolysis similar to those in our patient. The pathogenic IgG autoantibody is polyclonal and is likely formed because of immune stimulation, which is consistent with the predominance of nonmalignant triggers of this disease process.10 Hemolysis and associated symptoms are often exacerbated with cold exposure; both typically resolve within 2 weeks. In recurrent cases, which are a minority, immunosuppression may be considered.10

PCH remains an often-understated cause of hemolytic anemia particularly in children. Lacking obvious pathognomonic clinical symptoms, it may be overlooked for other forms of AIHA or MAHA. However, with a structured approach to evaluation, as with this patient who had hematuria and jaundice, early diagnosis can prevent an unnecessarily extensive workup and can provide reassurance to patient and parents. By understanding the basic categories of hemolytic anemia, the relevant blood testing available, and interpretation of Coombs test results, clinicians can ensure that PCH is a diagnosis that is not left out in the cold.

KEY TEACHING POINTS

  • Examination for schistocytes on a blood smear can help identify life-threatening causes of hemolytic anemia.
  • Characterization of cold AIHA includes defining the underlying etiology as primary cold agglutinin disease, secondary cold agglutinin disease, or PCH.
  • PCH is a cold AIHA that is an underrecognized cause of hemolytic anemia in children. The diagnosis of PCH is made by testing for the presence of the D-L antibody.
 

A previously healthy 4-year-old boy presented to his pediatrician for nasal congestion, left ear pain, and intermittent fevers, which he’d been experiencing for 2 days. His exam was consistent with acute otitis media. Cefdinir was prescribed given a rash allergy to amoxicillin. His fever, congestion, and otalgia improved the next day.

Three days later he developed abdominal pain, fever, and labored breathing; his mother brought him to the emergency department (ED). His temperature was 38.0 °C, heart rate 141 beats per minute, blood pressure 117/71 mm Hg, respiratory rate 22 breaths per minute; he had oxygen saturation of 96% on ambient air. Despite mild accessory muscle use, he appeared comfortable and interactive. His left tympanic membrane was bulging without erythema. His neck was supple and mucous membranes moist. He had neither cervical lymphadenopathy nor conjunctival pallor. The cardiopulmonary exam was normal except for tachycardia. His abdomen was soft and not distended without organomegaly or tenderness.

Upper respiratory tract symptoms are commonly encountered in pediatrics and most often result from self-limited viral processes. Evaluation of a child with upper respiratory tract symptoms aims to identify serious causes like meningitis, as well as assessing the need for antimicrobial therapy. Supportive management is often appropriate in otitis media. His new, more concerning symptoms portend either a progression of the original process causing his upper respiratory tract symptoms or a separate etiology. It is key to determine which signs and symptoms are associated with the primary process and which are compensatory or secondary. If he were to be more ill appearing, for example, it is possible that his respiratory distress may be related to an underlying systemic illness rather than a primary lung process. Respiratory distress, abdominal pain, and fever could be a result of sepsis from an intrabdominal process such as ruptured appendicitis, intussusception, or malrotation with volvulus. Other causes of sepsis, such as meningitis or severe mastoiditis, both rare complications of otitis media, should be considered, although he does not appear severely ill. Acute myelogenous leukemia or other malignancies and illnesses associated with immunodeficiency can present with sepsis and chloromas in the middle ear that can be misconstrued as otitis media.

A chest radiograph demonstrated left lower lobe patchy opacities concerning for pneumonia. Rapid respiratory syncytial virus and influenza antigen test results were negative. Laboratory testing for general bloodwork was not obtained. He was administered a single dose of intramuscular ceftriaxone, prescribed a 5-day course of azithromycin, and discharged home. The child’s breathing gradually improved, but he continued to have subjective fevers. Two days later, he developed dark red urine. His mother brought him back to the outpatient clinic.

At the time of the ED visit, a diagnosis of community-acquired pneumonia was plausible given fever, mildly increased work of breathing, and an opacification on chest radiography. Most community-acquired pneumonia is caused by viruses; common bacterial causes for his age include Streptococcus pneumoniae and Moraxella catarrhalis. The first-line treatment for uncomplicated community-acquired pneumonia in children is amoxicillin, but this was appropriately avoided given his allergy.

The persistent fevers are surprising. The improvement in breathing corresponds to the treatment (and resolution) of community-acquired pneumonia. However, the development of dark urine does not. Red urine—in the absence of ingested pigments (such as those found in beets)—usually results from hematuria, hemoglobinuria, or myoglobinuria. Gross hematuria can originate from the kidneys to the urethral meatus. Abdominal masses, kidney trauma, or underlying kidney disease may all present with gross hematuria (or microscopic hematuria, seen only on urinalysis). The urine should be examined for the presence of heme, protein, and for evidence of infection; microscopy should be performed to examine for cellular casts and dysmorphic red cells. Tests of renal function, a comprehensive metabolic panel, evaluation of hematologic indexes, and assessments of inflammatory markers should be performed.

The child lived with his parents and had no siblings. He experienced no physical trauma, and there was no family history of kidney disease or hematuria. His father had a persistent cough and fever for 1 month, but recovered around the time the patient began to experience his initial symptoms. This was the patient’s third diagnosis of pneumonia. He had not traveled and was up to date with immunizations. He attended day care.

The fact that this is not the first episode of “pneumonia” raises important possibilities. The most likely one is that the child has had multiple viral infections; however, he could have an underlying primary immunodeficiency (PI) that predisposes him to recurrent infections. More severe PIs often present with recurrent sepsis, bacteremia, and failure to thrive, none of which were present in this case. Less severe PIs (such as selective IgA deficiency) could be possible. Another possibility is that these recurrent episodes of pneumonia are a relapsing and remitting noninfectious process, such as an antineutrophil cytoplasmic antibodies–associated vasculitis or anti–glomerular basement membrane disease. The patient’s father’s recent prolonged respiratory symptoms may be suggestive of pertussis or a “walking pneumonia” potentially caused by Mycoplasma or another atypical bacterium.

His temperature was 36.9 °C, heart rate 107 beats per minute, blood pressure was 106/67 mm Hg, and respiratory rate was 24 breaths per minute with oxygen saturation of 100% on ambient air. He was well appearing. His mucous membranes were moist, and oropharynx was clear. He had scleral icterus. The cardiopulmonary exam was normal. He had no significant lymphadenopathy, hepatosplenomegaly, or rashes.

The finding of jaundice is an important diagnostic pivot point, especially when combined with hematuria. The next step is determining if the jaundice is resulting from unconjugated or conjugated hyperbilirubinemia; the former most often stems from hemolysis or impairment in conjugation, while the latter results from intrahepatic or extrahepatic biliary defects. Tests for hepatobiliary injury including evaluations of alanine and aspartate aminotransferases and alkaline phosphatase, as well as for hepatic function such as tests of coagulation, should be performed.

The patient was referred to the ED and admitted for further evaluation. A complete blood count revealed a white blood cell (WBC) count of 10,700/µL (61% polymorphonuclear neutrophils, 30% lymphocytes, 5% monocytes, 3% eosinophils, 1% basophils), hemoglobin count was 10.3 g/dL (reticulocyte 2% with absolute reticulocyte count 58,400/μL), and platelet count was 265,000/µL. Components of the basic metabolic panel were within reference ranges except for a mildly elevated blood urea nitrogen level of 14 mg/dL with normal creatinine level of 0.3 mg/dL. Total protein was 6.7 g/dL (reference range, 6.4-8.3) and albumin 3.9 g/dL (reference range, 3.4-4.8). Alkaline phosphatase level was 188 U/L (reference range, 44-147), aspartate aminotransferase level 76 U/L (reference range, 0-40), and alanine aminotransferase level 12 U/L (reference range, 7-40). Total bilirubin level was 2.4 mg/dL (reference range, less than 1.5) with direct bilirubin level of 0.4 mg/dL. His C-reactive protein level was 1.5 mg/mL (reference range, 0-0.75). Creatinine kinase (CK) level was 2,550 U/L (reference range, 2-198). International Normalized Ratio (INR) was 1.0. Urinalysis was notable for 2+ proteinuria, large hemoglobin pigment, and 6 red blood cells per high power field (reference range, 0-4).

His blood urea nitrogen is elevated, reflecting either prerenal azotemia or increased absorption of nitrogenous products. Unconjugated hyperbilirubinemia may result from impaired hepatic bilirubin uptake (such as in heart failure or portosystemic shunts), impaired bilirubin conjugation (resulting from genetic conditions or drugs), or excess bilirubin production (such as in hemolysis); his anemia and lack of other evidence of hepatic dysfunction point to hemolysis as the etiology. The reticulocyte production index is approximately 1%, which suggests that an increase in erythrocyte generation is present but inadequate. This, however, does not mean that an erythrocyte production abnormality is present since reticulocytosis can be delayed in many cases of acute hemolytic anemia. It is also possible that the same hemolytic process is affecting mature and immature erythrocytes. A peripheral blood smear should be reviewed for evidence of intravascular hemolysis and testing for autoimmune hemolysis should be performed. Notably, his white blood cell and platelet counts are preserved, which makes a bone marrow–involved malignancy or infiltrative process less likely. The alkaline phosphatase elevation may result from either intrahepatic or extrahepatic biliopathy; bone damage is also possible. The elevation of aspartate aminotransferase, CK, and potassium, along with marked urinary heme pigment, may indicate muscle damage; the most common myositis in children is benign acute childhood myositis resulting from viral infection. However, the moderate level of CK elevation seen in this case is nonspecific and can result from many different etiologies. A metabolic myopathy, such as carnitine palmitoyltransferase II deficiency, can be made worse by metabolic stress and result in rhabdomyolysis; the presentations of inborn errors of metabolism are varied and a planned-out, stepwise approach in evaluation is fundamental.

Lactic acid dehydrogenase (LDH) level was 1,457 U/L (reference range, 140-280), and haptoglobin level was less than 6 mg/dL (reference range, 30-200). Peripheral blood smear demonstrated occasional atypical, reactive-appearing lymphocytes with red cell clumping and agglutination, as well as rare target, burr, and fragmented red cells. Test results for urine myoglobin were negative. Results of urine culture were negative. No blood culture was collected.

The elevated LDH, decreased haptoglobin, and findings on the peripheral blood smear confirm hemolysis. The clumping of erythrocytes can be artifactual in the preparation of peripheral smears, but when considered in the context of hemolysis, may be clinically important. Clumping of erythrocytes on the peripheral smear indicates the binding of a protein to antigens on the erythrocyte membrane; when this occurs below body temperature, this is consistent with the presence of a “cold agglutinin,” usually an IgM antibody directed at erythrocyte surface antigens that causes agglutination and destruction, especially in cooler areas of the body. This is a well-known complication of Mycoplasma pneumoniae infections as well as Epstein-Barr virus (EBV) infections; it may also occur with lymphoid malignancies or autoimmune disease.

Direct Coombs IgG test findings were negative, direct Coombs C3 test was positive, and direct Coombs polyspecific test was positive. M pneumoniae IgG antibody level was 1.4 mg/dL (reference ranges: <0.9, negative; 0.91-1.09, equivocal; >1.1, positive); M pneumoniae IgM level was 529 U/mL (reference range: <770, negative). EBV capsid IgM and IgG levels were undetectable. EBV nuclear antigen IgG level was also undetectable. EBV viral load was fewer than 10 copies/mL. Antinuclear antibodies (ANA) level was negative. General IgE and IgM levels were normal, at 11 and 81 mg/dL, respectively. Repeat complete blood count showed WBC of 7,800/µL, hemoglobin of 8.7 g/dL, and platelet count of 341,000/µL. The patient’s hemoglobin remained stable during hospitalization.

This directed testing is helpful in further classifying the patient’s hemolytic anemia. Autoimmune hemolytic anemias are classified into warm antibody–mediated, cold antibody–mediated, and mixed-type forms; drug-induced and alloimmune hemolytic anemias also occur. In addition, both systemic lupus erythematosus and antiphospholipid antibody syndrome can have hemolytic anemia with variable Coombs testing results; neither fit well in this case. The absence of red blood cell–directed IgG antibodies substantially decreases the likelihood of warm antibody–mediated hemolytic anemia. In cold antibody–mediated hemolytic anemia, antibodies bind to the erythrocyte membrane and then adhere to complement C3, which leads to both intravascular and extravascular hemolysis. Important types of cold antibody–mediated hemolytic anemia in children are primary and secondary cold agglutinin disease, along with paroxysmal cold hemoglobinuria. The Donath-Landsteiner test can be helpful in differentiating these conditions. Antibodies to Mycoplasma may be delayed in response to acute infection, and a child who is reinfected may only produce IgG antibodies. Given the patient’s clinical stability and previous health, the most likely diagnosis is Mycoplasma-induced cold antibody–mediated hemolytic anemia. It may be helpful to check convalescent titers to Mycoplasma in 2 to 4 weeks.

Donath-Landsteiner (D-L) antibody test results were positive. Medication-derived hemolytic anemia testing was conducted, but the presence of positive D-L antibody makes the test results inconclusive. This ultimately led to a diagnosis of paroxysmal cold hemoglobinuria (PCH), presumably triggered by a viral syndrome. Convalescent titers to Mycoplasma were not checked given clinical improvement. Because the patient’s hemoglobin was stable during hospitalization, he was not treated with steroids. His parents were counseled on avoiding cold temperatures for several days. Within 1 month, his hemoglobin had recovered without further evidence of hemolysis.

DISCUSSION

Hemolytic anemia refers to the accelerated destruction of red blood cells and can be further classified as acquired or hereditary.1 Hereditary conditions causing hemolytic anemia include enzymopathies (eg, glucose-6-phosphate dehydrogenase deficiency), hemoglobinopathies (eg, sickle cell disease), and membrane abnormalities (eg, hereditary spherocytosis). Acquired pathologies include microangiopathic hemolytic anemia (MAHA), anemias directly caused by certain infections such as malaria, and immune-mediated (Coombs-positive) hemolytic anemias.

MAHA can sometimes be life-threatening and is therefore important to identify quickly. In the right clinical context, such processes may be rapidly recognized by the presence of schistocytes on blood smear in addition to an elevated serum LDH level. Schistocytes suggest mechanical destruction of erythrocytes in the vasculature, the hallmark of MAHA. Important MAHAs include thrombocytopenic purpura, hemolytic-uremic syndrome, and disseminated intravascular coagulation. Though this patient did have a mildly elevated LDH, MAHA was less likely because there were no schistocytes on the blood smear.

Autoimmune hemolytic anemias (AIHAs) are another important subset of acquired hemolytic anemias. AIHAs occur when there is antibody-mediated destruction of erythrocytes. The direct Coombs test evaluates for antibody- or complement-­coated erythrocytes. After administration of anti-IgG and anti-­C3 serum, the test evaluates for agglutination of the red cells caused by attached antibodies or complement. Coombs-­positive AIHA can also be categorized by the temperature of agglutination. “Warm” hemolysis often involves IgG autoantibodies (ie, warm agglutinins), while “cold” antibodies, usually IgM autoantibodies, bind at colder temperatures (0-4 °C) and activate complements, including C3. In this patient, the Coombs C3 was positive while the Coombs IgG was negative, which is more suggestive of a cold complement–mediated pathway.

Cold AIHA can be further categorized into primary cold agglutinin disease, secondary cold agglutinin disease, and PCH. Primary cold agglutinin disease is an autoimmune disorder that mostly occurs in adults. Secondary cold AIHA can often be triggered by bacterial infection (commonly M pneumoniae) or viruses including EBV, measles, and mumps.2 Medications, including penicillin and cephalosporins, can also be implicated. Secondary cold AIHA is also linked with autoimmune diseases, such as systemic lupus erythematosus and lymphoproliferative disorders. PCH can be identified with the unique presence of a specific autoantibody (ie, D-L autoantibody) that agglutinates at cold temperatures but dissociates on subsequent rewarming.3 Complement remains affixed and activates hemolysis.

The D-L antibody responsible for PCH is an IgG antibody to the P-antigen present on the erythrocyte surface. Since the Coombs test is conducted at normal temperature, it will be positive for the affixed complement but not for IgG. The underlying mechanism for PCH was proposed by Julius Donath, MD, and Karl Landsteiner, MD, in 1904 and is considered to be the first description of autoimmune disease being precipitated by antibodies.4 The D-L antibody test itself is uncommonly performed and somewhat difficult to interpret, particularly in adults, and may lead to false-negative results.5

PCH is an acquired, cold AIHA more common to children6,7 and may account for up to 33% of pediatric AIHA cases.8 Typical presentation is after an upper respiratory tract illness; however, the trigger is often not identified. Implicated triggers include a number of viruses.9 Clinical presentation includes findings of intravascular hemolysis similar to those in our patient. The pathogenic IgG autoantibody is polyclonal and is likely formed because of immune stimulation, which is consistent with the predominance of nonmalignant triggers of this disease process.10 Hemolysis and associated symptoms are often exacerbated with cold exposure; both typically resolve within 2 weeks. In recurrent cases, which are a minority, immunosuppression may be considered.10

PCH remains an often-understated cause of hemolytic anemia particularly in children. Lacking obvious pathognomonic clinical symptoms, it may be overlooked for other forms of AIHA or MAHA. However, with a structured approach to evaluation, as with this patient who had hematuria and jaundice, early diagnosis can prevent an unnecessarily extensive workup and can provide reassurance to patient and parents. By understanding the basic categories of hemolytic anemia, the relevant blood testing available, and interpretation of Coombs test results, clinicians can ensure that PCH is a diagnosis that is not left out in the cold.

KEY TEACHING POINTS

  • Examination for schistocytes on a blood smear can help identify life-threatening causes of hemolytic anemia.
  • Characterization of cold AIHA includes defining the underlying etiology as primary cold agglutinin disease, secondary cold agglutinin disease, or PCH.
  • PCH is a cold AIHA that is an underrecognized cause of hemolytic anemia in children. The diagnosis of PCH is made by testing for the presence of the D-L antibody.
 
References

1. Dhaliwal G, Cornett PA, Tierney LM Jr. Hemolytic anemia. Am Fam Physician. 2004;69(11):2599-2606.
2. Djaldetti M. Paroxysmal cold hemoglobinuria. CRC Crit Rev Clin Lab Sci. 1978;9(1):49-83. https://doi.org/10.3109/10408367809150915
3. Levine P, Celano MJ, Falkowski F. The specificity of the antibody in paroxysmal cold hemoglobinuria (P.C.H.). Transfusion. 1963;3(4):278-280. https://doi.org/10.1111/j.1537-2995.1963.tb04643.x
4. Donath J, Landsteiner K. Uber Paroxysmale Hamoglobinurie. Munch Med Wochenschr. 1904;51:1590-1593
5. Zeller MP, Arnold DM, Al Habsi K, et al. Paroxysmal cold hemoglobinuria: a difficult diagnosis in adult patients. Transfusion. 2017;57(1):137-143. https://doi.org/10.1111/trf.13888
6. Göttsche B, Salama A, Mueller-Eckhardt C. Donath-Landsteiner autoimmune hemolytic anemia in children. a study of 22 cases. Vox Sang. 1990;58(4):281-286. https://doi.org/10.1111/j.1423-0410.1990.tb05000.x
7. Sokol RJ, Booker DJ, Stamps R. Erythropoiesis: paroxysmal cold haemoglobinuria: a clinico-pathological study of patients with a positive Donath-­Landsteiner test. Hematology. 1999;4(2):137-164. https://doi.org/10.1080/10245332.1999.11746439
8. Petz LD. Cold antibody autoimmune hemolytic anemias. Blood Rev. 2008;22(1):1-15. https://doi.org/10.1016/j.blre.2007.08.002
9. Leibrandt R, Angelino K, Vizel-Schwartz M, Shapira I. Paroxysmal cold hemoglobinuria in an adult with respiratory syncytial virus. Case Rep Hematol. 2018;2018:1-3. https://doi.org/10.1155/2018/7586719
10. Gertz MA. Management of cold haemolytic syndrome. Br J Haematol. 2007;138(4):422-429. https://doi.org/10.1111/j.1365-2141.2007.06664.x

References

1. Dhaliwal G, Cornett PA, Tierney LM Jr. Hemolytic anemia. Am Fam Physician. 2004;69(11):2599-2606.
2. Djaldetti M. Paroxysmal cold hemoglobinuria. CRC Crit Rev Clin Lab Sci. 1978;9(1):49-83. https://doi.org/10.3109/10408367809150915
3. Levine P, Celano MJ, Falkowski F. The specificity of the antibody in paroxysmal cold hemoglobinuria (P.C.H.). Transfusion. 1963;3(4):278-280. https://doi.org/10.1111/j.1537-2995.1963.tb04643.x
4. Donath J, Landsteiner K. Uber Paroxysmale Hamoglobinurie. Munch Med Wochenschr. 1904;51:1590-1593
5. Zeller MP, Arnold DM, Al Habsi K, et al. Paroxysmal cold hemoglobinuria: a difficult diagnosis in adult patients. Transfusion. 2017;57(1):137-143. https://doi.org/10.1111/trf.13888
6. Göttsche B, Salama A, Mueller-Eckhardt C. Donath-Landsteiner autoimmune hemolytic anemia in children. a study of 22 cases. Vox Sang. 1990;58(4):281-286. https://doi.org/10.1111/j.1423-0410.1990.tb05000.x
7. Sokol RJ, Booker DJ, Stamps R. Erythropoiesis: paroxysmal cold haemoglobinuria: a clinico-pathological study of patients with a positive Donath-­Landsteiner test. Hematology. 1999;4(2):137-164. https://doi.org/10.1080/10245332.1999.11746439
8. Petz LD. Cold antibody autoimmune hemolytic anemias. Blood Rev. 2008;22(1):1-15. https://doi.org/10.1016/j.blre.2007.08.002
9. Leibrandt R, Angelino K, Vizel-Schwartz M, Shapira I. Paroxysmal cold hemoglobinuria in an adult with respiratory syncytial virus. Case Rep Hematol. 2018;2018:1-3. https://doi.org/10.1155/2018/7586719
10. Gertz MA. Management of cold haemolytic syndrome. Br J Haematol. 2007;138(4):422-429. https://doi.org/10.1111/j.1365-2141.2007.06664.x

Issue
Journal of Hospital Medicine 16(2)
Issue
Journal of Hospital Medicine 16(2)
Page Number
105-108. Published Online First August 19, 2020
Page Number
105-108. Published Online First August 19, 2020
Publications
Publications
Topics
Article Type
Sections
Article Source

© 2020 Society of Hospital Medicine

Disallow All Ads
Correspondence Location
Justin Berk, MD, MPH, MBA; Email: justin_berk@brown.edu; Telephone: 401-444-6118; Twitter: @justinberk.
Content Gating
Gated (full article locked unless allowed per User)
Alternative CME
Disqus Comments
Default
Use ProPublica
Hide sidebar & use full width
render the right sidebar.
Conference Recap Checkbox
Not Conference Recap
Clinical Edge
Display the Slideshow in this Article
Gating Strategy
First Peek Free
Medscape Article
Display survey writer
Reuters content
Article PDF Media
Image
Teambase ID
18001AD0.SIG
Disable zoom
Off

Hindsight Is 20/20

Article Type
Changed
Thu, 03/25/2021 - 13:51

A 38-year-old woman presented to her primary care clinic with 3 weeks of progressive numbness and tingling sensation, which began in both hands and then progressed to involve both feet, ascending from the legs to the chest while sparing her buttocks. She also noted weakness of her left leg, but no other motor symptoms were reported. She had no fevers, chills, weight loss, bladder dysfunction, nausea, vomiting, or diarrhea.

As with all neurological complaints, localization of the process will often inform a more specific differential diagnosis. If both sensory and motor findings are present, both central and peripheral nerve processes deserve consideration. The onset of paresthesia in the hands, rapid progression to the trunk, and unilateral leg weakness would be inconsistent with a length-dependent peripheral neuropathy. The distribution of complaints and the sacral sparing suggests a myelopathic process involving the cervical region rather than a cauda equina or conus lesions. In an otherwise healthy person of this age and gender, an inflammatory demyelinating disease affecting the cord including multiple sclerosis (MS) would be a strong consideration, although metabolic, vascular, infectious, compressive, or neoplastic disease of the spinal cord could also present with similar subacute onset and pattern of deficits.

Her medical history included morbid obesity, dry eyes, depression, iron deficiency anemia requiring recurrent intravenous replenishment, and abnormal uterine bleeding. Her surgical history included gastric band placement 7 years earlier with removal 5 years later due to persistent gastroesophageal reflux disease, dysphagia, nausea, and vomiting. The gastric band removal was complicated by chronic abdominal pain. Her medications consisted of duloxetine, intermittent iron infusions, artificial tears, loratadine, and pregabalin. She was sexually active with her husband. She consumed alcohol occasionally but did not smoke tobacco or use illicit drugs.

On exam, her temperature was 36.6°C (97.8°F), blood pressure 132/84 mm Hg, and heart rate 85 beats per minute. Body mass index was 39.5 kg/m2. The cardiac, pulmonary, and skin examinations were normal. The abdomen was soft with diffuse tenderness to palpation without rebound or guarding. Examination of cranial nerves 2-12 was normal. Cognition, strength, proprioception, deep tendon reflexes, and light touch were all normal. Her gait was normal, and the Romberg test was negative.

The normal neurologic exam is reassuring but imperfectly sensitive and does not eliminate the possibility of underlying neuropathology. Bariatric surgery may result in an array of nutritional deficiencies such as vitamin E, B12, and copper, which can cause myelopathy and/or neuropathy. However, these abnormalities occur less frequently with gastric banding procedures. If her dry eyes are part of the sicca syndrome, an underlying autoimmune diathesis may be present. Her unexplained chronic abdominal pain prompts considering nonmenstrual causes of iron deficiency anemia, such as celiac disease. Bariatric surgery may contribute to iron deficiency through impaired iron absorption. Her stable weight and lack of diarrhea argue against Crohn’s or celiac disease. Iron deficiency predisposes individuals to pica, most commonly described with ice chip ingestion. If lead pica had occurred, abdominal and neurological symptoms could result. Nevertheless, the abdominal pain is nonspecific, and its occurrence after gastric band removal makes its link to her neurologic syndrome unclear. An initial evaluation would include basic metabolic panel, complete blood count with differential, erythrocyte sedimentation rate, C-reactive protein (CRP), thyroid-stimulating hormone, vitamin B12, and copper levels.

A basic metabolic panel was normal. The white cell count was 5,710 per cubic millimeter, hemoglobin level 12.2 g per deciliter, mean corpuscular volume 85.2 fl, and platelet count 279,000 per cubic millimeter. The serum ferritin level was 18 ng per milliliter (normal range, 13-150), iron 28 µg per deciliter (normal range, 50-170), total iron-binding capacity 364 µg per deciliter (normal range, 250-450), and iron saturation 8% (normal range, 20-55). The vitamin B12 level was 621 pg per milliliter (normal range, 232-1,245) and thyroid-stimulating hormone level 1.87 units per milliliter (normal range, 0.50-4.50). Electrolyte and aminotransferase levels were within normal limits. CRP was 1.0 mg per deciliter (normal range, <0.5) and erythrocyte sedimentation rate 33 millimeters per hour (normal range, 4-25). Hepatitis C and HIV antibodies were nonreactive.

The ongoing iron deficiency despite parenteral iron replacement raises the question of ongoing gastrointestinal or genitourinary blood loss. While the level of vitamin B12 in the serum may be misleadingly normal with cobalamin deficiency, a methylmalonic acid level is indicated to evaluate whether tissue stores are depleted. Copper levels are warranted given the prior bariatric surgery. The mild elevations of inflammatory markers are nonspecific but reduce the likelihood of a highly inflammatory process to account for the neurological and abdominal symptoms. 

At her 3-month follow-up visit, she noted that the paresthesia had improved and was now limited to her bilateral lower extremities. During the same clinic visit, she experienced a 45-minute episode of ascending left upper extremity numbness. Her physical examination revealed normal strength and reflexes. She had diminished response to pinprick in both legs to the knees and in both hands to the wrists. Vibration sense was diminished in the bilateral lower extremities.

 

 

A glycosylated hemoglobin (HbA1c) level was 6.2%. Methylmalonic acid was 69 nmol per liter (normal range, 45-325). Antibodies to Borrelia burgdorferi and Treponema pallidum were absent. Impaired glucose metabolism was the leading diagnosis for her polyneuropathy, and it was recommended that she undergo an oral glucose tolerance test. Electromyography was not performed.

 

The neurological symptoms are now chronic, and importantly, the patient has developed sensory deficits on neurological examination, suggesting worsening of the underlying process. While the paresthesia is now limited to a “stocking/glove” distribution consistent with distal sensory polyneuropathy, there should still be a concern for spinal cord pathology given that the HbA1c level of 6.2 would not explain her initial distribution of symptoms. Myelopathy may mimic peripheral nerve disease if, for example, there is involvement of the dorsal columns leading to sensory deficits of vibration and proprioception. Additionally, the transient episode of upper extremity numbness raises the question of sensory nerve root involvement (ie, sensory radiculopathy). Unexplained abdominal pain could possibly represent the involvement of other nerve roots innervating the abdominal wall. The patient’s episode of focal arm numbness recalls the lancinating radicular pain of tabes dorsalis; however, the negative specific treponemal antibody test excludes neurosyphilis.

The differential diagnosis going forward will be strongly conditioned by the localization of the neurological lesion(s). To differentiate between myelopathy, radiculopathy, and peripheral neuropathy, I would perform nerve conduction studies, magnetic resonance imaging (MRI) of the spinal cord, and cerebrospinal fluid analysis.

The patient began taking a multivitamin, and after weeks her paresthesia had resolved. One month later, she developed an intermittent, throbbing left-sided headache and pain behind the left eye that was worsened with ocular movement. She then noted decreased visual acuity in her left eye that progressed the following month. She denied photophobia, flashers, or floaters.

In the emergency department, visual acuity was 20/25 in her right eye; in the left eye she was only able to count fingers. Extraocular movements of both eyes were normal as was her right pupillary reflex. Red desaturation and a relative afferent papillary defect were present in the left eye. Fundoscopic exam demonstrated left optic disc swelling. The remainder of her cranial nerves were normal. She had pronation of the left upper extremity and mild right finger-to-nose dysmetria. Muscle tone, strength, sensation, and deep tendon reflexes were normal.

The improvement in the sensory symptoms was unlikely to be related to the nutritional intervention and provides a clue to an underlying waxing and waning illness. That interpretation is supported by the subsequent development of new visual symptoms and signs, which point to optic nerve pathology. Optic neuropathy has a broad differential diagnosis that includes ischemic, metabolic, toxic, and compressive causes. Eye pain, swelling of the optic disc, and prominent impairment of color vision all point to the more specific syndrome of optic neuritis caused by infections (including both Treponema pallidum and Borrelia species), systemic autoimmune diseases (systemic lupus erythematosus or Sjogren’s syndrome), and central nervous system (CNS) demyelinating diseases. Of these, inflammatory demyelinating processes would be the likeliest explanation of intermittent and improving neurologic findings.

 

 

With relapsing symptoms and findings that are separate in distribution and time, two diagnoses become most likely, and both of these are most often diagnosed in young women. MS is common, and optic neuritis occurs in more than 50% of patients over the course of illness. Neuromyelitis optica spectrum disorder (NMOSD) is a rare condition that can exist in isolation or be associated with other autoimmune illnesses. While these entities are difficult to differentiate clinically, neuroimaging that demonstrates extensive intracerebral demyelinating lesions and cerebrospinal fluid with oligoclonal bands favor MS, whereas extensive, predominant spinal cord involvement is suggestive of NMOSD. Approximately 70% of NMO patients harbor an antibody directed against the aquaporin-4 channel, and these antibodies are not seen in patients with MS. A milder NMO-like disorder has also been associated with antimyelin oligodendrocyte antibodies (MOG).

Testing for antinuclear antibodies, anti–double-stranded DNA, anti-Ro (SSA), and anti-La (SSB) antibodies was negative. The level of C3 was 162 mg per deciliter (normal range 81-157) and C4 38 (normal range 13-39). T-spot testing for latent tuberculosis was negative.

There is no serological evidence of active systemic lupus erythematosus or Sjogren’s syndrome. The pretest probability of CNS tuberculosis was low in light of her presenting complaints, relatively protracted course, and overall clinical stability without antituberculous therapy. Tests for latent tuberculosis infection have significant limitations of both sensitivity and specificity for the diagnosis of active disease.

Optical coherence tomography showed optic disc edema in the left eye only. MRI of the head with contrast revealed abnormal signal intensity involving the posterior aspect of the pons, right middle cerebellar peduncle, anterior left temporal lobe, bilateral periventricular white matter, subcortical white matter of the frontal lobes bilaterally, and medulla with abnormal signal and enhancement of the left optic nerve (Figure, Panel A). MRI of the cervical and thoracic spine demonstrated multifocal demyelinating lesions at C3, C4, C7, T4, T5, T7, and T8 (Figure, Panel B). The lesions were not longitudinally extensive. There was no significant postcontrast enhancement to suggest active demyelination.

manesh03770219e_f1.jpg

The cerebrospinal fluid analysis revealed glucose of 105 mg per deciliter and a total protein of 26.1 mg per deciliter. In the fourth tube, there were 20 red cells per cubic and four white cells with a differential of 62% neutrophils, 35% lymphocytes, and 3% monocytes. Epstein-Barr and herpes simplex virus DNA were negative. A Venereal Disease Research Laboratory test was negative. Multiple oligoclonal IgG bands were identified only in the cerebrospinal fluid. Aquaporin-4 IgG and MOG antibodies were negative.

In addition to the expected finding of enhancement of the optic nerve, MRI demonstrated numerous multifocal white matter lesions throughout the cerebrum, brainstem, and spinal cord. Many of the lesions were in “silent” areas, which is not directly attributable to specific symptoms, but several did correlate with the subtler deficits of weakness and dysmetria that were noted on examination. Although such lesions may be seen with a diverse group of systemic diseases including adrenal leukodystrophy, sarcoidosis, Behcet’s, cerebral lupus, and vasculitis, primary CNS inflammatory demyelinating diseases are much more likely. The extensive distribution of demyelination argues against NMOSD. The negative aquaporin-4 and MOG assays support this conclusion. Not all multifocal CNS demyelination is caused by MS and can be seen in posterior reversible encephalopathy syndrome, cerebral autosomal dominant arteriopathy with subcortical infarcts and leukoencephalopathy, and adult polyglucosan body disease. Osmotic demyelination is increasingly being recognized as a process that can be more widespread rather than just being limited to the pons. Viral infections of the CNS such as the JC virus (PML) may also provoke multifocal demyelination. Acute disseminated encephalomyelitis is most often seen during childhood, usually after vaccination or after an infectious prodrome. The tempo of the progression of these other diseases tends to be much more rapid than this woman’s course, and often, the neurological deficits are more profound and debilitating. The clinical presentation of sensory-predominant myelopathy, followed by optic neuritis, absence of systemic inflammatory signs or laboratory markers, exclusion of other relevant diseases, multifocal white matter lesions on imaging, minimal pleocytosis, and presence of oligoclonal bands in cerebrospinal fluid, all point to a diagnosis of relapsing-remitting MS.

The patient was diagnosed with MS. She was admitted to the neurology service and treated with 1,000 mg IV methylprednisolone for 3 days with a prompt improvement in her vision. She was started on natalizumab without a relapse of symptoms over the past year.

 

 

COMMENTARY

Multiple sclerosis is a chronic demyelinating disease of the CNS.1 The diagnosis of MS has classically been based upon compatible clinical and radiographic evidence of pathology that is disseminated in space and time. Patients typically present with an initial clinically isolated syndrome—involving changes in vision, sensation, strength, mobility, or cognition—for which there is radiographic evidence of demyelination.2 A diagnosis of clinically definite MS is then often made based on a subsequent relapse of symptoms.3

An interval from initial symptoms has been central to the diagnosis of MS (“lesions disseminated in time”). However, recent evidence questions this diagnostic paradigm, and a more rapid diagnosis of MS has been recommended. This recommendation is reflected in the updated McDonald criteria, according to which, if a clinical presentation is supported by the presence of oligoclonal bands in the cerebrospinal fluid, a diagnosis can be made on the basis of radiographic evidence of dissemination of disease in space, without evidence of dissemination in time.4 The importance of such early diagnosis has been supported by numerous studies that have demonstrated improved clinical outcomes with early therapy.5-7

Despite the McDonald criteria, delays in definitive diagnosis are common in MS. Patients with MS in Spain were found to experience a 2-year delay from the first onset of symptoms to diagnosis.8 In this cohort, patients exhibited delays in presenting to a healthcare provider, as well as delays in diagnosis with an average time from seeing an initial provider to diagnosis of 6 months. When patients who were referred for a demyelinating episode were surveyed, over a third reported a prior suggestive event.9 The time from the first suggestive episode to referral to a neurologist for a recognized demyelinating event was 46 months. Other studies have shown that delays in diagnosis are especially common in younger patients, those with primary progressive MS, and those with comorbid disease.10,11

Misapplication of an MS diagnosis also occurs frequently. In one case series, such misapplication was found most often in cases involving migraine, fibromyalgia, psychogenic disorders, and NMOSD.12 NMOSD is distinguished from MS by the presence of typical brain and spine findings on MRI.13 Antibodies to aquaporin-4 are highly specific and moderately sensitive for the disease.14 It is important to distinguish NMOSD from MS as certain disease-modifying drugs used for MS might actually exacerbate NMOSD.15 A lesion that traverses over three or more contiguous vertebral segments with predominant involvement of central gray matter (ie, longitudinally extensive transverse myelitis) on MRI is the most distinct finding of NMOSD. In contrast, similar to our patient, short and often multiple lesions are demonstrated on spinal cord MRI in patients with MS. Sensitive and specific findings of brain MRI in patients with MS include the presence of lateral ventricle and inferior temporal lobe lesion, Dawson’s fingers, central vein sign, or an S-shaped U-fiber lesion. In NMOSD, brain MRI might reveal periependymal lesions surrounding the ventricular system.

This case highlights the diagnostic challenges related to presentations of a waxing and waning neurological process. At the time of the second evaluation, the presentation was interpreted as a length-dependent polyneuropathy due to glucose intolerance. Our patient’s relatively normal HbA1c, subacute onset of neuropathic symptoms (ie, <4 weeks), sensory and motor complaints, and onset in the upper extremities suggested an alternative diagnosis to prediabetes. Once the patient presented with optic neuritis, the cause of the initial symptoms was obvious, but then, hindsight is 20/20.

 

 

TEACHING POINTS

  • Early treatment of MS results in improved clinical outcomes.
  • Delays in the definitive diagnosis of MS are common, especially in younger patients, those with primary progressive MS, and those with comorbid disease.
  • If a clinical presentation is supported by the presence of oligoclonal bands in the cerebrospinal fluid, a diagnosis of MS can be made on the basis of radiographic evidence of dissemination of disease in space, without evidence of dissemination in time.

Acknowledgments

The authors wish to thank Rabih Geha, MD, and Gurpreet Dhaliwal, MD, for providing feedback on an earlier version of this manuscript.

References

1. Reich DS, Lucchinetti CF, Calabresi PA. Multiple sclerosis. N Engl J Med. 2018;378:169-180. https://doi.org/10.1056/NEJMra140148.
2. Brownlee WJ, Hardy TA, Fazekas F, Miller DH. Diagnosis of multiple sclerosis: progress and challenges. Lancet. 2017;389(10076):1336-1346. https://doi.org/10.1016/S0140-6736(16)30959-X.
3. Thompson AJ, Baranzini SE, Geurts J, Hemmer B, Ciccarelli O. Multiple sclerosis. Lancet. 2018;391(10130):1622-1636. https://doi.org/10.1016/S0140-6736(18)30481-1.
4. Thompson AJ, Banwell BL, Barkhof F, et al. Diagnosis of multiple sclerosis: 2017 revisions of the McDonald criteria. Lancet Neurol. 2018;17(2):162-173. https://doi.org/10.1016/S1474-4422(17)30470-2.
5. Comi G, Radaelli M, Soelberg Sørensen P. Evolving concepts in the treatment of relapsing multiple sclerosis. Lancet. 2017;389(10076):1347-1356. https://doi.org/10.1016/S0140-6736(16)32388-1.
6. Freedman MS, Comi G, De Stefano N, et al. Moving toward earlier treatment of multiple sclerosis: Findings from a decade of clinical trials and implications for clinical practice. Mult Scler Relat Disord. 2014;3(2):147-155. https://doi.org/10.1016/j.msard.2013.07.001.
7. Harding K, Williams O, Willis M, et al. Clinical outcomes of escalation vs early intensive disease-modifying therapy in patients with multiple sclerosis. JAMA Neurol. 2019;76(5):536-541. https://doi.org/10.1001/jamaneurol.2018.4905.
8. Fernández O, Fernández V, Arbizu T, et al. Characteristics of multiple sclerosis at onset and delay of diagnosis and treatment in Spain (the Novo Study). J Neurol. 257(9):1500-1507. https://doi.org/10.1007/s00415-010-5560-1.
9. Gout O, Lebrun-Frenay C, Labauge P, et al. Prior suggestive symptoms in one-third of patients consulting for a “first” demyelinating event. J Neurol Neurosurg Psychiatry 2011;82(3):323-325. https://doi.org/10.1136/jnnp.2008.166421.
10. Kingwell E, Leung A, Roger E, et al. Factors associated with delay to medical recognition in two Canadian multiple sclerosis cohorts. J Neurol Sci. 2010(1-2);292:57-62. https://doi.org/10.1016/j.jns.2010.02.007.
11. Marrie RA, Horwitz R, Cutter G, Tyry T, Campagnolo D, Vollmer T. Comorbidity delays diagnosis and increases disability at diagnosis in MS. Neurology. 2009;72(2):117-124. https://doi.org/10.1212/01.wnl.0000333252.78173.5f.
12. Solomon AJ, Bourdette DN, Cross AH, et al. The contemporary spectrum of multiple sclerosis misdiagnosis: A multicenter study. Neurology. 2016;87(13):1393-1399. https://doi.org/10.1212/WNL.0000000000003152.
13. Kim HJ, Paul F, Lana-Peixoto MA, et al. MRI characteristics of neuromyelitis optica spectrum disorder: An international update. Neurology. 2015;84(11):1165-1173. https://doi.org/10.1212/WNL.0000000000001367.
14. Wingerchuk DM, Banwell B, Bennett JL, et al. International consensus diagnostic criteria for neuromyelitis optica spectrum disorders. Neurology. 2015;85(2):177-189. https://doi.org/10.1212/WNL.0000000000001729.
15. Jacob A, Hutchinson M, Elsone L, et al. Does natalizumab therapy worsen neuromyelitis optica? Neurology. 2012;79(10):1065-1066. https://doi.org/10.1212/WNL.0b013e31826845fe.

Article PDF
Author and Disclosure Information

1Department of Medicine, Johns Hopkins University School of Medicine, Baltimore, Maryland; 2Department of Medicine, University of California San Francisco, San Francisco, California; 3Department of Neurology, Johns Hopkins University School of Medicine, Baltimore, Maryland.

Disclosures

The authors have nothing to disclose.

Issue
Journal of Hospital Medicine 15(4)
Publications
Topics
Page Number
245-249. Published online first February 19, 2020.
Sections
Author and Disclosure Information

1Department of Medicine, Johns Hopkins University School of Medicine, Baltimore, Maryland; 2Department of Medicine, University of California San Francisco, San Francisco, California; 3Department of Neurology, Johns Hopkins University School of Medicine, Baltimore, Maryland.

Disclosures

The authors have nothing to disclose.

Author and Disclosure Information

1Department of Medicine, Johns Hopkins University School of Medicine, Baltimore, Maryland; 2Department of Medicine, University of California San Francisco, San Francisco, California; 3Department of Neurology, Johns Hopkins University School of Medicine, Baltimore, Maryland.

Disclosures

The authors have nothing to disclose.

Article PDF
Article PDF
Related Articles

A 38-year-old woman presented to her primary care clinic with 3 weeks of progressive numbness and tingling sensation, which began in both hands and then progressed to involve both feet, ascending from the legs to the chest while sparing her buttocks. She also noted weakness of her left leg, but no other motor symptoms were reported. She had no fevers, chills, weight loss, bladder dysfunction, nausea, vomiting, or diarrhea.

As with all neurological complaints, localization of the process will often inform a more specific differential diagnosis. If both sensory and motor findings are present, both central and peripheral nerve processes deserve consideration. The onset of paresthesia in the hands, rapid progression to the trunk, and unilateral leg weakness would be inconsistent with a length-dependent peripheral neuropathy. The distribution of complaints and the sacral sparing suggests a myelopathic process involving the cervical region rather than a cauda equina or conus lesions. In an otherwise healthy person of this age and gender, an inflammatory demyelinating disease affecting the cord including multiple sclerosis (MS) would be a strong consideration, although metabolic, vascular, infectious, compressive, or neoplastic disease of the spinal cord could also present with similar subacute onset and pattern of deficits.

Her medical history included morbid obesity, dry eyes, depression, iron deficiency anemia requiring recurrent intravenous replenishment, and abnormal uterine bleeding. Her surgical history included gastric band placement 7 years earlier with removal 5 years later due to persistent gastroesophageal reflux disease, dysphagia, nausea, and vomiting. The gastric band removal was complicated by chronic abdominal pain. Her medications consisted of duloxetine, intermittent iron infusions, artificial tears, loratadine, and pregabalin. She was sexually active with her husband. She consumed alcohol occasionally but did not smoke tobacco or use illicit drugs.

On exam, her temperature was 36.6°C (97.8°F), blood pressure 132/84 mm Hg, and heart rate 85 beats per minute. Body mass index was 39.5 kg/m2. The cardiac, pulmonary, and skin examinations were normal. The abdomen was soft with diffuse tenderness to palpation without rebound or guarding. Examination of cranial nerves 2-12 was normal. Cognition, strength, proprioception, deep tendon reflexes, and light touch were all normal. Her gait was normal, and the Romberg test was negative.

The normal neurologic exam is reassuring but imperfectly sensitive and does not eliminate the possibility of underlying neuropathology. Bariatric surgery may result in an array of nutritional deficiencies such as vitamin E, B12, and copper, which can cause myelopathy and/or neuropathy. However, these abnormalities occur less frequently with gastric banding procedures. If her dry eyes are part of the sicca syndrome, an underlying autoimmune diathesis may be present. Her unexplained chronic abdominal pain prompts considering nonmenstrual causes of iron deficiency anemia, such as celiac disease. Bariatric surgery may contribute to iron deficiency through impaired iron absorption. Her stable weight and lack of diarrhea argue against Crohn’s or celiac disease. Iron deficiency predisposes individuals to pica, most commonly described with ice chip ingestion. If lead pica had occurred, abdominal and neurological symptoms could result. Nevertheless, the abdominal pain is nonspecific, and its occurrence after gastric band removal makes its link to her neurologic syndrome unclear. An initial evaluation would include basic metabolic panel, complete blood count with differential, erythrocyte sedimentation rate, C-reactive protein (CRP), thyroid-stimulating hormone, vitamin B12, and copper levels.

A basic metabolic panel was normal. The white cell count was 5,710 per cubic millimeter, hemoglobin level 12.2 g per deciliter, mean corpuscular volume 85.2 fl, and platelet count 279,000 per cubic millimeter. The serum ferritin level was 18 ng per milliliter (normal range, 13-150), iron 28 µg per deciliter (normal range, 50-170), total iron-binding capacity 364 µg per deciliter (normal range, 250-450), and iron saturation 8% (normal range, 20-55). The vitamin B12 level was 621 pg per milliliter (normal range, 232-1,245) and thyroid-stimulating hormone level 1.87 units per milliliter (normal range, 0.50-4.50). Electrolyte and aminotransferase levels were within normal limits. CRP was 1.0 mg per deciliter (normal range, <0.5) and erythrocyte sedimentation rate 33 millimeters per hour (normal range, 4-25). Hepatitis C and HIV antibodies were nonreactive.

The ongoing iron deficiency despite parenteral iron replacement raises the question of ongoing gastrointestinal or genitourinary blood loss. While the level of vitamin B12 in the serum may be misleadingly normal with cobalamin deficiency, a methylmalonic acid level is indicated to evaluate whether tissue stores are depleted. Copper levels are warranted given the prior bariatric surgery. The mild elevations of inflammatory markers are nonspecific but reduce the likelihood of a highly inflammatory process to account for the neurological and abdominal symptoms. 

At her 3-month follow-up visit, she noted that the paresthesia had improved and was now limited to her bilateral lower extremities. During the same clinic visit, she experienced a 45-minute episode of ascending left upper extremity numbness. Her physical examination revealed normal strength and reflexes. She had diminished response to pinprick in both legs to the knees and in both hands to the wrists. Vibration sense was diminished in the bilateral lower extremities.

 

 

A glycosylated hemoglobin (HbA1c) level was 6.2%. Methylmalonic acid was 69 nmol per liter (normal range, 45-325). Antibodies to Borrelia burgdorferi and Treponema pallidum were absent. Impaired glucose metabolism was the leading diagnosis for her polyneuropathy, and it was recommended that she undergo an oral glucose tolerance test. Electromyography was not performed.

 

The neurological symptoms are now chronic, and importantly, the patient has developed sensory deficits on neurological examination, suggesting worsening of the underlying process. While the paresthesia is now limited to a “stocking/glove” distribution consistent with distal sensory polyneuropathy, there should still be a concern for spinal cord pathology given that the HbA1c level of 6.2 would not explain her initial distribution of symptoms. Myelopathy may mimic peripheral nerve disease if, for example, there is involvement of the dorsal columns leading to sensory deficits of vibration and proprioception. Additionally, the transient episode of upper extremity numbness raises the question of sensory nerve root involvement (ie, sensory radiculopathy). Unexplained abdominal pain could possibly represent the involvement of other nerve roots innervating the abdominal wall. The patient’s episode of focal arm numbness recalls the lancinating radicular pain of tabes dorsalis; however, the negative specific treponemal antibody test excludes neurosyphilis.

The differential diagnosis going forward will be strongly conditioned by the localization of the neurological lesion(s). To differentiate between myelopathy, radiculopathy, and peripheral neuropathy, I would perform nerve conduction studies, magnetic resonance imaging (MRI) of the spinal cord, and cerebrospinal fluid analysis.

The patient began taking a multivitamin, and after weeks her paresthesia had resolved. One month later, she developed an intermittent, throbbing left-sided headache and pain behind the left eye that was worsened with ocular movement. She then noted decreased visual acuity in her left eye that progressed the following month. She denied photophobia, flashers, or floaters.

In the emergency department, visual acuity was 20/25 in her right eye; in the left eye she was only able to count fingers. Extraocular movements of both eyes were normal as was her right pupillary reflex. Red desaturation and a relative afferent papillary defect were present in the left eye. Fundoscopic exam demonstrated left optic disc swelling. The remainder of her cranial nerves were normal. She had pronation of the left upper extremity and mild right finger-to-nose dysmetria. Muscle tone, strength, sensation, and deep tendon reflexes were normal.

The improvement in the sensory symptoms was unlikely to be related to the nutritional intervention and provides a clue to an underlying waxing and waning illness. That interpretation is supported by the subsequent development of new visual symptoms and signs, which point to optic nerve pathology. Optic neuropathy has a broad differential diagnosis that includes ischemic, metabolic, toxic, and compressive causes. Eye pain, swelling of the optic disc, and prominent impairment of color vision all point to the more specific syndrome of optic neuritis caused by infections (including both Treponema pallidum and Borrelia species), systemic autoimmune diseases (systemic lupus erythematosus or Sjogren’s syndrome), and central nervous system (CNS) demyelinating diseases. Of these, inflammatory demyelinating processes would be the likeliest explanation of intermittent and improving neurologic findings.

 

 

With relapsing symptoms and findings that are separate in distribution and time, two diagnoses become most likely, and both of these are most often diagnosed in young women. MS is common, and optic neuritis occurs in more than 50% of patients over the course of illness. Neuromyelitis optica spectrum disorder (NMOSD) is a rare condition that can exist in isolation or be associated with other autoimmune illnesses. While these entities are difficult to differentiate clinically, neuroimaging that demonstrates extensive intracerebral demyelinating lesions and cerebrospinal fluid with oligoclonal bands favor MS, whereas extensive, predominant spinal cord involvement is suggestive of NMOSD. Approximately 70% of NMO patients harbor an antibody directed against the aquaporin-4 channel, and these antibodies are not seen in patients with MS. A milder NMO-like disorder has also been associated with antimyelin oligodendrocyte antibodies (MOG).

Testing for antinuclear antibodies, anti–double-stranded DNA, anti-Ro (SSA), and anti-La (SSB) antibodies was negative. The level of C3 was 162 mg per deciliter (normal range 81-157) and C4 38 (normal range 13-39). T-spot testing for latent tuberculosis was negative.

There is no serological evidence of active systemic lupus erythematosus or Sjogren’s syndrome. The pretest probability of CNS tuberculosis was low in light of her presenting complaints, relatively protracted course, and overall clinical stability without antituberculous therapy. Tests for latent tuberculosis infection have significant limitations of both sensitivity and specificity for the diagnosis of active disease.

Optical coherence tomography showed optic disc edema in the left eye only. MRI of the head with contrast revealed abnormal signal intensity involving the posterior aspect of the pons, right middle cerebellar peduncle, anterior left temporal lobe, bilateral periventricular white matter, subcortical white matter of the frontal lobes bilaterally, and medulla with abnormal signal and enhancement of the left optic nerve (Figure, Panel A). MRI of the cervical and thoracic spine demonstrated multifocal demyelinating lesions at C3, C4, C7, T4, T5, T7, and T8 (Figure, Panel B). The lesions were not longitudinally extensive. There was no significant postcontrast enhancement to suggest active demyelination.

manesh03770219e_f1.jpg

The cerebrospinal fluid analysis revealed glucose of 105 mg per deciliter and a total protein of 26.1 mg per deciliter. In the fourth tube, there were 20 red cells per cubic and four white cells with a differential of 62% neutrophils, 35% lymphocytes, and 3% monocytes. Epstein-Barr and herpes simplex virus DNA were negative. A Venereal Disease Research Laboratory test was negative. Multiple oligoclonal IgG bands were identified only in the cerebrospinal fluid. Aquaporin-4 IgG and MOG antibodies were negative.

In addition to the expected finding of enhancement of the optic nerve, MRI demonstrated numerous multifocal white matter lesions throughout the cerebrum, brainstem, and spinal cord. Many of the lesions were in “silent” areas, which is not directly attributable to specific symptoms, but several did correlate with the subtler deficits of weakness and dysmetria that were noted on examination. Although such lesions may be seen with a diverse group of systemic diseases including adrenal leukodystrophy, sarcoidosis, Behcet’s, cerebral lupus, and vasculitis, primary CNS inflammatory demyelinating diseases are much more likely. The extensive distribution of demyelination argues against NMOSD. The negative aquaporin-4 and MOG assays support this conclusion. Not all multifocal CNS demyelination is caused by MS and can be seen in posterior reversible encephalopathy syndrome, cerebral autosomal dominant arteriopathy with subcortical infarcts and leukoencephalopathy, and adult polyglucosan body disease. Osmotic demyelination is increasingly being recognized as a process that can be more widespread rather than just being limited to the pons. Viral infections of the CNS such as the JC virus (PML) may also provoke multifocal demyelination. Acute disseminated encephalomyelitis is most often seen during childhood, usually after vaccination or after an infectious prodrome. The tempo of the progression of these other diseases tends to be much more rapid than this woman’s course, and often, the neurological deficits are more profound and debilitating. The clinical presentation of sensory-predominant myelopathy, followed by optic neuritis, absence of systemic inflammatory signs or laboratory markers, exclusion of other relevant diseases, multifocal white matter lesions on imaging, minimal pleocytosis, and presence of oligoclonal bands in cerebrospinal fluid, all point to a diagnosis of relapsing-remitting MS.

The patient was diagnosed with MS. She was admitted to the neurology service and treated with 1,000 mg IV methylprednisolone for 3 days with a prompt improvement in her vision. She was started on natalizumab without a relapse of symptoms over the past year.

 

 

COMMENTARY

Multiple sclerosis is a chronic demyelinating disease of the CNS.1 The diagnosis of MS has classically been based upon compatible clinical and radiographic evidence of pathology that is disseminated in space and time. Patients typically present with an initial clinically isolated syndrome—involving changes in vision, sensation, strength, mobility, or cognition—for which there is radiographic evidence of demyelination.2 A diagnosis of clinically definite MS is then often made based on a subsequent relapse of symptoms.3

An interval from initial symptoms has been central to the diagnosis of MS (“lesions disseminated in time”). However, recent evidence questions this diagnostic paradigm, and a more rapid diagnosis of MS has been recommended. This recommendation is reflected in the updated McDonald criteria, according to which, if a clinical presentation is supported by the presence of oligoclonal bands in the cerebrospinal fluid, a diagnosis can be made on the basis of radiographic evidence of dissemination of disease in space, without evidence of dissemination in time.4 The importance of such early diagnosis has been supported by numerous studies that have demonstrated improved clinical outcomes with early therapy.5-7

Despite the McDonald criteria, delays in definitive diagnosis are common in MS. Patients with MS in Spain were found to experience a 2-year delay from the first onset of symptoms to diagnosis.8 In this cohort, patients exhibited delays in presenting to a healthcare provider, as well as delays in diagnosis with an average time from seeing an initial provider to diagnosis of 6 months. When patients who were referred for a demyelinating episode were surveyed, over a third reported a prior suggestive event.9 The time from the first suggestive episode to referral to a neurologist for a recognized demyelinating event was 46 months. Other studies have shown that delays in diagnosis are especially common in younger patients, those with primary progressive MS, and those with comorbid disease.10,11

Misapplication of an MS diagnosis also occurs frequently. In one case series, such misapplication was found most often in cases involving migraine, fibromyalgia, psychogenic disorders, and NMOSD.12 NMOSD is distinguished from MS by the presence of typical brain and spine findings on MRI.13 Antibodies to aquaporin-4 are highly specific and moderately sensitive for the disease.14 It is important to distinguish NMOSD from MS as certain disease-modifying drugs used for MS might actually exacerbate NMOSD.15 A lesion that traverses over three or more contiguous vertebral segments with predominant involvement of central gray matter (ie, longitudinally extensive transverse myelitis) on MRI is the most distinct finding of NMOSD. In contrast, similar to our patient, short and often multiple lesions are demonstrated on spinal cord MRI in patients with MS. Sensitive and specific findings of brain MRI in patients with MS include the presence of lateral ventricle and inferior temporal lobe lesion, Dawson’s fingers, central vein sign, or an S-shaped U-fiber lesion. In NMOSD, brain MRI might reveal periependymal lesions surrounding the ventricular system.

This case highlights the diagnostic challenges related to presentations of a waxing and waning neurological process. At the time of the second evaluation, the presentation was interpreted as a length-dependent polyneuropathy due to glucose intolerance. Our patient’s relatively normal HbA1c, subacute onset of neuropathic symptoms (ie, <4 weeks), sensory and motor complaints, and onset in the upper extremities suggested an alternative diagnosis to prediabetes. Once the patient presented with optic neuritis, the cause of the initial symptoms was obvious, but then, hindsight is 20/20.

 

 

TEACHING POINTS

  • Early treatment of MS results in improved clinical outcomes.
  • Delays in the definitive diagnosis of MS are common, especially in younger patients, those with primary progressive MS, and those with comorbid disease.
  • If a clinical presentation is supported by the presence of oligoclonal bands in the cerebrospinal fluid, a diagnosis of MS can be made on the basis of radiographic evidence of dissemination of disease in space, without evidence of dissemination in time.

Acknowledgments

The authors wish to thank Rabih Geha, MD, and Gurpreet Dhaliwal, MD, for providing feedback on an earlier version of this manuscript.

A 38-year-old woman presented to her primary care clinic with 3 weeks of progressive numbness and tingling sensation, which began in both hands and then progressed to involve both feet, ascending from the legs to the chest while sparing her buttocks. She also noted weakness of her left leg, but no other motor symptoms were reported. She had no fevers, chills, weight loss, bladder dysfunction, nausea, vomiting, or diarrhea.

As with all neurological complaints, localization of the process will often inform a more specific differential diagnosis. If both sensory and motor findings are present, both central and peripheral nerve processes deserve consideration. The onset of paresthesia in the hands, rapid progression to the trunk, and unilateral leg weakness would be inconsistent with a length-dependent peripheral neuropathy. The distribution of complaints and the sacral sparing suggests a myelopathic process involving the cervical region rather than a cauda equina or conus lesions. In an otherwise healthy person of this age and gender, an inflammatory demyelinating disease affecting the cord including multiple sclerosis (MS) would be a strong consideration, although metabolic, vascular, infectious, compressive, or neoplastic disease of the spinal cord could also present with similar subacute onset and pattern of deficits.

Her medical history included morbid obesity, dry eyes, depression, iron deficiency anemia requiring recurrent intravenous replenishment, and abnormal uterine bleeding. Her surgical history included gastric band placement 7 years earlier with removal 5 years later due to persistent gastroesophageal reflux disease, dysphagia, nausea, and vomiting. The gastric band removal was complicated by chronic abdominal pain. Her medications consisted of duloxetine, intermittent iron infusions, artificial tears, loratadine, and pregabalin. She was sexually active with her husband. She consumed alcohol occasionally but did not smoke tobacco or use illicit drugs.

On exam, her temperature was 36.6°C (97.8°F), blood pressure 132/84 mm Hg, and heart rate 85 beats per minute. Body mass index was 39.5 kg/m2. The cardiac, pulmonary, and skin examinations were normal. The abdomen was soft with diffuse tenderness to palpation without rebound or guarding. Examination of cranial nerves 2-12 was normal. Cognition, strength, proprioception, deep tendon reflexes, and light touch were all normal. Her gait was normal, and the Romberg test was negative.

The normal neurologic exam is reassuring but imperfectly sensitive and does not eliminate the possibility of underlying neuropathology. Bariatric surgery may result in an array of nutritional deficiencies such as vitamin E, B12, and copper, which can cause myelopathy and/or neuropathy. However, these abnormalities occur less frequently with gastric banding procedures. If her dry eyes are part of the sicca syndrome, an underlying autoimmune diathesis may be present. Her unexplained chronic abdominal pain prompts considering nonmenstrual causes of iron deficiency anemia, such as celiac disease. Bariatric surgery may contribute to iron deficiency through impaired iron absorption. Her stable weight and lack of diarrhea argue against Crohn’s or celiac disease. Iron deficiency predisposes individuals to pica, most commonly described with ice chip ingestion. If lead pica had occurred, abdominal and neurological symptoms could result. Nevertheless, the abdominal pain is nonspecific, and its occurrence after gastric band removal makes its link to her neurologic syndrome unclear. An initial evaluation would include basic metabolic panel, complete blood count with differential, erythrocyte sedimentation rate, C-reactive protein (CRP), thyroid-stimulating hormone, vitamin B12, and copper levels.

A basic metabolic panel was normal. The white cell count was 5,710 per cubic millimeter, hemoglobin level 12.2 g per deciliter, mean corpuscular volume 85.2 fl, and platelet count 279,000 per cubic millimeter. The serum ferritin level was 18 ng per milliliter (normal range, 13-150), iron 28 µg per deciliter (normal range, 50-170), total iron-binding capacity 364 µg per deciliter (normal range, 250-450), and iron saturation 8% (normal range, 20-55). The vitamin B12 level was 621 pg per milliliter (normal range, 232-1,245) and thyroid-stimulating hormone level 1.87 units per milliliter (normal range, 0.50-4.50). Electrolyte and aminotransferase levels were within normal limits. CRP was 1.0 mg per deciliter (normal range, <0.5) and erythrocyte sedimentation rate 33 millimeters per hour (normal range, 4-25). Hepatitis C and HIV antibodies were nonreactive.

The ongoing iron deficiency despite parenteral iron replacement raises the question of ongoing gastrointestinal or genitourinary blood loss. While the level of vitamin B12 in the serum may be misleadingly normal with cobalamin deficiency, a methylmalonic acid level is indicated to evaluate whether tissue stores are depleted. Copper levels are warranted given the prior bariatric surgery. The mild elevations of inflammatory markers are nonspecific but reduce the likelihood of a highly inflammatory process to account for the neurological and abdominal symptoms. 

At her 3-month follow-up visit, she noted that the paresthesia had improved and was now limited to her bilateral lower extremities. During the same clinic visit, she experienced a 45-minute episode of ascending left upper extremity numbness. Her physical examination revealed normal strength and reflexes. She had diminished response to pinprick in both legs to the knees and in both hands to the wrists. Vibration sense was diminished in the bilateral lower extremities.

 

 

A glycosylated hemoglobin (HbA1c) level was 6.2%. Methylmalonic acid was 69 nmol per liter (normal range, 45-325). Antibodies to Borrelia burgdorferi and Treponema pallidum were absent. Impaired glucose metabolism was the leading diagnosis for her polyneuropathy, and it was recommended that she undergo an oral glucose tolerance test. Electromyography was not performed.

 

The neurological symptoms are now chronic, and importantly, the patient has developed sensory deficits on neurological examination, suggesting worsening of the underlying process. While the paresthesia is now limited to a “stocking/glove” distribution consistent with distal sensory polyneuropathy, there should still be a concern for spinal cord pathology given that the HbA1c level of 6.2 would not explain her initial distribution of symptoms. Myelopathy may mimic peripheral nerve disease if, for example, there is involvement of the dorsal columns leading to sensory deficits of vibration and proprioception. Additionally, the transient episode of upper extremity numbness raises the question of sensory nerve root involvement (ie, sensory radiculopathy). Unexplained abdominal pain could possibly represent the involvement of other nerve roots innervating the abdominal wall. The patient’s episode of focal arm numbness recalls the lancinating radicular pain of tabes dorsalis; however, the negative specific treponemal antibody test excludes neurosyphilis.

The differential diagnosis going forward will be strongly conditioned by the localization of the neurological lesion(s). To differentiate between myelopathy, radiculopathy, and peripheral neuropathy, I would perform nerve conduction studies, magnetic resonance imaging (MRI) of the spinal cord, and cerebrospinal fluid analysis.

The patient began taking a multivitamin, and after weeks her paresthesia had resolved. One month later, she developed an intermittent, throbbing left-sided headache and pain behind the left eye that was worsened with ocular movement. She then noted decreased visual acuity in her left eye that progressed the following month. She denied photophobia, flashers, or floaters.

In the emergency department, visual acuity was 20/25 in her right eye; in the left eye she was only able to count fingers. Extraocular movements of both eyes were normal as was her right pupillary reflex. Red desaturation and a relative afferent papillary defect were present in the left eye. Fundoscopic exam demonstrated left optic disc swelling. The remainder of her cranial nerves were normal. She had pronation of the left upper extremity and mild right finger-to-nose dysmetria. Muscle tone, strength, sensation, and deep tendon reflexes were normal.

The improvement in the sensory symptoms was unlikely to be related to the nutritional intervention and provides a clue to an underlying waxing and waning illness. That interpretation is supported by the subsequent development of new visual symptoms and signs, which point to optic nerve pathology. Optic neuropathy has a broad differential diagnosis that includes ischemic, metabolic, toxic, and compressive causes. Eye pain, swelling of the optic disc, and prominent impairment of color vision all point to the more specific syndrome of optic neuritis caused by infections (including both Treponema pallidum and Borrelia species), systemic autoimmune diseases (systemic lupus erythematosus or Sjogren’s syndrome), and central nervous system (CNS) demyelinating diseases. Of these, inflammatory demyelinating processes would be the likeliest explanation of intermittent and improving neurologic findings.

 

 

With relapsing symptoms and findings that are separate in distribution and time, two diagnoses become most likely, and both of these are most often diagnosed in young women. MS is common, and optic neuritis occurs in more than 50% of patients over the course of illness. Neuromyelitis optica spectrum disorder (NMOSD) is a rare condition that can exist in isolation or be associated with other autoimmune illnesses. While these entities are difficult to differentiate clinically, neuroimaging that demonstrates extensive intracerebral demyelinating lesions and cerebrospinal fluid with oligoclonal bands favor MS, whereas extensive, predominant spinal cord involvement is suggestive of NMOSD. Approximately 70% of NMO patients harbor an antibody directed against the aquaporin-4 channel, and these antibodies are not seen in patients with MS. A milder NMO-like disorder has also been associated with antimyelin oligodendrocyte antibodies (MOG).

Testing for antinuclear antibodies, anti–double-stranded DNA, anti-Ro (SSA), and anti-La (SSB) antibodies was negative. The level of C3 was 162 mg per deciliter (normal range 81-157) and C4 38 (normal range 13-39). T-spot testing for latent tuberculosis was negative.

There is no serological evidence of active systemic lupus erythematosus or Sjogren’s syndrome. The pretest probability of CNS tuberculosis was low in light of her presenting complaints, relatively protracted course, and overall clinical stability without antituberculous therapy. Tests for latent tuberculosis infection have significant limitations of both sensitivity and specificity for the diagnosis of active disease.

Optical coherence tomography showed optic disc edema in the left eye only. MRI of the head with contrast revealed abnormal signal intensity involving the posterior aspect of the pons, right middle cerebellar peduncle, anterior left temporal lobe, bilateral periventricular white matter, subcortical white matter of the frontal lobes bilaterally, and medulla with abnormal signal and enhancement of the left optic nerve (Figure, Panel A). MRI of the cervical and thoracic spine demonstrated multifocal demyelinating lesions at C3, C4, C7, T4, T5, T7, and T8 (Figure, Panel B). The lesions were not longitudinally extensive. There was no significant postcontrast enhancement to suggest active demyelination.

manesh03770219e_f1.jpg

The cerebrospinal fluid analysis revealed glucose of 105 mg per deciliter and a total protein of 26.1 mg per deciliter. In the fourth tube, there were 20 red cells per cubic and four white cells with a differential of 62% neutrophils, 35% lymphocytes, and 3% monocytes. Epstein-Barr and herpes simplex virus DNA were negative. A Venereal Disease Research Laboratory test was negative. Multiple oligoclonal IgG bands were identified only in the cerebrospinal fluid. Aquaporin-4 IgG and MOG antibodies were negative.

In addition to the expected finding of enhancement of the optic nerve, MRI demonstrated numerous multifocal white matter lesions throughout the cerebrum, brainstem, and spinal cord. Many of the lesions were in “silent” areas, which is not directly attributable to specific symptoms, but several did correlate with the subtler deficits of weakness and dysmetria that were noted on examination. Although such lesions may be seen with a diverse group of systemic diseases including adrenal leukodystrophy, sarcoidosis, Behcet’s, cerebral lupus, and vasculitis, primary CNS inflammatory demyelinating diseases are much more likely. The extensive distribution of demyelination argues against NMOSD. The negative aquaporin-4 and MOG assays support this conclusion. Not all multifocal CNS demyelination is caused by MS and can be seen in posterior reversible encephalopathy syndrome, cerebral autosomal dominant arteriopathy with subcortical infarcts and leukoencephalopathy, and adult polyglucosan body disease. Osmotic demyelination is increasingly being recognized as a process that can be more widespread rather than just being limited to the pons. Viral infections of the CNS such as the JC virus (PML) may also provoke multifocal demyelination. Acute disseminated encephalomyelitis is most often seen during childhood, usually after vaccination or after an infectious prodrome. The tempo of the progression of these other diseases tends to be much more rapid than this woman’s course, and often, the neurological deficits are more profound and debilitating. The clinical presentation of sensory-predominant myelopathy, followed by optic neuritis, absence of systemic inflammatory signs or laboratory markers, exclusion of other relevant diseases, multifocal white matter lesions on imaging, minimal pleocytosis, and presence of oligoclonal bands in cerebrospinal fluid, all point to a diagnosis of relapsing-remitting MS.

The patient was diagnosed with MS. She was admitted to the neurology service and treated with 1,000 mg IV methylprednisolone for 3 days with a prompt improvement in her vision. She was started on natalizumab without a relapse of symptoms over the past year.

 

 

COMMENTARY

Multiple sclerosis is a chronic demyelinating disease of the CNS.1 The diagnosis of MS has classically been based upon compatible clinical and radiographic evidence of pathology that is disseminated in space and time. Patients typically present with an initial clinically isolated syndrome—involving changes in vision, sensation, strength, mobility, or cognition—for which there is radiographic evidence of demyelination.2 A diagnosis of clinically definite MS is then often made based on a subsequent relapse of symptoms.3

An interval from initial symptoms has been central to the diagnosis of MS (“lesions disseminated in time”). However, recent evidence questions this diagnostic paradigm, and a more rapid diagnosis of MS has been recommended. This recommendation is reflected in the updated McDonald criteria, according to which, if a clinical presentation is supported by the presence of oligoclonal bands in the cerebrospinal fluid, a diagnosis can be made on the basis of radiographic evidence of dissemination of disease in space, without evidence of dissemination in time.4 The importance of such early diagnosis has been supported by numerous studies that have demonstrated improved clinical outcomes with early therapy.5-7

Despite the McDonald criteria, delays in definitive diagnosis are common in MS. Patients with MS in Spain were found to experience a 2-year delay from the first onset of symptoms to diagnosis.8 In this cohort, patients exhibited delays in presenting to a healthcare provider, as well as delays in diagnosis with an average time from seeing an initial provider to diagnosis of 6 months. When patients who were referred for a demyelinating episode were surveyed, over a third reported a prior suggestive event.9 The time from the first suggestive episode to referral to a neurologist for a recognized demyelinating event was 46 months. Other studies have shown that delays in diagnosis are especially common in younger patients, those with primary progressive MS, and those with comorbid disease.10,11

Misapplication of an MS diagnosis also occurs frequently. In one case series, such misapplication was found most often in cases involving migraine, fibromyalgia, psychogenic disorders, and NMOSD.12 NMOSD is distinguished from MS by the presence of typical brain and spine findings on MRI.13 Antibodies to aquaporin-4 are highly specific and moderately sensitive for the disease.14 It is important to distinguish NMOSD from MS as certain disease-modifying drugs used for MS might actually exacerbate NMOSD.15 A lesion that traverses over three or more contiguous vertebral segments with predominant involvement of central gray matter (ie, longitudinally extensive transverse myelitis) on MRI is the most distinct finding of NMOSD. In contrast, similar to our patient, short and often multiple lesions are demonstrated on spinal cord MRI in patients with MS. Sensitive and specific findings of brain MRI in patients with MS include the presence of lateral ventricle and inferior temporal lobe lesion, Dawson’s fingers, central vein sign, or an S-shaped U-fiber lesion. In NMOSD, brain MRI might reveal periependymal lesions surrounding the ventricular system.

This case highlights the diagnostic challenges related to presentations of a waxing and waning neurological process. At the time of the second evaluation, the presentation was interpreted as a length-dependent polyneuropathy due to glucose intolerance. Our patient’s relatively normal HbA1c, subacute onset of neuropathic symptoms (ie, <4 weeks), sensory and motor complaints, and onset in the upper extremities suggested an alternative diagnosis to prediabetes. Once the patient presented with optic neuritis, the cause of the initial symptoms was obvious, but then, hindsight is 20/20.

 

 

TEACHING POINTS

  • Early treatment of MS results in improved clinical outcomes.
  • Delays in the definitive diagnosis of MS are common, especially in younger patients, those with primary progressive MS, and those with comorbid disease.
  • If a clinical presentation is supported by the presence of oligoclonal bands in the cerebrospinal fluid, a diagnosis of MS can be made on the basis of radiographic evidence of dissemination of disease in space, without evidence of dissemination in time.

Acknowledgments

The authors wish to thank Rabih Geha, MD, and Gurpreet Dhaliwal, MD, for providing feedback on an earlier version of this manuscript.

References

1. Reich DS, Lucchinetti CF, Calabresi PA. Multiple sclerosis. N Engl J Med. 2018;378:169-180. https://doi.org/10.1056/NEJMra140148.
2. Brownlee WJ, Hardy TA, Fazekas F, Miller DH. Diagnosis of multiple sclerosis: progress and challenges. Lancet. 2017;389(10076):1336-1346. https://doi.org/10.1016/S0140-6736(16)30959-X.
3. Thompson AJ, Baranzini SE, Geurts J, Hemmer B, Ciccarelli O. Multiple sclerosis. Lancet. 2018;391(10130):1622-1636. https://doi.org/10.1016/S0140-6736(18)30481-1.
4. Thompson AJ, Banwell BL, Barkhof F, et al. Diagnosis of multiple sclerosis: 2017 revisions of the McDonald criteria. Lancet Neurol. 2018;17(2):162-173. https://doi.org/10.1016/S1474-4422(17)30470-2.
5. Comi G, Radaelli M, Soelberg Sørensen P. Evolving concepts in the treatment of relapsing multiple sclerosis. Lancet. 2017;389(10076):1347-1356. https://doi.org/10.1016/S0140-6736(16)32388-1.
6. Freedman MS, Comi G, De Stefano N, et al. Moving toward earlier treatment of multiple sclerosis: Findings from a decade of clinical trials and implications for clinical practice. Mult Scler Relat Disord. 2014;3(2):147-155. https://doi.org/10.1016/j.msard.2013.07.001.
7. Harding K, Williams O, Willis M, et al. Clinical outcomes of escalation vs early intensive disease-modifying therapy in patients with multiple sclerosis. JAMA Neurol. 2019;76(5):536-541. https://doi.org/10.1001/jamaneurol.2018.4905.
8. Fernández O, Fernández V, Arbizu T, et al. Characteristics of multiple sclerosis at onset and delay of diagnosis and treatment in Spain (the Novo Study). J Neurol. 257(9):1500-1507. https://doi.org/10.1007/s00415-010-5560-1.
9. Gout O, Lebrun-Frenay C, Labauge P, et al. Prior suggestive symptoms in one-third of patients consulting for a “first” demyelinating event. J Neurol Neurosurg Psychiatry 2011;82(3):323-325. https://doi.org/10.1136/jnnp.2008.166421.
10. Kingwell E, Leung A, Roger E, et al. Factors associated with delay to medical recognition in two Canadian multiple sclerosis cohorts. J Neurol Sci. 2010(1-2);292:57-62. https://doi.org/10.1016/j.jns.2010.02.007.
11. Marrie RA, Horwitz R, Cutter G, Tyry T, Campagnolo D, Vollmer T. Comorbidity delays diagnosis and increases disability at diagnosis in MS. Neurology. 2009;72(2):117-124. https://doi.org/10.1212/01.wnl.0000333252.78173.5f.
12. Solomon AJ, Bourdette DN, Cross AH, et al. The contemporary spectrum of multiple sclerosis misdiagnosis: A multicenter study. Neurology. 2016;87(13):1393-1399. https://doi.org/10.1212/WNL.0000000000003152.
13. Kim HJ, Paul F, Lana-Peixoto MA, et al. MRI characteristics of neuromyelitis optica spectrum disorder: An international update. Neurology. 2015;84(11):1165-1173. https://doi.org/10.1212/WNL.0000000000001367.
14. Wingerchuk DM, Banwell B, Bennett JL, et al. International consensus diagnostic criteria for neuromyelitis optica spectrum disorders. Neurology. 2015;85(2):177-189. https://doi.org/10.1212/WNL.0000000000001729.
15. Jacob A, Hutchinson M, Elsone L, et al. Does natalizumab therapy worsen neuromyelitis optica? Neurology. 2012;79(10):1065-1066. https://doi.org/10.1212/WNL.0b013e31826845fe.

References

1. Reich DS, Lucchinetti CF, Calabresi PA. Multiple sclerosis. N Engl J Med. 2018;378:169-180. https://doi.org/10.1056/NEJMra140148.
2. Brownlee WJ, Hardy TA, Fazekas F, Miller DH. Diagnosis of multiple sclerosis: progress and challenges. Lancet. 2017;389(10076):1336-1346. https://doi.org/10.1016/S0140-6736(16)30959-X.
3. Thompson AJ, Baranzini SE, Geurts J, Hemmer B, Ciccarelli O. Multiple sclerosis. Lancet. 2018;391(10130):1622-1636. https://doi.org/10.1016/S0140-6736(18)30481-1.
4. Thompson AJ, Banwell BL, Barkhof F, et al. Diagnosis of multiple sclerosis: 2017 revisions of the McDonald criteria. Lancet Neurol. 2018;17(2):162-173. https://doi.org/10.1016/S1474-4422(17)30470-2.
5. Comi G, Radaelli M, Soelberg Sørensen P. Evolving concepts in the treatment of relapsing multiple sclerosis. Lancet. 2017;389(10076):1347-1356. https://doi.org/10.1016/S0140-6736(16)32388-1.
6. Freedman MS, Comi G, De Stefano N, et al. Moving toward earlier treatment of multiple sclerosis: Findings from a decade of clinical trials and implications for clinical practice. Mult Scler Relat Disord. 2014;3(2):147-155. https://doi.org/10.1016/j.msard.2013.07.001.
7. Harding K, Williams O, Willis M, et al. Clinical outcomes of escalation vs early intensive disease-modifying therapy in patients with multiple sclerosis. JAMA Neurol. 2019;76(5):536-541. https://doi.org/10.1001/jamaneurol.2018.4905.
8. Fernández O, Fernández V, Arbizu T, et al. Characteristics of multiple sclerosis at onset and delay of diagnosis and treatment in Spain (the Novo Study). J Neurol. 257(9):1500-1507. https://doi.org/10.1007/s00415-010-5560-1.
9. Gout O, Lebrun-Frenay C, Labauge P, et al. Prior suggestive symptoms in one-third of patients consulting for a “first” demyelinating event. J Neurol Neurosurg Psychiatry 2011;82(3):323-325. https://doi.org/10.1136/jnnp.2008.166421.
10. Kingwell E, Leung A, Roger E, et al. Factors associated with delay to medical recognition in two Canadian multiple sclerosis cohorts. J Neurol Sci. 2010(1-2);292:57-62. https://doi.org/10.1016/j.jns.2010.02.007.
11. Marrie RA, Horwitz R, Cutter G, Tyry T, Campagnolo D, Vollmer T. Comorbidity delays diagnosis and increases disability at diagnosis in MS. Neurology. 2009;72(2):117-124. https://doi.org/10.1212/01.wnl.0000333252.78173.5f.
12. Solomon AJ, Bourdette DN, Cross AH, et al. The contemporary spectrum of multiple sclerosis misdiagnosis: A multicenter study. Neurology. 2016;87(13):1393-1399. https://doi.org/10.1212/WNL.0000000000003152.
13. Kim HJ, Paul F, Lana-Peixoto MA, et al. MRI characteristics of neuromyelitis optica spectrum disorder: An international update. Neurology. 2015;84(11):1165-1173. https://doi.org/10.1212/WNL.0000000000001367.
14. Wingerchuk DM, Banwell B, Bennett JL, et al. International consensus diagnostic criteria for neuromyelitis optica spectrum disorders. Neurology. 2015;85(2):177-189. https://doi.org/10.1212/WNL.0000000000001729.
15. Jacob A, Hutchinson M, Elsone L, et al. Does natalizumab therapy worsen neuromyelitis optica? Neurology. 2012;79(10):1065-1066. https://doi.org/10.1212/WNL.0b013e31826845fe.

Issue
Journal of Hospital Medicine 15(4)
Issue
Journal of Hospital Medicine 15(4)
Page Number
245-249. Published online first February 19, 2020.
Page Number
245-249. Published online first February 19, 2020.
Publications
Publications
Topics
Article Type
Sections
Article Source

© 2020 Society of Hospital Medicine

Disallow All Ads
Correspondence Location
Reza Manesh, MD; E-mail: rsedigh1@jhmi.edu; Telephone: 412-708-6944; Twitter: @DxRxEdu
Content Gating
Gated (full article locked unless allowed per User)
Alternative CME
Disqus Comments
Default
Use ProPublica
Hide sidebar & use full width
render the right sidebar.
Conference Recap Checkbox
Not Conference Recap
Clinical Edge
Display the Slideshow in this Article
Gating Strategy
First Peek Free
Medscape Article
Display survey writer
Reuters content
Article PDF Media

Leadership & Professional Development: A Letter to the Future Teaching Physician

Article Type
Changed
Thu, 03/25/2021 - 11:29

“No one cares how much you know, until they know how much you care.”

—Theodore Roosevelt (attributed)

Like many early career clinician-educators, you are likely embarking on your teaching role with excitement and trepidation. Excitement accompanies the opportunity to develop the next generation of physicians. Trepidation arises from a fear of insufficient knowledge. This concern is understandable but misplaced: great teachers are great because of their emotional intelligence, not their medical intelligence. These five principles will help you establish an optimal learning environment.

Small-Talk before Med-Talk. “What do you like to do outside of the hospital?” “Where is your favorite place to eat?” These questions indicate that your interest in learners transcends clinical work. Leaders who are more relationship- than task-oriented achieve greater group cohesion and more team learning. Exemplary inpatient attending physicians use learners’ first names and get to know them on a personal level to signal that they care as much about the person as they do about the performance.1

Be Available. Medical educators balance supervision and autonomy while trainees engage in high-stakes decisions. The best teachers get this right by signaling “I have faith in you” and “I’m always available.” Clinician-educator Kimberly Manning, MD portrayed this balance in a recent Twitter thread. The resident called: “I am sorry to bother you.” Dr. Manning responded, “Never be sorry.” The resident was concerned about a patient with new abdominal pain but reassured Dr. Manning that she did not need to return to the hospital. She returned anyway. She assessed the patient and had nothing to add to the resident’s outstanding management. As the patient recovered from his operation for a perforated ulcer, Dr. Manning reflected, “On a perfect Saturday afternoon, I chose to return to the hospital. To make not one decision or write one single order. But instead to stand beside my resident and intentionally affirm her.”

Build from the Ground Up. Asking questions is the teacher’s core procedure. Strive to master the true Socratic method of starting with an elemental inquiry and then leading a conversation that poses questions of increasing difficulty until you reach the limits of the learner’s understanding. This method reinforces their hard-earned knowledge and sets the stage for growth. “What would be your first test to evaluate tachycardia?” Once the correct answer is firmly in hand, explore the margin of their knowledge. “Which regular, narrow complex tachycardias stop with adenosine?”

Never Judge. Never endorse an incorrect response—but do not disparage it either. A trainee must learn that their answer was wrong but should not feel defeated or embarrassed. Use judgment regarding whether constructive feedback should be delivered in public or in private.

I recall answering a question incorrectly in medical school. The attending responded, “How many years did you take off before starting third year?” I had not taken any time off. The attending was a phenomenal clinician but a lousy teacher. A master teacher would have accessed a foothold and built my knowledge without judgment.

 

 

Remain Humble. One of the most liberating phrases you will deploy as a teacher is “I don’t know.” Its utterance demonstrates the honesty and humility you hope to instill in learners. Be on the lookout for the many times your trainees will know more than you.

Recently my team evaluated a patient with blunted facial expression, bradykinesia, and a resting hand tremor. I disclosed to my team: “I don’t know the key maneuvers to distinguish the Parkinson plus syndromes from Parkinson disease.” The medical student had spent one year studying patients with neurodegenerative diseases (I learned this during the “small-talk before med-talk” phase). I invited him to demonstrate the neurologic exam, which he did admirably. That day I did not know the subject well, and we all learned because I freely admitted it.

Being a physician is the greatest job in the world. If you leverage your EQ (emotional quotient) as much as your IQ (intelligence quotient), your learners will conclude the same.

References

1. Houchens N, Harrod M, Moody S, Fowler KE, Saint S. Techniques and behaviors associated with exemplary inpatient general medicine teaching: an exploratory qualitative study. J Hosp Med. 2017;12(7):503-509. https://doi.org/10.12788/jhm.2763.

Article PDF
Author and Disclosure Information

1Department of Medicine, The Johns Hopkins Hospital, Baltimore, Maryland; 2Department of Medicine, University of California San Francisco, San Francisco, California; 3Medical Service, San Francisco VA Medical Center, San Francisco, California.

Disclosures

The authors have nothing to disclose.

Issue
Journal of Hospital Medicine 15(2)
Publications
Topics
Page Number
94
Sections
Author and Disclosure Information

1Department of Medicine, The Johns Hopkins Hospital, Baltimore, Maryland; 2Department of Medicine, University of California San Francisco, San Francisco, California; 3Medical Service, San Francisco VA Medical Center, San Francisco, California.

Disclosures

The authors have nothing to disclose.

Author and Disclosure Information

1Department of Medicine, The Johns Hopkins Hospital, Baltimore, Maryland; 2Department of Medicine, University of California San Francisco, San Francisco, California; 3Medical Service, San Francisco VA Medical Center, San Francisco, California.

Disclosures

The authors have nothing to disclose.

Article PDF
Article PDF
Related Articles

“No one cares how much you know, until they know how much you care.”

—Theodore Roosevelt (attributed)

Like many early career clinician-educators, you are likely embarking on your teaching role with excitement and trepidation. Excitement accompanies the opportunity to develop the next generation of physicians. Trepidation arises from a fear of insufficient knowledge. This concern is understandable but misplaced: great teachers are great because of their emotional intelligence, not their medical intelligence. These five principles will help you establish an optimal learning environment.

Small-Talk before Med-Talk. “What do you like to do outside of the hospital?” “Where is your favorite place to eat?” These questions indicate that your interest in learners transcends clinical work. Leaders who are more relationship- than task-oriented achieve greater group cohesion and more team learning. Exemplary inpatient attending physicians use learners’ first names and get to know them on a personal level to signal that they care as much about the person as they do about the performance.1

Be Available. Medical educators balance supervision and autonomy while trainees engage in high-stakes decisions. The best teachers get this right by signaling “I have faith in you” and “I’m always available.” Clinician-educator Kimberly Manning, MD portrayed this balance in a recent Twitter thread. The resident called: “I am sorry to bother you.” Dr. Manning responded, “Never be sorry.” The resident was concerned about a patient with new abdominal pain but reassured Dr. Manning that she did not need to return to the hospital. She returned anyway. She assessed the patient and had nothing to add to the resident’s outstanding management. As the patient recovered from his operation for a perforated ulcer, Dr. Manning reflected, “On a perfect Saturday afternoon, I chose to return to the hospital. To make not one decision or write one single order. But instead to stand beside my resident and intentionally affirm her.”

Build from the Ground Up. Asking questions is the teacher’s core procedure. Strive to master the true Socratic method of starting with an elemental inquiry and then leading a conversation that poses questions of increasing difficulty until you reach the limits of the learner’s understanding. This method reinforces their hard-earned knowledge and sets the stage for growth. “What would be your first test to evaluate tachycardia?” Once the correct answer is firmly in hand, explore the margin of their knowledge. “Which regular, narrow complex tachycardias stop with adenosine?”

Never Judge. Never endorse an incorrect response—but do not disparage it either. A trainee must learn that their answer was wrong but should not feel defeated or embarrassed. Use judgment regarding whether constructive feedback should be delivered in public or in private.

I recall answering a question incorrectly in medical school. The attending responded, “How many years did you take off before starting third year?” I had not taken any time off. The attending was a phenomenal clinician but a lousy teacher. A master teacher would have accessed a foothold and built my knowledge without judgment.

 

 

Remain Humble. One of the most liberating phrases you will deploy as a teacher is “I don’t know.” Its utterance demonstrates the honesty and humility you hope to instill in learners. Be on the lookout for the many times your trainees will know more than you.

Recently my team evaluated a patient with blunted facial expression, bradykinesia, and a resting hand tremor. I disclosed to my team: “I don’t know the key maneuvers to distinguish the Parkinson plus syndromes from Parkinson disease.” The medical student had spent one year studying patients with neurodegenerative diseases (I learned this during the “small-talk before med-talk” phase). I invited him to demonstrate the neurologic exam, which he did admirably. That day I did not know the subject well, and we all learned because I freely admitted it.

Being a physician is the greatest job in the world. If you leverage your EQ (emotional quotient) as much as your IQ (intelligence quotient), your learners will conclude the same.

“No one cares how much you know, until they know how much you care.”

—Theodore Roosevelt (attributed)

Like many early career clinician-educators, you are likely embarking on your teaching role with excitement and trepidation. Excitement accompanies the opportunity to develop the next generation of physicians. Trepidation arises from a fear of insufficient knowledge. This concern is understandable but misplaced: great teachers are great because of their emotional intelligence, not their medical intelligence. These five principles will help you establish an optimal learning environment.

Small-Talk before Med-Talk. “What do you like to do outside of the hospital?” “Where is your favorite place to eat?” These questions indicate that your interest in learners transcends clinical work. Leaders who are more relationship- than task-oriented achieve greater group cohesion and more team learning. Exemplary inpatient attending physicians use learners’ first names and get to know them on a personal level to signal that they care as much about the person as they do about the performance.1

Be Available. Medical educators balance supervision and autonomy while trainees engage in high-stakes decisions. The best teachers get this right by signaling “I have faith in you” and “I’m always available.” Clinician-educator Kimberly Manning, MD portrayed this balance in a recent Twitter thread. The resident called: “I am sorry to bother you.” Dr. Manning responded, “Never be sorry.” The resident was concerned about a patient with new abdominal pain but reassured Dr. Manning that she did not need to return to the hospital. She returned anyway. She assessed the patient and had nothing to add to the resident’s outstanding management. As the patient recovered from his operation for a perforated ulcer, Dr. Manning reflected, “On a perfect Saturday afternoon, I chose to return to the hospital. To make not one decision or write one single order. But instead to stand beside my resident and intentionally affirm her.”

Build from the Ground Up. Asking questions is the teacher’s core procedure. Strive to master the true Socratic method of starting with an elemental inquiry and then leading a conversation that poses questions of increasing difficulty until you reach the limits of the learner’s understanding. This method reinforces their hard-earned knowledge and sets the stage for growth. “What would be your first test to evaluate tachycardia?” Once the correct answer is firmly in hand, explore the margin of their knowledge. “Which regular, narrow complex tachycardias stop with adenosine?”

Never Judge. Never endorse an incorrect response—but do not disparage it either. A trainee must learn that their answer was wrong but should not feel defeated or embarrassed. Use judgment regarding whether constructive feedback should be delivered in public or in private.

I recall answering a question incorrectly in medical school. The attending responded, “How many years did you take off before starting third year?” I had not taken any time off. The attending was a phenomenal clinician but a lousy teacher. A master teacher would have accessed a foothold and built my knowledge without judgment.

 

 

Remain Humble. One of the most liberating phrases you will deploy as a teacher is “I don’t know.” Its utterance demonstrates the honesty and humility you hope to instill in learners. Be on the lookout for the many times your trainees will know more than you.

Recently my team evaluated a patient with blunted facial expression, bradykinesia, and a resting hand tremor. I disclosed to my team: “I don’t know the key maneuvers to distinguish the Parkinson plus syndromes from Parkinson disease.” The medical student had spent one year studying patients with neurodegenerative diseases (I learned this during the “small-talk before med-talk” phase). I invited him to demonstrate the neurologic exam, which he did admirably. That day I did not know the subject well, and we all learned because I freely admitted it.

Being a physician is the greatest job in the world. If you leverage your EQ (emotional quotient) as much as your IQ (intelligence quotient), your learners will conclude the same.

References

1. Houchens N, Harrod M, Moody S, Fowler KE, Saint S. Techniques and behaviors associated with exemplary inpatient general medicine teaching: an exploratory qualitative study. J Hosp Med. 2017;12(7):503-509. https://doi.org/10.12788/jhm.2763.

References

1. Houchens N, Harrod M, Moody S, Fowler KE, Saint S. Techniques and behaviors associated with exemplary inpatient general medicine teaching: an exploratory qualitative study. J Hosp Med. 2017;12(7):503-509. https://doi.org/10.12788/jhm.2763.

Issue
Journal of Hospital Medicine 15(2)
Issue
Journal of Hospital Medicine 15(2)
Page Number
94
Page Number
94
Publications
Publications
Topics
Article Type
Sections
Article Source

 

© 2020 Society of Hospital Medicine

Disallow All Ads
Correspondence Location
Reza Manesh, MD; E-mail: rsedigh1@jhmi.edu; Telephone: 412-708-6944
Content Gating
No Gating (article Unlocked/Free)
Alternative CME
Disqus Comments
Default
Use ProPublica
Hide sidebar & use full width
render the right sidebar.
Conference Recap Checkbox
Not Conference Recap
Clinical Edge
Display the Slideshow in this Article
Medscape Article
Display survey writer
Reuters content
Article PDF Media

The Right Frame

Article Type
Changed
Fri, 11/22/2019 - 12:48

A 65-year-old man was transferred to a tertiary academic medical center with one week of progressive shortness of breath, dry cough, and fevers. He reported no weight loss or night sweats but had experienced mild right upper quadrant pain and anorexia for the preceding three weeks. Several years had passed since he had consulted a physician, and he did not take any medications. He immigrated to the United States from Mexico four decades prior. He traveled back frequently to visit his family, most recently one month before his presentation. He worked as a farming supervisor in the Central Valley of California. He smoked tobacco and had a 30 pack-year history. He drank alcohol occasionally and denied any drug use.

Causes of subacute cough and dyspnea include bronchitis, pneumonia, heart failure, and asthma. Pneumonia and heart failure might cause right upper quadrant pain from diaphragmatic irritation and hepatic congestion, respectively. Metastatic cancer or infection may lead to synchronous pulmonary and hepatic involvement. The patient is at increased risk of lung cancer, given his extensive smoking history.

The patient’s place of residence in the Southwestern United States places him at risk of respiratory illness from coccidioidomycosis. His exact involvement with animals and their products should be further explored. For example, consumption of unpasteurized milk might result in pneumonia, hepatitis, or both from M. bovis, Brucella species, or C. burnetii. His travel to Mexico prompts consideration of tuberculosis, histoplasmosis, and paracoccidiomycosis as causes of respiratory and possible hepatic illness.

Two weeks prior, the patient had initially presented to another hospital with one week of intermittent right upper quadrant pain unrelated to eating. An abdominal ultrasound and hepatobiliary iminodiacetic acid (HIDA) scan were normal. Computed tomography (CT) of the chest, abdomen, and pelvis with contrast demonstrated a left upper lobe lung mass measuring 5.5 × 4.4 × 3.7 cm3 and scattered right-sided pulmonary nodules (Figure 1). He underwent CT-guided biopsy of the mass and was discharged with a presumed diagnosis of primary pulmonary malignancy with plans for outpatient follow-up.

jhm014040246_f1.jpg

Over the next four days, the patient developed progressive dyspnea with cough and subjective fevers. The patient was readmitted with a diagnosis of postobstructive pneumonia and acute kidney injury (creatinine increased from 0.7 mg/dL to 2.9 mg/dL between admissions), and this finding was attributed to contrast-induced nephropathy from his recent CT scan. He was treated with vancomycin and piperacillin/tazobactam for two days but wished to transfer to a tertiary care hospital for a second opinion.

 

 

Postobstructive pneumonia, pulmonary embolism, and pleural effusion are common causes of dyspnea in patients with lung cancer. The patient’s travel and occupational history, lung nodules, acute renal insufficiency, and rapidly progressive respiratory symptoms prompt consideration for radiographic mimickers of lung cancer. Tuberculosis might present as a lung mass (pulmonary tuberculoma) during primary infection or reactivation. Noninfectious causes of pulmonary masses and nodules include metastatic cancer (eg, colon cancer), sarcoidosis, IgG4-related disease, and granulomatous polyangiitis (GPA).

Contrast-induced nephropathy is unusual in patients with normal renal function. More probable explanations include hypovolemia or acute tubular necrosis (ATN) from underlying inflammation. The patient’s CT-negative right upper quadrant pain may be a distinct process or represent another facet of a disseminated illness such as hepatic infiltration from lymphoma.

Upon arrival, the patient’s temperature was 38°C, heart rate (HR) 107 beats per minute, blood pressure (BP) 159/89 mm Hg, respiratory rate 25 breaths per minute, and oxygen saturation 92% on 2 L of oxygen per minute. He showed no signs of distress. Mild scleral icterus was noted. The cardiac exam was normal. Auscultation revealed scattered wheezes and crackles in the left upper lobe. Mild right upper quadrant tenderness without hepatosplenomegaly was noted on the abdominal exam. The patient’s lower extremities exhibited bilateral trace edema. No rash was observed, and his neurologic exam was normal.

The white blood cell (WBC) count was 28,300 per cubic millimeter (87% neutrophils, 3.6% lymphocytes, and 0.03% eosinophils), hemoglobin 11.1 g per deciliter, and platelet count 789,000 per cubic millimeter. Sodium was 127 mmol per liter, potassium 4.6 mmol per liter, chloride 101 mmol per liter, bicarbonate 13 mmol per liter, blood urea nitrogen 60 mg per deciliter, and creatinine 3.4 mg per deciliter. Aspartate aminotransferase and alanine aminotransferase levels were normal. Alkaline phosphatase was 283 units per liter (normal range, 31-95), and total bilirubin was 4.5 mg per deciliter (normal range, 0.2­-1.3) with a direct bilirubin of 2.7 mg per deciliter. Urinalysis demonstrated urine protein of 30 mg/dL, specific gravity of 1.013, negative nitrites, 10­-21 white cells per high-powered field (normal, < 5), and 21­-50 red cells per high-powered field (normal, < 3). Urine microscopy revealed muddy brown casts but no cellular casts or dysmorphic red cells. A chest radiograph (CXR) showed patchy consolidations in the bilateral upper lobes and left lower lobe along with Kerley B lines, a small left pleural effusion, and thickened right horizontal fissure; the left upper lobe mass was re-demonstrated. Vancomycin, piperacillin-tazobactam, and azithromycin were administered.

At this point, the most likely source of sepsis is multifocal pneumonia. The patient is at risk for S. aureus and P. aeruginosa given his recent hospitalization. A severe form of leptospirosis (Weil’s disease) is associated with pulmonary disease, hyperbilirubinemia, and renal failure. Repeat abdominal imaging is necessary to evaluate for cholangitis given the patient’s right upper quadrant pain, fever, and jaundice. It would also help categorize his cholestatic pattern of liver injury as intrahepatic or extrahepatic (eg, stricture). An infiltrative disease such as sarcoidosis may cause both intrahepatic cholestasis and parenchymal lung disease, although the pleural pathology is uncommon.

 

 

His normal cardiac exam does not exclude cardiogenic pulmonary edema, a common cause of interstitial edema and pleural effusion. In this setting of systemic inflammation (neutrophilia, thrombocytosis, and hypoalbuminemia), the thickened right horizontal fissure and interlobular septa might represent an infiltrative process, such as lymphangitic carcinomatosis, lymphoma, or sarcoidosis.

Muddy brown casts are characteristic of ATN. The patient’s risk factors for ATN include sepsis and previously administered iodinated contrast. Fluid retention from oliguric renal failure is likely contributing to his hyponatremia and lower extremity edema. Pathology isolated to the tubules, however, would not cause hematuria and pyuria and suggests glomerular or interstitial disease. The lack of cellular casts on a single urinary specimen does not eliminate the likelihood of either disease. Hematuria and diffuse parenchymal lung disease prompt consideration of pulmonary-renal syndromes, such as anti-glomerular basement membrane disease, GPA, and systemic lupus erythematosus, which can all be triggered by infection.

On the night of transfer, the patient experienced acute respiratory distress. Heart rate was 130 beats per minute, BP 170/95 mm Hg, respiratory rate 40 breaths per minute, and oxygen saturation 88% on six liters of supplemental oxygen by nasal cannula. His arterial blood gas demonstrated a pH of 7.23, PaCO2 of 32 mm Hg, and PaO2 of 65 mm Hg. He was emergently intubated for progressive hypoxemic respiratory failure. A small amount of blood was noted in the endotracheal tube. A noncontrast CT of the chest demonstrated multifocal airspace opacities and bilateral pleural effusions. The previously noted left upper lobe mass was unchanged.

Rapid respiratory decline and diffuse alveolar disease commonly result from aspiration, flash pulmonary edema, and acute respiratory distress syndrome (ARDS). Necrotizing pneumonia (eg, S. aureus) and trauma during intubation are possible causes of blood in his endotracheal tube. However, in the setting of multifocal airspace opacity, renal insufficiency, hematuria, and rapid respiratory decline, the blood might represent diffuse alveolar hemorrhage (DAH). Bronchoscopy with bronchioalveolar lavage to evaluate for pulmonary edema, infection, and hemorrhage would be indicated.

The patient subsequently developed oliguria, requiring continuous renal replacement therapy. An echocardiogram demonstrated impaired left ventricular relaxation and a reduced ejection fraction of 45% without segmental wall motion abnormalities or valvular disease, and a right ventricular systolic pressure of 36 mm Hg. Over the next 12 hours, his respiratory status improved, and he was extubated to 15 L per minute of supplemental oxygen by high-flow nasal cannula (HFNC).

The pathology report of the lung biopsy from the other hospital disclosed chronic inflammation and fibrosis with ill-defined areas of necrosis and myxoid degeneration surrounded by nuclear palisading suggestive of granulomatous inflammation. Staining for acid-fast bacilli (AFB) and fungal organisms was negative.

The rapid pulmonary recovery is inconsistent with multifocal pneumonia or ARDS. Flash pulmonary edema might result in sudden hypoxemic respiratory failure that resolves with positive pressure ventilation and ultrafiltration. However, this condition would not explain the biopsy results. Granulomatous lung pathology often results from mycobacterial or fungal disease. Tuberculosis and fungal pneumonia are not excluded with negative staining alone. However, neither would cause self-limited respiratory failure. Histologic evidence of necrosis lessens the likelihood of sarcoidosis, which rarely causes fulminant pulmonary disease. Lymphoma can result in granulomatous inflammation but would not cause transient pulmonary disease. GPA, a cause of necrotizing granulomatous lung disease, might result in a lung mass and worsened hypoxemia through DAH.

The patient continued to require 15 L of oxygen per minute by HFNC. He had persistent bilateral perihilar alveolar and interstitial opacities on CXR. Repeat WBC count was 29,200 per cubic millimeter, hemoglobin 7.8 g per deciliter, and platelets 656,000 per cubic millimeter. The C-reactive protein was 300 mg per L (normal range, <6.3) and erythrocyte sedimentation rate 100 mm per hour (normal range, <10). Legionella urinary antigen, serum immunodiffusion for Coccidiodes imitus, human immunodeficiency virus antibody, respiratory viral panel, and beta-D glucan were negative. Rare acid-fast bacilli were visualized in one out of three concentrated AFB sputum smears. He was started on empiric antituberculous therapy with rifampin, isoniazid, pyrazinamide, and ethambutol.

The sputum sample is suggestive of pulmonary tuberculosis. The salient features of this case include systemic inflammation, pulmonary nodules and mass, necrotizing granulomatous lung pathology, renal insufficiency, and hematuria. Disseminated tuberculosis might explain all these findings. However, a positive AFB smear may signal the presence of a nontuberculous mycobacteria, which is less likely to cause this clinical syndrome.

M. tuberculosis complex polymerase chain reaction (MTB PCR) assay returned negative for M. tuberculosis. Antiproteinase 3 antibody was 1,930 units (normal range, <20). Antimyeloperoxidase and antiglomerular basement membrane antibodies were negative.

Tuberculosis and GPA share several overlapping features, such as necrotizing lung pathology and less commonly antineutrophil cytoplasmic autoantibody (ANCA)-associated antibodies. However, the lung mass, acute renal and respiratory failure, hematuria, and the degree of anti-proteinase 3 level elevation are highly suggestive of GPA. The negative MTB PCR raises the possibility that a nontuberculous mycobacterium was detected on the sputum smear. Nevertheless, continued treatment until finalization of culture results is appropriate given that tuberculosis is endemic in Mexico.

 

 

The patient’s presenting features of right upper quadrant tenderness, jaundice, and cholestatic hepatitis remain poorly explained by either of these diagnoses.  Neither tuberculosis nor GPA commonly presents with accompanying hepatic involvement, though both have been occasionally described as causing hepatitis. As the greatest concern in this patient remains his progressive renal failure and accompanying pulmonary hemorrhage, a renal biopsy to assess for glomerulonephritis associated with GPA is warranted before further investigation into the cause of his cholestatic hepatitis.

A core renal biopsy demonstrated pauci-immune focal crescentic and necrotizing glomerulonephritis with mixed tubulointerstitial inflammation (Figure 2). In conjunction with the pulmonary syndrome and positive antiproteinase 3 serology, a diagnosis of granulomatosis with polyangiitis was made. The patient was treated with pulse dose steroids, rituximab, and plasma exchange. Two weeks later, the sputum mycobacterial culture returned positive for Mycobacterium llatzerense and anti-tuberculous treatment was discontinued.

jhm014040246_f2.jpg

Over the following weeks, the patient improved and was transitioned off dialysis prior to hospital discharge. By six months later, he had resolution of his hemoptysis, shortness of breath, liver biochemical test abnormalities, and significant improvement in his renal function. Repeat sputum mycobacterial cultures were negative.

DISCUSSION

A 65-year-old man from Mexico with a significant smoking history presented with an apical lung mass and cough, prioritizing tuberculosis and pulmonary malignancy. As the case unfolded, renal failure, multifocal lung opacities, conflicting tuberculosis test results, positive anti-proteinase 3 antibody, and ultimately a renal biopsy led to the diagnosis of granulomatosis with polyangiitis (GPA).

The correct interpretation of occasionally conflicting mycobacterial testing is crucial. Mycobacterial cultures remain the gold standard for diagnosing tuberculosis. However, results take weeks to return. Rapid tests include acid-fast bacilli (AFB) smear microscopy and nucleic acid-amplification tests (NAAT) of sputum or bronchoalveolar samples.1 When three sputum smears are performed, the sensitivity of AFB smear microscopy for tuberculosis in immunocompetent hosts is 70%.1 The AFB smear does not distinguish between different mycobacterial organisms. Thus, a positive result must be interpreted with the relative prevalence of tuberculosis and nontuberculous mycobacteria (NTM) in mind. The addition of NAAT-based assays has allowed for enhanced sensitivity and specificity in the diagnosis of tuberculosis, such that a negative NAAT in a patient with a positive AFB smear strongly argues for the presence of a NTM.2-4

NTM are widely prevalent environmental microbes, with over 140 species described, and careful consideration is required to determine if an isolate is pathogenic.5 Given their ubiquitous nature, a high rate of asymptomatic respiratory and cutaneous colonization occurs. Correspondingly, the diagnosis of NTM disease requires multiple positive cultures or pathologic features on tissue biopsy, compatible clinical findings, and diligent exclusion of other causes.5 A retrospective study of all NTM isolates in Oregon from 2005­-2006 revealed that only 47% of patients met the guideline criteria for having symptomatic NTM disease.6 In our case, the patient’s sputum grew M. llatzerense, an aerobic, nonfermenting mycobacterium found in water sources that has only infrequently been implicated as a human pathogen.7,8 Subsequent AFB sputum cultures were negative, and serial imaging showed resolution of the pulmonary findings without additional antimycobacterial therapy, suggesting that this organism was not responsible for the disease process.

Along with microscopic polyangiitis (MPA) and eosinophilic granulomatosis with polyangiitis (EGPA), GPA is an antineutrophil cytoplasmic autoantibody (ANCA)-associated vasculitis that predominantly affects small to medium sized vessels. Although it can occur at any age, GPA most commonly afflicts older adults, with men and women being diagnosed at roughly equal rates.9 GPA is a multisystem disease with a wide array of clinical manifestations. The most frequently involved sites of disease are the respiratory tract and kidneys, although virtually any organ can be affected. Sino-nasal disease, such as destructive sinusitis, or ear involvement are nearly universal. Lower respiratory manifestations occur in 60% of patients, but are highly diverse and reflect the inherent difficulty in diagnosing this condition.9-11 Additionally, GPA is a frequent cause of the pulmonary-renal syndromes, with glomerulonephritis occurring in 80% of patients.9

The diagnosis of GPA in this case was delayed, in part, due to features suggestive of malignancy and pulmonary tuberculosis. While sino-nasal disease was not noted during this hospitalization, the patient had many different respiratory manifestations, including a dominant pulmonary mass, diffuse nodules, and hypoxemic respiratory failure due to suspected diffuse alveolar hemorrhage (DAH), all of which have been reported in GPA.12 Dysmorphic red cells and red blood cell casts are not sensitive for renal involvement in GPA; their absence does not exclude the possibility of an ANCA-associated vasculitis.13 Hematuria and rapid progression to oliguric renal failure are characteristic of a vasculitic process and should sway clinicians away from a working diagnosis of ATN.

The diagnosis of GPA involves the synthesis of clinical data, radiographic findings, serologic testing, and histopathology. ANCA testing is an essential step in the diagnosis of GPA but has limitations. Patients with GPA more commonly have ANCAs targeting the enzyme proteinase-3 (PR3-ANCA), with MPA being more closely associated with myeloperoxidase (MPO-ANCA), although cross-reactivity and antibody-negative disease can occur.14 Although 90% of patients with GPA with multiorgan involvement will have a positive ANCA, a negative test is more common in localized upper airway disease, where only 50% have a positive ANCA.15 A number of drugs, medications, infections, and nonvasculitic autoimmune diseases have been associated with positive ANCA serologies in the absence of systemic vasculitis.14,16,17 As such, pathologic demonstration of vasculitis is necessary for establishing the diagnosis. Typical sites for biopsy include the kidneys and lungs.9

This case illustrates how clinicians often find themselves at a diagnostic crossroads—being forced to choose which clinical elements to prioritize. At various points, our patient’s presentation could have been framed as “a man from a Tb-endemic country with hemoptysis and an apical opacity,” “an elderly man with extensive smoking history and lung mass,” or “a patient with elevated inflammatory markers and pulmonary-renal syndrome”. In such situations, it is incumbent on the clinician to evaluate how well a given problem representation encompasses or highlights the salient features of a case. As with painting or photography, an essential aspect of appreciating the whole picture involves carefully selecting the right frame.

 

 

KEY TEACHING POINTS

  • The diagnosis of tuberculosis relies on smear microscopy, nucleic-acid amplification testing (NAAT), and cultures. A positive AFB smear with negative NAAT suggests the presence of a nontuberculous mycobacteria (NTM).
  • NTM are common environmental organisms and often exist as nonpathogenic colonizers.6 The diagnosis of NTM disease requires exclusion of other causes and careful clinical, microbiologic, and radiographic correlation.
  • Granulomatosis with polyangiitis is a multisystem disease often involving the respiratory track and kidney. Pulmonary disease can present with airway involvement, parenchymal nodules, opacities, pleural findings, and diffuse alveolar hemorrhage.12

Disclosures

Drs. Minter, Geha, Boslett, Chung, and Ramani have no disclosures. Dr. Manesh is supported by the Jeremiah A. Barondess Fellowship in the Clinical Transaction of the New York Academy of Medicine, in collaboration with the Accreditation Council for Graduate Medical Education (ACGME).

 

References

1. Lewinsohn DM, Leonard MK, LoBue PA, et al. Official American Thoracic Society/Infectious Diseases Society of America/Centers for Disease Control and Prevention clinical practice guidelines: diagnosis of tuberculosis in adults and children. Clin Infect Dis. 2017;64(2):e1-e33. PubMed
2. Steingart KR, Sohn H, Schiller I, et al. Xpert(R) MTB/RIF assay for pulmonary tuberculosis and rifampicin resistance in adults. Cochrane Database Syst Rev. 2013;(1):Cd009593. PubMed
3. Luetkemeyer AF, Firnhaber C, Kendall MA, et al. Evaluation of Xpert MTB/RIF versus afb smear and culture to identify pulmonary tuberculosis in patients with suspected tuberculosis from low and higher prevalence settings. Clin Infect Dis. 2016;62(9):1081-1088. PubMed
4. Boehme CC, Nabeta P, Hillemann D, et al. Rapid molecular detection of tuberculosis and rifampin resistance. N Engl J Med. 2010;363(11):1005-1015. PubMed
5. Griffith DE, Aksamit T, Brown-Elliott BA, et al. An official ATS/IDSA statement: diagnosis, treatment, and prevention of nontuberculous mycobacterial diseases. Am J Respir Crit Care Med. 2007;175(4):367-416. PubMed
6. Winthrop KL, McNelley E, Kendall B, et al. Pulmonary nontuberculous mycobacterial disease prevalence and clinical features: an emerging public health disease. Am J Respir Crit Care Med. 2010;182(7):977-982. PubMed
7. Teixeira L, Avery RK, Iseman M, et al. Mycobacterium llatzerense lung infection in a liver transplant recipient: case report and review of the literature. Am J Transplant. 2013;13(8):2198-2200. PubMed
8. Cárdenas AM, Gomila M, Lalucat J, Edelstein PH. Abdominal abscess caused by Mycobacterium llatzerense. J Clin Microbiol. 2014;52(4):1287-1289. PubMed
9. Jennette JC, Falk RJ. Small-vessel vasculitis. N Engl J Med. 1997;337(21):1512-1523. PubMed
10. Mahr A, Katsahian S, Varet H, et al. Revisiting the classification of clinical phenotypes of anti-neutrophil cytoplasmic antibody-associated vasculitis: a cluster analysis. Ann Rheum Dis. 2013;72(6):1003-1010. PubMed
11. Holle JU, Gross WL, Latza U, et al. Improved outcome in 445 patients with Wegener’s granulomatosis in a German vasculitis center over four decades. Arthritis Rheum. 2011;63(1):257-266. PubMed
12. Cordier JF, Valeyre D, Guillevin L, Loire R, Brechot JM. Pulmonary Wegener’s granulomatosis. A clinical and imaging study of 77 cases. Chest. 1990;97(4):906-912. PubMed
13. Hamadah AM, Gharaibeh K, Mara KC, et al. Urinalysis for the diagnosis of glomerulonephritis: role of dysmorphic red blood cells. Nephrol Dial Transplant. 2018;33(8):1397-1403. PubMed
14. Jennette JC, Falk RJ. Pathogenesis of antineutrophil cytoplasmic autoantibody-mediated disease. Nat Rev Rheumatol. 2014;10(8):463-473. PubMed
15. Borner U, Landis BN, Banz Y, et al. Diagnostic value of biopsies in identifying cytoplasmic antineutrophil cytoplasmic antibody-negative localized Wegener’s granulomatosis presenting primarily with sinonasal disease. Am J Rhinol Allergy. 2012;26(6):475-480. PubMed
16. Mahr A, Batteux F, Tubiana S, et al. Brief report: prevalence of antineutrophil cytoplasmic antibodies in infective endocarditis. Arthritis Rheumatol. 2014;66(6):1672-1677. PubMed
17. Sherkat R, Mostafavizadeh K, Zeydabadi L, Shoaei P, Rostami S. Antineutrophil cytoplasmic antibodies in patients with pulmonary tuberculosis. Iran J Immunol. 2011;8(1):52-57. PubMed

Article PDF
Issue
Journal of Hospital Medicine 14(4)
Publications
Topics
Page Number
246-250
Sections
Article PDF
Article PDF

A 65-year-old man was transferred to a tertiary academic medical center with one week of progressive shortness of breath, dry cough, and fevers. He reported no weight loss or night sweats but had experienced mild right upper quadrant pain and anorexia for the preceding three weeks. Several years had passed since he had consulted a physician, and he did not take any medications. He immigrated to the United States from Mexico four decades prior. He traveled back frequently to visit his family, most recently one month before his presentation. He worked as a farming supervisor in the Central Valley of California. He smoked tobacco and had a 30 pack-year history. He drank alcohol occasionally and denied any drug use.

Causes of subacute cough and dyspnea include bronchitis, pneumonia, heart failure, and asthma. Pneumonia and heart failure might cause right upper quadrant pain from diaphragmatic irritation and hepatic congestion, respectively. Metastatic cancer or infection may lead to synchronous pulmonary and hepatic involvement. The patient is at increased risk of lung cancer, given his extensive smoking history.

The patient’s place of residence in the Southwestern United States places him at risk of respiratory illness from coccidioidomycosis. His exact involvement with animals and their products should be further explored. For example, consumption of unpasteurized milk might result in pneumonia, hepatitis, or both from M. bovis, Brucella species, or C. burnetii. His travel to Mexico prompts consideration of tuberculosis, histoplasmosis, and paracoccidiomycosis as causes of respiratory and possible hepatic illness.

Two weeks prior, the patient had initially presented to another hospital with one week of intermittent right upper quadrant pain unrelated to eating. An abdominal ultrasound and hepatobiliary iminodiacetic acid (HIDA) scan were normal. Computed tomography (CT) of the chest, abdomen, and pelvis with contrast demonstrated a left upper lobe lung mass measuring 5.5 × 4.4 × 3.7 cm3 and scattered right-sided pulmonary nodules (Figure 1). He underwent CT-guided biopsy of the mass and was discharged with a presumed diagnosis of primary pulmonary malignancy with plans for outpatient follow-up.

jhm014040246_f1.jpg

Over the next four days, the patient developed progressive dyspnea with cough and subjective fevers. The patient was readmitted with a diagnosis of postobstructive pneumonia and acute kidney injury (creatinine increased from 0.7 mg/dL to 2.9 mg/dL between admissions), and this finding was attributed to contrast-induced nephropathy from his recent CT scan. He was treated with vancomycin and piperacillin/tazobactam for two days but wished to transfer to a tertiary care hospital for a second opinion.

 

 

Postobstructive pneumonia, pulmonary embolism, and pleural effusion are common causes of dyspnea in patients with lung cancer. The patient’s travel and occupational history, lung nodules, acute renal insufficiency, and rapidly progressive respiratory symptoms prompt consideration for radiographic mimickers of lung cancer. Tuberculosis might present as a lung mass (pulmonary tuberculoma) during primary infection or reactivation. Noninfectious causes of pulmonary masses and nodules include metastatic cancer (eg, colon cancer), sarcoidosis, IgG4-related disease, and granulomatous polyangiitis (GPA).

Contrast-induced nephropathy is unusual in patients with normal renal function. More probable explanations include hypovolemia or acute tubular necrosis (ATN) from underlying inflammation. The patient’s CT-negative right upper quadrant pain may be a distinct process or represent another facet of a disseminated illness such as hepatic infiltration from lymphoma.

Upon arrival, the patient’s temperature was 38°C, heart rate (HR) 107 beats per minute, blood pressure (BP) 159/89 mm Hg, respiratory rate 25 breaths per minute, and oxygen saturation 92% on 2 L of oxygen per minute. He showed no signs of distress. Mild scleral icterus was noted. The cardiac exam was normal. Auscultation revealed scattered wheezes and crackles in the left upper lobe. Mild right upper quadrant tenderness without hepatosplenomegaly was noted on the abdominal exam. The patient’s lower extremities exhibited bilateral trace edema. No rash was observed, and his neurologic exam was normal.

The white blood cell (WBC) count was 28,300 per cubic millimeter (87% neutrophils, 3.6% lymphocytes, and 0.03% eosinophils), hemoglobin 11.1 g per deciliter, and platelet count 789,000 per cubic millimeter. Sodium was 127 mmol per liter, potassium 4.6 mmol per liter, chloride 101 mmol per liter, bicarbonate 13 mmol per liter, blood urea nitrogen 60 mg per deciliter, and creatinine 3.4 mg per deciliter. Aspartate aminotransferase and alanine aminotransferase levels were normal. Alkaline phosphatase was 283 units per liter (normal range, 31-95), and total bilirubin was 4.5 mg per deciliter (normal range, 0.2­-1.3) with a direct bilirubin of 2.7 mg per deciliter. Urinalysis demonstrated urine protein of 30 mg/dL, specific gravity of 1.013, negative nitrites, 10­-21 white cells per high-powered field (normal, < 5), and 21­-50 red cells per high-powered field (normal, < 3). Urine microscopy revealed muddy brown casts but no cellular casts or dysmorphic red cells. A chest radiograph (CXR) showed patchy consolidations in the bilateral upper lobes and left lower lobe along with Kerley B lines, a small left pleural effusion, and thickened right horizontal fissure; the left upper lobe mass was re-demonstrated. Vancomycin, piperacillin-tazobactam, and azithromycin were administered.

At this point, the most likely source of sepsis is multifocal pneumonia. The patient is at risk for S. aureus and P. aeruginosa given his recent hospitalization. A severe form of leptospirosis (Weil’s disease) is associated with pulmonary disease, hyperbilirubinemia, and renal failure. Repeat abdominal imaging is necessary to evaluate for cholangitis given the patient’s right upper quadrant pain, fever, and jaundice. It would also help categorize his cholestatic pattern of liver injury as intrahepatic or extrahepatic (eg, stricture). An infiltrative disease such as sarcoidosis may cause both intrahepatic cholestasis and parenchymal lung disease, although the pleural pathology is uncommon.

 

 

His normal cardiac exam does not exclude cardiogenic pulmonary edema, a common cause of interstitial edema and pleural effusion. In this setting of systemic inflammation (neutrophilia, thrombocytosis, and hypoalbuminemia), the thickened right horizontal fissure and interlobular septa might represent an infiltrative process, such as lymphangitic carcinomatosis, lymphoma, or sarcoidosis.

Muddy brown casts are characteristic of ATN. The patient’s risk factors for ATN include sepsis and previously administered iodinated contrast. Fluid retention from oliguric renal failure is likely contributing to his hyponatremia and lower extremity edema. Pathology isolated to the tubules, however, would not cause hematuria and pyuria and suggests glomerular or interstitial disease. The lack of cellular casts on a single urinary specimen does not eliminate the likelihood of either disease. Hematuria and diffuse parenchymal lung disease prompt consideration of pulmonary-renal syndromes, such as anti-glomerular basement membrane disease, GPA, and systemic lupus erythematosus, which can all be triggered by infection.

On the night of transfer, the patient experienced acute respiratory distress. Heart rate was 130 beats per minute, BP 170/95 mm Hg, respiratory rate 40 breaths per minute, and oxygen saturation 88% on six liters of supplemental oxygen by nasal cannula. His arterial blood gas demonstrated a pH of 7.23, PaCO2 of 32 mm Hg, and PaO2 of 65 mm Hg. He was emergently intubated for progressive hypoxemic respiratory failure. A small amount of blood was noted in the endotracheal tube. A noncontrast CT of the chest demonstrated multifocal airspace opacities and bilateral pleural effusions. The previously noted left upper lobe mass was unchanged.

Rapid respiratory decline and diffuse alveolar disease commonly result from aspiration, flash pulmonary edema, and acute respiratory distress syndrome (ARDS). Necrotizing pneumonia (eg, S. aureus) and trauma during intubation are possible causes of blood in his endotracheal tube. However, in the setting of multifocal airspace opacity, renal insufficiency, hematuria, and rapid respiratory decline, the blood might represent diffuse alveolar hemorrhage (DAH). Bronchoscopy with bronchioalveolar lavage to evaluate for pulmonary edema, infection, and hemorrhage would be indicated.

The patient subsequently developed oliguria, requiring continuous renal replacement therapy. An echocardiogram demonstrated impaired left ventricular relaxation and a reduced ejection fraction of 45% without segmental wall motion abnormalities or valvular disease, and a right ventricular systolic pressure of 36 mm Hg. Over the next 12 hours, his respiratory status improved, and he was extubated to 15 L per minute of supplemental oxygen by high-flow nasal cannula (HFNC).

The pathology report of the lung biopsy from the other hospital disclosed chronic inflammation and fibrosis with ill-defined areas of necrosis and myxoid degeneration surrounded by nuclear palisading suggestive of granulomatous inflammation. Staining for acid-fast bacilli (AFB) and fungal organisms was negative.

The rapid pulmonary recovery is inconsistent with multifocal pneumonia or ARDS. Flash pulmonary edema might result in sudden hypoxemic respiratory failure that resolves with positive pressure ventilation and ultrafiltration. However, this condition would not explain the biopsy results. Granulomatous lung pathology often results from mycobacterial or fungal disease. Tuberculosis and fungal pneumonia are not excluded with negative staining alone. However, neither would cause self-limited respiratory failure. Histologic evidence of necrosis lessens the likelihood of sarcoidosis, which rarely causes fulminant pulmonary disease. Lymphoma can result in granulomatous inflammation but would not cause transient pulmonary disease. GPA, a cause of necrotizing granulomatous lung disease, might result in a lung mass and worsened hypoxemia through DAH.

The patient continued to require 15 L of oxygen per minute by HFNC. He had persistent bilateral perihilar alveolar and interstitial opacities on CXR. Repeat WBC count was 29,200 per cubic millimeter, hemoglobin 7.8 g per deciliter, and platelets 656,000 per cubic millimeter. The C-reactive protein was 300 mg per L (normal range, <6.3) and erythrocyte sedimentation rate 100 mm per hour (normal range, <10). Legionella urinary antigen, serum immunodiffusion for Coccidiodes imitus, human immunodeficiency virus antibody, respiratory viral panel, and beta-D glucan were negative. Rare acid-fast bacilli were visualized in one out of three concentrated AFB sputum smears. He was started on empiric antituberculous therapy with rifampin, isoniazid, pyrazinamide, and ethambutol.

The sputum sample is suggestive of pulmonary tuberculosis. The salient features of this case include systemic inflammation, pulmonary nodules and mass, necrotizing granulomatous lung pathology, renal insufficiency, and hematuria. Disseminated tuberculosis might explain all these findings. However, a positive AFB smear may signal the presence of a nontuberculous mycobacteria, which is less likely to cause this clinical syndrome.

M. tuberculosis complex polymerase chain reaction (MTB PCR) assay returned negative for M. tuberculosis. Antiproteinase 3 antibody was 1,930 units (normal range, <20). Antimyeloperoxidase and antiglomerular basement membrane antibodies were negative.

Tuberculosis and GPA share several overlapping features, such as necrotizing lung pathology and less commonly antineutrophil cytoplasmic autoantibody (ANCA)-associated antibodies. However, the lung mass, acute renal and respiratory failure, hematuria, and the degree of anti-proteinase 3 level elevation are highly suggestive of GPA. The negative MTB PCR raises the possibility that a nontuberculous mycobacterium was detected on the sputum smear. Nevertheless, continued treatment until finalization of culture results is appropriate given that tuberculosis is endemic in Mexico.

 

 

The patient’s presenting features of right upper quadrant tenderness, jaundice, and cholestatic hepatitis remain poorly explained by either of these diagnoses.  Neither tuberculosis nor GPA commonly presents with accompanying hepatic involvement, though both have been occasionally described as causing hepatitis. As the greatest concern in this patient remains his progressive renal failure and accompanying pulmonary hemorrhage, a renal biopsy to assess for glomerulonephritis associated with GPA is warranted before further investigation into the cause of his cholestatic hepatitis.

A core renal biopsy demonstrated pauci-immune focal crescentic and necrotizing glomerulonephritis with mixed tubulointerstitial inflammation (Figure 2). In conjunction with the pulmonary syndrome and positive antiproteinase 3 serology, a diagnosis of granulomatosis with polyangiitis was made. The patient was treated with pulse dose steroids, rituximab, and plasma exchange. Two weeks later, the sputum mycobacterial culture returned positive for Mycobacterium llatzerense and anti-tuberculous treatment was discontinued.

jhm014040246_f2.jpg

Over the following weeks, the patient improved and was transitioned off dialysis prior to hospital discharge. By six months later, he had resolution of his hemoptysis, shortness of breath, liver biochemical test abnormalities, and significant improvement in his renal function. Repeat sputum mycobacterial cultures were negative.

DISCUSSION

A 65-year-old man from Mexico with a significant smoking history presented with an apical lung mass and cough, prioritizing tuberculosis and pulmonary malignancy. As the case unfolded, renal failure, multifocal lung opacities, conflicting tuberculosis test results, positive anti-proteinase 3 antibody, and ultimately a renal biopsy led to the diagnosis of granulomatosis with polyangiitis (GPA).

The correct interpretation of occasionally conflicting mycobacterial testing is crucial. Mycobacterial cultures remain the gold standard for diagnosing tuberculosis. However, results take weeks to return. Rapid tests include acid-fast bacilli (AFB) smear microscopy and nucleic acid-amplification tests (NAAT) of sputum or bronchoalveolar samples.1 When three sputum smears are performed, the sensitivity of AFB smear microscopy for tuberculosis in immunocompetent hosts is 70%.1 The AFB smear does not distinguish between different mycobacterial organisms. Thus, a positive result must be interpreted with the relative prevalence of tuberculosis and nontuberculous mycobacteria (NTM) in mind. The addition of NAAT-based assays has allowed for enhanced sensitivity and specificity in the diagnosis of tuberculosis, such that a negative NAAT in a patient with a positive AFB smear strongly argues for the presence of a NTM.2-4

NTM are widely prevalent environmental microbes, with over 140 species described, and careful consideration is required to determine if an isolate is pathogenic.5 Given their ubiquitous nature, a high rate of asymptomatic respiratory and cutaneous colonization occurs. Correspondingly, the diagnosis of NTM disease requires multiple positive cultures or pathologic features on tissue biopsy, compatible clinical findings, and diligent exclusion of other causes.5 A retrospective study of all NTM isolates in Oregon from 2005­-2006 revealed that only 47% of patients met the guideline criteria for having symptomatic NTM disease.6 In our case, the patient’s sputum grew M. llatzerense, an aerobic, nonfermenting mycobacterium found in water sources that has only infrequently been implicated as a human pathogen.7,8 Subsequent AFB sputum cultures were negative, and serial imaging showed resolution of the pulmonary findings without additional antimycobacterial therapy, suggesting that this organism was not responsible for the disease process.

Along with microscopic polyangiitis (MPA) and eosinophilic granulomatosis with polyangiitis (EGPA), GPA is an antineutrophil cytoplasmic autoantibody (ANCA)-associated vasculitis that predominantly affects small to medium sized vessels. Although it can occur at any age, GPA most commonly afflicts older adults, with men and women being diagnosed at roughly equal rates.9 GPA is a multisystem disease with a wide array of clinical manifestations. The most frequently involved sites of disease are the respiratory tract and kidneys, although virtually any organ can be affected. Sino-nasal disease, such as destructive sinusitis, or ear involvement are nearly universal. Lower respiratory manifestations occur in 60% of patients, but are highly diverse and reflect the inherent difficulty in diagnosing this condition.9-11 Additionally, GPA is a frequent cause of the pulmonary-renal syndromes, with glomerulonephritis occurring in 80% of patients.9

The diagnosis of GPA in this case was delayed, in part, due to features suggestive of malignancy and pulmonary tuberculosis. While sino-nasal disease was not noted during this hospitalization, the patient had many different respiratory manifestations, including a dominant pulmonary mass, diffuse nodules, and hypoxemic respiratory failure due to suspected diffuse alveolar hemorrhage (DAH), all of which have been reported in GPA.12 Dysmorphic red cells and red blood cell casts are not sensitive for renal involvement in GPA; their absence does not exclude the possibility of an ANCA-associated vasculitis.13 Hematuria and rapid progression to oliguric renal failure are characteristic of a vasculitic process and should sway clinicians away from a working diagnosis of ATN.

The diagnosis of GPA involves the synthesis of clinical data, radiographic findings, serologic testing, and histopathology. ANCA testing is an essential step in the diagnosis of GPA but has limitations. Patients with GPA more commonly have ANCAs targeting the enzyme proteinase-3 (PR3-ANCA), with MPA being more closely associated with myeloperoxidase (MPO-ANCA), although cross-reactivity and antibody-negative disease can occur.14 Although 90% of patients with GPA with multiorgan involvement will have a positive ANCA, a negative test is more common in localized upper airway disease, where only 50% have a positive ANCA.15 A number of drugs, medications, infections, and nonvasculitic autoimmune diseases have been associated with positive ANCA serologies in the absence of systemic vasculitis.14,16,17 As such, pathologic demonstration of vasculitis is necessary for establishing the diagnosis. Typical sites for biopsy include the kidneys and lungs.9

This case illustrates how clinicians often find themselves at a diagnostic crossroads—being forced to choose which clinical elements to prioritize. At various points, our patient’s presentation could have been framed as “a man from a Tb-endemic country with hemoptysis and an apical opacity,” “an elderly man with extensive smoking history and lung mass,” or “a patient with elevated inflammatory markers and pulmonary-renal syndrome”. In such situations, it is incumbent on the clinician to evaluate how well a given problem representation encompasses or highlights the salient features of a case. As with painting or photography, an essential aspect of appreciating the whole picture involves carefully selecting the right frame.

 

 

KEY TEACHING POINTS

  • The diagnosis of tuberculosis relies on smear microscopy, nucleic-acid amplification testing (NAAT), and cultures. A positive AFB smear with negative NAAT suggests the presence of a nontuberculous mycobacteria (NTM).
  • NTM are common environmental organisms and often exist as nonpathogenic colonizers.6 The diagnosis of NTM disease requires exclusion of other causes and careful clinical, microbiologic, and radiographic correlation.
  • Granulomatosis with polyangiitis is a multisystem disease often involving the respiratory track and kidney. Pulmonary disease can present with airway involvement, parenchymal nodules, opacities, pleural findings, and diffuse alveolar hemorrhage.12

Disclosures

Drs. Minter, Geha, Boslett, Chung, and Ramani have no disclosures. Dr. Manesh is supported by the Jeremiah A. Barondess Fellowship in the Clinical Transaction of the New York Academy of Medicine, in collaboration with the Accreditation Council for Graduate Medical Education (ACGME).

 

A 65-year-old man was transferred to a tertiary academic medical center with one week of progressive shortness of breath, dry cough, and fevers. He reported no weight loss or night sweats but had experienced mild right upper quadrant pain and anorexia for the preceding three weeks. Several years had passed since he had consulted a physician, and he did not take any medications. He immigrated to the United States from Mexico four decades prior. He traveled back frequently to visit his family, most recently one month before his presentation. He worked as a farming supervisor in the Central Valley of California. He smoked tobacco and had a 30 pack-year history. He drank alcohol occasionally and denied any drug use.

Causes of subacute cough and dyspnea include bronchitis, pneumonia, heart failure, and asthma. Pneumonia and heart failure might cause right upper quadrant pain from diaphragmatic irritation and hepatic congestion, respectively. Metastatic cancer or infection may lead to synchronous pulmonary and hepatic involvement. The patient is at increased risk of lung cancer, given his extensive smoking history.

The patient’s place of residence in the Southwestern United States places him at risk of respiratory illness from coccidioidomycosis. His exact involvement with animals and their products should be further explored. For example, consumption of unpasteurized milk might result in pneumonia, hepatitis, or both from M. bovis, Brucella species, or C. burnetii. His travel to Mexico prompts consideration of tuberculosis, histoplasmosis, and paracoccidiomycosis as causes of respiratory and possible hepatic illness.

Two weeks prior, the patient had initially presented to another hospital with one week of intermittent right upper quadrant pain unrelated to eating. An abdominal ultrasound and hepatobiliary iminodiacetic acid (HIDA) scan were normal. Computed tomography (CT) of the chest, abdomen, and pelvis with contrast demonstrated a left upper lobe lung mass measuring 5.5 × 4.4 × 3.7 cm3 and scattered right-sided pulmonary nodules (Figure 1). He underwent CT-guided biopsy of the mass and was discharged with a presumed diagnosis of primary pulmonary malignancy with plans for outpatient follow-up.

jhm014040246_f1.jpg

Over the next four days, the patient developed progressive dyspnea with cough and subjective fevers. The patient was readmitted with a diagnosis of postobstructive pneumonia and acute kidney injury (creatinine increased from 0.7 mg/dL to 2.9 mg/dL between admissions), and this finding was attributed to contrast-induced nephropathy from his recent CT scan. He was treated with vancomycin and piperacillin/tazobactam for two days but wished to transfer to a tertiary care hospital for a second opinion.

 

 

Postobstructive pneumonia, pulmonary embolism, and pleural effusion are common causes of dyspnea in patients with lung cancer. The patient’s travel and occupational history, lung nodules, acute renal insufficiency, and rapidly progressive respiratory symptoms prompt consideration for radiographic mimickers of lung cancer. Tuberculosis might present as a lung mass (pulmonary tuberculoma) during primary infection or reactivation. Noninfectious causes of pulmonary masses and nodules include metastatic cancer (eg, colon cancer), sarcoidosis, IgG4-related disease, and granulomatous polyangiitis (GPA).

Contrast-induced nephropathy is unusual in patients with normal renal function. More probable explanations include hypovolemia or acute tubular necrosis (ATN) from underlying inflammation. The patient’s CT-negative right upper quadrant pain may be a distinct process or represent another facet of a disseminated illness such as hepatic infiltration from lymphoma.

Upon arrival, the patient’s temperature was 38°C, heart rate (HR) 107 beats per minute, blood pressure (BP) 159/89 mm Hg, respiratory rate 25 breaths per minute, and oxygen saturation 92% on 2 L of oxygen per minute. He showed no signs of distress. Mild scleral icterus was noted. The cardiac exam was normal. Auscultation revealed scattered wheezes and crackles in the left upper lobe. Mild right upper quadrant tenderness without hepatosplenomegaly was noted on the abdominal exam. The patient’s lower extremities exhibited bilateral trace edema. No rash was observed, and his neurologic exam was normal.

The white blood cell (WBC) count was 28,300 per cubic millimeter (87% neutrophils, 3.6% lymphocytes, and 0.03% eosinophils), hemoglobin 11.1 g per deciliter, and platelet count 789,000 per cubic millimeter. Sodium was 127 mmol per liter, potassium 4.6 mmol per liter, chloride 101 mmol per liter, bicarbonate 13 mmol per liter, blood urea nitrogen 60 mg per deciliter, and creatinine 3.4 mg per deciliter. Aspartate aminotransferase and alanine aminotransferase levels were normal. Alkaline phosphatase was 283 units per liter (normal range, 31-95), and total bilirubin was 4.5 mg per deciliter (normal range, 0.2­-1.3) with a direct bilirubin of 2.7 mg per deciliter. Urinalysis demonstrated urine protein of 30 mg/dL, specific gravity of 1.013, negative nitrites, 10­-21 white cells per high-powered field (normal, < 5), and 21­-50 red cells per high-powered field (normal, < 3). Urine microscopy revealed muddy brown casts but no cellular casts or dysmorphic red cells. A chest radiograph (CXR) showed patchy consolidations in the bilateral upper lobes and left lower lobe along with Kerley B lines, a small left pleural effusion, and thickened right horizontal fissure; the left upper lobe mass was re-demonstrated. Vancomycin, piperacillin-tazobactam, and azithromycin were administered.

At this point, the most likely source of sepsis is multifocal pneumonia. The patient is at risk for S. aureus and P. aeruginosa given his recent hospitalization. A severe form of leptospirosis (Weil’s disease) is associated with pulmonary disease, hyperbilirubinemia, and renal failure. Repeat abdominal imaging is necessary to evaluate for cholangitis given the patient’s right upper quadrant pain, fever, and jaundice. It would also help categorize his cholestatic pattern of liver injury as intrahepatic or extrahepatic (eg, stricture). An infiltrative disease such as sarcoidosis may cause both intrahepatic cholestasis and parenchymal lung disease, although the pleural pathology is uncommon.

 

 

His normal cardiac exam does not exclude cardiogenic pulmonary edema, a common cause of interstitial edema and pleural effusion. In this setting of systemic inflammation (neutrophilia, thrombocytosis, and hypoalbuminemia), the thickened right horizontal fissure and interlobular septa might represent an infiltrative process, such as lymphangitic carcinomatosis, lymphoma, or sarcoidosis.

Muddy brown casts are characteristic of ATN. The patient’s risk factors for ATN include sepsis and previously administered iodinated contrast. Fluid retention from oliguric renal failure is likely contributing to his hyponatremia and lower extremity edema. Pathology isolated to the tubules, however, would not cause hematuria and pyuria and suggests glomerular or interstitial disease. The lack of cellular casts on a single urinary specimen does not eliminate the likelihood of either disease. Hematuria and diffuse parenchymal lung disease prompt consideration of pulmonary-renal syndromes, such as anti-glomerular basement membrane disease, GPA, and systemic lupus erythematosus, which can all be triggered by infection.

On the night of transfer, the patient experienced acute respiratory distress. Heart rate was 130 beats per minute, BP 170/95 mm Hg, respiratory rate 40 breaths per minute, and oxygen saturation 88% on six liters of supplemental oxygen by nasal cannula. His arterial blood gas demonstrated a pH of 7.23, PaCO2 of 32 mm Hg, and PaO2 of 65 mm Hg. He was emergently intubated for progressive hypoxemic respiratory failure. A small amount of blood was noted in the endotracheal tube. A noncontrast CT of the chest demonstrated multifocal airspace opacities and bilateral pleural effusions. The previously noted left upper lobe mass was unchanged.

Rapid respiratory decline and diffuse alveolar disease commonly result from aspiration, flash pulmonary edema, and acute respiratory distress syndrome (ARDS). Necrotizing pneumonia (eg, S. aureus) and trauma during intubation are possible causes of blood in his endotracheal tube. However, in the setting of multifocal airspace opacity, renal insufficiency, hematuria, and rapid respiratory decline, the blood might represent diffuse alveolar hemorrhage (DAH). Bronchoscopy with bronchioalveolar lavage to evaluate for pulmonary edema, infection, and hemorrhage would be indicated.

The patient subsequently developed oliguria, requiring continuous renal replacement therapy. An echocardiogram demonstrated impaired left ventricular relaxation and a reduced ejection fraction of 45% without segmental wall motion abnormalities or valvular disease, and a right ventricular systolic pressure of 36 mm Hg. Over the next 12 hours, his respiratory status improved, and he was extubated to 15 L per minute of supplemental oxygen by high-flow nasal cannula (HFNC).

The pathology report of the lung biopsy from the other hospital disclosed chronic inflammation and fibrosis with ill-defined areas of necrosis and myxoid degeneration surrounded by nuclear palisading suggestive of granulomatous inflammation. Staining for acid-fast bacilli (AFB) and fungal organisms was negative.

The rapid pulmonary recovery is inconsistent with multifocal pneumonia or ARDS. Flash pulmonary edema might result in sudden hypoxemic respiratory failure that resolves with positive pressure ventilation and ultrafiltration. However, this condition would not explain the biopsy results. Granulomatous lung pathology often results from mycobacterial or fungal disease. Tuberculosis and fungal pneumonia are not excluded with negative staining alone. However, neither would cause self-limited respiratory failure. Histologic evidence of necrosis lessens the likelihood of sarcoidosis, which rarely causes fulminant pulmonary disease. Lymphoma can result in granulomatous inflammation but would not cause transient pulmonary disease. GPA, a cause of necrotizing granulomatous lung disease, might result in a lung mass and worsened hypoxemia through DAH.

The patient continued to require 15 L of oxygen per minute by HFNC. He had persistent bilateral perihilar alveolar and interstitial opacities on CXR. Repeat WBC count was 29,200 per cubic millimeter, hemoglobin 7.8 g per deciliter, and platelets 656,000 per cubic millimeter. The C-reactive protein was 300 mg per L (normal range, <6.3) and erythrocyte sedimentation rate 100 mm per hour (normal range, <10). Legionella urinary antigen, serum immunodiffusion for Coccidiodes imitus, human immunodeficiency virus antibody, respiratory viral panel, and beta-D glucan were negative. Rare acid-fast bacilli were visualized in one out of three concentrated AFB sputum smears. He was started on empiric antituberculous therapy with rifampin, isoniazid, pyrazinamide, and ethambutol.

The sputum sample is suggestive of pulmonary tuberculosis. The salient features of this case include systemic inflammation, pulmonary nodules and mass, necrotizing granulomatous lung pathology, renal insufficiency, and hematuria. Disseminated tuberculosis might explain all these findings. However, a positive AFB smear may signal the presence of a nontuberculous mycobacteria, which is less likely to cause this clinical syndrome.

M. tuberculosis complex polymerase chain reaction (MTB PCR) assay returned negative for M. tuberculosis. Antiproteinase 3 antibody was 1,930 units (normal range, <20). Antimyeloperoxidase and antiglomerular basement membrane antibodies were negative.

Tuberculosis and GPA share several overlapping features, such as necrotizing lung pathology and less commonly antineutrophil cytoplasmic autoantibody (ANCA)-associated antibodies. However, the lung mass, acute renal and respiratory failure, hematuria, and the degree of anti-proteinase 3 level elevation are highly suggestive of GPA. The negative MTB PCR raises the possibility that a nontuberculous mycobacterium was detected on the sputum smear. Nevertheless, continued treatment until finalization of culture results is appropriate given that tuberculosis is endemic in Mexico.

 

 

The patient’s presenting features of right upper quadrant tenderness, jaundice, and cholestatic hepatitis remain poorly explained by either of these diagnoses.  Neither tuberculosis nor GPA commonly presents with accompanying hepatic involvement, though both have been occasionally described as causing hepatitis. As the greatest concern in this patient remains his progressive renal failure and accompanying pulmonary hemorrhage, a renal biopsy to assess for glomerulonephritis associated with GPA is warranted before further investigation into the cause of his cholestatic hepatitis.

A core renal biopsy demonstrated pauci-immune focal crescentic and necrotizing glomerulonephritis with mixed tubulointerstitial inflammation (Figure 2). In conjunction with the pulmonary syndrome and positive antiproteinase 3 serology, a diagnosis of granulomatosis with polyangiitis was made. The patient was treated with pulse dose steroids, rituximab, and plasma exchange. Two weeks later, the sputum mycobacterial culture returned positive for Mycobacterium llatzerense and anti-tuberculous treatment was discontinued.

jhm014040246_f2.jpg

Over the following weeks, the patient improved and was transitioned off dialysis prior to hospital discharge. By six months later, he had resolution of his hemoptysis, shortness of breath, liver biochemical test abnormalities, and significant improvement in his renal function. Repeat sputum mycobacterial cultures were negative.

DISCUSSION

A 65-year-old man from Mexico with a significant smoking history presented with an apical lung mass and cough, prioritizing tuberculosis and pulmonary malignancy. As the case unfolded, renal failure, multifocal lung opacities, conflicting tuberculosis test results, positive anti-proteinase 3 antibody, and ultimately a renal biopsy led to the diagnosis of granulomatosis with polyangiitis (GPA).

The correct interpretation of occasionally conflicting mycobacterial testing is crucial. Mycobacterial cultures remain the gold standard for diagnosing tuberculosis. However, results take weeks to return. Rapid tests include acid-fast bacilli (AFB) smear microscopy and nucleic acid-amplification tests (NAAT) of sputum or bronchoalveolar samples.1 When three sputum smears are performed, the sensitivity of AFB smear microscopy for tuberculosis in immunocompetent hosts is 70%.1 The AFB smear does not distinguish between different mycobacterial organisms. Thus, a positive result must be interpreted with the relative prevalence of tuberculosis and nontuberculous mycobacteria (NTM) in mind. The addition of NAAT-based assays has allowed for enhanced sensitivity and specificity in the diagnosis of tuberculosis, such that a negative NAAT in a patient with a positive AFB smear strongly argues for the presence of a NTM.2-4

NTM are widely prevalent environmental microbes, with over 140 species described, and careful consideration is required to determine if an isolate is pathogenic.5 Given their ubiquitous nature, a high rate of asymptomatic respiratory and cutaneous colonization occurs. Correspondingly, the diagnosis of NTM disease requires multiple positive cultures or pathologic features on tissue biopsy, compatible clinical findings, and diligent exclusion of other causes.5 A retrospective study of all NTM isolates in Oregon from 2005­-2006 revealed that only 47% of patients met the guideline criteria for having symptomatic NTM disease.6 In our case, the patient’s sputum grew M. llatzerense, an aerobic, nonfermenting mycobacterium found in water sources that has only infrequently been implicated as a human pathogen.7,8 Subsequent AFB sputum cultures were negative, and serial imaging showed resolution of the pulmonary findings without additional antimycobacterial therapy, suggesting that this organism was not responsible for the disease process.

Along with microscopic polyangiitis (MPA) and eosinophilic granulomatosis with polyangiitis (EGPA), GPA is an antineutrophil cytoplasmic autoantibody (ANCA)-associated vasculitis that predominantly affects small to medium sized vessels. Although it can occur at any age, GPA most commonly afflicts older adults, with men and women being diagnosed at roughly equal rates.9 GPA is a multisystem disease with a wide array of clinical manifestations. The most frequently involved sites of disease are the respiratory tract and kidneys, although virtually any organ can be affected. Sino-nasal disease, such as destructive sinusitis, or ear involvement are nearly universal. Lower respiratory manifestations occur in 60% of patients, but are highly diverse and reflect the inherent difficulty in diagnosing this condition.9-11 Additionally, GPA is a frequent cause of the pulmonary-renal syndromes, with glomerulonephritis occurring in 80% of patients.9

The diagnosis of GPA in this case was delayed, in part, due to features suggestive of malignancy and pulmonary tuberculosis. While sino-nasal disease was not noted during this hospitalization, the patient had many different respiratory manifestations, including a dominant pulmonary mass, diffuse nodules, and hypoxemic respiratory failure due to suspected diffuse alveolar hemorrhage (DAH), all of which have been reported in GPA.12 Dysmorphic red cells and red blood cell casts are not sensitive for renal involvement in GPA; their absence does not exclude the possibility of an ANCA-associated vasculitis.13 Hematuria and rapid progression to oliguric renal failure are characteristic of a vasculitic process and should sway clinicians away from a working diagnosis of ATN.

The diagnosis of GPA involves the synthesis of clinical data, radiographic findings, serologic testing, and histopathology. ANCA testing is an essential step in the diagnosis of GPA but has limitations. Patients with GPA more commonly have ANCAs targeting the enzyme proteinase-3 (PR3-ANCA), with MPA being more closely associated with myeloperoxidase (MPO-ANCA), although cross-reactivity and antibody-negative disease can occur.14 Although 90% of patients with GPA with multiorgan involvement will have a positive ANCA, a negative test is more common in localized upper airway disease, where only 50% have a positive ANCA.15 A number of drugs, medications, infections, and nonvasculitic autoimmune diseases have been associated with positive ANCA serologies in the absence of systemic vasculitis.14,16,17 As such, pathologic demonstration of vasculitis is necessary for establishing the diagnosis. Typical sites for biopsy include the kidneys and lungs.9

This case illustrates how clinicians often find themselves at a diagnostic crossroads—being forced to choose which clinical elements to prioritize. At various points, our patient’s presentation could have been framed as “a man from a Tb-endemic country with hemoptysis and an apical opacity,” “an elderly man with extensive smoking history and lung mass,” or “a patient with elevated inflammatory markers and pulmonary-renal syndrome”. In such situations, it is incumbent on the clinician to evaluate how well a given problem representation encompasses or highlights the salient features of a case. As with painting or photography, an essential aspect of appreciating the whole picture involves carefully selecting the right frame.

 

 

KEY TEACHING POINTS

  • The diagnosis of tuberculosis relies on smear microscopy, nucleic-acid amplification testing (NAAT), and cultures. A positive AFB smear with negative NAAT suggests the presence of a nontuberculous mycobacteria (NTM).
  • NTM are common environmental organisms and often exist as nonpathogenic colonizers.6 The diagnosis of NTM disease requires exclusion of other causes and careful clinical, microbiologic, and radiographic correlation.
  • Granulomatosis with polyangiitis is a multisystem disease often involving the respiratory track and kidney. Pulmonary disease can present with airway involvement, parenchymal nodules, opacities, pleural findings, and diffuse alveolar hemorrhage.12

Disclosures

Drs. Minter, Geha, Boslett, Chung, and Ramani have no disclosures. Dr. Manesh is supported by the Jeremiah A. Barondess Fellowship in the Clinical Transaction of the New York Academy of Medicine, in collaboration with the Accreditation Council for Graduate Medical Education (ACGME).

 

References

1. Lewinsohn DM, Leonard MK, LoBue PA, et al. Official American Thoracic Society/Infectious Diseases Society of America/Centers for Disease Control and Prevention clinical practice guidelines: diagnosis of tuberculosis in adults and children. Clin Infect Dis. 2017;64(2):e1-e33. PubMed
2. Steingart KR, Sohn H, Schiller I, et al. Xpert(R) MTB/RIF assay for pulmonary tuberculosis and rifampicin resistance in adults. Cochrane Database Syst Rev. 2013;(1):Cd009593. PubMed
3. Luetkemeyer AF, Firnhaber C, Kendall MA, et al. Evaluation of Xpert MTB/RIF versus afb smear and culture to identify pulmonary tuberculosis in patients with suspected tuberculosis from low and higher prevalence settings. Clin Infect Dis. 2016;62(9):1081-1088. PubMed
4. Boehme CC, Nabeta P, Hillemann D, et al. Rapid molecular detection of tuberculosis and rifampin resistance. N Engl J Med. 2010;363(11):1005-1015. PubMed
5. Griffith DE, Aksamit T, Brown-Elliott BA, et al. An official ATS/IDSA statement: diagnosis, treatment, and prevention of nontuberculous mycobacterial diseases. Am J Respir Crit Care Med. 2007;175(4):367-416. PubMed
6. Winthrop KL, McNelley E, Kendall B, et al. Pulmonary nontuberculous mycobacterial disease prevalence and clinical features: an emerging public health disease. Am J Respir Crit Care Med. 2010;182(7):977-982. PubMed
7. Teixeira L, Avery RK, Iseman M, et al. Mycobacterium llatzerense lung infection in a liver transplant recipient: case report and review of the literature. Am J Transplant. 2013;13(8):2198-2200. PubMed
8. Cárdenas AM, Gomila M, Lalucat J, Edelstein PH. Abdominal abscess caused by Mycobacterium llatzerense. J Clin Microbiol. 2014;52(4):1287-1289. PubMed
9. Jennette JC, Falk RJ. Small-vessel vasculitis. N Engl J Med. 1997;337(21):1512-1523. PubMed
10. Mahr A, Katsahian S, Varet H, et al. Revisiting the classification of clinical phenotypes of anti-neutrophil cytoplasmic antibody-associated vasculitis: a cluster analysis. Ann Rheum Dis. 2013;72(6):1003-1010. PubMed
11. Holle JU, Gross WL, Latza U, et al. Improved outcome in 445 patients with Wegener’s granulomatosis in a German vasculitis center over four decades. Arthritis Rheum. 2011;63(1):257-266. PubMed
12. Cordier JF, Valeyre D, Guillevin L, Loire R, Brechot JM. Pulmonary Wegener’s granulomatosis. A clinical and imaging study of 77 cases. Chest. 1990;97(4):906-912. PubMed
13. Hamadah AM, Gharaibeh K, Mara KC, et al. Urinalysis for the diagnosis of glomerulonephritis: role of dysmorphic red blood cells. Nephrol Dial Transplant. 2018;33(8):1397-1403. PubMed
14. Jennette JC, Falk RJ. Pathogenesis of antineutrophil cytoplasmic autoantibody-mediated disease. Nat Rev Rheumatol. 2014;10(8):463-473. PubMed
15. Borner U, Landis BN, Banz Y, et al. Diagnostic value of biopsies in identifying cytoplasmic antineutrophil cytoplasmic antibody-negative localized Wegener’s granulomatosis presenting primarily with sinonasal disease. Am J Rhinol Allergy. 2012;26(6):475-480. PubMed
16. Mahr A, Batteux F, Tubiana S, et al. Brief report: prevalence of antineutrophil cytoplasmic antibodies in infective endocarditis. Arthritis Rheumatol. 2014;66(6):1672-1677. PubMed
17. Sherkat R, Mostafavizadeh K, Zeydabadi L, Shoaei P, Rostami S. Antineutrophil cytoplasmic antibodies in patients with pulmonary tuberculosis. Iran J Immunol. 2011;8(1):52-57. PubMed

References

1. Lewinsohn DM, Leonard MK, LoBue PA, et al. Official American Thoracic Society/Infectious Diseases Society of America/Centers for Disease Control and Prevention clinical practice guidelines: diagnosis of tuberculosis in adults and children. Clin Infect Dis. 2017;64(2):e1-e33. PubMed
2. Steingart KR, Sohn H, Schiller I, et al. Xpert(R) MTB/RIF assay for pulmonary tuberculosis and rifampicin resistance in adults. Cochrane Database Syst Rev. 2013;(1):Cd009593. PubMed
3. Luetkemeyer AF, Firnhaber C, Kendall MA, et al. Evaluation of Xpert MTB/RIF versus afb smear and culture to identify pulmonary tuberculosis in patients with suspected tuberculosis from low and higher prevalence settings. Clin Infect Dis. 2016;62(9):1081-1088. PubMed
4. Boehme CC, Nabeta P, Hillemann D, et al. Rapid molecular detection of tuberculosis and rifampin resistance. N Engl J Med. 2010;363(11):1005-1015. PubMed
5. Griffith DE, Aksamit T, Brown-Elliott BA, et al. An official ATS/IDSA statement: diagnosis, treatment, and prevention of nontuberculous mycobacterial diseases. Am J Respir Crit Care Med. 2007;175(4):367-416. PubMed
6. Winthrop KL, McNelley E, Kendall B, et al. Pulmonary nontuberculous mycobacterial disease prevalence and clinical features: an emerging public health disease. Am J Respir Crit Care Med. 2010;182(7):977-982. PubMed
7. Teixeira L, Avery RK, Iseman M, et al. Mycobacterium llatzerense lung infection in a liver transplant recipient: case report and review of the literature. Am J Transplant. 2013;13(8):2198-2200. PubMed
8. Cárdenas AM, Gomila M, Lalucat J, Edelstein PH. Abdominal abscess caused by Mycobacterium llatzerense. J Clin Microbiol. 2014;52(4):1287-1289. PubMed
9. Jennette JC, Falk RJ. Small-vessel vasculitis. N Engl J Med. 1997;337(21):1512-1523. PubMed
10. Mahr A, Katsahian S, Varet H, et al. Revisiting the classification of clinical phenotypes of anti-neutrophil cytoplasmic antibody-associated vasculitis: a cluster analysis. Ann Rheum Dis. 2013;72(6):1003-1010. PubMed
11. Holle JU, Gross WL, Latza U, et al. Improved outcome in 445 patients with Wegener’s granulomatosis in a German vasculitis center over four decades. Arthritis Rheum. 2011;63(1):257-266. PubMed
12. Cordier JF, Valeyre D, Guillevin L, Loire R, Brechot JM. Pulmonary Wegener’s granulomatosis. A clinical and imaging study of 77 cases. Chest. 1990;97(4):906-912. PubMed
13. Hamadah AM, Gharaibeh K, Mara KC, et al. Urinalysis for the diagnosis of glomerulonephritis: role of dysmorphic red blood cells. Nephrol Dial Transplant. 2018;33(8):1397-1403. PubMed
14. Jennette JC, Falk RJ. Pathogenesis of antineutrophil cytoplasmic autoantibody-mediated disease. Nat Rev Rheumatol. 2014;10(8):463-473. PubMed
15. Borner U, Landis BN, Banz Y, et al. Diagnostic value of biopsies in identifying cytoplasmic antineutrophil cytoplasmic antibody-negative localized Wegener’s granulomatosis presenting primarily with sinonasal disease. Am J Rhinol Allergy. 2012;26(6):475-480. PubMed
16. Mahr A, Batteux F, Tubiana S, et al. Brief report: prevalence of antineutrophil cytoplasmic antibodies in infective endocarditis. Arthritis Rheumatol. 2014;66(6):1672-1677. PubMed
17. Sherkat R, Mostafavizadeh K, Zeydabadi L, Shoaei P, Rostami S. Antineutrophil cytoplasmic antibodies in patients with pulmonary tuberculosis. Iran J Immunol. 2011;8(1):52-57. PubMed

Issue
Journal of Hospital Medicine 14(4)
Issue
Journal of Hospital Medicine 14(4)
Page Number
246-250
Page Number
246-250
Publications
Publications
Topics
Article Type
Sections
Article Source

© 2019 Society of Hospital Medicine

Disallow All Ads
Correspondence Location
Daniel Minter, MD; E-mail: daniel.minter@ucsf.edu; Telephone: 253-948-2047
Content Gating
Gated (full article locked unless allowed per User)
Alternative CME
Disqus Comments
Default
Use ProPublica
Hide sidebar & use full width
render the right sidebar.
Gating Strategy
First Peek Free
Article PDF Media

Egad!

Article Type
Changed
Wed, 03/27/2019 - 18:21

A 69-year-old woman presented to the clinic with pain in the right great toe lasting several days. She was prescribed colchicine and indomethacin empirically for gout. She took one tablet of colchicine (0.6 mg) every hour until her stools became loose after the eighth tablet. Her toe pain resolved, but two days later she developed bilateral lower extremity pruritus and paresthesia and presented to the emergency department (ED). On physical examination, no rash, weakness, or sensory deficits were observed, and she was able to ambulate without assistance. Her patellar reflexes were normal. The complete blood count was notable for an absolute lymphocyte count of 6,120/µL (normal: 1,100-4,800), and the comprehensive metabolic panel was normal. Serum creatine kinase (CK) was 341 U/L (normal: 24-170) and uric acid 7.7 mg/dL (normal: 2.4-6.4). Her lower extremity symptoms were attributed to colchicine, which was discontinued. She was prescribed diphenhydramine and discharged home.

Monoarthritis of the hallux is the classic manifestation of gout, although other considerations include pseudogout, sesamoiditis, and trauma. The typical side effects of colchicine include diarrhea and myositis. Colchicine-induced muscle injury often results in a modest elevation of CK levels and is associated with myalgia.

Paresthesia is defined as abnormal sensory symptoms that most commonly localize to the peripheral nerves or spinal cord. Acute neuropathies or myelopathies might result from vasculitis, heavy metal toxicity, vitamin deficiencies, and paraneoplastic neurologic syndromes. The normal motor, sensory, and reflex examination, however, make these unlikely.

The neuro-anatomic localization of pruritus is poorly understood but is proposed to include peripheral nerves, spinothalamic tracts, and thalami. Acute pruritus (lasting <6 weeks) typically results from a primary dermatologic process such as a drug reaction, eczema, or xerosis. Less common causes include uremia, cholestasis, and thyroid disease. Pruritus can also be seen with malignancy, most commonly hematologic or paraneoplastic syndromes, or with connective tissue diseases. At this stage, it is unclear whether her pruritus and paresthesia are part of a unifying disease process.

Five days later she re-presented to the ED with nausea and emesis after eating at a restaurant. Her symptoms improved with intravenous fluids, and she was discharged. Four days later she returned with difficulty ambulating, bilateral leg cramping, and continued pruritus and paresthesia. The chemistry panel was normal except for a potassium level of 2.6 mmol/L and a bicarbonate level of 32 mmol/L. She was admitted to the hospital because of severe hypokalemia and impaired ability to ambulate. Her potassium was replenished. Her CK was elevated (3,551 U/L on hospital day 7). She was given cyclobenzaprine, gabapentin, oxycodone, acetaminophen, and prednisone (40 mg); her cramping only mildly improved, and she remained unable to walk. On hospital day five she had visual hallucinations and confusion, which did not resolve with administration of haloperidol; a head CT was unremarkable. On hospital day eight the patient, with her family’s support, left the hospital and presented to a different ED for a second opinion.

Difficulty ambulating often results from weakness, sensory impairment, cerebellar ataxia, extrapyramidal dysfunction (eg, parkinsonism), and pain. In this patient, leg cramping suggests pain or true weakness due to a myopathic process as a contributing factor. Symptoms of muscle disease include cramps, myalgia, and difficulty walking. Causes of elevated CK and myalgia include inflammatory myopathies, endocrinopathies, drugs, infections, and electrolyte abnormalities (eg, hypokalemia). Her age and acuity of presentation decrease the likelihood of a metabolic myopathy due to a disorder of glycogen storage, lipid metabolism, or mitochondrial function. Her hypokalemic metabolic alkalosis likely resulted from vomiting. Hypokalemic periodic paralysis is unlikely as exacerbations typically only last hours to days. As such, her difficulty ambulating, muscle cramps, and elevated CK strongly support a primary myopathic disorder, although additional information regarding the neurologic examination is still required.

 

 

Acute changes in mental status without corresponding changes in cranial nerve, motor, or sensory function are common in the hospital setting and frequently relate to delirium, which is the most likely explanation for her confusion. Her age and exposure to muscle relaxants, opiates, and corticosteroids increase her risk considerably. Other possible explanations for isolated changes in mental status include nonconvulsive seizures, central nervous system (CNS) infection, and strokes that involve the thalamus, nondominant parietal lobe, and reticular activating system. A shower of emboli resulting in small multifocal strokes can have the same effect.

She was re-evaluated by her new providers. Her only prior medical history was hypertension, which was treated at home with atenolol and amlodipine. She had emigrated from Nigeria to the US many years prior. She occasionally consumed alcohol and never smoked tobacco or used illicit drugs. She was unsure if she had received a tetanus booster in the past 10 years.

On physical examination, her temperature was 36°C, blood pressure 149/70 mm Hg, pulse 56 beats per minute, respiratory rate 18 breaths per minute, and oxygen saturation 98% on ambient air. She was diaphoretic and appeared anxious, grabbing both bedrails out of fear of falling. Cardiovascular, pulmonary, abdominal, and skin examinations were normal. She was alert and oriented to her identity, her location, and the time. Cranial nerves II to XII were normal. Tone was normal in her upper extremities but markedly increased in her lower extremities and back. There were spontaneous and stimulus-induced painful spasms, predominantly involving her axial muscles and distal lower extremities. Muscle bulk was normal. Strength was normal in the upper extremities and could not be assessed in the lower extremities due to rigidity. Reflexes were 2+ and symmetric throughout with downgoing toes on Babinski testing. A sensory examination was normal. Gait could not be tested because of the severe muscle spasms. The patient was admitted to the hospital.

Localized muscle spasms may be caused by muscle overuse, but more generalized spasms are associated with systemic diseases such as electrolyte disturbances, toxidromes, tetanus, peripheral nerve hyperexcitability syndromes (including Isaacs syndrome and Morvan syndrome), or stiff person syndrome (SPS). Hypokalemia is unlikely the cause as its correction did not improve her symptoms. Although tetanus is rare in the United States, it remains endemic in the developing world and can cause focal as well as generalized stimulus-induced spasms. The patient should be asked about potential exposure to Clostridium tetani infection, such as incurring a puncture wound. It is also important to consider neuroleptic malignant syndrome and serotonin syndrome, which can cause confusion, elevated CK, and increased muscle tone. Her confusion, however, was transient and the elevated CK preceded the administration of haloperidol.

SPS and progressive encephalomyelitis with rigidity and myoclonus (PERM) provide better explanations for her presentation. Both diseases cause severe spasms, impaired ambulation, and stiffness. They differ in their acuity of onset, accompanying symptoms, antibody associations, and responses to treatment. The rapid onset, paresthesia, and confusion seen in this patient are atypical of SPS. SPS usually presents with subacute-to-chronic stiffness or soreness of muscles in the back and lower extremities, followed by the upper extremities. Rigidity, stimulation-provoked spasms, hyperlordosis, and difficulty ambulating are typically later-stage findings. Her rapid escalation of symptoms is more consistent with PERM, which is often more acute and progressive than typical SPS; however, unlike this patient, PERM commonly causes widespread CNS dysfunction, including persistent encephalopathy, cranial neuropathies, hyperreflexia, and autonomic instability. Both are rare diagnoses that can manifest as a paraneoplastic neurologic syndrome.

 

 

Blood tests showed a leukocyte count of 17,350/µL, neutrophils 8,720/µL (normal: 1,500–7,800), lymphocytes 6,130/µL, hemoglobin 11.3 g/dL, and platelets 231,000/µL. The basic metabolic panel was normal. Serum total protein was 6.7 g/dL with albumin 3.5 g/dL. Aspartate aminotransferase (AST) was 94 U/L (normal: 0-31), alanine aminotransferase (ALT) 56 U/L (normal: 0-31), alkaline phosphatase 45 U/L, and total bilirubin 1.1 mg/dL. Vitamin B12 was 868 pg/mL. Hemoglobin A1c and thyrotropin levels were normal. Creatine kinase was 3,757 U/L and lactate dehydrogenase (LDH) 435 U/L (normal: 122-220). The syphilis treponemal test and hepatitis B surface antigen were negative. HIV and hepatitis C antibodies were nonreactive. The anti-nuclear antibody screen was negative and complement C3 and C4 were normal.

Neutrophilia likely reflects glucocorticoid-induced demargination, as opposed to an infectious process, given the temporal association with steroid administration. Persistent mild lymphocytosis is nonspecific but more likely to reflect a reactive rather than a clonal process. Elevated LDH and CK, as well as a greater increase of AST relative to ALT, suggest muscle injury, although mild concomitant hepatic injury cannot be excluded. Normal or negative serum studies for TSH, HIV, ANA, peripheral blood smear, and creatinine eliminate many of the systemic causes of her pruritus, but malignancy and associated paraneoplastic etiologies remain considerations.

The initial work-up for SPS includes electromyography (EMG) which would show spontaneous muscle activity. Her poorly localized sensory abnormalities, transient vestibular symptoms, and confusion warrant an MRI of the brain and spine to evaluate for inflammation (eg, encephalomyelitis), which could be consistent with PERM.

An MRI of the brain and cervicothoracic spine without contrast was significantly limited by motion artifact but without obvious intracranial or cord signal abnormalities. Electromyography demonstrated spontaneous muscle activity in both lower extremities with co-contraction of agonist and antagonist muscles (hamstrings and quadriceps as well as medial gastrocnemius and tibialis anterior). Sensory and motor nerve conductions were normal. Cerebral spinal fluid (CSF) contained six leukocytes (96% lymphocytes) and three red blood cells per microliter; glucose was 67 mg/dL and protein 24 mg/dL. There were two oligoclonal bands unique to the CSF. Cytology was negative for malignant cells.

The EMG narrows the differential diagnosis considerably. Co-contraction of opposing flexor and extensor groups (with predominance of extensors) on EMG is a diagnostic criterion for SPS and explains the myalgia and elevated CK. Her normal MRI studies effectively ruled out any focal lesion and did not show signs of encephalitis. Oligoclonal bands in the CSF are a sensitive marker of intrathecal inflammation, although not specific to one diagnosis. The mildly elevated cell count also supports CNS inflammation. In the setting of a lymphocytic pleocytosis and unique oligoclonal bands, it is important to consider infectious, neoplastic, autoimmune, and paraneoplastic causes of neuroinflammatory disorders.

Serum analyses, including antiglutamic acid decarboxylase 65 (GAD65) antibody and anti-amphiphysin antibody, should be ordered. The anti-GAD65 antibody is most commonly elevated in the setting of autoimmune diabetes mellitus; the titer, however, is usually dramatically higher in SPS. The CSF titer of anti-GAD65 antibodies is more specific than the serum titer for SPS. Antibodies against amphiphysin are typically elevated in paraneoplastic SPS, and anti-glycine receptor antibodies are associated with PERM, which commonly does not have elevated anti-GAD65 antibodies.

 

 

The serum GAD65 antibody level was greater than 265,000 × 103 IU/µL (normal <5,000), and the CSF level was 11.2 nmol/L (normal: ≤0.02). Serum amphiphysin antibody testing was negative.

Significantly elevated serum and CSF anti-GAD65 antibody levels are highly suggestive of SPS. Stiff person syndrome with rapidly progressive clinical symptoms raises the concern of a paraneoplastic neurologic syndrome. Although anti-amphiphysin antibody – the antibody classically associated with breast cancer and SPS – was negative, anti-GAD65 antibody has been implicated in paraneoplastic SPS with thymoma, lymphoma, and thyroid carcinoma. Paraneoplastic neurologic syndrome can predate a detectable malignancy by several years. As SPS and lymphoma are associated with pruritus and lymphocytosis, imaging is indicated to search for malignancy. Antiglycine receptor antibody, associated with PERM, is not routinely available commercially.

Computed tomography of the chest, abdomen, and pelvis with intravenous contrast revealed a 3.9 × 8.0 × 7.0 cm anterior mediastinal mass (Figure 1, Panel A). Biopsy of the mass demonstrated a thymoma. Given that the patient exhibited no further signs of CNS involvement, her initial transiently altered mental status was attributed to opioids and steroids. As she did not meet the clinical criteria for PERM, testing of antiglycine antibodies was not pursued.

jhm014030181_f1.jpg

She received scheduled baclofen and diazepam with as needed cyclobenzaprine for continued muscle spasms. Over the next several days, her stiffness, spasms, and myoclonic jerks slowly improved, and she was able to attempt physical therapy (Appendix Video 1; https://youtu.be/d0gLpTgqaCs). She subsequently received intravenous immunoglobulin (IVIG) with further improvement. After five months of scheduled diazepam and baclofen, she was able to ambulate with minimal assistance (Appendix Video 2; https://youtu.be/I00i638u00o). Given the absence of safe tissue planes for resection, the patient received neoadjuvant chemotherapy with four cycles of cyclophosphamide, doxorubicin, and cisplatin. Tumor size decreased to 1.7 × 6.5 × 5.2 cm (Figure 1, Panel B), and she subsequently underwent resection (Figure 2). Pathological analysis demonstrated a type B1 thymoma.

jhm014030181_f2.jpg

COMMENTARY

SPS is a condition of muscle stiffness and spasticity. Diagnosis is difficult and often delayed due to its rarity, with an approximate prevalence of one to two cases per million people.1 SPS typically occurs in middle age, and women are diagnosed twice as often as men. Classic SPS is characterized by axial and limb muscle stiffness, episodic spasms precipitated by tactile or auditory stimuli, continuous motor unit activity in agonist and antagonist muscles on EMG, high-titer antibody to GAD65 or amphiphysin, and the absence of an alternate diagnosis.2 Variant syndromes have been described, including a milder variant limited to the limbs, a severe variant with brainstem and spinal cord involvement, and a paraneoplastic variant.3 This patient’s clinical presentation, EMG findings, and extraordinarily high anti-GAD titers in the serum and CSF were diagnostic of SPS.

The pathophysiology of SPS is associated with autoantibodies targeting proteins such as GAD65, amphiphysin, gephyrin, and GABAA receptor-associated protein (GABARAP). These proteins are critical to gamma-aminobutyric acid (GABA) signaling, the primary inhibitory neurotransmitter pathway in the CNS (Figure 3).4 The formation of GABA from glutamate is catalyzed by GAD65. Gamma-aminobutyric acid is loaded into secretory vesicles, and amphiphysin facilitates vesicle recycling from the synaptic space.5 In the postsynaptic neuron, GABA binds the GABAA receptor, leading to neuronal hyperpolarization and resistance to excitation. The GABAA receptor is clustered on the plasma membrane through a scaffold formed by gephyrin. GABARAP facilitates this clustering, in part by linking GABAA receptors and gephyrin.6 Autoantibodies to these proteins may be pathogenic; however, the direct effects on their targets are unclear. The end result is decreased GABAergic activity, leading to continuous activation of opposing muscle groups. The resulting stiffness is characteristic of this disorder. Colchicine is known to antagonize GABAA receptor signaling, and this may have brought the underlying diagnosis of SPS to clinical attention.7,8

jhm014030181_f3.jpg


Symptomatic treatment of SPS targets the GABAergic system. Typically, high doses of scheduled benzodiazepines9 and baclofen10 are necessary. When symptoms are not controlled by GABAergic drugs, immunosuppression with corticosteroids and IVIG has been used, as have plasmapheresis and rituximab.11 The efficacy of the latter, however, was not supported by a randomized, placebo-controlled trial.12 This patient experienced significant improvement with benzodiazepines, baclofen, IVIG, and neoadjuvant chemotherapy prior to thymoma resection. The pruritus, paresthesia, and lymphocytosis also resolved with medical therapy. Interestingly, GABA signaling suppresses itch, suggesting that loss of GABAA signaling may have contributed to the development of pruritus.

SPS occasionally occurs as a paraneoplastic neurologic syndrome. Breast cancer is the most commonly associated malignancy, although associations between thymomas and SPS13 with anti-GAD65 antibodies14 have also been described. The presentation of thymomas is variable, with approximately one-third discovered incidentally on imaging, one-third producing symptoms of local compression, and one-third identified in the setting of another syndrome, most commonly myasthenia gravis. In addition to myasthenia gravis, thymomas have been associated with conditions such as hypogammaglobulinemia, pure red cell aplasia, and agranulocytosis. Stiff person syndrome is a known, albeit infrequently associated, condition.15

A critical step in arriving at the relevant differential diagnosis requires correctly framing the patient’s case.16 The treatment team’s initial frame was “a 69-year-old woman with weakness and elevated CK,” which prioritized causes of weakness and myositis. Stiff person syndrome does not cause weakness, but rather impaired movement from marked stiffness and spasms. The patient’s elevated CK was a result of continual muscle contractions. The physical exam and lack of motor deficit on EMG led the treatment team to reframe as “a 69-year-old woman with severe stiffness and spasms.” Egad! This correct frame was the key to diagnosis and confirmed by EMG and GAD65 antibody testing.

 

 

KEY LEARNING POINTS

  • Classic SPS is characterized by axial and limb muscle stiffness, episodic spasms precipitated by tactile or auditory stimuli, continuous motor unit activity in agonist and antagonist muscles on EMG, and high-titer antibody to GAD65 or amphiphysin.
  • SPS typically occurs in middle age, and women are diagnosed twice as often as men.
  • Symptomatic treatment of SPS targets the GABAergic system. Typically, high doses of scheduled benzodiazepines and baclofenare necessary.
  • SPS occasionally occurs as a paraneoplastic neurologic syndrome, most commonly in association with breast cancer.

Acknowledgments

The authors wish to thank Jason Kern, MD for his preparation and interpretation of the pathologic image; and the Jeremiah A. Barondess Fellowship in the Clinical Transaction of the New York Academy of Medicine, in collaboration with the Accreditation Council for Graduate Medical Education, for supporting Reza Manesh, MD.

Disclosures

The authors have nothing to disclose.

Appendix Video 1: This video was taken during a physical therapy session after 1 week of scheduled benzodiazepine and 2 days of intravenous immunoglobulin. It was difficult for the patient to stand without assistance due to severe stiffness. (https://youtu.be/d0gLpTgqaCs)

Appendix Video 2: This video was taken 5 months after scheduled diazepam and baclofen, and 1 week prior to thymectomy. (https://youtu.be/I00i638u00o)

 

References

1. Hadavi S, Noyce AJ, Leslie RD, Giovannoni G. Stiff person syndrome. Pract Neurol. 2011;11(5):272-282. doi: 10.1136/practneurol-2011-000071. PubMed
2. Dalakas MC. Stiff person syndrome: advances in pathogenesis and therapeutic interventions. Curr Treat Options Neurol. 2009;11(2):102-110. doi: 10.1007/s11940-009-0013-9
PubMed
3. Murinson BB. Stiff-person syndrome. Neurologist. 2004;10(3):131-137. doi: 10.1097/01.nrl.0000126587.37087.1a
PubMed
4. Rakocevic G, Floeter MK. Autoimmune stiff person syndrome and related myelopathies: understanding of electrophysiological and immunological processes. Muscle Nerve. 2012;45(5):623-634. doi: 10.1002/mus.23234
PubMed
5. Zhang B, Zelhof AC. Amphiphysins: raising the BAR for synaptic vesicle recycling and membrane dynamics. Bin-Amphiphysin-Rvsp. Traffic. 2002;3(7):452-460. doi: 10.1034/j.1600-0854.2002.30702.x
PubMed
6. Tyagarajan SK, Fritschy JM. Gephyrin: a master regulator of neuronal function? Nat Rev Neurosci. 2014;15(3):141-156. doi: 10.1038/nrn3670
PubMed
7. Bueno OF, Leidenheimer NJ. Colchicine inhibits GABA(A) receptors independently of microtubule depolymerization. Neuropharmacology. 1998;37(3):383-390. doi: 10.1016/S0028-3908(98)00020-3
PubMed
8. Weiner JL, Buhler AV, Whatley VJ, Harris RA, Dunwiddie TV. Colchicine is a competitive antagonist at human recombinant γ-aminobutyric acidA receptors. J Pharmacol Exp Ther. 1998;284(1):95-102 . PubMed
9. Lorish TR, Thorsteinsson G, Howard FM Jr. Stiff-man syndrome updated. Mayo Clin Proc. 1989;64(6):629-636. doi: 10.1016/S0025-6196(12)65339-7
PubMed
10. McKeon A, Robinson MT, McEvoy KM, et al. Stiff-man syndrome and variants: clinical course, treatments, and outcomes. Arch Neurol. 2012;69(2):230-238. doi: 10.1001/archneurol.2011.991
PubMed
11. Dalakas MC, Li M, Fujii M, Jacobowitz DM. Stiff person syndrome: quantification, specificity, and intrathecal synthesis of GAD65 antibodies. Neurology. 2001;57(5):780-784. doi: 10.1212/WNL.57.5.780
PubMed
12. Dalakas MC, Rakocevic G, Dambrosia JM, Alexopoulos H, McElroy B. A double-blind, placebo-controlled study of rituximab in patients with stiff person syndrome. Ann Neurol. 2017;82(2):271-277. doi: 10.1002/ana.25002
PubMed
13. Hagiwara H, Enomoto-Nakatani S, Sakai K, et al. Stiff-person syndrome associated with invasive thymoma: a case report. J Neurol Sci. 2001;193(1):59-62. doi: 10.1016/S0022-510X(01)00602-5
PubMed
14. Vernino S, Lennon VA. Autoantibody profiles and neurological correlations of thymoma. Clin Cancer Res. 2004;10(21):7270-7275. doi: 10.1158/1078-0432.CCR-04-0735 PubMed
15. Thomas CR, Wright CD, Loehrer PJ. Thymoma: state of the art. J Clin Oncol. 1999;17(7):2280-2289. doi: 10.1200/JCO.1999.17.7.2280 PubMed
16. Stuart S, Hartig JR, Willett L. The importance of framing. J Gen Intern Med. 2017;32(6):706-710. doi: 10.1007/s11606-016-3964-z PubMed

Article PDF
Issue
Journal of Hospital Medicine 14(3)
Publications
Topics
Page Number
181-185
Sections
Article PDF
Article PDF

A 69-year-old woman presented to the clinic with pain in the right great toe lasting several days. She was prescribed colchicine and indomethacin empirically for gout. She took one tablet of colchicine (0.6 mg) every hour until her stools became loose after the eighth tablet. Her toe pain resolved, but two days later she developed bilateral lower extremity pruritus and paresthesia and presented to the emergency department (ED). On physical examination, no rash, weakness, or sensory deficits were observed, and she was able to ambulate without assistance. Her patellar reflexes were normal. The complete blood count was notable for an absolute lymphocyte count of 6,120/µL (normal: 1,100-4,800), and the comprehensive metabolic panel was normal. Serum creatine kinase (CK) was 341 U/L (normal: 24-170) and uric acid 7.7 mg/dL (normal: 2.4-6.4). Her lower extremity symptoms were attributed to colchicine, which was discontinued. She was prescribed diphenhydramine and discharged home.

Monoarthritis of the hallux is the classic manifestation of gout, although other considerations include pseudogout, sesamoiditis, and trauma. The typical side effects of colchicine include diarrhea and myositis. Colchicine-induced muscle injury often results in a modest elevation of CK levels and is associated with myalgia.

Paresthesia is defined as abnormal sensory symptoms that most commonly localize to the peripheral nerves or spinal cord. Acute neuropathies or myelopathies might result from vasculitis, heavy metal toxicity, vitamin deficiencies, and paraneoplastic neurologic syndromes. The normal motor, sensory, and reflex examination, however, make these unlikely.

The neuro-anatomic localization of pruritus is poorly understood but is proposed to include peripheral nerves, spinothalamic tracts, and thalami. Acute pruritus (lasting <6 weeks) typically results from a primary dermatologic process such as a drug reaction, eczema, or xerosis. Less common causes include uremia, cholestasis, and thyroid disease. Pruritus can also be seen with malignancy, most commonly hematologic or paraneoplastic syndromes, or with connective tissue diseases. At this stage, it is unclear whether her pruritus and paresthesia are part of a unifying disease process.

Five days later she re-presented to the ED with nausea and emesis after eating at a restaurant. Her symptoms improved with intravenous fluids, and she was discharged. Four days later she returned with difficulty ambulating, bilateral leg cramping, and continued pruritus and paresthesia. The chemistry panel was normal except for a potassium level of 2.6 mmol/L and a bicarbonate level of 32 mmol/L. She was admitted to the hospital because of severe hypokalemia and impaired ability to ambulate. Her potassium was replenished. Her CK was elevated (3,551 U/L on hospital day 7). She was given cyclobenzaprine, gabapentin, oxycodone, acetaminophen, and prednisone (40 mg); her cramping only mildly improved, and she remained unable to walk. On hospital day five she had visual hallucinations and confusion, which did not resolve with administration of haloperidol; a head CT was unremarkable. On hospital day eight the patient, with her family’s support, left the hospital and presented to a different ED for a second opinion.

Difficulty ambulating often results from weakness, sensory impairment, cerebellar ataxia, extrapyramidal dysfunction (eg, parkinsonism), and pain. In this patient, leg cramping suggests pain or true weakness due to a myopathic process as a contributing factor. Symptoms of muscle disease include cramps, myalgia, and difficulty walking. Causes of elevated CK and myalgia include inflammatory myopathies, endocrinopathies, drugs, infections, and electrolyte abnormalities (eg, hypokalemia). Her age and acuity of presentation decrease the likelihood of a metabolic myopathy due to a disorder of glycogen storage, lipid metabolism, or mitochondrial function. Her hypokalemic metabolic alkalosis likely resulted from vomiting. Hypokalemic periodic paralysis is unlikely as exacerbations typically only last hours to days. As such, her difficulty ambulating, muscle cramps, and elevated CK strongly support a primary myopathic disorder, although additional information regarding the neurologic examination is still required.

 

 

Acute changes in mental status without corresponding changes in cranial nerve, motor, or sensory function are common in the hospital setting and frequently relate to delirium, which is the most likely explanation for her confusion. Her age and exposure to muscle relaxants, opiates, and corticosteroids increase her risk considerably. Other possible explanations for isolated changes in mental status include nonconvulsive seizures, central nervous system (CNS) infection, and strokes that involve the thalamus, nondominant parietal lobe, and reticular activating system. A shower of emboli resulting in small multifocal strokes can have the same effect.

She was re-evaluated by her new providers. Her only prior medical history was hypertension, which was treated at home with atenolol and amlodipine. She had emigrated from Nigeria to the US many years prior. She occasionally consumed alcohol and never smoked tobacco or used illicit drugs. She was unsure if she had received a tetanus booster in the past 10 years.

On physical examination, her temperature was 36°C, blood pressure 149/70 mm Hg, pulse 56 beats per minute, respiratory rate 18 breaths per minute, and oxygen saturation 98% on ambient air. She was diaphoretic and appeared anxious, grabbing both bedrails out of fear of falling. Cardiovascular, pulmonary, abdominal, and skin examinations were normal. She was alert and oriented to her identity, her location, and the time. Cranial nerves II to XII were normal. Tone was normal in her upper extremities but markedly increased in her lower extremities and back. There were spontaneous and stimulus-induced painful spasms, predominantly involving her axial muscles and distal lower extremities. Muscle bulk was normal. Strength was normal in the upper extremities and could not be assessed in the lower extremities due to rigidity. Reflexes were 2+ and symmetric throughout with downgoing toes on Babinski testing. A sensory examination was normal. Gait could not be tested because of the severe muscle spasms. The patient was admitted to the hospital.

Localized muscle spasms may be caused by muscle overuse, but more generalized spasms are associated with systemic diseases such as electrolyte disturbances, toxidromes, tetanus, peripheral nerve hyperexcitability syndromes (including Isaacs syndrome and Morvan syndrome), or stiff person syndrome (SPS). Hypokalemia is unlikely the cause as its correction did not improve her symptoms. Although tetanus is rare in the United States, it remains endemic in the developing world and can cause focal as well as generalized stimulus-induced spasms. The patient should be asked about potential exposure to Clostridium tetani infection, such as incurring a puncture wound. It is also important to consider neuroleptic malignant syndrome and serotonin syndrome, which can cause confusion, elevated CK, and increased muscle tone. Her confusion, however, was transient and the elevated CK preceded the administration of haloperidol.

SPS and progressive encephalomyelitis with rigidity and myoclonus (PERM) provide better explanations for her presentation. Both diseases cause severe spasms, impaired ambulation, and stiffness. They differ in their acuity of onset, accompanying symptoms, antibody associations, and responses to treatment. The rapid onset, paresthesia, and confusion seen in this patient are atypical of SPS. SPS usually presents with subacute-to-chronic stiffness or soreness of muscles in the back and lower extremities, followed by the upper extremities. Rigidity, stimulation-provoked spasms, hyperlordosis, and difficulty ambulating are typically later-stage findings. Her rapid escalation of symptoms is more consistent with PERM, which is often more acute and progressive than typical SPS; however, unlike this patient, PERM commonly causes widespread CNS dysfunction, including persistent encephalopathy, cranial neuropathies, hyperreflexia, and autonomic instability. Both are rare diagnoses that can manifest as a paraneoplastic neurologic syndrome.

 

 

Blood tests showed a leukocyte count of 17,350/µL, neutrophils 8,720/µL (normal: 1,500–7,800), lymphocytes 6,130/µL, hemoglobin 11.3 g/dL, and platelets 231,000/µL. The basic metabolic panel was normal. Serum total protein was 6.7 g/dL with albumin 3.5 g/dL. Aspartate aminotransferase (AST) was 94 U/L (normal: 0-31), alanine aminotransferase (ALT) 56 U/L (normal: 0-31), alkaline phosphatase 45 U/L, and total bilirubin 1.1 mg/dL. Vitamin B12 was 868 pg/mL. Hemoglobin A1c and thyrotropin levels were normal. Creatine kinase was 3,757 U/L and lactate dehydrogenase (LDH) 435 U/L (normal: 122-220). The syphilis treponemal test and hepatitis B surface antigen were negative. HIV and hepatitis C antibodies were nonreactive. The anti-nuclear antibody screen was negative and complement C3 and C4 were normal.

Neutrophilia likely reflects glucocorticoid-induced demargination, as opposed to an infectious process, given the temporal association with steroid administration. Persistent mild lymphocytosis is nonspecific but more likely to reflect a reactive rather than a clonal process. Elevated LDH and CK, as well as a greater increase of AST relative to ALT, suggest muscle injury, although mild concomitant hepatic injury cannot be excluded. Normal or negative serum studies for TSH, HIV, ANA, peripheral blood smear, and creatinine eliminate many of the systemic causes of her pruritus, but malignancy and associated paraneoplastic etiologies remain considerations.

The initial work-up for SPS includes electromyography (EMG) which would show spontaneous muscle activity. Her poorly localized sensory abnormalities, transient vestibular symptoms, and confusion warrant an MRI of the brain and spine to evaluate for inflammation (eg, encephalomyelitis), which could be consistent with PERM.

An MRI of the brain and cervicothoracic spine without contrast was significantly limited by motion artifact but without obvious intracranial or cord signal abnormalities. Electromyography demonstrated spontaneous muscle activity in both lower extremities with co-contraction of agonist and antagonist muscles (hamstrings and quadriceps as well as medial gastrocnemius and tibialis anterior). Sensory and motor nerve conductions were normal. Cerebral spinal fluid (CSF) contained six leukocytes (96% lymphocytes) and three red blood cells per microliter; glucose was 67 mg/dL and protein 24 mg/dL. There were two oligoclonal bands unique to the CSF. Cytology was negative for malignant cells.

The EMG narrows the differential diagnosis considerably. Co-contraction of opposing flexor and extensor groups (with predominance of extensors) on EMG is a diagnostic criterion for SPS and explains the myalgia and elevated CK. Her normal MRI studies effectively ruled out any focal lesion and did not show signs of encephalitis. Oligoclonal bands in the CSF are a sensitive marker of intrathecal inflammation, although not specific to one diagnosis. The mildly elevated cell count also supports CNS inflammation. In the setting of a lymphocytic pleocytosis and unique oligoclonal bands, it is important to consider infectious, neoplastic, autoimmune, and paraneoplastic causes of neuroinflammatory disorders.

Serum analyses, including antiglutamic acid decarboxylase 65 (GAD65) antibody and anti-amphiphysin antibody, should be ordered. The anti-GAD65 antibody is most commonly elevated in the setting of autoimmune diabetes mellitus; the titer, however, is usually dramatically higher in SPS. The CSF titer of anti-GAD65 antibodies is more specific than the serum titer for SPS. Antibodies against amphiphysin are typically elevated in paraneoplastic SPS, and anti-glycine receptor antibodies are associated with PERM, which commonly does not have elevated anti-GAD65 antibodies.

 

 

The serum GAD65 antibody level was greater than 265,000 × 103 IU/µL (normal <5,000), and the CSF level was 11.2 nmol/L (normal: ≤0.02). Serum amphiphysin antibody testing was negative.

Significantly elevated serum and CSF anti-GAD65 antibody levels are highly suggestive of SPS. Stiff person syndrome with rapidly progressive clinical symptoms raises the concern of a paraneoplastic neurologic syndrome. Although anti-amphiphysin antibody – the antibody classically associated with breast cancer and SPS – was negative, anti-GAD65 antibody has been implicated in paraneoplastic SPS with thymoma, lymphoma, and thyroid carcinoma. Paraneoplastic neurologic syndrome can predate a detectable malignancy by several years. As SPS and lymphoma are associated with pruritus and lymphocytosis, imaging is indicated to search for malignancy. Antiglycine receptor antibody, associated with PERM, is not routinely available commercially.

Computed tomography of the chest, abdomen, and pelvis with intravenous contrast revealed a 3.9 × 8.0 × 7.0 cm anterior mediastinal mass (Figure 1, Panel A). Biopsy of the mass demonstrated a thymoma. Given that the patient exhibited no further signs of CNS involvement, her initial transiently altered mental status was attributed to opioids and steroids. As she did not meet the clinical criteria for PERM, testing of antiglycine antibodies was not pursued.

jhm014030181_f1.jpg

She received scheduled baclofen and diazepam with as needed cyclobenzaprine for continued muscle spasms. Over the next several days, her stiffness, spasms, and myoclonic jerks slowly improved, and she was able to attempt physical therapy (Appendix Video 1; https://youtu.be/d0gLpTgqaCs). She subsequently received intravenous immunoglobulin (IVIG) with further improvement. After five months of scheduled diazepam and baclofen, she was able to ambulate with minimal assistance (Appendix Video 2; https://youtu.be/I00i638u00o). Given the absence of safe tissue planes for resection, the patient received neoadjuvant chemotherapy with four cycles of cyclophosphamide, doxorubicin, and cisplatin. Tumor size decreased to 1.7 × 6.5 × 5.2 cm (Figure 1, Panel B), and she subsequently underwent resection (Figure 2). Pathological analysis demonstrated a type B1 thymoma.

jhm014030181_f2.jpg

COMMENTARY

SPS is a condition of muscle stiffness and spasticity. Diagnosis is difficult and often delayed due to its rarity, with an approximate prevalence of one to two cases per million people.1 SPS typically occurs in middle age, and women are diagnosed twice as often as men. Classic SPS is characterized by axial and limb muscle stiffness, episodic spasms precipitated by tactile or auditory stimuli, continuous motor unit activity in agonist and antagonist muscles on EMG, high-titer antibody to GAD65 or amphiphysin, and the absence of an alternate diagnosis.2 Variant syndromes have been described, including a milder variant limited to the limbs, a severe variant with brainstem and spinal cord involvement, and a paraneoplastic variant.3 This patient’s clinical presentation, EMG findings, and extraordinarily high anti-GAD titers in the serum and CSF were diagnostic of SPS.

The pathophysiology of SPS is associated with autoantibodies targeting proteins such as GAD65, amphiphysin, gephyrin, and GABAA receptor-associated protein (GABARAP). These proteins are critical to gamma-aminobutyric acid (GABA) signaling, the primary inhibitory neurotransmitter pathway in the CNS (Figure 3).4 The formation of GABA from glutamate is catalyzed by GAD65. Gamma-aminobutyric acid is loaded into secretory vesicles, and amphiphysin facilitates vesicle recycling from the synaptic space.5 In the postsynaptic neuron, GABA binds the GABAA receptor, leading to neuronal hyperpolarization and resistance to excitation. The GABAA receptor is clustered on the plasma membrane through a scaffold formed by gephyrin. GABARAP facilitates this clustering, in part by linking GABAA receptors and gephyrin.6 Autoantibodies to these proteins may be pathogenic; however, the direct effects on their targets are unclear. The end result is decreased GABAergic activity, leading to continuous activation of opposing muscle groups. The resulting stiffness is characteristic of this disorder. Colchicine is known to antagonize GABAA receptor signaling, and this may have brought the underlying diagnosis of SPS to clinical attention.7,8

jhm014030181_f3.jpg


Symptomatic treatment of SPS targets the GABAergic system. Typically, high doses of scheduled benzodiazepines9 and baclofen10 are necessary. When symptoms are not controlled by GABAergic drugs, immunosuppression with corticosteroids and IVIG has been used, as have plasmapheresis and rituximab.11 The efficacy of the latter, however, was not supported by a randomized, placebo-controlled trial.12 This patient experienced significant improvement with benzodiazepines, baclofen, IVIG, and neoadjuvant chemotherapy prior to thymoma resection. The pruritus, paresthesia, and lymphocytosis also resolved with medical therapy. Interestingly, GABA signaling suppresses itch, suggesting that loss of GABAA signaling may have contributed to the development of pruritus.

SPS occasionally occurs as a paraneoplastic neurologic syndrome. Breast cancer is the most commonly associated malignancy, although associations between thymomas and SPS13 with anti-GAD65 antibodies14 have also been described. The presentation of thymomas is variable, with approximately one-third discovered incidentally on imaging, one-third producing symptoms of local compression, and one-third identified in the setting of another syndrome, most commonly myasthenia gravis. In addition to myasthenia gravis, thymomas have been associated with conditions such as hypogammaglobulinemia, pure red cell aplasia, and agranulocytosis. Stiff person syndrome is a known, albeit infrequently associated, condition.15

A critical step in arriving at the relevant differential diagnosis requires correctly framing the patient’s case.16 The treatment team’s initial frame was “a 69-year-old woman with weakness and elevated CK,” which prioritized causes of weakness and myositis. Stiff person syndrome does not cause weakness, but rather impaired movement from marked stiffness and spasms. The patient’s elevated CK was a result of continual muscle contractions. The physical exam and lack of motor deficit on EMG led the treatment team to reframe as “a 69-year-old woman with severe stiffness and spasms.” Egad! This correct frame was the key to diagnosis and confirmed by EMG and GAD65 antibody testing.

 

 

KEY LEARNING POINTS

  • Classic SPS is characterized by axial and limb muscle stiffness, episodic spasms precipitated by tactile or auditory stimuli, continuous motor unit activity in agonist and antagonist muscles on EMG, and high-titer antibody to GAD65 or amphiphysin.
  • SPS typically occurs in middle age, and women are diagnosed twice as often as men.
  • Symptomatic treatment of SPS targets the GABAergic system. Typically, high doses of scheduled benzodiazepines and baclofenare necessary.
  • SPS occasionally occurs as a paraneoplastic neurologic syndrome, most commonly in association with breast cancer.

Acknowledgments

The authors wish to thank Jason Kern, MD for his preparation and interpretation of the pathologic image; and the Jeremiah A. Barondess Fellowship in the Clinical Transaction of the New York Academy of Medicine, in collaboration with the Accreditation Council for Graduate Medical Education, for supporting Reza Manesh, MD.

Disclosures

The authors have nothing to disclose.

Appendix Video 1: This video was taken during a physical therapy session after 1 week of scheduled benzodiazepine and 2 days of intravenous immunoglobulin. It was difficult for the patient to stand without assistance due to severe stiffness. (https://youtu.be/d0gLpTgqaCs)

Appendix Video 2: This video was taken 5 months after scheduled diazepam and baclofen, and 1 week prior to thymectomy. (https://youtu.be/I00i638u00o)

 

A 69-year-old woman presented to the clinic with pain in the right great toe lasting several days. She was prescribed colchicine and indomethacin empirically for gout. She took one tablet of colchicine (0.6 mg) every hour until her stools became loose after the eighth tablet. Her toe pain resolved, but two days later she developed bilateral lower extremity pruritus and paresthesia and presented to the emergency department (ED). On physical examination, no rash, weakness, or sensory deficits were observed, and she was able to ambulate without assistance. Her patellar reflexes were normal. The complete blood count was notable for an absolute lymphocyte count of 6,120/µL (normal: 1,100-4,800), and the comprehensive metabolic panel was normal. Serum creatine kinase (CK) was 341 U/L (normal: 24-170) and uric acid 7.7 mg/dL (normal: 2.4-6.4). Her lower extremity symptoms were attributed to colchicine, which was discontinued. She was prescribed diphenhydramine and discharged home.

Monoarthritis of the hallux is the classic manifestation of gout, although other considerations include pseudogout, sesamoiditis, and trauma. The typical side effects of colchicine include diarrhea and myositis. Colchicine-induced muscle injury often results in a modest elevation of CK levels and is associated with myalgia.

Paresthesia is defined as abnormal sensory symptoms that most commonly localize to the peripheral nerves or spinal cord. Acute neuropathies or myelopathies might result from vasculitis, heavy metal toxicity, vitamin deficiencies, and paraneoplastic neurologic syndromes. The normal motor, sensory, and reflex examination, however, make these unlikely.

The neuro-anatomic localization of pruritus is poorly understood but is proposed to include peripheral nerves, spinothalamic tracts, and thalami. Acute pruritus (lasting <6 weeks) typically results from a primary dermatologic process such as a drug reaction, eczema, or xerosis. Less common causes include uremia, cholestasis, and thyroid disease. Pruritus can also be seen with malignancy, most commonly hematologic or paraneoplastic syndromes, or with connective tissue diseases. At this stage, it is unclear whether her pruritus and paresthesia are part of a unifying disease process.

Five days later she re-presented to the ED with nausea and emesis after eating at a restaurant. Her symptoms improved with intravenous fluids, and she was discharged. Four days later she returned with difficulty ambulating, bilateral leg cramping, and continued pruritus and paresthesia. The chemistry panel was normal except for a potassium level of 2.6 mmol/L and a bicarbonate level of 32 mmol/L. She was admitted to the hospital because of severe hypokalemia and impaired ability to ambulate. Her potassium was replenished. Her CK was elevated (3,551 U/L on hospital day 7). She was given cyclobenzaprine, gabapentin, oxycodone, acetaminophen, and prednisone (40 mg); her cramping only mildly improved, and she remained unable to walk. On hospital day five she had visual hallucinations and confusion, which did not resolve with administration of haloperidol; a head CT was unremarkable. On hospital day eight the patient, with her family’s support, left the hospital and presented to a different ED for a second opinion.

Difficulty ambulating often results from weakness, sensory impairment, cerebellar ataxia, extrapyramidal dysfunction (eg, parkinsonism), and pain. In this patient, leg cramping suggests pain or true weakness due to a myopathic process as a contributing factor. Symptoms of muscle disease include cramps, myalgia, and difficulty walking. Causes of elevated CK and myalgia include inflammatory myopathies, endocrinopathies, drugs, infections, and electrolyte abnormalities (eg, hypokalemia). Her age and acuity of presentation decrease the likelihood of a metabolic myopathy due to a disorder of glycogen storage, lipid metabolism, or mitochondrial function. Her hypokalemic metabolic alkalosis likely resulted from vomiting. Hypokalemic periodic paralysis is unlikely as exacerbations typically only last hours to days. As such, her difficulty ambulating, muscle cramps, and elevated CK strongly support a primary myopathic disorder, although additional information regarding the neurologic examination is still required.

 

 

Acute changes in mental status without corresponding changes in cranial nerve, motor, or sensory function are common in the hospital setting and frequently relate to delirium, which is the most likely explanation for her confusion. Her age and exposure to muscle relaxants, opiates, and corticosteroids increase her risk considerably. Other possible explanations for isolated changes in mental status include nonconvulsive seizures, central nervous system (CNS) infection, and strokes that involve the thalamus, nondominant parietal lobe, and reticular activating system. A shower of emboli resulting in small multifocal strokes can have the same effect.

She was re-evaluated by her new providers. Her only prior medical history was hypertension, which was treated at home with atenolol and amlodipine. She had emigrated from Nigeria to the US many years prior. She occasionally consumed alcohol and never smoked tobacco or used illicit drugs. She was unsure if she had received a tetanus booster in the past 10 years.

On physical examination, her temperature was 36°C, blood pressure 149/70 mm Hg, pulse 56 beats per minute, respiratory rate 18 breaths per minute, and oxygen saturation 98% on ambient air. She was diaphoretic and appeared anxious, grabbing both bedrails out of fear of falling. Cardiovascular, pulmonary, abdominal, and skin examinations were normal. She was alert and oriented to her identity, her location, and the time. Cranial nerves II to XII were normal. Tone was normal in her upper extremities but markedly increased in her lower extremities and back. There were spontaneous and stimulus-induced painful spasms, predominantly involving her axial muscles and distal lower extremities. Muscle bulk was normal. Strength was normal in the upper extremities and could not be assessed in the lower extremities due to rigidity. Reflexes were 2+ and symmetric throughout with downgoing toes on Babinski testing. A sensory examination was normal. Gait could not be tested because of the severe muscle spasms. The patient was admitted to the hospital.

Localized muscle spasms may be caused by muscle overuse, but more generalized spasms are associated with systemic diseases such as electrolyte disturbances, toxidromes, tetanus, peripheral nerve hyperexcitability syndromes (including Isaacs syndrome and Morvan syndrome), or stiff person syndrome (SPS). Hypokalemia is unlikely the cause as its correction did not improve her symptoms. Although tetanus is rare in the United States, it remains endemic in the developing world and can cause focal as well as generalized stimulus-induced spasms. The patient should be asked about potential exposure to Clostridium tetani infection, such as incurring a puncture wound. It is also important to consider neuroleptic malignant syndrome and serotonin syndrome, which can cause confusion, elevated CK, and increased muscle tone. Her confusion, however, was transient and the elevated CK preceded the administration of haloperidol.

SPS and progressive encephalomyelitis with rigidity and myoclonus (PERM) provide better explanations for her presentation. Both diseases cause severe spasms, impaired ambulation, and stiffness. They differ in their acuity of onset, accompanying symptoms, antibody associations, and responses to treatment. The rapid onset, paresthesia, and confusion seen in this patient are atypical of SPS. SPS usually presents with subacute-to-chronic stiffness or soreness of muscles in the back and lower extremities, followed by the upper extremities. Rigidity, stimulation-provoked spasms, hyperlordosis, and difficulty ambulating are typically later-stage findings. Her rapid escalation of symptoms is more consistent with PERM, which is often more acute and progressive than typical SPS; however, unlike this patient, PERM commonly causes widespread CNS dysfunction, including persistent encephalopathy, cranial neuropathies, hyperreflexia, and autonomic instability. Both are rare diagnoses that can manifest as a paraneoplastic neurologic syndrome.

 

 

Blood tests showed a leukocyte count of 17,350/µL, neutrophils 8,720/µL (normal: 1,500–7,800), lymphocytes 6,130/µL, hemoglobin 11.3 g/dL, and platelets 231,000/µL. The basic metabolic panel was normal. Serum total protein was 6.7 g/dL with albumin 3.5 g/dL. Aspartate aminotransferase (AST) was 94 U/L (normal: 0-31), alanine aminotransferase (ALT) 56 U/L (normal: 0-31), alkaline phosphatase 45 U/L, and total bilirubin 1.1 mg/dL. Vitamin B12 was 868 pg/mL. Hemoglobin A1c and thyrotropin levels were normal. Creatine kinase was 3,757 U/L and lactate dehydrogenase (LDH) 435 U/L (normal: 122-220). The syphilis treponemal test and hepatitis B surface antigen were negative. HIV and hepatitis C antibodies were nonreactive. The anti-nuclear antibody screen was negative and complement C3 and C4 were normal.

Neutrophilia likely reflects glucocorticoid-induced demargination, as opposed to an infectious process, given the temporal association with steroid administration. Persistent mild lymphocytosis is nonspecific but more likely to reflect a reactive rather than a clonal process. Elevated LDH and CK, as well as a greater increase of AST relative to ALT, suggest muscle injury, although mild concomitant hepatic injury cannot be excluded. Normal or negative serum studies for TSH, HIV, ANA, peripheral blood smear, and creatinine eliminate many of the systemic causes of her pruritus, but malignancy and associated paraneoplastic etiologies remain considerations.

The initial work-up for SPS includes electromyography (EMG) which would show spontaneous muscle activity. Her poorly localized sensory abnormalities, transient vestibular symptoms, and confusion warrant an MRI of the brain and spine to evaluate for inflammation (eg, encephalomyelitis), which could be consistent with PERM.

An MRI of the brain and cervicothoracic spine without contrast was significantly limited by motion artifact but without obvious intracranial or cord signal abnormalities. Electromyography demonstrated spontaneous muscle activity in both lower extremities with co-contraction of agonist and antagonist muscles (hamstrings and quadriceps as well as medial gastrocnemius and tibialis anterior). Sensory and motor nerve conductions were normal. Cerebral spinal fluid (CSF) contained six leukocytes (96% lymphocytes) and three red blood cells per microliter; glucose was 67 mg/dL and protein 24 mg/dL. There were two oligoclonal bands unique to the CSF. Cytology was negative for malignant cells.

The EMG narrows the differential diagnosis considerably. Co-contraction of opposing flexor and extensor groups (with predominance of extensors) on EMG is a diagnostic criterion for SPS and explains the myalgia and elevated CK. Her normal MRI studies effectively ruled out any focal lesion and did not show signs of encephalitis. Oligoclonal bands in the CSF are a sensitive marker of intrathecal inflammation, although not specific to one diagnosis. The mildly elevated cell count also supports CNS inflammation. In the setting of a lymphocytic pleocytosis and unique oligoclonal bands, it is important to consider infectious, neoplastic, autoimmune, and paraneoplastic causes of neuroinflammatory disorders.

Serum analyses, including antiglutamic acid decarboxylase 65 (GAD65) antibody and anti-amphiphysin antibody, should be ordered. The anti-GAD65 antibody is most commonly elevated in the setting of autoimmune diabetes mellitus; the titer, however, is usually dramatically higher in SPS. The CSF titer of anti-GAD65 antibodies is more specific than the serum titer for SPS. Antibodies against amphiphysin are typically elevated in paraneoplastic SPS, and anti-glycine receptor antibodies are associated with PERM, which commonly does not have elevated anti-GAD65 antibodies.

 

 

The serum GAD65 antibody level was greater than 265,000 × 103 IU/µL (normal <5,000), and the CSF level was 11.2 nmol/L (normal: ≤0.02). Serum amphiphysin antibody testing was negative.

Significantly elevated serum and CSF anti-GAD65 antibody levels are highly suggestive of SPS. Stiff person syndrome with rapidly progressive clinical symptoms raises the concern of a paraneoplastic neurologic syndrome. Although anti-amphiphysin antibody – the antibody classically associated with breast cancer and SPS – was negative, anti-GAD65 antibody has been implicated in paraneoplastic SPS with thymoma, lymphoma, and thyroid carcinoma. Paraneoplastic neurologic syndrome can predate a detectable malignancy by several years. As SPS and lymphoma are associated with pruritus and lymphocytosis, imaging is indicated to search for malignancy. Antiglycine receptor antibody, associated with PERM, is not routinely available commercially.

Computed tomography of the chest, abdomen, and pelvis with intravenous contrast revealed a 3.9 × 8.0 × 7.0 cm anterior mediastinal mass (Figure 1, Panel A). Biopsy of the mass demonstrated a thymoma. Given that the patient exhibited no further signs of CNS involvement, her initial transiently altered mental status was attributed to opioids and steroids. As she did not meet the clinical criteria for PERM, testing of antiglycine antibodies was not pursued.

jhm014030181_f1.jpg

She received scheduled baclofen and diazepam with as needed cyclobenzaprine for continued muscle spasms. Over the next several days, her stiffness, spasms, and myoclonic jerks slowly improved, and she was able to attempt physical therapy (Appendix Video 1; https://youtu.be/d0gLpTgqaCs). She subsequently received intravenous immunoglobulin (IVIG) with further improvement. After five months of scheduled diazepam and baclofen, she was able to ambulate with minimal assistance (Appendix Video 2; https://youtu.be/I00i638u00o). Given the absence of safe tissue planes for resection, the patient received neoadjuvant chemotherapy with four cycles of cyclophosphamide, doxorubicin, and cisplatin. Tumor size decreased to 1.7 × 6.5 × 5.2 cm (Figure 1, Panel B), and she subsequently underwent resection (Figure 2). Pathological analysis demonstrated a type B1 thymoma.

jhm014030181_f2.jpg

COMMENTARY

SPS is a condition of muscle stiffness and spasticity. Diagnosis is difficult and often delayed due to its rarity, with an approximate prevalence of one to two cases per million people.1 SPS typically occurs in middle age, and women are diagnosed twice as often as men. Classic SPS is characterized by axial and limb muscle stiffness, episodic spasms precipitated by tactile or auditory stimuli, continuous motor unit activity in agonist and antagonist muscles on EMG, high-titer antibody to GAD65 or amphiphysin, and the absence of an alternate diagnosis.2 Variant syndromes have been described, including a milder variant limited to the limbs, a severe variant with brainstem and spinal cord involvement, and a paraneoplastic variant.3 This patient’s clinical presentation, EMG findings, and extraordinarily high anti-GAD titers in the serum and CSF were diagnostic of SPS.

The pathophysiology of SPS is associated with autoantibodies targeting proteins such as GAD65, amphiphysin, gephyrin, and GABAA receptor-associated protein (GABARAP). These proteins are critical to gamma-aminobutyric acid (GABA) signaling, the primary inhibitory neurotransmitter pathway in the CNS (Figure 3).4 The formation of GABA from glutamate is catalyzed by GAD65. Gamma-aminobutyric acid is loaded into secretory vesicles, and amphiphysin facilitates vesicle recycling from the synaptic space.5 In the postsynaptic neuron, GABA binds the GABAA receptor, leading to neuronal hyperpolarization and resistance to excitation. The GABAA receptor is clustered on the plasma membrane through a scaffold formed by gephyrin. GABARAP facilitates this clustering, in part by linking GABAA receptors and gephyrin.6 Autoantibodies to these proteins may be pathogenic; however, the direct effects on their targets are unclear. The end result is decreased GABAergic activity, leading to continuous activation of opposing muscle groups. The resulting stiffness is characteristic of this disorder. Colchicine is known to antagonize GABAA receptor signaling, and this may have brought the underlying diagnosis of SPS to clinical attention.7,8

jhm014030181_f3.jpg


Symptomatic treatment of SPS targets the GABAergic system. Typically, high doses of scheduled benzodiazepines9 and baclofen10 are necessary. When symptoms are not controlled by GABAergic drugs, immunosuppression with corticosteroids and IVIG has been used, as have plasmapheresis and rituximab.11 The efficacy of the latter, however, was not supported by a randomized, placebo-controlled trial.12 This patient experienced significant improvement with benzodiazepines, baclofen, IVIG, and neoadjuvant chemotherapy prior to thymoma resection. The pruritus, paresthesia, and lymphocytosis also resolved with medical therapy. Interestingly, GABA signaling suppresses itch, suggesting that loss of GABAA signaling may have contributed to the development of pruritus.

SPS occasionally occurs as a paraneoplastic neurologic syndrome. Breast cancer is the most commonly associated malignancy, although associations between thymomas and SPS13 with anti-GAD65 antibodies14 have also been described. The presentation of thymomas is variable, with approximately one-third discovered incidentally on imaging, one-third producing symptoms of local compression, and one-third identified in the setting of another syndrome, most commonly myasthenia gravis. In addition to myasthenia gravis, thymomas have been associated with conditions such as hypogammaglobulinemia, pure red cell aplasia, and agranulocytosis. Stiff person syndrome is a known, albeit infrequently associated, condition.15

A critical step in arriving at the relevant differential diagnosis requires correctly framing the patient’s case.16 The treatment team’s initial frame was “a 69-year-old woman with weakness and elevated CK,” which prioritized causes of weakness and myositis. Stiff person syndrome does not cause weakness, but rather impaired movement from marked stiffness and spasms. The patient’s elevated CK was a result of continual muscle contractions. The physical exam and lack of motor deficit on EMG led the treatment team to reframe as “a 69-year-old woman with severe stiffness and spasms.” Egad! This correct frame was the key to diagnosis and confirmed by EMG and GAD65 antibody testing.

 

 

KEY LEARNING POINTS

  • Classic SPS is characterized by axial and limb muscle stiffness, episodic spasms precipitated by tactile or auditory stimuli, continuous motor unit activity in agonist and antagonist muscles on EMG, and high-titer antibody to GAD65 or amphiphysin.
  • SPS typically occurs in middle age, and women are diagnosed twice as often as men.
  • Symptomatic treatment of SPS targets the GABAergic system. Typically, high doses of scheduled benzodiazepines and baclofenare necessary.
  • SPS occasionally occurs as a paraneoplastic neurologic syndrome, most commonly in association with breast cancer.

Acknowledgments

The authors wish to thank Jason Kern, MD for his preparation and interpretation of the pathologic image; and the Jeremiah A. Barondess Fellowship in the Clinical Transaction of the New York Academy of Medicine, in collaboration with the Accreditation Council for Graduate Medical Education, for supporting Reza Manesh, MD.

Disclosures

The authors have nothing to disclose.

Appendix Video 1: This video was taken during a physical therapy session after 1 week of scheduled benzodiazepine and 2 days of intravenous immunoglobulin. It was difficult for the patient to stand without assistance due to severe stiffness. (https://youtu.be/d0gLpTgqaCs)

Appendix Video 2: This video was taken 5 months after scheduled diazepam and baclofen, and 1 week prior to thymectomy. (https://youtu.be/I00i638u00o)

 

References

1. Hadavi S, Noyce AJ, Leslie RD, Giovannoni G. Stiff person syndrome. Pract Neurol. 2011;11(5):272-282. doi: 10.1136/practneurol-2011-000071. PubMed
2. Dalakas MC. Stiff person syndrome: advances in pathogenesis and therapeutic interventions. Curr Treat Options Neurol. 2009;11(2):102-110. doi: 10.1007/s11940-009-0013-9
PubMed
3. Murinson BB. Stiff-person syndrome. Neurologist. 2004;10(3):131-137. doi: 10.1097/01.nrl.0000126587.37087.1a
PubMed
4. Rakocevic G, Floeter MK. Autoimmune stiff person syndrome and related myelopathies: understanding of electrophysiological and immunological processes. Muscle Nerve. 2012;45(5):623-634. doi: 10.1002/mus.23234
PubMed
5. Zhang B, Zelhof AC. Amphiphysins: raising the BAR for synaptic vesicle recycling and membrane dynamics. Bin-Amphiphysin-Rvsp. Traffic. 2002;3(7):452-460. doi: 10.1034/j.1600-0854.2002.30702.x
PubMed
6. Tyagarajan SK, Fritschy JM. Gephyrin: a master regulator of neuronal function? Nat Rev Neurosci. 2014;15(3):141-156. doi: 10.1038/nrn3670
PubMed
7. Bueno OF, Leidenheimer NJ. Colchicine inhibits GABA(A) receptors independently of microtubule depolymerization. Neuropharmacology. 1998;37(3):383-390. doi: 10.1016/S0028-3908(98)00020-3
PubMed
8. Weiner JL, Buhler AV, Whatley VJ, Harris RA, Dunwiddie TV. Colchicine is a competitive antagonist at human recombinant γ-aminobutyric acidA receptors. J Pharmacol Exp Ther. 1998;284(1):95-102 . PubMed
9. Lorish TR, Thorsteinsson G, Howard FM Jr. Stiff-man syndrome updated. Mayo Clin Proc. 1989;64(6):629-636. doi: 10.1016/S0025-6196(12)65339-7
PubMed
10. McKeon A, Robinson MT, McEvoy KM, et al. Stiff-man syndrome and variants: clinical course, treatments, and outcomes. Arch Neurol. 2012;69(2):230-238. doi: 10.1001/archneurol.2011.991
PubMed
11. Dalakas MC, Li M, Fujii M, Jacobowitz DM. Stiff person syndrome: quantification, specificity, and intrathecal synthesis of GAD65 antibodies. Neurology. 2001;57(5):780-784. doi: 10.1212/WNL.57.5.780
PubMed
12. Dalakas MC, Rakocevic G, Dambrosia JM, Alexopoulos H, McElroy B. A double-blind, placebo-controlled study of rituximab in patients with stiff person syndrome. Ann Neurol. 2017;82(2):271-277. doi: 10.1002/ana.25002
PubMed
13. Hagiwara H, Enomoto-Nakatani S, Sakai K, et al. Stiff-person syndrome associated with invasive thymoma: a case report. J Neurol Sci. 2001;193(1):59-62. doi: 10.1016/S0022-510X(01)00602-5
PubMed
14. Vernino S, Lennon VA. Autoantibody profiles and neurological correlations of thymoma. Clin Cancer Res. 2004;10(21):7270-7275. doi: 10.1158/1078-0432.CCR-04-0735 PubMed
15. Thomas CR, Wright CD, Loehrer PJ. Thymoma: state of the art. J Clin Oncol. 1999;17(7):2280-2289. doi: 10.1200/JCO.1999.17.7.2280 PubMed
16. Stuart S, Hartig JR, Willett L. The importance of framing. J Gen Intern Med. 2017;32(6):706-710. doi: 10.1007/s11606-016-3964-z PubMed

References

1. Hadavi S, Noyce AJ, Leslie RD, Giovannoni G. Stiff person syndrome. Pract Neurol. 2011;11(5):272-282. doi: 10.1136/practneurol-2011-000071. PubMed
2. Dalakas MC. Stiff person syndrome: advances in pathogenesis and therapeutic interventions. Curr Treat Options Neurol. 2009;11(2):102-110. doi: 10.1007/s11940-009-0013-9
PubMed
3. Murinson BB. Stiff-person syndrome. Neurologist. 2004;10(3):131-137. doi: 10.1097/01.nrl.0000126587.37087.1a
PubMed
4. Rakocevic G, Floeter MK. Autoimmune stiff person syndrome and related myelopathies: understanding of electrophysiological and immunological processes. Muscle Nerve. 2012;45(5):623-634. doi: 10.1002/mus.23234
PubMed
5. Zhang B, Zelhof AC. Amphiphysins: raising the BAR for synaptic vesicle recycling and membrane dynamics. Bin-Amphiphysin-Rvsp. Traffic. 2002;3(7):452-460. doi: 10.1034/j.1600-0854.2002.30702.x
PubMed
6. Tyagarajan SK, Fritschy JM. Gephyrin: a master regulator of neuronal function? Nat Rev Neurosci. 2014;15(3):141-156. doi: 10.1038/nrn3670
PubMed
7. Bueno OF, Leidenheimer NJ. Colchicine inhibits GABA(A) receptors independently of microtubule depolymerization. Neuropharmacology. 1998;37(3):383-390. doi: 10.1016/S0028-3908(98)00020-3
PubMed
8. Weiner JL, Buhler AV, Whatley VJ, Harris RA, Dunwiddie TV. Colchicine is a competitive antagonist at human recombinant γ-aminobutyric acidA receptors. J Pharmacol Exp Ther. 1998;284(1):95-102 . PubMed
9. Lorish TR, Thorsteinsson G, Howard FM Jr. Stiff-man syndrome updated. Mayo Clin Proc. 1989;64(6):629-636. doi: 10.1016/S0025-6196(12)65339-7
PubMed
10. McKeon A, Robinson MT, McEvoy KM, et al. Stiff-man syndrome and variants: clinical course, treatments, and outcomes. Arch Neurol. 2012;69(2):230-238. doi: 10.1001/archneurol.2011.991
PubMed
11. Dalakas MC, Li M, Fujii M, Jacobowitz DM. Stiff person syndrome: quantification, specificity, and intrathecal synthesis of GAD65 antibodies. Neurology. 2001;57(5):780-784. doi: 10.1212/WNL.57.5.780
PubMed
12. Dalakas MC, Rakocevic G, Dambrosia JM, Alexopoulos H, McElroy B. A double-blind, placebo-controlled study of rituximab in patients with stiff person syndrome. Ann Neurol. 2017;82(2):271-277. doi: 10.1002/ana.25002
PubMed
13. Hagiwara H, Enomoto-Nakatani S, Sakai K, et al. Stiff-person syndrome associated with invasive thymoma: a case report. J Neurol Sci. 2001;193(1):59-62. doi: 10.1016/S0022-510X(01)00602-5
PubMed
14. Vernino S, Lennon VA. Autoantibody profiles and neurological correlations of thymoma. Clin Cancer Res. 2004;10(21):7270-7275. doi: 10.1158/1078-0432.CCR-04-0735 PubMed
15. Thomas CR, Wright CD, Loehrer PJ. Thymoma: state of the art. J Clin Oncol. 1999;17(7):2280-2289. doi: 10.1200/JCO.1999.17.7.2280 PubMed
16. Stuart S, Hartig JR, Willett L. The importance of framing. J Gen Intern Med. 2017;32(6):706-710. doi: 10.1007/s11606-016-3964-z PubMed

Issue
Journal of Hospital Medicine 14(3)
Issue
Journal of Hospital Medicine 14(3)
Page Number
181-185
Page Number
181-185
Publications
Publications
Topics
Article Type
Sections
Article Source

© 2019 Society of Hospital Medicine

Disallow All Ads
Correspondence Location
Reza Manesh, MD; E-mail: rsedigh1@jhmi.edu Telephone: 412-708-6944
Content Gating
Gated (full article locked unless allowed per User)
Alternative CME
Disqus Comments
Default
Use ProPublica
Hide sidebar & use full width
render the right sidebar.
Gating Strategy
First Peek Free
Article PDF Media

The Basement Flight

Article Type
Changed
Sun, 01/20/2019 - 16:13

A 14-year-old girl with a history of asthma presented to the Emergency Department (ED) with three months of persistent, nonproductive cough, and progressive shortness of breath. She reported fatigue, chest tightness, orthopnea, and dyspnea with exertion. She denied fever, rhinorrhea, congestion, hemoptysis, or paroxysmal nocturnal dyspnea.

Her age and past medical history of asthma are incongruent with her new symptoms, as asthma is typified by intermittent exacerbations, not progressive symptoms. Thus, another process, in addition to asthma, is most likely present; it is also important to question the accuracy of previous diagnoses in light of new information. Her symptoms may signify an underlying cardiopulmonary process, such as infiltrative diseases (eg, lymphoma or sarcoidosis), atypical infections, genetic conditions (eg, variant cystic fibrosis), autoimmune conditions, or cardiomyopathy. A detailed symptom history, family history, and careful physical examination will help expand and then refine the differential diagnosis. At this stage, typical infections are less likely.

She had presented two months prior with nonproductive cough and dyspnea. At that presentation, her temperature was 36.3°C, heart rate 110 beats per minute, blood pressure 119/63 mm Hg, respiratory rate 43 breaths per minute, and oxygen saturation 86% while breathing ambient air. A chest CT with contrast demonstrated diffuse patchy multifocal ground-glass opacities in the bilateral lungs as well as a mixture of atelectasis and lobular emphysema in the dependent lobes bilaterally (Figure 1). Her main pulmonary artery was dilated at 3.6 cm (mean of 2.42 cm with SD 0.22). She was diagnosed with atypical pneumonia. She was administered azithromycin, weaned off oxygen, and discharged after a seven-day hospitalization.

manesh02181128e_f1.jpg


Two months prior, she had marked tachypnea, tachycardia, and hypoxemia, and imaging revealed diffuse ground-glass opacities. The differential diagnosis for this constellation of symptoms is extensive and includes many conditions that have an inflammatory component, such as atypical pneumonia caused by Mycoplasma or Chlamydia pneumoniae or a common respiratory virus such as rhinovirus or human metapneumovirus. However, two findings make an acute pneumonia unlikely to be the sole cause of her symptoms: underlying emphysema and an enlarged pulmonary artery. Emphysema is an uncommon finding in children and can be related to congenital or acquired causes; congenital lobar emphysema most often presents earlier in life and is focal, not diffuse. Alpha-1-anti-trypin deficiency and mutations in connective tissue genes such as those encoding for elastin and fibrillin can lead to pulmonary disease. While not diagnostic of pulmonary hypertension, her dilated pulmonary artery, coupled with her history, makes pulmonary hypertension a strong possibility. While her pulmonary hypertension is most likely secondary to chronic lung disease based on the emphysematous changes on CT, it could still be related to a cardiac etiology.

The patient had a history of seasonal allergies and well-controlled asthma. She was hospitalized at age six for an asthma exacerbation associated with a respiratory infection. She was discharged with an albuterol inhaler, but seldom used it. Her parents denied any regular coughing during the day or night. She was morbidly obese. Her tonsils and adenoids were removed to treat obstructive sleep apnea (OSA) at age seven, and a subsequent polysomnography was normal. Her medications included intranasal fluticasone propionate and oral iron supplementation. She had no known allergies or recent travels. She had never smoked. She had two pet cats and a dog. Her mother had a history of obesity, OSA, and eczema. Her father had diabetes and eczema.

The patient’s history prior to the recent few months sheds little light on the cause of her current symptoms. While it is possible that her current symptoms are related to the worsening of a process that had been present for many years which mimicked asthma, this seems implausible given the long period of time between her last asthma exacerbation and her present symptoms. Similarly, while tonsillar and adenoidal hypertrophy can be associated with infiltrative diseases (such as lymphoma), this is less common than the usual (and normal) disproportionate increase in size of the adenoids compared to other airway structures during growth in children.

She was admitted to the hospital. On initial examination, her temperature was 37.4°C, heart rate 125 beats per minute, blood pressure 143/69 mm Hg, respiratory rate 48 breaths per minute, and oxygen saturation 86% breathing ambient air. Her BMI was 58 kg/m2. Her exam demonstrated increased work of breathing with accessory muscle use, and decreased breath sounds at the bases. There were no wheezes or crackles. Cardiovascular, abdominal, and skin exams were normal except for tachycardia. At rest, later in the hospitalization, her oxygen saturation was 97% breathing ambient air and heart rate 110 bpm. After two minutes of walking, her oxygen saturation was 77% and heart rate 132 bpm. Two minutes after resting, her oxygen saturation increased to 91%.

 

 

 

Her white blood cell count was 11.9 x 10 9 /L (67% neutrophils, 24.2% lymphocytes, 6% monocytes, and 2% eosinophils), hemoglobin 11.2 g/dL, and platelet count 278,000/mm 3 . Her complete metabolic panel was normal. The C-reactive protein (CRP) was 24 mg/L (normal range, < 4.9) and erythrocyte sedimentation rate (ESR) 103 mm/hour (normal range, 0-32). A venous blood gas (VBG) showed a pH of 7.42 and pCO2 39. An EKG demonstrated sinus tachycardia.

The combination of the patient’s tachypnea, hypoxemia, respiratory distress, and obesity is striking. Her lack of adventitious lung sounds is surprising given her CT findings, but the sensitivity of chest auscultation may be limited in obese patients. Her laboratory findings help narrow the diagnostic frame: she has mild anemia and leukocytosis along with significant inflammation. The normal CO2 concentration on VBG is concerning given the degree of her tachypnea and reflects significant alveolar hypoventilation.

This marked inflammation with diffuse lung findings again raises the possibility of an inflammatory or, less likely, infectious disorder. Sjogren’s syndrome, systemic lupus erythematosus (SLE), and juvenile dermatomyositis can present in young women with interstitial lung disease. She does have exposure to pets and hypersensitivity pneumonitis can worsen rapidly with continued exposure. Another possibility is that she has an underlying immunodeficiency such as common variable immunodeficiency, although a history of recurrent infections such as pneumonia, bacteremia, or sinusitis is lacking.

An echocardiogram should be performed. In addition, laboratory evaluation for the aforementioned autoimmune causes of interstitial lung disease, immunoglobulin levels, pulmonary function testing (if available as an inpatient), and potentially a bronchoscopy with bronchoalveolar lavage (BAL), and biopsy should be pursued. The BAL and biopsy would be helpful in evaluating for infection and interstitial lung disease in an expeditious manner.

A chest CT without contrast was done and compared to the scan from two months prior. New diffuse, ill-defined centrilobular ground-glass opacities were evident throughout the lung fields; dilation of the main pulmonary artery was unchanged, and previously seen ground-glass opacities had resolved. There were patchy areas of air-trapping and mosaic attenuation in the lower lobes (Figure 2).

manesh02181128e_f2.jpg

Transthoracic echocardiogram demonstrated a right ventricular systolic pressure of 58 mm Hg with flattened intraventricular septum during systole. Left and right ventricular systolic function were normal. The left ventricular diastolic function was normal. Pulmonary function testing demonstrated a FEV1/FVC ratio of 100 (112% predicted), FVC 1.07 L (35 % predicted) and FEV1 1.07 L (39% predicted), and total lung capacity was 2.7L (56% predicted) (Figure 3). Single-breath carbon monoxide uptake in the lung was not interpretable based on 2017 European Respiratory Society (ERS)/American Thoracic Society (ATS) technical standards.

manesh02181128e_f3.jpg


This information is helpful in classifying whether this patient’s primary condition is cardiac or pulmonary in nature. Her normal left ventricular systolic and diastolic function make a cardiac etiology for her pulmonary hypertension less likely. Further, the combination of pulmonary hypertension, a restrictive pattern on pulmonary function testing, and findings consistent with interstitial lung disease on cross-sectional imaging all suggest a primary pulmonary etiology rather than a cardiac, infectious, or thromboembolic condition. While chronic thromboembolic hypertension can result in nonspecific mosaic attenuation, it typically would not cause centrilobular ground-glass opacities nor restrictive lung disease. Thus, it seems most likely that this patient has a progressive pulmonary process resulting in hypoxia, pulmonary hypertension, centrilobular opacities, and lower-lobe mosaic attenuation. Considerations for this process can be broadly categorized as one of the childhood interstitial lung disease (chILD). While this differential diagnosis is broad, strong consideration should be given to hypersensitivity pneumonitis, chronic aspiration, sarcoidosis, and Sjogren’s syndrome. An intriguing possibility is that the patient’s “response to azithromycin” two months prior was due to the avoidance of an inhaled antigen while she was in the hospital; a detailed environmental history should be explored. The normal polysomnography after tonsilloadenoidectomy makes it unlikely that OSA is a major contributor to her current presentation. However, since the surgery was seven years ago, and her BMI is presently 58 kg/m2 she remains at risk for OSA and obesity-hypoventilation syndrome. Polysomnography should be done after her acute symptoms improve.

She was started on 5 mm Hg of continuous positive airway pressure (CPAP) at night after a sleep study on room air demonstrated severe OSA with a respiratory disturbance index of 13 events per hour. Antinuclear antibodies (ANA), anti-neutrophil cytoplasmic antibody (ANCA), anti-Jo-1 antibody, anti-RNP antibody, anti-Smith antibody, anti-Ro/SSA and anti-La/SSB antibody were negative as was the histoplasmin antibody. Serum angiotensin-converting enzyme (ACE) level was normal. Mycoplasma IgM and IgG were negative. IgE was 529 kU/L (normal range, <114).

This evaluation reduces the likelihood the patient has Sjogren’s syndrome, SLE, dermatomyositis, or ANCA-associated pulmonary disease. While many patients with dermatomyositis may have negative serologic evaluations, other findings usually present such as rash and myositis are lacking. The negative ANCA evaluation makes granulomatosis with polyangiitis and microscopic polyangiitis very unlikely given the high sensitivity of the ANCA assay for these conditions. ANCA assays are less sensitive for eosinophilic granulomatosis with polyangiitis (EGPA), but the lack of eosinophilia significantly decreases the likelihood of EGPA. ACE levels have relatively poor operating characteristics in the evaluation of sarcoidosis; however, sarcoidosis seems unlikely in this case, especially as patients with sarcoidosis tend to have low or normal IgE levels. Patients with asthma can have elevated IgE levels. However, very elevated IgE levels are more common in other conditions, including allergic bronchopulmonary aspergillosis (ABPA) and the Hyper-IgE syndrome. The latter manifests with recurrent infections and eczema, and is inherited in an autosomal dominant manner. However, both the Hyper-IgE syndrome and ABPA have much higher IgE levels than seen in this case. Allergen-specific IgE testing (including for antibodies to Aspergillus) should be sent. It seems that an interstitial lung disease is present; the waxing and waning pattern and clinical presentation, along with the lack of other systemic findings, make hypersensitivity pneumonitis most likely.

The family lived in an apartment building. Her symptoms started when the family’s neighbor recently moved his outdoor pigeon coop into his basement. The patient often smelled the pigeons and noted feathers coming through the holes in the wall.

One of the key diagnostic features of hypersensitivity pneumonitis (HP) is the history of exposure to a potential offending antigen—in this case likely bird feathers—along with worsening upon reexposure to that antigen. HP is primarily a clinical diagnosis, and testing for serum precipitants has limited value, given the high false negative rate and the frequent lack of clinical symptoms accompanying positive testing. Bronchoalveolar lavage fluid may reveal lymphocytosis and reduced CD4:CD8 ratio. Crackles are commonly heard on examination, but in this case were likely not auscultated due to her obese habitus. The most important treatment is withdrawal of the offending antigen. Limited data suggest that corticosteroid therapy may be helpful in certain HP cases, including subacute, chronic and severe cases as well as patients with hypoxemia, significant imaging findings, and those with significant abnormalities on pulmonary function testing (PFT).

A hypersensitivity pneumonitis precipitins panel was sent with positive antibodies to M. faeni, T. Vulgaris, A. Fumigatus 1 and 6, A. Flavus, and pigeon serum. Her symptoms gradually improved within five days of oral prednisone (60 mg). She was discharged home without dyspnea and normal oxygen saturation while breathing ambient air. A repeat echocardiogram after nighttime CPAP for 1 week demonstrated a right ventricular systolic pressure of 17 mm Hg consistent with improved pulmonary hypertension.

 

 

Three weeks later, she returned to clinic for follow up. She had re-experienced dyspnea, cough, and wheezing, which improved when she was outdoors. She was afebrile, tachypneic, tachycardic, and her oxygen saturation was 92% on ambient air.

Her steroid-responsive interstitial lung disease and rapid improvement upon avoidance of the offending antigen is consistent with HP. The positive serum precipitins assay lends further credence to the diagnosis of HP, although serologic analysis with such antibody assays is limited by false positives and false negatives; further, individuals exposed to pigeons often have antibodies present without evidence of HP. History taking at this visit should ask specifically about further pigeon exposure: were the pigeons removed from the home completely, were heating-cooling filters changed, carpets cleaned, and bedding laundered? An in-home evaluation may be helpful before conducting further diagnostic testing.

She was admitted for oxygen therapy and a bronchoscopy, which showed mucosal friability and cobblestoning, suggesting inflammation. BAL revealed a normal CD4:CD8 ratio of 3; BAL cultures were sterile. Her shortness of breath significantly improved following a prolonged course of systemic steroids and removal from the triggering environment. PFTs improved with a FEV1/FVC ratio of 94 (105% predicted), FVC of 2.00 L (66% predicted), FEV1 of 1.88L (69% predicted) (Figure 3B). Her presenting symptoms of persistent cough and progressive dyspnea on exertion, characteristic CT, sterile BAL cultures, positive serum precipitants against pigeon serum, and resolution of her symptoms with withdrawal of the offending antigen were diagnostic of hypersensitivity pneumonitis due to pigeon exposure, also known as bird fancier’s disease.

COMMENTARY

The patient’s original presentation of dyspnea, tachypnea, and hypoxia is commonly associated with pediatric pneumonia and asthma exacerbations.1 However, an alternative diagnosis was suggested by the lack of wheezing, absence of fever, and recurrent presentations with progressive symptoms.

Hypersensitivity pneumonitis (HP) represents an exaggerated T-cell meditated immune response to inhalation of an offending antigen that results in a restrictive ventilatory defect and interstitial infiltrates.2 Bird pneumonitis (also known as bird fancier’s disease) is a frequent cause of HP, accounting for approximately 65-70% of cases.3 HP, however, only manifests in a small number of subjects exposed to culprit antigens, suggesting an underlying genetic susceptibility.4 Prevalence estimates vary depending on bird species, county, climate, and other possible factors.

There are no standard criteria for the diagnosis of HP, though a combination of findings is suggestive. A recent prospective multicenter study created a scoring system for HP based on factors associated with the disease to aid in accurate diagnosis. The most relevant criteria included antigen exposure, recurrent symptoms noted within 4-8 hours after antigen exposure, weight loss, presence of specific IgG antibodies to avian antigens, and inspiratory crackles on exam. Using this rule, the probability that our patient has HP based on clinical characteristics was 93% with an area under the receiver operating curve of 0.93 (96% confidence interval: 0.90-0.95)5. Chest imaging (high resolution CT) often consists of a mosaic pattern of air trapping, as seen in this patient in combination with ground-glass opacities6. Bronchoalveolar lavage (BAL) is sensitive in detecting lung inflammation in a patient with suspected HP. On BAL, a lymphocytic alveolitis can be seen, but absence of this finding does not exclude HP.5,7,8 Pulmonary function tests (PFTs) may be normal in acute HP. When abnormal, PFTs may reveal a restrictive pattern and reduction in carbon monoxide diffusing capacity.7 However, BAL and PFT results are neither specific nor diagnostic of HP; it is important to consider results in the context of the clinical picture.

The respiratory response to inhalation of the avian antigen has traditionally been classified as acute, subacute, or chronic.9 The acute response occurs within hours of exposure to the offending agent and usually resolves within 24 hours after antigen withdrawal. The subacute presentation involves cough and dyspnea over several days to weeks, and can progress to chronic and permanent lung damage if unrecognized and untreated. In chronic presentations, lung abnormalities may persist despite antigen avoidance and pharmacologic interventions.4,10 The patient’s symptoms occurred over a six-month period which coincided with pigeon exposure and resolved during each hospitalization with steroid treatment and removal from the offending agent. Her presentation was consistent with a subacute time course of HP.

The dilated pulmonary artery, elevated right systolic ventricular pressure, and normal right ventricular function in our patient suggested pulmonary hypertension of chronic duration. Her risk factors for pulmonary hypertension included asthma, sleep apnea, possible obesity-hypoventilation syndrome, and HP-associated interstitial lung disease.11

The most important intervention in HP is avoidance of the causative antigen. Medical therapy without removal of antigen is inadequate. Systemic corticosteroids can help ameliorate acute symptoms though dosing and duration remains unclear. For chronic patients unresponsive to steroid therapy, lung transplantation can be considered.4

The key to diagnosis of HP in this patient—and to minimizing repeat testing upon the patient’s recrudescence of symptoms—was the clinician’s consideration that the major impetus for the patient’s improvement in the hospital was removal from the offending antigen in her home environment. As in this case, taking time to delve deeply into a patient’s environment—even by descending the basement stairs—may lead to the diagnosis.

 

 

LEARNING POINTS

  • Consider hypersensitivity pneumonitis (HP) in patients with recurrent respiratory distress, offending exposure, and resolution of symptoms with removal of culprit antigen.
  • The most important treatment of HP is removal of offending antigen; systemic and/or inhaled corticosteroids are indicated until the full resolution of respiratory symptoms.
  • Prognosis is dependent on early diagnosis and removal of offending exposures.
  • Failure to treat HP might result in end-stage lung disease from pulmonary fibrosis secondary to long-term inflammation.

Disclosures

Dr. Manesh is supported by the Jeremiah A. Barondess Fellowship in the Clinical Transaction of the New York Academy of Medicine, in collaboration with the Accreditation Council for Graduate Medical Education (ACGME). The authors declare no conflicts of interests.

 

References

1. Ebell MH. Clinical diagnosis of pneumonia in children. Am Fam Physician. 2010;82(2):192-193. PubMed
2. Cormier Y, Lacasse Y. Hypersensitivity pneumonitis and organic dust toxic syndrome. In: Malo J-L, Chan-Yeung M, Bernstein DI, eds. Asthma in the Workplace. Vol 32. Boca Raton, FL: Fourth Informa Healthcare; 2013:392-405. 
3. Chan AL, Juarez MM, Leslie KO, Ismail HA, Albertson TE. Bird fancier’s lung: a state-of-the-art review. Clin Rev Allergy Immunol. 2012;43(1-2):69-83. doi: 10.1007/s12016-011-8282-y. PubMed
4. Camarena A, Juárez A, Mejía M, et al. Major histocompatibility complex and tumor necrosis factor-α polymorphisms in pigeon breeder’s disease. Am J Respir Crit Care Med. 2001;163(7):1528-1533. https:/doi.org/10.1164/ajrccm.163.7.2004023. PubMed
5. Lacasse Y, Selman M, Costabel U, et al. Clinical diagnosis of hypersensitivity pneumonitis. Am J Respir Crit Care Med. 2003;168(8):952-958. doi: 10.1164/rccm.200301-137OC. PubMed
6. Glazer CS, Rose CS, Lynch DA. Clinical and radiologic manifestations of hypersensitivity pneumonitis. J Thorac Imaging. 2002;17(4):261-272. PubMed
7. Selman M, Pardo A, King TE Jr. Hypersensitivity pneumonitis: insights in diagnosis and pathobiology. Am J Respir Crit Care Med. 2012;186(4):314-324. doi: 10.1164/rccm.201203-0513CI. PubMed
8. Calillad DM, Vergnon, JM, Madroszyk A, et al. Bronchoalveolar lavage in hypersensitivity pneumonitis: a series of 139 patients. Inflamm Allergy Drug Targets. 2012;11(1):15-19. doi: 10.2174/187152812798889330. PubMed
9. Richerson HB, Bernstein IL, Fink JN, et al. Guidelines for the clinical evaluation of hypersensitivity pneumonitis. Report of the Subcommittee on Hypersensitivity Pneumonitis. J Allergy Clin Immunol. 1989;84(5 Pt 2):839-844. doi: 10.1016/0091-6749(89)90349-7. PubMed
10. Zacharisen MC, Schlueter DP, Kurup VP, Fink JN. The long-term outcome in acute, subacute, and chronic forms of pigeon breeder’s disease hypersensitivity pneumonitis. Ann Allergy Asthma Immunol. 2002;88(2):175-182. doi: 10.1016/S1081-1206(10)61993-X. PubMed
11. Raymond TE, Khabbaza JE, Yadav R, Tonelli AR. Significance of main pulmonary artery dilation on imaging studies. Ann Am Thorac Soc. 2014;11(10):1623-1632. doi: 10.1513/AnnalsATS.201406-253PP. PubMed

Article PDF
Issue
Journal of Hospital Medicine 14(1)
Publications
Topics
Page Number
51-55. Published online first November 28, 2018.
Sections
Article PDF
Article PDF

A 14-year-old girl with a history of asthma presented to the Emergency Department (ED) with three months of persistent, nonproductive cough, and progressive shortness of breath. She reported fatigue, chest tightness, orthopnea, and dyspnea with exertion. She denied fever, rhinorrhea, congestion, hemoptysis, or paroxysmal nocturnal dyspnea.

Her age and past medical history of asthma are incongruent with her new symptoms, as asthma is typified by intermittent exacerbations, not progressive symptoms. Thus, another process, in addition to asthma, is most likely present; it is also important to question the accuracy of previous diagnoses in light of new information. Her symptoms may signify an underlying cardiopulmonary process, such as infiltrative diseases (eg, lymphoma or sarcoidosis), atypical infections, genetic conditions (eg, variant cystic fibrosis), autoimmune conditions, or cardiomyopathy. A detailed symptom history, family history, and careful physical examination will help expand and then refine the differential diagnosis. At this stage, typical infections are less likely.

She had presented two months prior with nonproductive cough and dyspnea. At that presentation, her temperature was 36.3°C, heart rate 110 beats per minute, blood pressure 119/63 mm Hg, respiratory rate 43 breaths per minute, and oxygen saturation 86% while breathing ambient air. A chest CT with contrast demonstrated diffuse patchy multifocal ground-glass opacities in the bilateral lungs as well as a mixture of atelectasis and lobular emphysema in the dependent lobes bilaterally (Figure 1). Her main pulmonary artery was dilated at 3.6 cm (mean of 2.42 cm with SD 0.22). She was diagnosed with atypical pneumonia. She was administered azithromycin, weaned off oxygen, and discharged after a seven-day hospitalization.

manesh02181128e_f1.jpg


Two months prior, she had marked tachypnea, tachycardia, and hypoxemia, and imaging revealed diffuse ground-glass opacities. The differential diagnosis for this constellation of symptoms is extensive and includes many conditions that have an inflammatory component, such as atypical pneumonia caused by Mycoplasma or Chlamydia pneumoniae or a common respiratory virus such as rhinovirus or human metapneumovirus. However, two findings make an acute pneumonia unlikely to be the sole cause of her symptoms: underlying emphysema and an enlarged pulmonary artery. Emphysema is an uncommon finding in children and can be related to congenital or acquired causes; congenital lobar emphysema most often presents earlier in life and is focal, not diffuse. Alpha-1-anti-trypin deficiency and mutations in connective tissue genes such as those encoding for elastin and fibrillin can lead to pulmonary disease. While not diagnostic of pulmonary hypertension, her dilated pulmonary artery, coupled with her history, makes pulmonary hypertension a strong possibility. While her pulmonary hypertension is most likely secondary to chronic lung disease based on the emphysematous changes on CT, it could still be related to a cardiac etiology.

The patient had a history of seasonal allergies and well-controlled asthma. She was hospitalized at age six for an asthma exacerbation associated with a respiratory infection. She was discharged with an albuterol inhaler, but seldom used it. Her parents denied any regular coughing during the day or night. She was morbidly obese. Her tonsils and adenoids were removed to treat obstructive sleep apnea (OSA) at age seven, and a subsequent polysomnography was normal. Her medications included intranasal fluticasone propionate and oral iron supplementation. She had no known allergies or recent travels. She had never smoked. She had two pet cats and a dog. Her mother had a history of obesity, OSA, and eczema. Her father had diabetes and eczema.

The patient’s history prior to the recent few months sheds little light on the cause of her current symptoms. While it is possible that her current symptoms are related to the worsening of a process that had been present for many years which mimicked asthma, this seems implausible given the long period of time between her last asthma exacerbation and her present symptoms. Similarly, while tonsillar and adenoidal hypertrophy can be associated with infiltrative diseases (such as lymphoma), this is less common than the usual (and normal) disproportionate increase in size of the adenoids compared to other airway structures during growth in children.

She was admitted to the hospital. On initial examination, her temperature was 37.4°C, heart rate 125 beats per minute, blood pressure 143/69 mm Hg, respiratory rate 48 breaths per minute, and oxygen saturation 86% breathing ambient air. Her BMI was 58 kg/m2. Her exam demonstrated increased work of breathing with accessory muscle use, and decreased breath sounds at the bases. There were no wheezes or crackles. Cardiovascular, abdominal, and skin exams were normal except for tachycardia. At rest, later in the hospitalization, her oxygen saturation was 97% breathing ambient air and heart rate 110 bpm. After two minutes of walking, her oxygen saturation was 77% and heart rate 132 bpm. Two minutes after resting, her oxygen saturation increased to 91%.

 

 

 

Her white blood cell count was 11.9 x 10 9 /L (67% neutrophils, 24.2% lymphocytes, 6% monocytes, and 2% eosinophils), hemoglobin 11.2 g/dL, and platelet count 278,000/mm 3 . Her complete metabolic panel was normal. The C-reactive protein (CRP) was 24 mg/L (normal range, < 4.9) and erythrocyte sedimentation rate (ESR) 103 mm/hour (normal range, 0-32). A venous blood gas (VBG) showed a pH of 7.42 and pCO2 39. An EKG demonstrated sinus tachycardia.

The combination of the patient’s tachypnea, hypoxemia, respiratory distress, and obesity is striking. Her lack of adventitious lung sounds is surprising given her CT findings, but the sensitivity of chest auscultation may be limited in obese patients. Her laboratory findings help narrow the diagnostic frame: she has mild anemia and leukocytosis along with significant inflammation. The normal CO2 concentration on VBG is concerning given the degree of her tachypnea and reflects significant alveolar hypoventilation.

This marked inflammation with diffuse lung findings again raises the possibility of an inflammatory or, less likely, infectious disorder. Sjogren’s syndrome, systemic lupus erythematosus (SLE), and juvenile dermatomyositis can present in young women with interstitial lung disease. She does have exposure to pets and hypersensitivity pneumonitis can worsen rapidly with continued exposure. Another possibility is that she has an underlying immunodeficiency such as common variable immunodeficiency, although a history of recurrent infections such as pneumonia, bacteremia, or sinusitis is lacking.

An echocardiogram should be performed. In addition, laboratory evaluation for the aforementioned autoimmune causes of interstitial lung disease, immunoglobulin levels, pulmonary function testing (if available as an inpatient), and potentially a bronchoscopy with bronchoalveolar lavage (BAL), and biopsy should be pursued. The BAL and biopsy would be helpful in evaluating for infection and interstitial lung disease in an expeditious manner.

A chest CT without contrast was done and compared to the scan from two months prior. New diffuse, ill-defined centrilobular ground-glass opacities were evident throughout the lung fields; dilation of the main pulmonary artery was unchanged, and previously seen ground-glass opacities had resolved. There were patchy areas of air-trapping and mosaic attenuation in the lower lobes (Figure 2).

manesh02181128e_f2.jpg

Transthoracic echocardiogram demonstrated a right ventricular systolic pressure of 58 mm Hg with flattened intraventricular septum during systole. Left and right ventricular systolic function were normal. The left ventricular diastolic function was normal. Pulmonary function testing demonstrated a FEV1/FVC ratio of 100 (112% predicted), FVC 1.07 L (35 % predicted) and FEV1 1.07 L (39% predicted), and total lung capacity was 2.7L (56% predicted) (Figure 3). Single-breath carbon monoxide uptake in the lung was not interpretable based on 2017 European Respiratory Society (ERS)/American Thoracic Society (ATS) technical standards.

manesh02181128e_f3.jpg


This information is helpful in classifying whether this patient’s primary condition is cardiac or pulmonary in nature. Her normal left ventricular systolic and diastolic function make a cardiac etiology for her pulmonary hypertension less likely. Further, the combination of pulmonary hypertension, a restrictive pattern on pulmonary function testing, and findings consistent with interstitial lung disease on cross-sectional imaging all suggest a primary pulmonary etiology rather than a cardiac, infectious, or thromboembolic condition. While chronic thromboembolic hypertension can result in nonspecific mosaic attenuation, it typically would not cause centrilobular ground-glass opacities nor restrictive lung disease. Thus, it seems most likely that this patient has a progressive pulmonary process resulting in hypoxia, pulmonary hypertension, centrilobular opacities, and lower-lobe mosaic attenuation. Considerations for this process can be broadly categorized as one of the childhood interstitial lung disease (chILD). While this differential diagnosis is broad, strong consideration should be given to hypersensitivity pneumonitis, chronic aspiration, sarcoidosis, and Sjogren’s syndrome. An intriguing possibility is that the patient’s “response to azithromycin” two months prior was due to the avoidance of an inhaled antigen while she was in the hospital; a detailed environmental history should be explored. The normal polysomnography after tonsilloadenoidectomy makes it unlikely that OSA is a major contributor to her current presentation. However, since the surgery was seven years ago, and her BMI is presently 58 kg/m2 she remains at risk for OSA and obesity-hypoventilation syndrome. Polysomnography should be done after her acute symptoms improve.

She was started on 5 mm Hg of continuous positive airway pressure (CPAP) at night after a sleep study on room air demonstrated severe OSA with a respiratory disturbance index of 13 events per hour. Antinuclear antibodies (ANA), anti-neutrophil cytoplasmic antibody (ANCA), anti-Jo-1 antibody, anti-RNP antibody, anti-Smith antibody, anti-Ro/SSA and anti-La/SSB antibody were negative as was the histoplasmin antibody. Serum angiotensin-converting enzyme (ACE) level was normal. Mycoplasma IgM and IgG were negative. IgE was 529 kU/L (normal range, <114).

This evaluation reduces the likelihood the patient has Sjogren’s syndrome, SLE, dermatomyositis, or ANCA-associated pulmonary disease. While many patients with dermatomyositis may have negative serologic evaluations, other findings usually present such as rash and myositis are lacking. The negative ANCA evaluation makes granulomatosis with polyangiitis and microscopic polyangiitis very unlikely given the high sensitivity of the ANCA assay for these conditions. ANCA assays are less sensitive for eosinophilic granulomatosis with polyangiitis (EGPA), but the lack of eosinophilia significantly decreases the likelihood of EGPA. ACE levels have relatively poor operating characteristics in the evaluation of sarcoidosis; however, sarcoidosis seems unlikely in this case, especially as patients with sarcoidosis tend to have low or normal IgE levels. Patients with asthma can have elevated IgE levels. However, very elevated IgE levels are more common in other conditions, including allergic bronchopulmonary aspergillosis (ABPA) and the Hyper-IgE syndrome. The latter manifests with recurrent infections and eczema, and is inherited in an autosomal dominant manner. However, both the Hyper-IgE syndrome and ABPA have much higher IgE levels than seen in this case. Allergen-specific IgE testing (including for antibodies to Aspergillus) should be sent. It seems that an interstitial lung disease is present; the waxing and waning pattern and clinical presentation, along with the lack of other systemic findings, make hypersensitivity pneumonitis most likely.

The family lived in an apartment building. Her symptoms started when the family’s neighbor recently moved his outdoor pigeon coop into his basement. The patient often smelled the pigeons and noted feathers coming through the holes in the wall.

One of the key diagnostic features of hypersensitivity pneumonitis (HP) is the history of exposure to a potential offending antigen—in this case likely bird feathers—along with worsening upon reexposure to that antigen. HP is primarily a clinical diagnosis, and testing for serum precipitants has limited value, given the high false negative rate and the frequent lack of clinical symptoms accompanying positive testing. Bronchoalveolar lavage fluid may reveal lymphocytosis and reduced CD4:CD8 ratio. Crackles are commonly heard on examination, but in this case were likely not auscultated due to her obese habitus. The most important treatment is withdrawal of the offending antigen. Limited data suggest that corticosteroid therapy may be helpful in certain HP cases, including subacute, chronic and severe cases as well as patients with hypoxemia, significant imaging findings, and those with significant abnormalities on pulmonary function testing (PFT).

A hypersensitivity pneumonitis precipitins panel was sent with positive antibodies to M. faeni, T. Vulgaris, A. Fumigatus 1 and 6, A. Flavus, and pigeon serum. Her symptoms gradually improved within five days of oral prednisone (60 mg). She was discharged home without dyspnea and normal oxygen saturation while breathing ambient air. A repeat echocardiogram after nighttime CPAP for 1 week demonstrated a right ventricular systolic pressure of 17 mm Hg consistent with improved pulmonary hypertension.

 

 

Three weeks later, she returned to clinic for follow up. She had re-experienced dyspnea, cough, and wheezing, which improved when she was outdoors. She was afebrile, tachypneic, tachycardic, and her oxygen saturation was 92% on ambient air.

Her steroid-responsive interstitial lung disease and rapid improvement upon avoidance of the offending antigen is consistent with HP. The positive serum precipitins assay lends further credence to the diagnosis of HP, although serologic analysis with such antibody assays is limited by false positives and false negatives; further, individuals exposed to pigeons often have antibodies present without evidence of HP. History taking at this visit should ask specifically about further pigeon exposure: were the pigeons removed from the home completely, were heating-cooling filters changed, carpets cleaned, and bedding laundered? An in-home evaluation may be helpful before conducting further diagnostic testing.

She was admitted for oxygen therapy and a bronchoscopy, which showed mucosal friability and cobblestoning, suggesting inflammation. BAL revealed a normal CD4:CD8 ratio of 3; BAL cultures were sterile. Her shortness of breath significantly improved following a prolonged course of systemic steroids and removal from the triggering environment. PFTs improved with a FEV1/FVC ratio of 94 (105% predicted), FVC of 2.00 L (66% predicted), FEV1 of 1.88L (69% predicted) (Figure 3B). Her presenting symptoms of persistent cough and progressive dyspnea on exertion, characteristic CT, sterile BAL cultures, positive serum precipitants against pigeon serum, and resolution of her symptoms with withdrawal of the offending antigen were diagnostic of hypersensitivity pneumonitis due to pigeon exposure, also known as bird fancier’s disease.

COMMENTARY

The patient’s original presentation of dyspnea, tachypnea, and hypoxia is commonly associated with pediatric pneumonia and asthma exacerbations.1 However, an alternative diagnosis was suggested by the lack of wheezing, absence of fever, and recurrent presentations with progressive symptoms.

Hypersensitivity pneumonitis (HP) represents an exaggerated T-cell meditated immune response to inhalation of an offending antigen that results in a restrictive ventilatory defect and interstitial infiltrates.2 Bird pneumonitis (also known as bird fancier’s disease) is a frequent cause of HP, accounting for approximately 65-70% of cases.3 HP, however, only manifests in a small number of subjects exposed to culprit antigens, suggesting an underlying genetic susceptibility.4 Prevalence estimates vary depending on bird species, county, climate, and other possible factors.

There are no standard criteria for the diagnosis of HP, though a combination of findings is suggestive. A recent prospective multicenter study created a scoring system for HP based on factors associated with the disease to aid in accurate diagnosis. The most relevant criteria included antigen exposure, recurrent symptoms noted within 4-8 hours after antigen exposure, weight loss, presence of specific IgG antibodies to avian antigens, and inspiratory crackles on exam. Using this rule, the probability that our patient has HP based on clinical characteristics was 93% with an area under the receiver operating curve of 0.93 (96% confidence interval: 0.90-0.95)5. Chest imaging (high resolution CT) often consists of a mosaic pattern of air trapping, as seen in this patient in combination with ground-glass opacities6. Bronchoalveolar lavage (BAL) is sensitive in detecting lung inflammation in a patient with suspected HP. On BAL, a lymphocytic alveolitis can be seen, but absence of this finding does not exclude HP.5,7,8 Pulmonary function tests (PFTs) may be normal in acute HP. When abnormal, PFTs may reveal a restrictive pattern and reduction in carbon monoxide diffusing capacity.7 However, BAL and PFT results are neither specific nor diagnostic of HP; it is important to consider results in the context of the clinical picture.

The respiratory response to inhalation of the avian antigen has traditionally been classified as acute, subacute, or chronic.9 The acute response occurs within hours of exposure to the offending agent and usually resolves within 24 hours after antigen withdrawal. The subacute presentation involves cough and dyspnea over several days to weeks, and can progress to chronic and permanent lung damage if unrecognized and untreated. In chronic presentations, lung abnormalities may persist despite antigen avoidance and pharmacologic interventions.4,10 The patient’s symptoms occurred over a six-month period which coincided with pigeon exposure and resolved during each hospitalization with steroid treatment and removal from the offending agent. Her presentation was consistent with a subacute time course of HP.

The dilated pulmonary artery, elevated right systolic ventricular pressure, and normal right ventricular function in our patient suggested pulmonary hypertension of chronic duration. Her risk factors for pulmonary hypertension included asthma, sleep apnea, possible obesity-hypoventilation syndrome, and HP-associated interstitial lung disease.11

The most important intervention in HP is avoidance of the causative antigen. Medical therapy without removal of antigen is inadequate. Systemic corticosteroids can help ameliorate acute symptoms though dosing and duration remains unclear. For chronic patients unresponsive to steroid therapy, lung transplantation can be considered.4

The key to diagnosis of HP in this patient—and to minimizing repeat testing upon the patient’s recrudescence of symptoms—was the clinician’s consideration that the major impetus for the patient’s improvement in the hospital was removal from the offending antigen in her home environment. As in this case, taking time to delve deeply into a patient’s environment—even by descending the basement stairs—may lead to the diagnosis.

 

 

LEARNING POINTS

  • Consider hypersensitivity pneumonitis (HP) in patients with recurrent respiratory distress, offending exposure, and resolution of symptoms with removal of culprit antigen.
  • The most important treatment of HP is removal of offending antigen; systemic and/or inhaled corticosteroids are indicated until the full resolution of respiratory symptoms.
  • Prognosis is dependent on early diagnosis and removal of offending exposures.
  • Failure to treat HP might result in end-stage lung disease from pulmonary fibrosis secondary to long-term inflammation.

Disclosures

Dr. Manesh is supported by the Jeremiah A. Barondess Fellowship in the Clinical Transaction of the New York Academy of Medicine, in collaboration with the Accreditation Council for Graduate Medical Education (ACGME). The authors declare no conflicts of interests.

 

A 14-year-old girl with a history of asthma presented to the Emergency Department (ED) with three months of persistent, nonproductive cough, and progressive shortness of breath. She reported fatigue, chest tightness, orthopnea, and dyspnea with exertion. She denied fever, rhinorrhea, congestion, hemoptysis, or paroxysmal nocturnal dyspnea.

Her age and past medical history of asthma are incongruent with her new symptoms, as asthma is typified by intermittent exacerbations, not progressive symptoms. Thus, another process, in addition to asthma, is most likely present; it is also important to question the accuracy of previous diagnoses in light of new information. Her symptoms may signify an underlying cardiopulmonary process, such as infiltrative diseases (eg, lymphoma or sarcoidosis), atypical infections, genetic conditions (eg, variant cystic fibrosis), autoimmune conditions, or cardiomyopathy. A detailed symptom history, family history, and careful physical examination will help expand and then refine the differential diagnosis. At this stage, typical infections are less likely.

She had presented two months prior with nonproductive cough and dyspnea. At that presentation, her temperature was 36.3°C, heart rate 110 beats per minute, blood pressure 119/63 mm Hg, respiratory rate 43 breaths per minute, and oxygen saturation 86% while breathing ambient air. A chest CT with contrast demonstrated diffuse patchy multifocal ground-glass opacities in the bilateral lungs as well as a mixture of atelectasis and lobular emphysema in the dependent lobes bilaterally (Figure 1). Her main pulmonary artery was dilated at 3.6 cm (mean of 2.42 cm with SD 0.22). She was diagnosed with atypical pneumonia. She was administered azithromycin, weaned off oxygen, and discharged after a seven-day hospitalization.

manesh02181128e_f1.jpg


Two months prior, she had marked tachypnea, tachycardia, and hypoxemia, and imaging revealed diffuse ground-glass opacities. The differential diagnosis for this constellation of symptoms is extensive and includes many conditions that have an inflammatory component, such as atypical pneumonia caused by Mycoplasma or Chlamydia pneumoniae or a common respiratory virus such as rhinovirus or human metapneumovirus. However, two findings make an acute pneumonia unlikely to be the sole cause of her symptoms: underlying emphysema and an enlarged pulmonary artery. Emphysema is an uncommon finding in children and can be related to congenital or acquired causes; congenital lobar emphysema most often presents earlier in life and is focal, not diffuse. Alpha-1-anti-trypin deficiency and mutations in connective tissue genes such as those encoding for elastin and fibrillin can lead to pulmonary disease. While not diagnostic of pulmonary hypertension, her dilated pulmonary artery, coupled with her history, makes pulmonary hypertension a strong possibility. While her pulmonary hypertension is most likely secondary to chronic lung disease based on the emphysematous changes on CT, it could still be related to a cardiac etiology.

The patient had a history of seasonal allergies and well-controlled asthma. She was hospitalized at age six for an asthma exacerbation associated with a respiratory infection. She was discharged with an albuterol inhaler, but seldom used it. Her parents denied any regular coughing during the day or night. She was morbidly obese. Her tonsils and adenoids were removed to treat obstructive sleep apnea (OSA) at age seven, and a subsequent polysomnography was normal. Her medications included intranasal fluticasone propionate and oral iron supplementation. She had no known allergies or recent travels. She had never smoked. She had two pet cats and a dog. Her mother had a history of obesity, OSA, and eczema. Her father had diabetes and eczema.

The patient’s history prior to the recent few months sheds little light on the cause of her current symptoms. While it is possible that her current symptoms are related to the worsening of a process that had been present for many years which mimicked asthma, this seems implausible given the long period of time between her last asthma exacerbation and her present symptoms. Similarly, while tonsillar and adenoidal hypertrophy can be associated with infiltrative diseases (such as lymphoma), this is less common than the usual (and normal) disproportionate increase in size of the adenoids compared to other airway structures during growth in children.

She was admitted to the hospital. On initial examination, her temperature was 37.4°C, heart rate 125 beats per minute, blood pressure 143/69 mm Hg, respiratory rate 48 breaths per minute, and oxygen saturation 86% breathing ambient air. Her BMI was 58 kg/m2. Her exam demonstrated increased work of breathing with accessory muscle use, and decreased breath sounds at the bases. There were no wheezes or crackles. Cardiovascular, abdominal, and skin exams were normal except for tachycardia. At rest, later in the hospitalization, her oxygen saturation was 97% breathing ambient air and heart rate 110 bpm. After two minutes of walking, her oxygen saturation was 77% and heart rate 132 bpm. Two minutes after resting, her oxygen saturation increased to 91%.

 

 

 

Her white blood cell count was 11.9 x 10 9 /L (67% neutrophils, 24.2% lymphocytes, 6% monocytes, and 2% eosinophils), hemoglobin 11.2 g/dL, and platelet count 278,000/mm 3 . Her complete metabolic panel was normal. The C-reactive protein (CRP) was 24 mg/L (normal range, < 4.9) and erythrocyte sedimentation rate (ESR) 103 mm/hour (normal range, 0-32). A venous blood gas (VBG) showed a pH of 7.42 and pCO2 39. An EKG demonstrated sinus tachycardia.

The combination of the patient’s tachypnea, hypoxemia, respiratory distress, and obesity is striking. Her lack of adventitious lung sounds is surprising given her CT findings, but the sensitivity of chest auscultation may be limited in obese patients. Her laboratory findings help narrow the diagnostic frame: she has mild anemia and leukocytosis along with significant inflammation. The normal CO2 concentration on VBG is concerning given the degree of her tachypnea and reflects significant alveolar hypoventilation.

This marked inflammation with diffuse lung findings again raises the possibility of an inflammatory or, less likely, infectious disorder. Sjogren’s syndrome, systemic lupus erythematosus (SLE), and juvenile dermatomyositis can present in young women with interstitial lung disease. She does have exposure to pets and hypersensitivity pneumonitis can worsen rapidly with continued exposure. Another possibility is that she has an underlying immunodeficiency such as common variable immunodeficiency, although a history of recurrent infections such as pneumonia, bacteremia, or sinusitis is lacking.

An echocardiogram should be performed. In addition, laboratory evaluation for the aforementioned autoimmune causes of interstitial lung disease, immunoglobulin levels, pulmonary function testing (if available as an inpatient), and potentially a bronchoscopy with bronchoalveolar lavage (BAL), and biopsy should be pursued. The BAL and biopsy would be helpful in evaluating for infection and interstitial lung disease in an expeditious manner.

A chest CT without contrast was done and compared to the scan from two months prior. New diffuse, ill-defined centrilobular ground-glass opacities were evident throughout the lung fields; dilation of the main pulmonary artery was unchanged, and previously seen ground-glass opacities had resolved. There were patchy areas of air-trapping and mosaic attenuation in the lower lobes (Figure 2).

manesh02181128e_f2.jpg

Transthoracic echocardiogram demonstrated a right ventricular systolic pressure of 58 mm Hg with flattened intraventricular septum during systole. Left and right ventricular systolic function were normal. The left ventricular diastolic function was normal. Pulmonary function testing demonstrated a FEV1/FVC ratio of 100 (112% predicted), FVC 1.07 L (35 % predicted) and FEV1 1.07 L (39% predicted), and total lung capacity was 2.7L (56% predicted) (Figure 3). Single-breath carbon monoxide uptake in the lung was not interpretable based on 2017 European Respiratory Society (ERS)/American Thoracic Society (ATS) technical standards.

manesh02181128e_f3.jpg


This information is helpful in classifying whether this patient’s primary condition is cardiac or pulmonary in nature. Her normal left ventricular systolic and diastolic function make a cardiac etiology for her pulmonary hypertension less likely. Further, the combination of pulmonary hypertension, a restrictive pattern on pulmonary function testing, and findings consistent with interstitial lung disease on cross-sectional imaging all suggest a primary pulmonary etiology rather than a cardiac, infectious, or thromboembolic condition. While chronic thromboembolic hypertension can result in nonspecific mosaic attenuation, it typically would not cause centrilobular ground-glass opacities nor restrictive lung disease. Thus, it seems most likely that this patient has a progressive pulmonary process resulting in hypoxia, pulmonary hypertension, centrilobular opacities, and lower-lobe mosaic attenuation. Considerations for this process can be broadly categorized as one of the childhood interstitial lung disease (chILD). While this differential diagnosis is broad, strong consideration should be given to hypersensitivity pneumonitis, chronic aspiration, sarcoidosis, and Sjogren’s syndrome. An intriguing possibility is that the patient’s “response to azithromycin” two months prior was due to the avoidance of an inhaled antigen while she was in the hospital; a detailed environmental history should be explored. The normal polysomnography after tonsilloadenoidectomy makes it unlikely that OSA is a major contributor to her current presentation. However, since the surgery was seven years ago, and her BMI is presently 58 kg/m2 she remains at risk for OSA and obesity-hypoventilation syndrome. Polysomnography should be done after her acute symptoms improve.

She was started on 5 mm Hg of continuous positive airway pressure (CPAP) at night after a sleep study on room air demonstrated severe OSA with a respiratory disturbance index of 13 events per hour. Antinuclear antibodies (ANA), anti-neutrophil cytoplasmic antibody (ANCA), anti-Jo-1 antibody, anti-RNP antibody, anti-Smith antibody, anti-Ro/SSA and anti-La/SSB antibody were negative as was the histoplasmin antibody. Serum angiotensin-converting enzyme (ACE) level was normal. Mycoplasma IgM and IgG were negative. IgE was 529 kU/L (normal range, <114).

This evaluation reduces the likelihood the patient has Sjogren’s syndrome, SLE, dermatomyositis, or ANCA-associated pulmonary disease. While many patients with dermatomyositis may have negative serologic evaluations, other findings usually present such as rash and myositis are lacking. The negative ANCA evaluation makes granulomatosis with polyangiitis and microscopic polyangiitis very unlikely given the high sensitivity of the ANCA assay for these conditions. ANCA assays are less sensitive for eosinophilic granulomatosis with polyangiitis (EGPA), but the lack of eosinophilia significantly decreases the likelihood of EGPA. ACE levels have relatively poor operating characteristics in the evaluation of sarcoidosis; however, sarcoidosis seems unlikely in this case, especially as patients with sarcoidosis tend to have low or normal IgE levels. Patients with asthma can have elevated IgE levels. However, very elevated IgE levels are more common in other conditions, including allergic bronchopulmonary aspergillosis (ABPA) and the Hyper-IgE syndrome. The latter manifests with recurrent infections and eczema, and is inherited in an autosomal dominant manner. However, both the Hyper-IgE syndrome and ABPA have much higher IgE levels than seen in this case. Allergen-specific IgE testing (including for antibodies to Aspergillus) should be sent. It seems that an interstitial lung disease is present; the waxing and waning pattern and clinical presentation, along with the lack of other systemic findings, make hypersensitivity pneumonitis most likely.

The family lived in an apartment building. Her symptoms started when the family’s neighbor recently moved his outdoor pigeon coop into his basement. The patient often smelled the pigeons and noted feathers coming through the holes in the wall.

One of the key diagnostic features of hypersensitivity pneumonitis (HP) is the history of exposure to a potential offending antigen—in this case likely bird feathers—along with worsening upon reexposure to that antigen. HP is primarily a clinical diagnosis, and testing for serum precipitants has limited value, given the high false negative rate and the frequent lack of clinical symptoms accompanying positive testing. Bronchoalveolar lavage fluid may reveal lymphocytosis and reduced CD4:CD8 ratio. Crackles are commonly heard on examination, but in this case were likely not auscultated due to her obese habitus. The most important treatment is withdrawal of the offending antigen. Limited data suggest that corticosteroid therapy may be helpful in certain HP cases, including subacute, chronic and severe cases as well as patients with hypoxemia, significant imaging findings, and those with significant abnormalities on pulmonary function testing (PFT).

A hypersensitivity pneumonitis precipitins panel was sent with positive antibodies to M. faeni, T. Vulgaris, A. Fumigatus 1 and 6, A. Flavus, and pigeon serum. Her symptoms gradually improved within five days of oral prednisone (60 mg). She was discharged home without dyspnea and normal oxygen saturation while breathing ambient air. A repeat echocardiogram after nighttime CPAP for 1 week demonstrated a right ventricular systolic pressure of 17 mm Hg consistent with improved pulmonary hypertension.

 

 

Three weeks later, she returned to clinic for follow up. She had re-experienced dyspnea, cough, and wheezing, which improved when she was outdoors. She was afebrile, tachypneic, tachycardic, and her oxygen saturation was 92% on ambient air.

Her steroid-responsive interstitial lung disease and rapid improvement upon avoidance of the offending antigen is consistent with HP. The positive serum precipitins assay lends further credence to the diagnosis of HP, although serologic analysis with such antibody assays is limited by false positives and false negatives; further, individuals exposed to pigeons often have antibodies present without evidence of HP. History taking at this visit should ask specifically about further pigeon exposure: were the pigeons removed from the home completely, were heating-cooling filters changed, carpets cleaned, and bedding laundered? An in-home evaluation may be helpful before conducting further diagnostic testing.

She was admitted for oxygen therapy and a bronchoscopy, which showed mucosal friability and cobblestoning, suggesting inflammation. BAL revealed a normal CD4:CD8 ratio of 3; BAL cultures were sterile. Her shortness of breath significantly improved following a prolonged course of systemic steroids and removal from the triggering environment. PFTs improved with a FEV1/FVC ratio of 94 (105% predicted), FVC of 2.00 L (66% predicted), FEV1 of 1.88L (69% predicted) (Figure 3B). Her presenting symptoms of persistent cough and progressive dyspnea on exertion, characteristic CT, sterile BAL cultures, positive serum precipitants against pigeon serum, and resolution of her symptoms with withdrawal of the offending antigen were diagnostic of hypersensitivity pneumonitis due to pigeon exposure, also known as bird fancier’s disease.

COMMENTARY

The patient’s original presentation of dyspnea, tachypnea, and hypoxia is commonly associated with pediatric pneumonia and asthma exacerbations.1 However, an alternative diagnosis was suggested by the lack of wheezing, absence of fever, and recurrent presentations with progressive symptoms.

Hypersensitivity pneumonitis (HP) represents an exaggerated T-cell meditated immune response to inhalation of an offending antigen that results in a restrictive ventilatory defect and interstitial infiltrates.2 Bird pneumonitis (also known as bird fancier’s disease) is a frequent cause of HP, accounting for approximately 65-70% of cases.3 HP, however, only manifests in a small number of subjects exposed to culprit antigens, suggesting an underlying genetic susceptibility.4 Prevalence estimates vary depending on bird species, county, climate, and other possible factors.

There are no standard criteria for the diagnosis of HP, though a combination of findings is suggestive. A recent prospective multicenter study created a scoring system for HP based on factors associated with the disease to aid in accurate diagnosis. The most relevant criteria included antigen exposure, recurrent symptoms noted within 4-8 hours after antigen exposure, weight loss, presence of specific IgG antibodies to avian antigens, and inspiratory crackles on exam. Using this rule, the probability that our patient has HP based on clinical characteristics was 93% with an area under the receiver operating curve of 0.93 (96% confidence interval: 0.90-0.95)5. Chest imaging (high resolution CT) often consists of a mosaic pattern of air trapping, as seen in this patient in combination with ground-glass opacities6. Bronchoalveolar lavage (BAL) is sensitive in detecting lung inflammation in a patient with suspected HP. On BAL, a lymphocytic alveolitis can be seen, but absence of this finding does not exclude HP.5,7,8 Pulmonary function tests (PFTs) may be normal in acute HP. When abnormal, PFTs may reveal a restrictive pattern and reduction in carbon monoxide diffusing capacity.7 However, BAL and PFT results are neither specific nor diagnostic of HP; it is important to consider results in the context of the clinical picture.

The respiratory response to inhalation of the avian antigen has traditionally been classified as acute, subacute, or chronic.9 The acute response occurs within hours of exposure to the offending agent and usually resolves within 24 hours after antigen withdrawal. The subacute presentation involves cough and dyspnea over several days to weeks, and can progress to chronic and permanent lung damage if unrecognized and untreated. In chronic presentations, lung abnormalities may persist despite antigen avoidance and pharmacologic interventions.4,10 The patient’s symptoms occurred over a six-month period which coincided with pigeon exposure and resolved during each hospitalization with steroid treatment and removal from the offending agent. Her presentation was consistent with a subacute time course of HP.

The dilated pulmonary artery, elevated right systolic ventricular pressure, and normal right ventricular function in our patient suggested pulmonary hypertension of chronic duration. Her risk factors for pulmonary hypertension included asthma, sleep apnea, possible obesity-hypoventilation syndrome, and HP-associated interstitial lung disease.11

The most important intervention in HP is avoidance of the causative antigen. Medical therapy without removal of antigen is inadequate. Systemic corticosteroids can help ameliorate acute symptoms though dosing and duration remains unclear. For chronic patients unresponsive to steroid therapy, lung transplantation can be considered.4

The key to diagnosis of HP in this patient—and to minimizing repeat testing upon the patient’s recrudescence of symptoms—was the clinician’s consideration that the major impetus for the patient’s improvement in the hospital was removal from the offending antigen in her home environment. As in this case, taking time to delve deeply into a patient’s environment—even by descending the basement stairs—may lead to the diagnosis.

 

 

LEARNING POINTS

  • Consider hypersensitivity pneumonitis (HP) in patients with recurrent respiratory distress, offending exposure, and resolution of symptoms with removal of culprit antigen.
  • The most important treatment of HP is removal of offending antigen; systemic and/or inhaled corticosteroids are indicated until the full resolution of respiratory symptoms.
  • Prognosis is dependent on early diagnosis and removal of offending exposures.
  • Failure to treat HP might result in end-stage lung disease from pulmonary fibrosis secondary to long-term inflammation.

Disclosures

Dr. Manesh is supported by the Jeremiah A. Barondess Fellowship in the Clinical Transaction of the New York Academy of Medicine, in collaboration with the Accreditation Council for Graduate Medical Education (ACGME). The authors declare no conflicts of interests.

 

References

1. Ebell MH. Clinical diagnosis of pneumonia in children. Am Fam Physician. 2010;82(2):192-193. PubMed
2. Cormier Y, Lacasse Y. Hypersensitivity pneumonitis and organic dust toxic syndrome. In: Malo J-L, Chan-Yeung M, Bernstein DI, eds. Asthma in the Workplace. Vol 32. Boca Raton, FL: Fourth Informa Healthcare; 2013:392-405. 
3. Chan AL, Juarez MM, Leslie KO, Ismail HA, Albertson TE. Bird fancier’s lung: a state-of-the-art review. Clin Rev Allergy Immunol. 2012;43(1-2):69-83. doi: 10.1007/s12016-011-8282-y. PubMed
4. Camarena A, Juárez A, Mejía M, et al. Major histocompatibility complex and tumor necrosis factor-α polymorphisms in pigeon breeder’s disease. Am J Respir Crit Care Med. 2001;163(7):1528-1533. https:/doi.org/10.1164/ajrccm.163.7.2004023. PubMed
5. Lacasse Y, Selman M, Costabel U, et al. Clinical diagnosis of hypersensitivity pneumonitis. Am J Respir Crit Care Med. 2003;168(8):952-958. doi: 10.1164/rccm.200301-137OC. PubMed
6. Glazer CS, Rose CS, Lynch DA. Clinical and radiologic manifestations of hypersensitivity pneumonitis. J Thorac Imaging. 2002;17(4):261-272. PubMed
7. Selman M, Pardo A, King TE Jr. Hypersensitivity pneumonitis: insights in diagnosis and pathobiology. Am J Respir Crit Care Med. 2012;186(4):314-324. doi: 10.1164/rccm.201203-0513CI. PubMed
8. Calillad DM, Vergnon, JM, Madroszyk A, et al. Bronchoalveolar lavage in hypersensitivity pneumonitis: a series of 139 patients. Inflamm Allergy Drug Targets. 2012;11(1):15-19. doi: 10.2174/187152812798889330. PubMed
9. Richerson HB, Bernstein IL, Fink JN, et al. Guidelines for the clinical evaluation of hypersensitivity pneumonitis. Report of the Subcommittee on Hypersensitivity Pneumonitis. J Allergy Clin Immunol. 1989;84(5 Pt 2):839-844. doi: 10.1016/0091-6749(89)90349-7. PubMed
10. Zacharisen MC, Schlueter DP, Kurup VP, Fink JN. The long-term outcome in acute, subacute, and chronic forms of pigeon breeder’s disease hypersensitivity pneumonitis. Ann Allergy Asthma Immunol. 2002;88(2):175-182. doi: 10.1016/S1081-1206(10)61993-X. PubMed
11. Raymond TE, Khabbaza JE, Yadav R, Tonelli AR. Significance of main pulmonary artery dilation on imaging studies. Ann Am Thorac Soc. 2014;11(10):1623-1632. doi: 10.1513/AnnalsATS.201406-253PP. PubMed

References

1. Ebell MH. Clinical diagnosis of pneumonia in children. Am Fam Physician. 2010;82(2):192-193. PubMed
2. Cormier Y, Lacasse Y. Hypersensitivity pneumonitis and organic dust toxic syndrome. In: Malo J-L, Chan-Yeung M, Bernstein DI, eds. Asthma in the Workplace. Vol 32. Boca Raton, FL: Fourth Informa Healthcare; 2013:392-405. 
3. Chan AL, Juarez MM, Leslie KO, Ismail HA, Albertson TE. Bird fancier’s lung: a state-of-the-art review. Clin Rev Allergy Immunol. 2012;43(1-2):69-83. doi: 10.1007/s12016-011-8282-y. PubMed
4. Camarena A, Juárez A, Mejía M, et al. Major histocompatibility complex and tumor necrosis factor-α polymorphisms in pigeon breeder’s disease. Am J Respir Crit Care Med. 2001;163(7):1528-1533. https:/doi.org/10.1164/ajrccm.163.7.2004023. PubMed
5. Lacasse Y, Selman M, Costabel U, et al. Clinical diagnosis of hypersensitivity pneumonitis. Am J Respir Crit Care Med. 2003;168(8):952-958. doi: 10.1164/rccm.200301-137OC. PubMed
6. Glazer CS, Rose CS, Lynch DA. Clinical and radiologic manifestations of hypersensitivity pneumonitis. J Thorac Imaging. 2002;17(4):261-272. PubMed
7. Selman M, Pardo A, King TE Jr. Hypersensitivity pneumonitis: insights in diagnosis and pathobiology. Am J Respir Crit Care Med. 2012;186(4):314-324. doi: 10.1164/rccm.201203-0513CI. PubMed
8. Calillad DM, Vergnon, JM, Madroszyk A, et al. Bronchoalveolar lavage in hypersensitivity pneumonitis: a series of 139 patients. Inflamm Allergy Drug Targets. 2012;11(1):15-19. doi: 10.2174/187152812798889330. PubMed
9. Richerson HB, Bernstein IL, Fink JN, et al. Guidelines for the clinical evaluation of hypersensitivity pneumonitis. Report of the Subcommittee on Hypersensitivity Pneumonitis. J Allergy Clin Immunol. 1989;84(5 Pt 2):839-844. doi: 10.1016/0091-6749(89)90349-7. PubMed
10. Zacharisen MC, Schlueter DP, Kurup VP, Fink JN. The long-term outcome in acute, subacute, and chronic forms of pigeon breeder’s disease hypersensitivity pneumonitis. Ann Allergy Asthma Immunol. 2002;88(2):175-182. doi: 10.1016/S1081-1206(10)61993-X. PubMed
11. Raymond TE, Khabbaza JE, Yadav R, Tonelli AR. Significance of main pulmonary artery dilation on imaging studies. Ann Am Thorac Soc. 2014;11(10):1623-1632. doi: 10.1513/AnnalsATS.201406-253PP. PubMed

Issue
Journal of Hospital Medicine 14(1)
Issue
Journal of Hospital Medicine 14(1)
Page Number
51-55. Published online first November 28, 2018.
Page Number
51-55. Published online first November 28, 2018.
Publications
Publications
Topics
Article Type
Sections
Teambase XML
<?xml version="1.0" encoding="UTF-8"?>
<!--$RCSfile: InCopy_agile.xsl,v $ $Revision: 1.35 $-->
<!--$RCSfile: drupal.xsl,v $ $Revision: 1.7 $-->
<root generator="drupal.xsl" gversion="1.7"> <header> <fileName>Manesh0218</fileName> <TBEID>0C0181CE.SIG</TBEID> <TBUniqueIdentifier>NJ_0C0181CE</TBUniqueIdentifier> <newsOrJournal>Journal</newsOrJournal> <publisherName>Frontline Medical Communications Inc.</publisherName> <storyname/> <articleType>1</articleType> <TBLocation>Copyfitting-JHM</TBLocation> <QCDate/> <firstPublished>20181117T153124</firstPublished> <LastPublished>20181117T153124</LastPublished> <pubStatus qcode="stat:"/> <embargoDate/> <killDate/> <CMSDate>20181117T153124</CMSDate> <articleSource/> <facebookInfo/> <meetingNumber/> <byline/> <bylineText>The approach to clinical conundrums an expert clinician is revealed through the presentation of an actual patient’s case in an approach typical of a morning report. Similar to patient care, sequential pieces of information are provided to the clinician, who is unfamiliar with the case. The focus is on the thought processes of both the clinical team caring for the patient and the discussant.</bylineText> <bylineFull/> <bylineTitleText/> <USOrGlobal/> <wireDocType/> <newsDocType/> <journalDocType/> <linkLabel/> <pageRange/> <citation/> <quizID/> <indexIssueDate/> <itemClass qcode="ninat:text"/> <provider qcode="provider:"> <name/> <rightsInfo> <copyrightHolder> <name/> </copyrightHolder> <copyrightNotice/> </rightsInfo> </provider> <abstract/> <metaDescription>*Address for correspondence: Reza Manesh, MD, Assistant Professor of Medicine, Division of General Internal Medicine, Johns Hopkins Hospital, 600 N. Wolfe Stree</metaDescription> <articlePDF/> <teaserImage/> <title>The Basement Flight</title> <deck/> <eyebrow>CLINICAL CARE CONUNDRUM</eyebrow> <disclaimer/> <AuthorList/> <articleURL/> <doi>10.12788/jhm.3096 &#13;</doi> <pubMedID/> <publishXMLStatus/> <publishXMLVersion>1</publishXMLVersion> <useEISSN>0</useEISSN> <urgency/> <pubPubdateYear/> <pubPubdateMonth/> <pubPubdateDay/> <pubVolume/> <pubNumber/> <wireChannels/> <primaryCMSID/> <CMSIDs/> <keywords/> <seeAlsos/> <publications_g> <publicationData> <publicationCode>jhm</publicationCode> <pubIssueName/> <pubArticleType/> <pubTopics/> <pubCategories/> <pubSections/> <journalTitle/> <journalFullTitle/> <copyrightStatement/> </publicationData> </publications_g> <publications> <term canonical="true">27312</term> </publications> <sections> <term>27622</term> <term canonical="true">28090</term> </sections> <topics> <term canonical="true">327</term> </topics> <links/> </header> <itemSet> <newsItem> <itemMeta> <itemRole>Main</itemRole> <itemClass>text</itemClass> <title>The Basement Flight</title> <deck/> </itemMeta> <itemContent> <p class="affiliation"><sup>1</sup>Department of Pediatrics, The Johns Hopkins Hospital, Baltimore, Maryland; <sup>2</sup>Department of Medicine, The Johns Hopkins Hospital, Baltimore, Maryland;<sup> 3</sup>Departments of Medicine and <sup>4</sup>Pediatrics, University of Minnesota Medical School, Minneapolis, Minnesota.</p> <p>*Address for correspondence: Reza Manesh, MD, Assistant Professor of Medicine, Division of General Internal Medicine, Johns Hopkins Hospital, 600 N. Wolfe Street / Meyer 8-34D, Baltimore, MD 21287; Telephone: 412-708-6944; E-mail: rsedigh1@jhmi.edu</p> <p>Received: April 13, 2018; Revised: August 13, 2018; Accepted: September 7, 2018<br/><br/><strong>©</strong><strong> 2018 Society of Hospital Medicine DOI 10.12788/jhm.3096</strong></p> <p>A 14-year-old girl with a history of asthma presented to the Emergency Department (ED) with three months of persistent, nonproductive cough, and progressive shortness of breath. She reported fatigue, chest tightness, orthopnea, and dyspnea with exertion. She denied fever, rhinorrhea, congestion, hemoptysis, or paroxysmal nocturnal dyspnea. <br/><br/>Her age and past medical history of asthma are incongruent with her new symptoms, as asthma is typified by intermittent exacerbations, not progressive symptoms. Thus, another process, in addition to asthma, is most likely present; it is also important to question the accuracy of previous diagnoses in light of new information. Her symptoms may signify an underlying cardiopulmonary process, such as infiltrative diseases (eg, lymphoma or sarcoidosis), atypical infections, genetic conditions (eg, variant cystic fibrosis), autoimmune conditions, or cardiomyopathy. A detailed symptom history, family history, and careful physical examination will help expand and then refine the differential diagnosis. At this stage, typical infections are less likely. <br/><br/>She had presented two months prior with nonproductive cough and dyspnea. At that presentation, her temperature was 36.3°C, heart rate 110 beats per minute, blood pressure 119/63 mm Hg, respiratory rate 43 breaths per minute, and oxygen saturation 86% while breathing ambient air. A chest CT with contrast demonstrated diffuse patchy multifocal ground-glass opacities in the bilateral lungs as well as a mixture of atelectasis and lobular emphysema in the dependent lobes bilaterally (Figure 1). Her main pulmonary artery was dilated at 3.6 cm (mean of 2.42 cm with SD 0.22). She was diagnosed with atypical pneumonia. She was administered azithromycin, weaned off oxygen, and discharged after a seven-day hospitalization. <br/><br/>Two months prior, she had marked tachypnea, tachycardia, and hypoxemia, and imaging revealed diffuse ground-glass opacities. The differential diagnosis for this constellation of symptoms is extensive and includes many conditions that have an inflammatory component, such as atypical pneumonia caused by Mycoplasma or Chlamydia pneumoniae or a common respiratory virus such as rhinovirus or human metapneumovirus. However, two findings make an acute pneumonia unlikely to be the sole cause of her symptoms: underlying emphysema and an enlarged pulmonary artery. Emphysema is an uncommon finding in children and can be related to congenital or acquired causes; congenital lobar emphysema most often presents earlier in life and is focal, not diffuse. Alpha-1-anti-trypin deficiency and mutations in connective tissue genes such as those encoding for elastin and fibrillin can lead to pulmonary disease. While not diagnostic of pulmonary hypertension, her dilated pulmonary artery, coupled with her history, makes pulmonary hypertension a strong possibility. While her pulmonary hypertension is most likely secondary to chronic lung disease based on the emphysematous changes on CT, it could still be related to a cardiac etiology. <br/><br/>The patient had a history of seasonal allergies and well-controlled asthma. She was hospitalized at age six for an asthma exacerbation associated with a respiratory infection. She was discharged with an albuterol inhaler, but seldom used it. Her parents denied any regular coughing during the day or night. She was morbidly obese. Her tonsils and adenoids were removed to treat obstructive sleep apnea (OSA) at age seven, and a subsequent polysomnography was normal. Her medications included intranasal fluticasone propionate and oral iron supplementation. She had no known allergies or recent travels. She had never smoked. She had two pet cats and a dog. Her mother had a history of obesity, OSA, and eczema. Her father had diabetes and eczema.<br/><br/>The patient’s history prior to the recent few months sheds little light on the cause of her current symptoms. While it is possible that her current symptoms are related to the worsening of a process that had been present for many years which mimicked asthma, this seems implausible given the long period of time between her last asthma exacerbation and her present symptoms. Similarly, while tonsillar and adenoidal hypertrophy can be associated with infiltrative diseases (such as lymphoma), this is less common than the usual (and normal) disproportionate increase in size of the adenoids compared to other airway structures during growth in children. <br/><br/>She was admitted to the hospital. On initial examination, her temperature was 37.4°C, heart rate 125 beats per minute, blood pressure 143/69 mm Hg, respiratory rate 48 breaths per minute, and oxygen saturation 86% breathing ambient air. Her BMI was 58 kg/m2. Her exam demonstrated increased work of breathing with accessory muscle use, and decreased breath sounds at the bases. There were no wheezes or crackles. Cardiovascular, abdominal, and skin exams were normal except for tachycardia. At rest, later in the hospitalization, her oxygen saturation was 97% breathing ambient air and heart rate 110 bpm. After two minutes of walking, her oxygen saturation was 77% and heart rate 132 bpm. Two minutes after resting, her oxygen saturation increased to 91%. </p> <p> <strong>Her white blood cell count was 11.9 x 10</strong> <strong> <sup>9</sup> </strong> <strong>/L (67% neutrophils, 24.2% lymphocytes, 6% monocytes, and 2% eosinophils), hemoglobin 11.2 g/dL, and platelet count 278,000/mm</strong> <strong> <sup>3</sup> </strong> <strong>. Her complete metabolic panel was normal. The C-reactive protein (CRP) was 24 mg/L (normal range, &lt; 4.9) and erythrocyte sedimentation rate (ESR) 103 mm/hour (normal range, 0-32). A venous blood gas (VBG) showed a pH of 7.42 and pCO2 39. An EKG demonstrated sinus tachycardia.</strong> </p> <p>The combination of the patient’s tachypnea, hypoxemia, respiratory distress, and obesity is striking. Her lack of adventitious lung sounds is surprising given her CT findings, but the sensitivity of chest auscultation may be limited in obese patients. Her laboratory findings help narrow the diagnostic frame: she has mild anemia and leukocytosis along with significant inflammation. The normal CO<sub>2</sub> concentration on VBG is concerning given the degree of her tachypnea and reflects significant alveolar hypoventilation. </p> <p>This marked inflammation with diffuse lung findings again raises the possibility of an inflammatory or, less likely, infectious disorder. Sjogren’s syndrome, systemic lupus erythematosus (SLE), and juvenile dermatomyositis can present in young women with interstitial lung disease. She does have exposure to pets and hypersensitivity pneumonitis can worsen rapidly with continued exposure. Another possibility is that she has an underlying immunodeficiency such as common variable immunodeficiency, although a history of recurrent infections such as pneumonia, bacteremia, or sinusitis is lacking. <br/><br/>An echocardiogram should be performed. In addition, laboratory evaluation for the aforementioned autoimmune causes of interstitial lung disease, immunoglobulin levels, pulmonary function testing (if available as an inpatient), and potentially a bronchoscopy with bronchoalveolar lavage (BAL), and biopsy should be pursued. The BAL and biopsy would be helpful in evaluating for infection and interstitial lung disease in an expeditious manner.</p> <p>A chest CT without contrast was done and compared to the scan from two months prior. New diffuse, ill-defined centrilobular ground-glass opacities were evident throughout the lung fields; dilation of the main pulmonary artery was unchanged, and previously seen ground-glass opacities had resolved. There were patchy areas of air-trapping and mosaic attenuation in the lower lobes (Figure 2). Transthoracic echocardiogram demonstrated a right ventricular systolic pressure of 58 mm Hg with flattened intraventricular septum during systole. Left and right ventricular systolic function were normal. The left ventricular diastolic function was normal. Pulmonary function testing demonstrated a FEV1/FVC ratio of 100 (112% predicted), FVC 1.07 L (35 % predicted) and FEV1 1.07 L (39% predicted), and total lung capacity was 2.7L (56% predicted) (Figure 3). Single-breath carbon monoxide uptake in the lung was not interpretable based on 2017 European Respiratory Society (ERS)/American Thoracic Society (ATS) technical standards.<br/><br/>This information is helpful in classifying whether this patient’s primary condition is cardiac or pulmonary in nature. Her normal left ventricular systolic and diastolic function make a cardiac etiology for her pulmonary hypertension less likely. Further, the combination of pulmonary hypertension, a restrictive pattern on pulmonary function testing, and findings consistent with interstitial lung disease on cross-sectional imaging all suggest a primary pulmonary etiology rather than a cardiac, infectious, or thromboembolic condition. While chronic thromboembolic hypertension can result in nonspecific mosaic attenuation, it typically would not cause centrilobular ground-glass opacities nor restrictive lung disease. Thus, it seems most likely that this patient has a progressive pulmonary process resulting in hypoxia, pulmonary hypertension, centrilobular opacities, and lower-lobe mosaic attenuation. Considerations for this process can be broadly categorized as one of the childhood interstitial lung disease (chILD). While this differential diagnosis is broad, strong consideration should be given to hypersensitivity pneumonitis, chronic aspiration, sarcoidosis, and Sjogren’s syndrome. An intriguing possibility is that the patient’s “response to azithromycin” two months prior was due to the avoidance of an inhaled antigen while she was in the hospital; a detailed environmental history should be explored. The normal polysomnography after tonsilloadenoidectomy makes it unlikely that OSA is a major contributor to her current presentation. However, since the surgery was seven years ago, and her BMI is presently 58 kg/m<sup>2</sup> she remains at risk for OSA and obesity-hypoventilation syndrome. Polysomnography should be done after her acute symptoms improve. <br/><br/>She was started on 5 mm Hg of continuous positive airway pressure (CPAP) at night after a sleep study on room air demonstrated severe OSA with a respiratory disturbance index of 13 events per hour. Antinuclear antibodies (ANA), anti-neutrophil cytoplasmic antibody (ANCA), anti-Jo-1 antibody, anti-RNP antibody, anti-Smith antibody, anti-Ro/SSA and anti-La/SSB antibody were negative as was the histoplasmin antibody. Serum angiotensin-converting enzyme (ACE) level was normal. Mycoplasma IgM and IgG were negative. IgE was 529 kU/L (normal range, &lt;114).<br/><br/>This evaluation reduces the likelihood the patient has Sjogren’s syndrome, SLE, dermatomyositis, or ANCA-associated pulmonary disease. While many patients with dermatomyositis may have negative serologic evaluations, other findings usually present such as rash and myositis are lacking. The negative ANCA evaluation makes granulomatosis with polyangiitis and microscopic polyangiitis very unlikely given the high sensitivity of the ANCA assay for these conditions. ANCA assays are less sensitive for eosinophilic granulomatosis with polyangiitis (EGPA), but the lack of eosinophilia significantly decreases the likelihood of EGPA. ACE levels have relatively poor operating characteristics in the evaluation of sarcoidosis; however, sarcoidosis seems unlikely in this case, especially as patients with sarcoidosis tend to have low or normal IgE levels. Patients with asthma can have elevated IgE levels. However, very elevated IgE levels are more common in other conditions, including allergic bronchopulmonary aspergillosis (ABPA) and the Hyper-IgE syndrome. The latter manifests with recurrent infections and eczema, and is inherited in an autosomal dominant manner. However, both the Hyper-IgE syndrome and ABPA have much higher IgE levels than seen in this case. Allergen-specific IgE testing (including for antibodies to Aspergillus) should be sent. It seems that an interstitial lung disease is present; the waxing and waning pattern and clinical presentation, along with the lack of other systemic findings, make hypersensitivity pneumonitis most likely.<br/><br/>The family lived in an apartment building. Her symptoms started when the family’s neighbor recently moved his outdoor pigeon coop into his basement. The patient often smelled the pigeons and noted feathers coming through the holes in the wall. <br/><br/>One of the key diagnostic features of hypersensitivity pneumonitis (HP) is the history of exposure to a potential offending antigen—in this case likely bird feathers—along with worsening upon reexposure to that antigen. HP is primarily a clinical diagnosis, and testing for serum precipitants has limited value, given the high false negative rate and the frequent lack of clinical symptoms accompanying positive testing. Bronchoalveolar lavage fluid may reveal lymphocytosis and reduced CD4:CD8 ratio. Crackles are commonly heard on examination, but in this case were likely not auscultated due to her obese habitus. The most important treatment is withdrawal of the offending antigen. Limited data suggest that corticosteroid therapy may be helpful in certain HP cases, including subacute, chronic and severe cases as well as patients with hypoxemia, significant imaging findings, and those with significant abnormalities on pulmonary function testing (PFT). <br/><br/>A hypersensitivity pneumonitis precipitins panel was sent with positive antibodies to M. faeni, T. Vulgaris, A. Fumigatus 1 and 6, A. Flavus, and pigeon serum. Her symptoms gradually improved within five days of oral prednisone (60 mg). She was discharged home without dyspnea and normal oxygen saturation while breathing ambient air. A repeat echocardiogram after nighttime CPAP for 1 week demonstrated a right ventricular systolic pressure of 17 mm Hg consistent with improved pulmonary hypertension. </p> <p>Three weeks later, she returned to clinic for follow up. She had re-experienced dyspnea, cough, and wheezing, which improved when she was outdoors. She was afebrile, tachypneic, tachycardic, and her oxygen saturation was 92% on ambient air. </p> <p>Her steroid-responsive interstitial lung disease and rapid improvement upon avoidance of the offending antigen is consistent with HP. The positive serum precipitins assay lends further credence to the diagnosis of HP, although serologic analysis with such antibody assays is limited by false positives and false negatives; further, individuals exposed to pigeons often have antibodies present without evidence of HP. History taking at this visit should ask specifically about further pigeon exposure: were the pigeons removed from the home completely, were heating-cooling filters changed, carpets cleaned, and bedding laundered? An in-home evaluation may be helpful before conducting further diagnostic testing. <br/><br/>She was admitted for oxygen therapy and a bronchoscopy, which showed mucosal friability and cobblestoning, suggesting inflammation. BAL revealed a normal CD4:CD8 ratio of 3; BAL cultures were sterile. Her shortness of breath significantly improved following a prolonged course of systemic steroids and removal from the triggering environment. PFTs improved with a FEV1/FVC ratio of 94 (105% predicted), FVC of 2.00 L (66% predicted), FEV1 of 1.88L (69% predicted) (Figure 3B). Her presenting symptoms of persistent cough and progressive dyspnea on exertion, characteristic CT, sterile BAL cultures, positive serum precipitants against pigeon serum, and resolution of her symptoms with withdrawal of the offending antigen were diagnostic of hypersensitivity pneumonitis due to pigeon exposure, also known as bird fancier’s disease. </p> <h2>Commentary</h2> <p>The patient’s original presentation of dyspnea, tachypnea, and hypoxia is commonly associated with pediatric pneumonia and asthma exacerbations.<sup><a href="https://paperpile.com/c/pTSVGK/Sph3">1</a></sup> However, an alternative diagnosis was suggested by the lack of wheezing, absence of fever, and recurrent presentations with progressive symptoms.</p> <p>Hypersensitivity pneumonitis (HP) represents an exaggerated T-cell meditated immune response to inhalation of an offending antigen that results in a restrictive ventilatory defect and interstitial infiltrates.<sup>2</sup> Bird pneumonitis (also known as bird fancier’s disease) is a frequent cause of HP, accounting for approximately 65-70% of cases.<sup><a href="https://paperpile.com/c/pTSVGK/aq4N">3</a></sup> HP, however, only manifests in a small number of subjects exposed to culprit antigens, suggesting an underlying genetic susceptibility.<sup><a href="https://paperpile.com/c/pTSVGK/OOp8">4</a></sup> Prevalence estimates vary depending on bird species, county, climate, and other possible factors. <br/><br/>There are no standard criteria for the diagnosis of HP, though a combination of findings is suggestive. A recent prospective multicenter study created a scoring system for HP based on factors associated with the disease to aid in accurate diagnosis. The most relevant criteria included antigen exposure, recurrent symptoms noted within 4-8 hours after antigen exposure, weight loss, presence of specific IgG antibodies to avian antigens, and inspiratory crackles on exam. Using this rule, the probability that our patient has HP based on clinical characteristics was 93% with an area under the receiver operating curve of 0.93 (96% confidence interval: 0.90-0.95)<sup>5</sup>. Chest imaging (high resolution CT) often consists of a mosaic pattern of air trapping, as seen in this patient in combination with ground-glass opacities<span class="Annotation"><sup>6</sup></span>. Bronchoalveolar lavage (BAL) is sensitive in detecting lung inflammation in a patient with suspected HP. On BAL, a lymphocytic alveolitis can be seen, but absence of this finding does not exclude HP.<sup>5,7,8</sup> Pulmonary function tests (PFTs) may be normal in acute HP. When abnormal, PFTs may reveal a restrictive pattern and reduction in carbon monoxide diffusing capacity.<sup>7</sup> However, BAL and PFT results are neither specific nor diagnostic of HP; it is important to consider results in the context of the clinical picture. <br/><br/>The respiratory response to inhalation of the avian antigen has traditionally been classified as acute, subacute, or chronic.<sup>9</sup> The acute response occurs within hours of exposure to the offending agent and usually resolves within 24 hours after antigen withdrawal. The subacute presentation involves cough and dyspnea over several days to weeks, and can progress to chronic and permanent lung damage if unrecognized and untreated. In chronic presentations, lung abnormalities may persist despite antigen avoidance and pharmacologic interventions.<sup>4,10</sup> The patient’s symptoms occurred over a six-month period which coincided with pigeon exposure and resolved during each hospitalization with steroid treatment and removal from the offending agent. Her presentation was consistent with a subacute time course of HP. <br/><br/>The dilated pulmonary artery, elevated right systolic ventricular pressure, and normal right ventricular function in our patient suggested pulmonary hypertension of chronic duration. Her risk factors for pulmonary hypertension included asthma, sleep apnea, possible obesity-hypoventilation syndrome, and HP-associated interstitial lung disease.<sup>11<br/><br/></sup>The most important intervention in HP is avoidance of the causative antigen. Medical therapy without removal of antigen is inadequate. Systemic corticosteroids can help ameliorate acute symptoms though dosing and duration remains unclear. For chronic patients unresponsive to steroid therapy, lung transplantation can be considered.<sup><a href="https://paperpile.com/c/pTSVGK/aq4N">4</a></sup><br/><br/>The key to diagnosis of HP in this patient—and to minimizing repeat testing upon the patient’s recrudescence of symptoms—was the clinician’s consideration that the major impetus for the patient’s improvement in the hospital was removal from the offending antigen in her home environment. As in this case, taking time to delve deeply into a patient’s environment—even by descending the basement stairs—may lead to the diagnosis. </p> <h2>Learning Points</h2> <ul class="body"> <li>Consider hypersensitivity pneumonitis (HP) in patients with recurrent respiratory distress, offending exposure, and resolution of symptoms with removal of culprit antigen.</li> <li>The most important treatment of HP is removal of offending antigen; systemic and/or inhaled corticosteroids are indicated until the full resolution of respiratory symptoms.</li> <li>Prognosis is dependent on early diagnosis and removal of offending exposures.</li> <li>Failure to treat HP might result in end-stage lung disease from pulmonary fibrosis secondary to long-term inflammation.</li> </ul> <p>Disclosures: Dr. Manesh is supported by the Jeremiah A. Barondess Fellowship in the Clinical Transaction of the New York Academy of Medicine, in collaboration with the Accreditation Council for Graduate Medical Education (ACGME). The authors declare no conflicts of interests.</p> <p class="references">1. <a href="http://paperpile.com/b/pTSVGK/Sph3">Ebell MH. Clinical diagnosis of pneumonia in children. Am Fam Physician. 2010;82(2):192-193.</a><br/><br/>2. <a href="http://paperpile.com/b/pTSVGK/T3Et">Cormier Y, Lacasse Y. Hypersensitivity pneumonitis and organic dust toxic syndrome. In: Malo J-L, Chan-Yeung M, Bernstein DI, eds. Asthma in the Workplace. Vol 32. Boca Raton, FL: Fourth Informa Healthcare; 2013:392-405.</a><br/><br/>3. <a href="http://paperpile.com/b/pTSVGK/aq4N">Chan AL, Juarez MM, Leslie KO, Ismail HA, Albertson TE. Bird fancier’s lung: a state-of-the-art review. Clin Rev Allergy Immunol. 2012;43(1-2):69-83.</a> doi: 10.1007/s12016-011-8282-y.<br/><br/>4. <a href="http://paperpile.com/b/pTSVGK/OOp8">Camarena A, Juárez A, Mejía M, et al. Major histocompatibility complex and tumor necrosis factor-α polymorphisms in pigeon breeder’s disease. Am J Respir Crit Care Med. 2001;163(7):1528-1533.</a> https:/doi.org/10.1164/ajrccm.163.7.2004023.<br/><br/>5. <a href="http://paperpile.com/b/pTSVGK/fG40">Lacasse Y, Selman M, Costabel U, et al. Clinical diagnosis of hypersensitivity pneumonitis. Am J Respir Crit Care Med. 2003;168(8):952-958.</a> doi: 10.1164/rccm.200301-137OC.<br/><br/>6. <a href="http://paperpile.com/b/pTSVGK/stWc">Glazer CS, Rose CS, Lynch DA. Clinical and radiologic manifestations of hypersensitivity pneumonitis. J Thorac Imaging. 2002;17(4):261-272.</a><br/><br/>7. Selman M, Pardo A, King TE Jr. Hypersensitivity pneumonitis: insights in diagnosis and pathobiology. Am J Respir Crit Care Med. 2012;186(4):314-324. doi: 10.1164/rccm.201203-0513CI.<br/><br/>8. Calillad DM, Vergnon, JM, Madroszyk A, et al. Bronchoalveolar lavage in hypersensitivity pneumonitis: a series of 139 patients. Inflamm Allergy Drug Targets. 2012;11(1):15-19. doi: 10.2174/187152812798889330.<br/><br/>9. <a href="http://paperpile.com/b/pTSVGK/poiJ">Richerson HB, Bernstein IL, Fink JN, et al. Guidelines for the clinical evaluation of hypersensitivity pneumonitis. Report of the Subcommittee on Hypersensitivity Pneumonitis. J Allergy Clin Immunol. 1989;84(5 Pt 2):839-844.</a> doi: 10.1016/0091-6749(89)90349-7.<br/><br/>10. <a href="http://paperpile.com/b/pTSVGK/rcBe">Zacharisen MC, Schlueter DP, Kurup VP, Fink JN. The long-term outcome in acute, subacute, and chronic forms of pigeon breeder’s disease hypersensitivity pneumonitis. Ann Allergy Asthma Immunol. 2002;88(2):175-182.</a> doi: 10.1016/S1081-1206(10)61993-X.<br/><br/>11. Raymond TE, Khabbaza JE, Yadav R, Tonelli AR. Significance of main pulmonary artery dilation on imaging studies. Ann Am Thorac Soc. 2014;11(10):1623-1632. doi: 10.1513/AnnalsATS.201406-253PP.</p> </itemContent> </newsItem> </itemSet></root>
Article Source

© 2019 Society of Hospital Medicine

Disallow All Ads
Correspondence Location
Reza Manesh, MD, Assistant Professor of Medicine, Division of General Internal Medicine, Johns Hopkins Hospital, 600 N. Wolfe Street / Meyer 8-34D, Baltimore, MD 21287; Telephone: 412-708-6944; E-mail: rsedigh1@jhmi.edu
Content Gating
Gated (full article locked unless allowed per User)
Alternative CME
Disqus Comments
Default
Use ProPublica
Gating Strategy
First Peek Free
Article PDF Media